Các phương pháp chứng minh bất đẳng thức – Nguyễn Tất Thu

Tài liệu gồm 174 trang, được biên soạn bởi thầy giáo Nguyễn Tất Thu (giáo viên Toán trường THPT chuyên Lương Thế Vinh, tỉnh Đồng Nai), hướng dẫn các phương pháp chứng minh bất đẳng thức, giúp học sinh học tốt chương trình Đại số 10 chương 4: bất đẳng thức và bất phương trình và ôn thi chọn học sinh giỏi môn Toán bậc THPT.a

Mục lục
1 Các bất đẳng thức cổ điển 3
1 Bất đẳng thức AM - GM . . . . . . . . . . . . . . . . . . . . . . . . . . . . . . . . 3
I. Bất đẳng thức AM - GM . . . . . . . . . . . . . . . . . . . . . . . . . . . . 3
II. Một số dụ áp dụng . . . . . . . . . . . . . . . . . . . . . . . . . . . . . . 5
III. Bài tập . . . . . . . . . . . . . . . . . . . . . . . . . . . . . . . . . . . . . 13
2 Bất đẳng thức Cauchy - Schwarz . . . . . . . . . . . . . . . . . . . . . . . . . . . 19
I. Bất đẳng thức Cauchy-Schwarz dạng đa thức . . . . . . . . . . . . . . . . 19
II. Bất đẳng thức Cauchy-Schwarz dạng phân thức . . . . . . . . . . . . . . . 19
III. Các dụ minh họa . . . . . . . . . . . . . . . . . . . . . . . . . . . . . . . 19
IV. Bài tập . . . . . . . . . . . . . . . . . . . . . . . . . . . . . . . . . . . . . 27
3 Một số bất đẳng thức khác . . . . . . . . . . . . . . . . . . . . . . . . . . . . . . . 31
I. Bất đẳng thức Schur . . . . . . . . . . . . . . . . . . . . . . . . . . . . . . 31
1. Bất đẳng thức Schur . . . . . . . . . . . . . . . . . . . . . . . . . 31
2. Các trường hợp đặc biệt . . . . . . . . . . . . . . . . . . . . . . . 31
3. Bất đẳng thức Schur mở rộng . . . . . . . . . . . . . . . . . . . . 31
4. Các dụ . . . . . . . . . . . . . . . . . . . . . . . . . . . . . . . 31
II. Bất đẳng thức Holder . . . . . . . . . . . . . . . . . . . . . . . . . . . . . 34
1. Bất đẳng thức Holder . . . . . . . . . . . . . . . . . . . . . . . . 34
2. Trường hợp đặc biệt . . . . . . . . . . . . . . . . . . . . . . . . . 34
3. dụ minh họa . . . . . . . . . . . . . . . . . . . . . . . . . . . . 34
III. Bất đẳng thức Chebyshev . . . . . . . . . . . . . . . . . . . . . . . . . . . 35
1. Bất đẳng thức Chebyshev . . . . . . . . . . . . . . . . . . . . . . 35
2. dụ minh họa . . . . . . . . . . . . . . . . . . . . . . . . . . . . 36
IV. Bài tập . . . . . . . . . . . . . . . . . . . . . . . . . . . . . . . . . . . . . 36
4 Phương pháp quy nạp . . . . . . . . . . . . . . . . . . . . . . . . . . . . . . . . . 38
I. thuyết . . . . . . . . . . . . . . . . . . . . . . . . . . . . . . . . . . . . 38
II. dụ minh họa . . . . . . . . . . . . . . . . . . . . . . . . . . . . . . . . . 38
5 Phương pháp phân tích bình phương SOS . . . . . . . . . . . . . . . . . . . . . . 42
I. thuyết . . . . . . . . . . . . . . . . . . . . . . . . . . . . . . . . . . . . 42
1. Một số tiêu chuẩn đánh giá . . . . . . . . . . . . . . . . . . . . . 42
2. Một số biểu diễn sở . . . . . . . . . . . . . . . . . . . . . . . . 42
II. Các dụ . . . . . . . . . . . . . . . . . . . . . . . . . . . . . . . . . . . . 43
III. Bài tập . . . . . . . . . . . . . . . . . . . . . . . . . . . . . . . . . . . . . 46
6 Phương pháp dồn biến . . . . . . . . . . . . . . . . . . . . . . . . . . . . . . . . . 48
I. thuyết . . . . . . . . . . . . . . . . . . . . . . . . . . . . . . . . . . . . 48
II. dụ minh họa . . . . . . . . . . . . . . . . . . . . . . . . . . . . . . . . . 48
III. Bài tập . . . . . . . . . . . . . . . . . . . . . . . . . . . . . . . . . . . . . 51
2 Các phương pháp chứng minh bất đẳng thức hiện đại 53
1 Phương pháp p, q, r . . . . . . . . . . . . . . . . . . . . . . . . . . . . . . . . . . 54
I. thuyết . . . . . . . . . . . . . . . . . . . . . . . . . . . . . . . . . . . . 54
1. Bất đẳng thức Schur . . . . . . . . . . . . . . . . . . . . . . . . . 54
1
MỤC LỤC
2. Một số biểu diễn đa thức đối xứng ba biến qua p, q, r . . . . . . 54
3. Một số đánh giá giữa p, q, r . . . . . . . . . . . . . . . . . . . . . 55
II. Một số dụ . . . . . . . . . . . . . . . . . . . . . . . . . . . . . . . . . . . 55
III. Bài tập . . . . . . . . . . . . . . . . . . . . . . . . . . . . . . . . . . . . . 56
2 Phương pháp sử dụng tiếp tuyến và cát tuyến . . . . . . . . . . . . . . . . . . . . 58
I. thuyết . . . . . . . . . . . . . . . . . . . . . . . . . . . . . . . . . . . . 58
1. Hàm lồi - Dấu hiệu hàm lồi . . . . . . . . . . . . . . . . . . . . . 58
2. Bất đẳng thức tiếp tuyến - Bất đẳng thức cát tuyến . . . . . . . 58
II. Các dụ minh họa . . . . . . . . . . . . . . . . . . . . . . . . . . . . . . . 59
III. Bài tập . . . . . . . . . . . . . . . . . . . . . . . . . . . . . . . . . . . . . 66
3 Một số chuyên đề 68
1 Ứng dụng điều kiện nghiệm của phương trình bậc ba trong chứng minh bất
đẳng thức . . . . . . . . . . . . . . . . . . . . . . . . . . . . . . . . . . . . . . . . 68
I. thuyết . . . . . . . . . . . . . . . . . . . . . . . . . . . . . . . . . . . . 68
1. Mở đầu . . . . . . . . . . . . . . . . . . . . . . . . . . . . . . . . 68
2. Một số kết quả . . . . . . . . . . . . . . . . . . . . . . . . . . . . 68
II. dụ minh họa . . . . . . . . . . . . . . . . . . . . . . . . . . . . . . . . . 70
III. Bài tập . . . . . . . . . . . . . . . . . . . . . . . . . . . . . . . . . . . . . 74
2 Bài toán tìm hằng số tốt nhất trong bất đẳng thức . . . . . . . . . . . . . . . . . 75
I. thuyết . . . . . . . . . . . . . . . . . . . . . . . . . . . . . . . . . . . . 75
II. dụ minh họa . . . . . . . . . . . . . . . . . . . . . . . . . . . . . . . . . 75
III. Bài tập . . . . . . . . . . . . . . . . . . . . . . . . . . . . . . . . . . . . . 82
1 Các bất đẳng thức cổ điển 86
1 Bất đẳng thức AM-GM . . . . . . . . . . . . . . . . . . . . . . . . . . . . . . . . . 86
2 Bất đẳng thức Cauchy-Schwarz . . . . . . . . . . . . . . . . . . . . . . . . . . . . 109
3 Một số bất đẳng thức khác . . . . . . . . . . . . . . . . . . . . . . . . . . . . . . . 124
2 Một số phương pháp chứng minh bất đẳng thức 129
1 Phương pháp quy nạp . . . . . . . . . . . . . . . . . . . . . . . . . . . . . . . . . 129
2 Phương pháp phân tích bình phương SOS . . . . . . . . . . . . . . . . . . . . . . 130
3 Phương pháp dồn biến . . . . . . . . . . . . . . . . . . . . . . . . . . . . . . . . . 135
4 Phương pháp p, q, r . . . . . . . . . . . . . . . . . . . . . . . . . . . . . . . . . . 148
5 Phương pháp tiếp tuyến và cát tuyến . . . . . . . . . . . . . . . . . . . . . . . . . 150
3 Một số chuyên đề 156
1 Ứng dụng đều kiện nghiệm của phương trình bậc ba . . . . . . . . . . . . . . . 156
2 Bài toán tìm hằng số tốt nhất . . . . . . . . . . . . . . . . . . . . . . . . . . . . . 159
2
Chương 1
Các bất đẳng thức cổ điển
§1. Bất đẳng thức AM - GM
Bất đẳng thức AM GM bất đẳng thức cổ điển được sử dụng nhiều trong các bài toán chứng
minh bất đẳng thức. Ta biết trung bình cộng của nsố thực a
1
,a
2
, ··· ,a
n
số
a
1
+ a
2
+ ··· + a
n
n
và trung bình nhân của n số đó
n
a
1
a
2
···a
n
(với điều kiện
n
a
1
a
2
···a
n
tồn tại). Bất đẳng
thức AM GM cho chúng ta đánh giá giữa trung bình cộng của các số thực không âm và trung
bình nhân của chúng. Cụ thể như sau:
I. Bất đẳng thức AM - GM
Định 1. Cho n số thực không âm a
1
, a
2
, ···, a
n
. ta có
a
1
+ a
2
+ ··· + a
n
n
n
a
1
· a
2
···a
n
.
Đẳng thức xảy ra khi a
1
= a
2
= ··· = a
n
.
Chứng minh. nhiều cách đề chứng minh bất đẳng thức AM GM, dưới đây ta sẽ chứng
minh bất đẳng thức AM GM bằng phương pháp quy nạp.
Trước hết ta chứng minh bất đẳng thức AM GM cho trường hợp n = 2. Tức là, cần chứng
minh
a
1
+ a
2
2
a
1
· a
2
Bất đẳng thức y tương đương với
a
1
+ a
2
2
a
1
a
2
(
a
1
a
2
)
2
0.
Bất đẳng thức cuối hiển nhiên đúng. Đẳng thức xảy ra khi a
1
= a
2
.
Tiếp theo ta chứng minh cho trường hợp n = 4. Tức cần chứng minh
a
1
+ a
2
+ a
3
+ a
4
4
4
a
1
· a
2
· a
3
· a
4
.
Áp dụng trường hợp n = 2 ta
a
1
+ a
2
2
a
1
· a
2
và
a
3
+ a
4
2
a
3
· a
4
.
Do đó
a
1
+ a
2
+ a
3
+ a
4
4
=
a
1
+ a
2
2
+
a
3
+ a
4
2
2
a
1
a
2
+
a
3
a
4
2
4
a
1
a
2
a
3
a
4
.
3
1. BẤT ĐẲNG THỨC AM - GM
Nên trường hợp n = 4 được chứng minh.
Tiếp đến ta chứng minh trường hợp n = 3, tức chứng minh
a
1
+ a
2
+ a
3
3
3
a
1
· a
2
· a
3
Đặt a
4
=
a
1
+ a
2
+ a
3
3
. Áp dụng cho trường hợp n = 4 ta
a
1
+ a
2
+ a
3
+ a
4
4
4
a
1
· a
2
· a
3
· a
4
,
hay
a
1
+ a
2
+ a
3
+
a
1
+ a
2
+ a
3
3
4
4
r
a
1
· a
2
· a
3
·
a
1
+ a
2
+ a
3
3
Suy ra
a
1
+ a
2
+ a
3
3
3
a
1
· a
2
· a
3
(đpcm).
Để chứng minh cho trường hợp tổng quát ta chứng minh theo hai bước sau:
Bước 1: Ta chứng minh bất đẳng thức đúng với n = 2
m
+) Với m = 1, ta n = 2nên bất đẳng thức đúng với m = 1
+) Giả sử bất đẳng thức đúng với n = 2
m1
, ta chứng minh bất đẳng thức đúng với n = 2
m
.
Tức
a
1
+ a
2
+ ··· + a
2
m1
+ ··· + a
n
n
n
a
1
a
2
···a
n
. (1)
Đặt
x =
a
1
+ a
2
+ ··· + a
2
m1
2
m1
, y =
a
2
m1
+1
+ a
2
m1
+2
+ ··· + a
2
m
2
m1
Theo giả thiết quy nạp ta
x
2
m1
a
1
a
2
···a
2
m1
,y
2
m1
a
2
m1
+1
···a
n
.
Áp dụng cho trường hợp n = 2 ta có:
x + y
2
xy
hay
a
1
+ a
2
+ ··· + a
2
m1
+ a
2
m1
+1
+ ··· + a
n
2
m
2
m
a
1
a
2
···a
n
Hay (1) được chứng minh.
Bước 2: Ta chứng minh nếu bất đẳng thức đúng với n 2 thì cũng đúng với n 1
Gải sử
a
1
+ a
2
+ ··· + a
n
n
n
a
1
a
2
···a
n
Ta chứng minh
a
1
+ a
2
+ ··· + a
n1
n 1
n1
a
1
· a
2
···a
n1
.
Thật vy: Đặt a
n
=
a
1
+ a
2
+ ··· + a
n1
n 1
. ÁP dụng bất đẳng thức AM-GM cho n số ta
a
1
+ a
2
+ ··· + a
n
n
n
a
1
a
2
···a
n
,
hay
a
1
+ a
2
+ ··· +
a
1
+ a
2
+ ··· + a
n1
n 1
n
n
r
a
1
a
2
···a
n1
·
a
1
+ a
2
+ ··· + a
n1
n 1
.
4
1. BẤT ĐẲNG THỨC AM - GM
Suy ra
a
1
+ a
2
+ ··· + a
n1
n 1
n1
a
1
· a
2
···a
n1
(đpcm).
Từ hai bước trên ta bất đẳng thức AM GM được chứng minh.
Hệ quả 1. Cho các số thực dương a
1
,a
2
, ··· ,a
n
. Ta
1
a
1
+
1
a
2
+ ··· +
1
a
n
n
2
a
1
+ a
2
+ ··· + a
n
.
Đẳng thức xảy ra khi a
1
= a
2
= ··· = a
n
.
II. Một số dụ áp dụng
dụ 1.1. Cho a,b,c > 0 thỏa a
2
+ b
2
+ c
2
= 3. Chứng minh rằng
a
5
+ b
5
+ c
5
3.
Áp dụng bất đẳng thức AM-GM ta
a
5
+ a
5
+ 1 + 1 + 1 3a
2
hay 2a
5
+ 3 3a
2
.
Tương tự
2b
5
+ 3 3b
2
và 2c
5
+ 3 3c
2
.
Cộng ba bất đẳng thức trên ta đpcm.
Nhận xét 1. Ta bài toán tổng quát như sau:
Cho a,b,c > 0 thỏa mãn a + b + c = 3 (hoặc abc = 1) và m,n N,m n. Khi đó
a
m
+ b
m
+ c
m
a
n
+ b
n
+ c
n
(1).
Bất đẳng thức (1) còn đúng khi m,n các số hữu tỉ dương. Và ta thể tổng quát 3 biến thành
k biến.
dụ 1.2. Cho a,b,c > 0 thỏa a + 4b + 9c = 6.Chứng minh rằng
a
3
+ b
3
+ c
3
1
6
.
Xét x, y, z các số thực dương. Áp dụng bất đẳng thức AM-GM ta
a
3
+ 2x
3
= a
3
+ x
3
+ x
3
3x
2
a,
đẳng thức xảy ra khi a = x.
Tương tự ta cũng có:
b
3
+ 2y
3
3y
2
b, c
3
+ 2z
3
3y
2
c.
Đẳng thức xảy ra khi b = y, c = z. Cộng các bất đẳng thức trên theo vế ta được
a
3
+ b
3
+ c
3
3(x
2
a + y
2
b + z
2
c) 2(x
3
+ y
3
+ z
3
).
5
1. BẤT ĐẲNG THỨC AM - GM
Ta chọn x, y, z sao cho
x + 4y + 9z = a + 4b + 9c = 6
x
2
1
=
y
2
4
=
z
2
9
= t
2
x =
1
6
y =
1
3
z =
1
2
.
Do đó
a
3
+ b
3
+ c
3
3t
2
(a + 4b + 9c) 2(x
3
+ y
3
+ z
3
) =
1
6
.
dụ 1.3. Cho a, b, c > 0 thỏa ab + bc + ca = 3. Chứng minh rằng
a
3
+ b
3
+ c
3
3.
Áp dụng bất đẳng thức AM-GM ta
a
3
+ b
3
+ 1 3ab
b
3
+ c
3
+ 1 3bc
c
3
+ a
3
+ 1 3ca.
Cộng ba bất đẳng thức trên ta đpcm.
dụ 1.4. Cho các số thực dương a, b, c tổng bình phương bằng 3. Chứng minh rằng
ab
c
+
bc
a
+
ca
b
3.
Gọi P vế trái của bất đẳng thức cần chứng minh, ta
P
2
=
ab
c
+
bc
a
+
ca
b
2
=
a
2
b
2
c
2
+
c
2
b
2
a
2
+
c
2
a
2
b
2
+ 2(a
2
+ b
2
+ c
2
)
=
1
2
a
2
b
2
c
2
+
c
2
b
2
a
2
+
1
2
c
2
b
2
a
2
+
c
2
a
2
b
2
+
1
2
a
2
b
2
c
2
+
c
2
a
2
b
2
+ 6
b
2
+ c
2
+ a
2
+ 6 = 9.
Suy ra P 3. Đẳng thức xảy ra khi a = b = c = 1.
dụ 1.5. Cho a, b, c > 0 và a + b + c = abc. Chứng minh rằng :
a
b
3
+
b
c
3
+
c
a
3
1.
Ta bất đẳng thức cần chứng minh tương đương với:
abc
a
b
3
+
b
c
3
+
c
a
3
a + b + c.
6
1. BẤT ĐẲNG THỨC AM - GM
Hay
a
2
c
b
2
+
b
2
a
c
2
+
c
2
b
a
2
a + b + c. (1)
Áp dụng bất đẳng thức si cho ba số ta được :
a
2
c
b
2
+
b
2
a
c
2
+ c 3.
3
r
a
2
c
b
2
.
b
2
a
c
2
.c = 3a.
Tương tự :
b
2
a
c
2
+
c
2
b
a
2
+ a 3b ;
c
2
b
a
2
+
a
2
c
b
2
+ b 3c.
Cộng ba bất đẳng thức trên ta được bất đẳng thức (1).
Bài toán được chứng minh. Đẳng thức xảy ra a = b = c =
1
3
.
dụ 1.6. Cho a, b, c > 0. Chứng minh rằng :
a
5
b
2
+
b
5
c
2
+
c
5
a
2
a
3
+ b
3
+ c
3
.
Áp dụng bất đẳng thức si :
a
5
b
2
+ ab
2
2
r
a
5
b
2
ab
2
= 2a
3
.
Tương tự :
b
5
c
2
+ bc
2
2b
3
;
c
5
a
2
+ ca
2
2c
3
.
Công 3 bất đẳng thức trên lại với nhau ta được :
a
5
b
2
+
b
5
c
2
+
c
5
a
2
a
3
+ b
3
+ c
3
+
a
3
+ b
3
+ c
3
ab
2
bc
2
ca
2
.
Nên ta cần chứng minh :
a
3
+ b
3
+ c
3
ab
2
bc
2
ca
2
0 a
3
+ b
3
+ c
3
ab
2
+ bc
2
+ ca
2
. (1)
Áp dụng bất đẳng thức si :
a
3
+ b
3
+ b
3
3
3
a
3
b
3
b
3
= 3ab
2
a
3
+ 2b
3
3ab
2
Tương tự :
b
3
+ 2c
3
3bc
2
; c
3
+ 2a
3
3ca
2
.
Công 3 bất đẳng thức trên lại với nhau ta (1).
Vy bài toán được chứng minh.
dụ 1.7. Cho các số thực dương a,b,c. Chứng minh rằng
a
4
b
2
(c + a)
+
b
4
c
2
(a + b)
+
c
4
a
2
(b + c)
a + b + c
2
.
7
1. BẤT ĐẲNG THỨC AM - GM
Áp dụng bất đẳng thức AM-GM ta
a
4
b
2
(c + a)
+
b
2
+
b
2
+
c + a
4
2a
hay
a
4
b
2
(c + a)
+ b +
c + a
4
2a.
Tương tự, ta cũng
b
4
c
2
(a + b)
+ c +
a + b
4
2b và
c
4
a
2
(b + c)
+ a +
b + c
4
2c.
Cộng ba bất đẳng thức trên theo vế ta đpcm.
dụ 1.8 (BĐT Nesbit cho 3 số). Cho a, b, c > 0. Chứng minh rằng
a
b + c
+
b
c + a
+
c
a + b
3
2
.
Bất đẳng thức cần chứng minh tương đương với
a
b + c
+ 1
+
b
c + a
+ 1
+
c
a + b
+ 1
9
2
Hay
(a + b + c)
1
a + b
+
1
b + c
+
1
c + a
9
2
(1).
Ta
1
a + b
+
1
b + c
+
1
c + a
9
a + b + b + c + c + a
=
9
2 (a + b + c)
Nên (1) đúng.
dụ 1.9. Cho các số thực dương a, b, c thỏa a + b + c = 1. Chứng minh rằng
1
a
2
+ b
2
+ c
2
+
1
ab
+
1
bc
+
1
ca
30.
Ta có:
ab + bc + ca
(a + b + c)
2
3
=
1
3
1
ab
+
1
bc
+
1
ca
9
ab + bc + ca
1
a
2
+ b
2
+ c
2
+
1
ab + bc + ca
+
1
ab + bc + ca
9
(a + b + c)
2
= 9.
Do đó
V T
1
a
2
+ b
2
+ c
2
+
9
ab + bc + ca
=
1
a
2
+ b
2
+ c
2
+
1
ab + bc + ca
+
1
ab + bc + ca
+
7
ab + bc + ca
9 +
7
1
3
= 30.
Ta điều phải chứng minh.
8
1. BẤT ĐẲNG THỨC AM - GM
dụ 1.10. Cho các số thực dương x,y,z thỏa mãn : xy + yz + zx = 3.Chứng minh rằng:
1
xyz
+
4
(x + y)(y + z)(z + x)
3
2
.
Ta có:
3
p
xyz (x + y) (y + z) (z + x)
x (y + z) + y (z + x) + z (x + y)
3
= 2.
Suy ra
4
(x + y) (y + z) (z + x)
xyz
2
Do đó
V T
1
xyz
+
xyz
2
1
2xyz
+
xyz
2
+
1
2xyz
1 +
1
2
=
3
2
.
Bài toán được chứng minh.
dụ 1.11. (IMO 2012) Cho n 3 và các số thực dương a
2
, a
3
, . . . , a
n
thỏa mãn
a
2
a
3
···a
n
= 1. Chứng minh rằng
(1 + a
2
)
2
(1 + a
3
)
3
···(1 + a
n
)
n
> n
n
.
Áp dụng bất đẳng thức AM-GM ta
(1 + a
k
)
k
=
1
k 1
+
1
k 1
+ ··· +
1
k 1
+ a
k
k
k
k
a
k
(k 1)
k1
.
Suy ra
(1 + a
2
)
2
. (1 + a
3
)
3
···(1 + a
n
)
n
2
2
1
1
·
3
3
2
2
·
4
4
3
3
···
n
n
(n 1)
n
a
1
a
2
···a
n
= n
n
.
Ta thấy không đẳng thức xảy ra. Vậy bài toán được chứng minh.
dụ 1.12. Cho các số thực dương a, b, c tích bằng 1. Chứng minh rằng
1 +
3
a + b + c
6
ab + bc + ca
.
Bất đẳng thức cần chứng minh tương đương với
ab + bc + ca +
3(ab + bc + ca)
a + b + c
6. (1)
Áp dụng bất đẳng thức AM-GM ta
ab + bc + ca +
3(ab + bc + ca)
a + b + c
2
s
3(ab + bc + ca)
2
a + b + c
.
9
1. BẤT ĐẲNG THỨC AM - GM
Mặt khác
(ab + bc + ca)
2
3(ab · bc + bc · ca + ca · ab) = 3abc(a + b + c) = 3(a + b + c).
Suy ra
ab + bc + ca +
3(ab + bc + ca)
a + b + c
6.
Vy bài toán được chứng minh.
dụ 1.13. (Moldova TST 2014) Cho các số thực dương a,b,c thỏa mãn abc = 1.
Chứng minh rằng
a
3
+ b
3
+ c
3
+
ab
a
2
+ b
2
+
bc
b
2
+ c
2
+
ca
c
2
+ a
2
9
2
.
Bất đẳng thức cần chứng minh tương đương với
2
a
3
+ b
3
+ c
3
+
2ab
a
2
+ b
2
+
2bc
b
2
+ c
2
+
2ca
c
2
+ a
2
9 (1).
Ta x
3
+ y
3
x
2
y + y
2
x với mọi x,y > 0 nên
a
3
+ b
3
+ c
3
c (a
2
+ b
2
)
2
+
b (c
2
+ a
2
)
2
+
a (b
2
+ c
2
)
2
Suy ra
V T (1)
c (a
2
+ b
2
)
2
+
2ab
a
2
+ b
2
+
b (c
2
+ a
2
)
2
+
2bc
b
2
+ c
2
+
a (b
2
+ c
2
)
2
+
2ca
c
2
+ a
2
+3abc 9.
Bài toán được chứng minh.
dụ 1.14. Chứng minh rằng mỗi số thực dương a,b,c ta luôn có:
ab
a + 3b + 2c
+
bc
b + 3c + 2a
+
ca
c + 3a + 2b
a + b + c
6
.
Ta :
ab
a + 3b + 2c
=
ab
(a + c) + (b + c) + 2b
ab
9
.
1
a + c
+
1
b + c
+
1
2b
.
Tương tự :
bc
b + 3c + 2a
bc
9
1
a + b
+
1
a + c
+
1
2c
,
ac
c + 3a + 2b
ac
9
1
b + c
+
1
a + b
+
1
2a
.
Cộng vế theo vế ta được
ab
a + 3b + 2c
+
bc
b + 3c + 2a
+
ca
c + 3a + 2b
1
9
bc + ac
a + b
+
bc + ab
a + c
+
ab + ac
b + c
+
1
18
(a + b + c) .
Hay
ab
a + 3b + 2c
+
bc
b + 3c + 2a
+
ca
c + 3a + 2b
1
9
(a + b + c) +
1
18
(a + b + c) =
a + b + c
6
.
10
1. BẤT ĐẲNG THỨC AM - GM
dụ 1.15. Cho các số thực dương a,b,c thỏa a + b + c = 3. Chứng minh rằng
ab
c
2
+ 3
+
bc
a
2
+ 3
+
ca
b
2
+ 3
3
2
.
Ta
3 (ab + bc + ca) (a + b + c)
2
= 9 ab + bc + ca 3
Suy ra
1
c
2
+ 3
1
c
2
+ ab + bc + ca
=
1
p
(a + c)(b + c)
1
2
1
a + c
+
1
b + c
.
Do đó:
ab
c
2
+ 3
1
2
ab
a + c
+
ab
b + c
Tương tự:
bc
a
2
+ 3
1
2
bc
a + b
+
bc
a + b
và
ca
b
2
+ 3
1
2
ca
b + a
+
ca
b + c
Cộng ba bất đẳng thức trên theo vế ta có:
ab
c
2
+ 3
+
bc
a
2
+ 3
+
ca
b
2
+ 3
1
2
(a + b + c) =
3
2
.
dụ 1.16. (IMO 2005) Cho các số thực dương x,y,z thỏa xyz 1. Chứng minh rằng
x
5
x
2
x
5
+ y
2
+ z
2
+
y
5
y
2
y
5
+ z
2
+ x
2
+
z
5
z
2
z
5
+ x
2
+ y
2
0.
Bất đẳng thức cần chứng minh tương đương với
1
x
5
x
2
x
5
+ y
2
+ z
2
+ 1
y
5
y
2
y
5
+ z
2
+ x
2
+ 1
z
5
z
2
z
5
+ x
2
+ y
2
3
1
x
5
+ y
2
+ z
2
+
1
y
5
+ z
2
+ x
2
+
1
z
5
+ x
2
+ y
2
3
x
2
+ y
2
+ z
2
. (1)
Ta
x
5
+ y
2
+ z
2
x
4
yz
+ y
2
+ z
2
2x
4
+ (y
2
+ z
2
)
2
y
2
+ z
2
2
3
.
(x
2
+ y
2
+ z
2
)
2
y
2
+ z
2
.
Do đó
1
x
5
+ y
2
+ z
2
3
2
y
2
+ z
2
(x
2
+ y
2
+ z
2
)
2
.
Chứng minh tương tự
1
y
5
+ z
2
+ x
2
3
2
z
2
+ x
2
(x
2
+ y
2
+ z
2
)
2
và
1
z
5
+ x
2
+ y
2
3
2
x
2
+ y
2
x
2
+ y
2
+ z
2
.
Suy ra
1
x
5
+ y
2
+ z
2
+
1
y
5
+ z
2
+ x
2
+
1
z
5
+ x
2
+ y
2
3
x
2
+ y
2
+ z
2
Hay (1) đúng.
11
1. BẤT ĐẲNG THỨC AM - GM
dụ 1.17. (IMO Shortlist 2009) Cho các số thực dương a,b,c thỏa ab+bc+ca 3abc.
Chứng minh rằng
s
a
2
+ b
2
a + b
+
s
b
2
+ c
2
b + c
+
s
c
2
+ a
2
c + a
+ 3
2
a + b +
b + c +
c + a
.
Ta
p
2(a + b) =
s
2 (a + b)
2
a + b
=
s
2
a
2
+ b
2
a + b
+
2ab
a + b
s
a
2
+ b
2
a + b
+
r
2ab
a + b
.
Suy ra
V P
r
2ab
a + b
+
r
2bc
b + c
+
r
2ca
c + a
+
s
a
2
+ b
2
a + b
+
s
b
2
+ c
2
b + c
+
s
c
2
+ a
2
c + a
.
Mặt khác áp dụng bất đẳng thức
1
x
2
+
1
y
2
+
1
z
2
(x + y + z)
2
27
ta suy ra
x + y + z 3
3
v
u
u
t
1
1
x
2
+
1
y
2
+
1
z
2
Do đó
r
2ab
a + b
+
r
2bc
b + c
+
r
2ca
c + a
3
3
v
u
u
u
u
t
1
r
a + b
2ab
!
2
+
r
b + c
2bc
!
2
+
r
c + a
2ca
2
= 3
r
3abc
ab + bc + ca
= 3.
Từ đó, ta đpcm.
dụ 1.18. Cho các số thực dương a,b,c. Chứng minh rằng
a
3
a
2
+ b
2
+
b
3
b
2
+ c
2
+
c
3
c
2
+ a
2
a + b + c
2
.
Ta
a
3
a
2
+ b
2
=
a (a
2
+ b
2
) ab
2
a
2
+ b
2
= a
ab
2
a
2
+ b
2
a
b
2
.
Tương tự
b
3
b
2
+ c
2
b
c
2
và
c
3
c
2
+ a
2
c
a
2
.
Cộng các bất đẳng thức trên theo vế ta đpcm.
12
1. BẤT ĐẲNG THỨC AM - GM
dụ 1.19. Cho các số thực a,b,c thỏa abc < 0 và a + b + c = 0. Chứng minh rằng:
1
a
+
1
b
+
1
c
(1 ab bc ca) +
12abc 8
ab + bc + ca
16.
Gọi P vế trái của bất đẳng thức.
Đặt m = (ab + bc + ca) ,n = abc
Do a + b + c = 0 2(ab + bc + ca) = (a
2
+ b
2
+ c
2
) < 0 m,n > 0
Khi đó:
P =
m(1 + m)
n
+
12n + 8
m
Áp dụng bất đẳng thức sita có:
m
3
+ 8n
2
+ 8n 12mn và m
2
+ 4n
2
4mn
Suy ra
m
3
+ m
2
+ 12n
2
+ 8n 16mn
Do đó:
P =
m(1 + m)
n
+
12n + 8
m
16
Đẳng thức xảy ra khi và chỉ khi m = 2,n = 1, tức a,b,c ba nghiệm của phương trình
x
3
2x + 1 = 0 (x 1)(x
2
+ x 1) = 0 x = 1,x =
1 ±
5
2
.
III. Bài tập
Bài 1.1. Cho các số thực dương a,b,c. Chứng minh rằng
a) (1 + a) (1 + b) (1 + c)
1 +
3
abc
3
.
b)
1 +
a
b
1 +
b
c
1 +
c
a
2
1 +
a + b + c
3
abc
.
Bài 1.2. Cho các số thực dương a
1
, a
2
, ··· , a
n
. Chứng minh rằng
(1 + a
1
)(1 + a
2
) ···(1 + a
n
) (1 +
n
a
1
· a
2
···a
n
)
n
.
Bài 1.3. Cho các số thực a, b, c thỏa mãn a + b + c = 1. Chứng minh rằng
(1 + a) (1 + b) (1 + c) 64abc.
Bài 1.4. Cho 2n số thực dương a
1
,a
2
, . . . ,a
n
,b
1
,b
2
, . . . ,b
n
. Chứng minh rằng
n
p
(a
1
+ b
1
) (a
2
+ b
2
) ···(a
n
+ b
n
)
n
a
1
a
2
···a
n
+
n
p
b
1
b
2
···b
n
.
13
1. BẤT ĐẲNG THỨC AM - GM
Bài 1.5. (BĐT AM-GM suy rộng) Cho a
i
0 (i = 1,n) và các số hữu tỉ dương α
i
thỏa mãn
n
P
i=1
α
i
= 1. Chứng minh rằng:
n
X
i=1
α
i
a
i
a
α
1
1
· a
α
2
2
···a
α
n
n
.
Bài 1.6. Cho n số thực dương a
1
, a
2
, ··· , a
n
và số nguyên dương k. Chứng minh rằng
a
k
1
+ a
k
2
+ ··· + a
k
n
n
a
1
+ a
2
+ ··· + a
n
n
k
.
Bài 1.7. Cho a, b, c > 0. Chứng minh rằng
1
a + 3b
+
1
b + 3c
+
1
c + 3a
1
a + 2b + c
+
1
b + 2c + a
+
1
c + 2a + b
.
Bài 1.8. Cho các số thực a,b,c > 0 thỏa ab + bc + ca 3abc. Chứng minh rằng
1
4
a +
4
b
4
+
1
4
b +
4
c
4
+
1
(
4
c +
4
a)
4
3
16
.
Bài 1.9. Cho a,b,c > 0. Chứng minh rằng
2
a
b + 2c
+
b
c + 2a
+
c
a + 2b
1 +
b
b + 2a
+
c
c + 2b
+
a
a + 2c
.
Bài 1.10. Cho x, y, z các số thực dương thoả mãn x
2
+ y
2
+ z
2
= 12.Tìm giá trị nhỏ nhất
của biểu thức
P =
1
1 + x
3
+
1
p
1 + y
3
+
1
1 + z
3
.
Bài 1.11. Cho 3 số thực dương a,b,c thoả mãn a + b + c = 3 . Chứng minh rằng :
a
1 + b
2
+
b
1 + c
2
+
c
1 + a
2
3
2
.
Bài 1.12. Cho các số thực dương a,b,c thỏa a + b + c =
3
2
. Chứng minh rằng:
a
2
a + 2b
2
+
b
2
b + 2c
2
+
c
2
c + 2a
2
3
4
.
Bài 1.13. Chứng minh rằng nếu xy + yz + zx = 5 thì 3x
2
+ 3y
2
+ z
2
10
Bài 1.14. Cho a,b,c > 0. Chứng minh bất đẳng thức
a
3
(a + 2b) (b + 2c)
+
b
3
(b + 2c) (c + 2a)
+
c
3
(c + 2a) (a + 2b)
a + b + c
9
.
14
1. BẤT ĐẲNG THỨC AM - GM
Bài 1.15. Cho các số thực dương a,b,c > 0 thỏa abc = 1. Chứng minh rằng
a
4
b
2
(c + 2)
+
b
4
c
2
(a + 2)
+
c
4
a
2
(b + 2)
1.
Bài 1.16. Chứng minh rằng nếu a, b, c > 0 thì :
r
a + b
c
+
r
b + c
a
+
r
c + a
b
2
r
c
a + b
+
r
a
b + c
+
r
b
a + c
!
.
Bài 1.17. Cho các số thực a,b,cthỏa a
2
+ b
2
+ c
2
= 3. Chứng minh rằng
a
4
b + 2
+
b
4
c + 2
+
c
4
a + 2
1.
Bài 1.18. Cho các số thực dương a,b,c thỏa a
2
+ b
2
+ c
2
= 3 . Chứng minh rằng
4
a
2
+ b
2
+ 1
4
b
2
+ c
2
+ 1
4
c
2
+ a
2
+ 1
3 (a + b + c)
2
.
Bài 1.19. Cho các số thực dương a,b,c thỏa:
a
2
+ b
2
+
b
2
+ c
2
+
c
2
+ a
2
=
7 abc
2
.
Chứng minh rằng:
a
2
b + c
+
b
2
c + a
+
c
2
a + b
3
2
.
Bài 1.20. Chứng minh rằng nếu a,b,c > 0 và thỏa mãn a.b.c = 1 thì
1
a
2
+ 2b
2
+ 3
+
1
b
2
+ 2c
2
+ 3
+
1
c
2
+ 2a
2
+ 3
1
2
.
Bài 1.21. (Baltic Way 2014) Cho các số thực dương a,b,c thỏa
1
a
+
1
b
+
1
c
= 3. Chứng minh
rằng
1
a
3
+ b
+
1
b
3
+ c
+
1
c
3
+ a
3
2
.
Bài 1.22. (USA 2011) Với a, b, c các số thực dương thỏa a
2
+ b
2
+ c
2
+ (a + b + c)
2
4,
chứng minh rằng
ab + 1
(a + b)
2
+
bc + 1
(b + c)
2
+
ca + 1
(c + a)
2
3.
Bài 1.23. Cho a, b, c > 0. Chứng minh rằng
3
s
2a
b + c
2
+
3
s
2b
c + a
2
+
3
s
2c
a + b
2
3.
15
1. BẤT ĐẲNG THỨC AM - GM
Bài 1.24. Cho các số thực dương a, b, c thỏa abc = 1. Chứng minh rằng
3
r
a
3
+ b
3
2
+
3
r
b
3
+ c
3
2
+
3
r
c
3
+ a
3
2
+ 6 3 (a + b + c) .
Bài 1.25. Cho các số thực a,b,c thỏa a + b + c = 0. Chứng minh rằng
13a
2
b
2
c
2
2abc 2
(a
2
+ b
2
+ c
2
)
3
1
4
.
Bài 1.26. Cho các số thực không âm a,b,c. Chứng minh rằng:
q
(a + b + c)
3
6
3 (a b) (b c) (c a) .
Bài 1.27. Cho các số thực a,b,c phân biệt thỏa a + b + c = 1 và ab + bc + ca > 0. Tìm giá trị
nhỏ nhất
P =
2
|a b|
+
2
|b c|
+
2
|c a|
+
5
ab + bc + ca
.
Bài 1.28. (JBMO 2014) Cho các số thực dương a, b, c thỏa mãn abc = 1. Chứng minh rằng
a +
1
b
2
+
b +
1
c
2
+
c +
1
a
2
3(a + b + c + 1).
Bài 1.29. Cho các số thực dương a, b thỏa mãn ab 1. Chứng minh rằng
a + 2b +
2
a + 1
b + 2a +
2
b + 1
16.
Bài 1.30. (IMO Shortlist 2009) Cho các số thực dương a,b,c thỏa a + b + c =
1
a
+
1
b
+
1
c
.
Chứng minh rằng
1
(2a + b + c)
2
+
1
(2b + c + a)
2
+
1
(2c + a + b)
2
3
16
.
Bài 1.31. Cho a, b, c các số thực dương thỏa mãn:
3
a
3
+ b
3
+
3
b
3
+ c
3
+
3
c
3
+ a
3
+ abc = 3.
Chứng minh rằng giá trị nhỏ nhất của biểu thức:
P =
a
3
b
2
+ c
2
+
b
3
c
2
+ a
2
+
c
3
a
2
+ b
2
bằng
6
32m, trong đó m nghiệm của phương trình t
3
+ 54t 162 = 0.
Bài 1.32 (Đề thi chọn đội tuyển, vòng 1, Tĩnh, năm học 2017-20178). Cho các số
thực không âm a, b, c thoả mãn điều kiện a
2
+ b
2
+ c
2
3. Chứng minh rằng
(a + b + c)(a + b + c abc) 2(a
2
b + b
2
c + c
2
a).
16
1. BẤT ĐẲNG THỨC AM - GM
Bài 1.33 (Đề thi chọn đội tuyển, vòng 2, Nam Định, năm học 2017-2018). Xét các số
thực a,b,c [0; 1]. Tìm giá trị lớn nhất của biểu thức
P =
a
b + c + 1
+
b
c + a + 1
+
c
a + b + 1
+ (1 a) (1 b) (1 c)
Bài 1.34 (Đề thi chọn đội tuyển, vòng 2, Quảng Ngãi, năm học 2017-2018). Cho a, b, c
các số thực dương thỏa mãn 3bc + 4ac + 5ab 6abc . Tìm giá trị lớn nhất của biểu thức
P =
3a + 2b + c
(a + b)(b + c)(c + a)
.
Bài 1.35 (ĐỀ THI HSG TỈNH TÂY NINH,VÒNG 1, 2017-2018). Cho ba số thực dương
x, y, z thỏa mãn xyz = 1. Chứng minh rằng:
1
4
p
x
3
+ 2y
3
+ 6
+
1
4
p
y
3
+ 2z
3
+ 6
+
1
4
z
3
+ 2x
3
+ 6
3.
Bài 1.36 (Đề thi chọn đội tuyển, Lâm Đồng, năm học 2017-2018). Cho x,y,z các số
thực dương thỏa mãn điều kiện x
3
+ y
2
+ z = 2
3 + 1.
Tìm giá trị nhỏ nhất của biểu thức P =
1
x
+
1
y
2
+
1
z
3
.
Bài 1.37 (Đề thi chọn đội tuyển, vòng 1, Tĩnh, năm học 2016-2017). Cho các số thực
a,b,c dương và thỏa a
5
+ b
5
+ c
5
= 3. Chứng minh rằng:
a
6
b
6
+ b
6
c
6
+ c
6
a
6
3.
Bài 1.38. Tìm số nguyên dương k nhỏ nhất sao cho bất đẳng thức x
k
y
k
z
k
(x
3
+ y
3
+ z
3
) 3
đúng với mọi số thực dương x, y, z thỏa mãn điều kiện x + y + z = 3.
Bài 1.39. (VN TST 2010) Cho các số thực dương a, b, c thỏa mãn 16 (a + b + c)
1
a
+
1
b
+
1
c
.
Chứng minh rằng
1
a + b +
p
2 (a + c)
3
+
1
b + c +
p
2 (b + a)
3
+
1
c + a +
p
2 (c + b)
3
8
9
.
Bài 1.40. (IMO 2001) Cho các số thực dương a, b, c. Chứng minh rằng
a
2
a
2
+ 8bc
+
b
2
b
2
+ 8ca
+
c
2
c
2
+ 8ab
1.
Bài 1.41 (Turkey TST 2017). Cho các số thực dương a, b, c thỏa mãn a + b + c = 3. Chứng
minh rằng
a
3
b + b
3
c + c
3
a + 9 4(ab + bc + ca).
Bài 1.42 (IMO Shortlits A5-2008). Cho các số thực dương a,b,c,d thỏa mãn abcd = 1 và
a + b + c + d
a
b
+
b
c
+
c
d
+
d
a
.
Chứng minh rằng a + b + c + d
b
a
+
c
b
+
d
c
+
a
d
.
17
1. BẤT ĐẲNG THỨC AM - GM
Bài 1.43. Cho các số thực không âm a, b, c thỏa mãn a + b + c = 2. Chứng minh rằng
a
3
+ b
3
b
3
+ c
3
c
3
+ a
3
2.
Bài 1.44. (Hàn Quốc MO 2016) Cho các số thực x, y, z thỏa mãn x
2
+ y
2
+ z
2
= 1. Tìm
GTLN của biểu thức
P = (x
2
yz)(y
2
zx)(z
2
xy).
Bài 1.45. Cho các số thực dương x, y, z thỏa mãn x + y + z = 1. Chứng minh rằng
x
2
y
2
1 z
+
y
2
z
2
1 x
+
z
2
x
2
1 y
+ 3xyz
1
6
Bài 1.46. Cho các số thực dương a,b,c thỏa mãn:
9
a
4
+ b
4
+ c
4
25
a
2
+ b
2
+ c
2
+ 48 = 0.
Tìm giá trị nhỏ nhất của biểu thức:
F =
a
2
b + 2c
+
b
2
c + 2a
+
c
2
a + 2b
.
Bài 1.47. (TST Quảng Nam 2014-2015) Cho các số thực dương a, b, c. Chứng minh rằng
5a
2
+ 4bc +
5b
2
+ 4ca +
5c
2
+ 4ab
p
3 (a
2
+ b
2
+ c
2
) + 2
ab +
bc +
ca
.
Bài 1.48. Cho các số thực dương a, b, c thỏa mãn a + b + c = 3. Chứng minh rằng
a
2
b
1 + a + b
+
b
2
c
1 + b + c
+
c
2
a
1 + c + a
1.
Bài 1.49 (P122, Tạp chí Pi, tháng 12 năm 2017). Chứng minh rằng, với mọi số thực dương
a, b, c ta luôn bất đẳng thức:
s
a
2
+ bc
a(b + c)
+
s
b
2
+ ca
b(c + a)
+
s
c
2
+ ab
c(a + b)
3.
Hỏi đẳng thức xảy ra khi nào?
Bài 1.50. Cho 2018 số dương a
1
,a
2
, . . . ,a
2018
thỏa: a
1
+ a
2
+ ···+ a
2018
=
1
a
1
+
1
a
2
+ ···+
1
a
2018
.
Chứng minh rằng: a
1
+ a
2
+ ··· + a
2018
2018.
18
2. BẤT ĐẲNG THỨC CAUCHY - SCHWARZ
§2. Bất đẳng thức Cauchy - Schwarz
I. Bất đẳng thức Cauchy-Schwarz dạng đa thức
Định 1. Cho 2n số thực a
1
,a
2
, ··· ,a
n
,b
1
,b
2
, ··· ,b
n
. Khi đó, ta có
a
2
1
+ a
2
2
+ ··· + a
2
n
b
2
1
+ b
2
2
+ ··· + b
2
n
(a
1
b
1
+ a
2
b
2
+ ··· + a
n
b
n
)
2
.
Đẳng thức xảy ra khi a
i
= kb
i
với mọi i = 1,2, ··· ,n.
Chứng minh. Nếu a
i
= 0 i = 1,n thì bất đẳng thức hiển nhiên đúng.
Nếu
n
P
i=1
a
2
i
> 0, ta xét tam thức
f(x) =
n
X
i=1
a
2
i
!
x
2
2
n
X
i=1
a
i
.b
i
!
x +
n
X
i=1
b
2
i
Ta
f(x) =
n
X
i=1
(a
i
x b
i
)
2
x R
Do đó
0
=
n
X
i=1
a
i
b
i
!
2
n
X
i=1
a
2
i
!
n
X
i=1
b
2
i
!
0
Hay bất đẳng thức được chứng minh.
Đẳng thức xảy ra khi a
i
x b
i
= 0 a
i
= k.b
i
.
II. Bất đẳng thức Cauchy-Schwarz dạng phân thức
Định 2. Cho các n số thực a
1
,a
2
, ··· , a
n
n số thực dương b
1
,b
2
, ··· ,b
n
. Khi đó, ta có
a
2
1
b
1
+
a
2
2
b
2
+ ··· +
a
2
n
b
n
(a
1
+ a
2
+ ··· + a
n
)
2
b
1
+ b
2
+ ··· + b
n
.
Đẳng thức xảy ra khi chỉ khi
a
1
b
1
=
a
2
b
2
= ··· =
a
n
b
n
.
Chứng minh. Áp dụng bất đẳng thức Cauchy-Schwarz dạng đa thức ta
n
X
i=1
a
i
!
2
=
n
X
i=1
p
b
i
.
a
i
b
i
!
2
n
X
i=1
b
i
!
n
X
i=1
a
2
i
b
i
!
Hay
a
2
1
b
1
+
a
2
2
b
2
+ ··· +
a
2
n
b
n
(a
1
+ a
2
+ ··· + a
n
)
2
b
1
+ b
2
+ ··· + b
n
(đpcm).
III. Các dụ minh họa
dụ 2.1. Cho a, b, c > 0 thỏa mãn a + b + c = 1. Chứng minh rằng
r
a
2
+
1
b
2
+
r
b
2
+
1
c
2
+
r
c
2
+
1
a
2
82
19
2. BẤT ĐẲNG THỨC CAUCHY - SCHWARZ
Áp dụng bất đẳng thức Cauchy Schwarz ta
a
2
+
1
b
2
1
9
+ 9
a
3
+
3
b
2
hay
r
a
2
+
1
b
2
3
82
a
3
+
3
b
.
Tương tự, ta cũng
r
b
2
+
1
c
2
3
82
b
3
+
3
c
và
r
c
2
+
1
a
2
3
82
c
3
+
3
a
.
Công ba bất đẳng thức theo vế ta
r
a
2
+
1
b
2
+
r
b
2
+
1
c
2
+
r
c
2
+
1
a
2
3
82
a + b + c
3
+ 3
1
a
+
1
b
+
1
c

.
Lại
1
a
+
1
b
+
1
c
9
a + b + c
= 9 nên ta suy ra được
r
a
2
+
1
b
2
+
r
b
2
+
1
c
2
+
r
c
2
+
1
a
2
3
82
1
3
+ 27
=
82.
Đẳng thức xảy ra khi a = b = c =
1
3
.
dụ 2.2. Cho các số thực dương a,b,c thỏa abc = 1. Chứng minh rằng
1 + a
2
1 + b
2
1 + c
2
4
3
p
(a + b) (b + c) (c + a).
Áp dụng bất đẳng thức Bunhiacopsky cho hai b số (1; a) và (b; 1) ta
1 + a
2
1 + b
2
=
1 + a
2
b
2
+ 1
(a + b)
2
.
Tương tự
1 + b
2
1 + c
2
(b + c)
2
,
1 + c
2
1 + a
2
(a + c)
2
.
Nhận các bất đẳng thức trên theo vế ta được
1 + a
2
1 + b
2
1 + c
2
(a + b) (b + c) (c + a) .
Mặt khác
(a + b) (b + c) (c + a) 2
ab.2
bc.2
ca = 8abc = 8.
Suy ra
(a + b) (b + c) (c + a) =
3
p
(a + b) (b + c) (c + a).
3
q
[(a + b) (b + c) (c + a)]
2
3
p
(a + b) (b + c) (c + a).
3
8
2
= 4
3
p
(a + b) (b + c) (c + a) (đpcm).
20
2. BẤT ĐẲNG THỨC CAUCHY - SCHWARZ
dụ 2.3. Cho a,b,c > 0 thỏa
1
a
2
+ b
2
+ 1
+
1
b
2
+ c
2
+ 1
+
1
c
2
+ a
2
+ 1
1.
Chứng minh rằng ab + bc + ca 3.
Áp dụng bất đẳng thức Cauchy-Schwarz cho hai b số (a; b; 1) và (1; 1; c) ta
a
2
+ b
2
+ 1
1 + 1 + c
2
(a + b + c)
2
.
Suy ra
1
a
2
+ b
2
+ 1
2 + c
2
(a + b + c)
2
.
Tương tự:
1
b
2
+ c
2
+ 1
2 + a
2
(a + b + c)
2
,
1
c
2
+ a
2
+ 1
2 + b
2
(a + b + c)
2
.
Suy ra
2 + a
2
(a + b + c)
2
+
2 + b
2
(a + b + c)
2
+
2 + c
2
(a + b + c)
2
1,
Do đó ta
6 + a
2
+ b
2
+ c
2
(a + b + c)
2
ab + bc + ca 3 (đpcm).
dụ 2.4. Cho các số thực dương a,b,c thỏa mãn a + b + c =
1
a
+
1
b
+
1
c
. Chứng minh
rằng
a
2
+ 1 +
b
2
+ 1 +
c
2
+ 1
2 (a + b + c) .
Áp dụng bất đẳng thức Cauchy Schwarz ta
a
2
+ 1 +
b
2
+ 1 +
c
2
+ 1 =
a.
r
a +
1
a
+
b.
r
b +
1
b
+
c.
r
c +
1
c
s
(a + b + c)
a +
1
a
+ b +
1
b
+ c +
1
c
=
2 (a + b + c) .
Đẳng thức xảy ra khi a = b = c = 1.
dụ 2.5. Cho các số thực a, b, c, x, y, z. Chứng minh rằng
ax + by + cz +
p
(a
2
+ b
2
+ c
2
)(x
2
+ y
2
+ z
2
)
2
3
(a + b + c)(x + y + z).
Ta
2
3
(a + b + c)(x + y + z) (ax + by + cz)
= a ·
2y + 2z x
3
+ b ·
2z + 2x y
3
+ c ·
2x + 2y z
3
v
u
u
t
(a
2
+ b
2
+ c
2
)
2y + 2z x
3
2
+
2z + 2x y
3
2
+
2x + 2y z
3
2
!
.
21
2. BẤT ĐẲNG THỨC CAUCHY - SCHWARZ
Hơn nữa
2y + 2z x
3
2
+
2z + 2x y
3
2
+
2x + 2y z
3
2
= x
2
+ y
2
+ z
2
.
Nên ta đpcm.
dụ 2.6. Cho các số thực a,b,c thỏa a
2
+ b
2
+ c
2
= 9. Chứng minh rằng
2 (a + b + c) abc 10.
Không mất tính tổng quát, ta giả sử |a| |b| |c|
3
a
2
+ b
2
2
a
2
+ b
2
+ c
2
= 18 a
2
+ b
2
6.
Áp dụng bất đẳng thức Cauchy-Schwarz cho hai b số
2 (a + b + c) abc = 2 (a + b) + c (2 ab)
q
4 + (2 ab)
2
(a + b)
2
+ c
2
=
p
(8 4ab + a
2
b
2
) (a
2
+ b
2
+ c
2
+ 2ab)
=
p
(8 4ab + a
2
b
2
) (9 + 2ab).
Do đó, ta chỉ cần chứng minh
p
(8 4ab + a
2
b
2
) (9 + 2ab) 10
2a
3
b
3
+ a
2
b
2
20ab 28 0
(ab + 2)
2
(2ab 7) 0. (*)
2ab a
2
+ b
2
6 2ab 7 < 0, do đó (*) đúng.
dụ 2.7 (VQB Cẩn). Cho các số thực dương a,b,c thỏa mãn a + b + c = 6 và a
2
+ b
2
+
c
2
= 14. Chứng minh rằng
2
4a + b
c
31
2
.
Ta
4a + b
c
2 4a b + 2c 0 3a + 6b + 9c 7 (a + b + c) = 42 (1).
Áp dụng bất đẳng thức Cauchy Schwarz ta
3a + 6b + 9c
p
(3
2
+ 6
2
+ 9
2
) (a
2
+ b
2
+ c
2
) = 42.
Suy ra (1) đúng. Đẳng thức xảy ra khi a = 1,b = 2,c = 3.
Tương tự
4a + b
c
31
2
8a + 2b 31c 0 57a + 51b + 18c 49 (a + b + c) = 294 (2).
Áp dụng bất đẳng thức Cauchy Schwarz ta
57a + 51b + 18c
p
(57
2
+ 51
2
+ 18
2
) (a
2
+ b
2
+ c
2
) = 294.
Hay (2) được chứng minh. Đẳng thức xảy ra khi a =
19
7
,b =
17
7
,c =
6
7
.
22
2. BẤT ĐẲNG THỨC CAUCHY - SCHWARZ
dụ 2.8. Cho các số thực dương a, b, c. Chứng minh rằng
r
2a
a + b
+
r
2b
b + c
+
r
2c
c + a
3.
Áp dụng bất đẳng thức Cauchy Schwarz ta
V T
2
=
a + c
r
a
(a + b)(a + c)
+
b + a
s
b
(b + c)(b + a)
+
c + b
r
c
(c + a)(c + b)
!
2
2 (a + b + c)
a
(a + b) (a + c)
+
b
(b + a) (b + c)
+
c
(c + a) (c + b)
=
4 (a + b + c) [ab + bc + ca]
(a + b) (b + c) (c + a)
.
Do đó, ta chỉ cần chứng minh
4 (a + b + c) (ab + bc + ca)
(a + b) (b + c) (c + a)
9
2
(a + b + c) (ab + bc + ca)
(a + b) (b + c) (c + a)
9
8
.
Đây một kết quả quen thuộc.
dụ 2.9. Cho các số thực dương a,b,c thỏa
1
a
2
+ 2
+
1
b
2
+ 2
+
1
c
2
+ 2
= 1.
Chứng minh rằng: ab + bc + ca 3.
Từ giả thiết suy ra:
1 =
a
2
a
2
+ 2
+
b
2
b
2
+ 2
+
c
2
c
2
+ 2
(a + b + c)
2
a
2
+ b
2
+ c
2
+ 6
Do đó:
a
2
+ b
2
+ c
2
+ 6 (a + b + c)
2
ab + bc + ca 3 (đpcm).
dụ 2.10. Cho các số thực a,b,c > 0 thỏa mãn a + b + c = 3. Chứng minh rằng
a
2
a + 2b
2
+
b
2
b + 2c
2
+
c
2
c + 2a
2
1.
Gọi P vế trái của bất đẳng thức cần chứng minh. Áp dụng bất đẳng thức Cauchy Schwarz
ta
P =
a
4
a
3
+ 2a
2
b
2
+
b
4
b
3
+ 2b
2
c
2
+
c
4
c
3
+ 2c
2
a
2
(a
2
+ b
2
+ c
2
)
2
a
3
+ b
3
+ c
3
+ 2 (a
2
b
2
+ b
2
c
2
+ c
2
a
2
)
.
Với a + b + c = 3 ta
a
4
+ b
4
+ c
4
a
2
+ b
2
+ c
2
a
3
+ b
3
+ c
3
2
a
3
+ b
3
+ c
3
(a + b + c)
a
2
+ b
2
+ c
2
2
3
a
2
+ b
2
+ c
2
(a + b + c)
2
.
23
2. BẤT ĐẲNG THỨC CAUCHY - SCHWARZ
Nhân ba bất đẳng thức trên theo vế ta được
3
a
4
+ b
4
+ c
3
(a + b + c)
a
3
+ b
3
+ c
3
Hay a
4
+ b
4
+ c
4
a
3
+ b
3
+ c
3
. Do đó
a
2
+ b
2
+ c
2
2
= a
4
+ b
4
+ c
4
+ +2
a
2
b
2
+ b
2
c
2
+ c
2
a
2
a
3
+ b
3
+ c
3
+ 2
a
2
b
2
+ b
2
c
2
+ c
2
a
2
.
Vy P 1. Đẳng thức xảy ra khi a = b = c = 1.
dụ 2.11. Cho các số thực dương a,b,c. Chứng minh rằng:
a
a + b
2
+
b
b + c
2
+
c
c + a
2
3
4
.
b
a
.
c
b
.
a
c
= 1 nên tồn tại các số thực dương x,y,z sao cho
b
a
=
yz
x
2
,
c
b
=
zx
y
2
,
a
c
=
xy
z
2
.
Bất đẳng thức cần chứng minh trở thành
x
4
(x
2
+ yz)
2
+
y
4
(y
2
+ zx)
2
+
z
4
(z
2
+ xy)
2
3
4
.
Áp dụng bất đẳng thức Cauchy Schwarz ta
x
4
(x
2
+ yz)
2
+
y
4
(y
2
+ zx)
2
+
z
4
(z
2
+ xy)
2
(x
2
+ y
2
+ z
2
)
2
(x
2
+ yz)
2
+ (y
2
+ zx)
2
+ (z
2
+ xy)
2
.
Ta chứng minh
(x
2
+ y
2
+ z
2
)
2
(x
2
+ yz)
2
+ (y
2
+ zx)
2
+ (z
2
+ xy)
2
3
4
Biến đổi và rút gọn ta thu được bất đẳng thức
x
4
+ y
4
+ z
4
+ 5
x
2
y
2
+ y
2
z
2
+ z
2
x
2
6xyz (x + y + z) ().
Ta
x
4
+ y
4
+ z
4
x
2
y
2
+ y
2
z
2
+ z
2
x
2
xyz (x + y + z) .
Nên suy ra () đúng. Vy bài toán được chứng minh.
dụ 2.12 (P61, Tạp chí Pi, tháng 6 năm 2017). Cho a, b, c độ dài ba cạnh của
một tam giác. Chứng minh rằng
a
b + c
+
b
c + a
+
c
a + b
+
2(ab + bc + ca)
a
2
+ b
2
+ c
2
7
2
.
Hỏi dấu bằng xảy ra khi và chỉ khi nào?
Ta biết rằng với a, b, c ba số thực tùy ý, luôn
ab + bc + ca a
2
+ b
2
+ c
2
24
2. BẤT ĐẲNG THỨC CAUCHY - SCHWARZ
Do đó
2(ab + bc + ca)
a
2
+ b
2
+ c
2
ab + bc + ca
a
2
+ b
2
+ c
2
+ 1. (1)
Tiếp theo ta sẽ chứng minh
a
b + c
+
b
c + a
+
c
a + b
+
ab + bc + ca
a
2
+ b
2
+ c
2
5
2
(2)
Thật vy, ta
(2)
1
a
b + c
+
1
b
c + a
+
1
c
a + b
1
2
+
ab + bc + ca
a
2
+ b
2
+ c
2
.
b + c a
b + c
+
c + a b
c + a
+
a + b c
a + b
(a + b + c)
2
2(a
2
+ b
2
+ c
2
)
(b + c a)
2
(b + c a)(b + c)
+
(c + a b)
2
(c + a b)(c + a)
+
(a + b c)
2
(a + b c)(a + b)
(a + b + c)
2
2(a
2
+ b
2
+ c
2
)
. (3)
Do đó a, b, c độ dài 3 cạnh của một tam giác nên b + c > a, a + c > b và a + b > c. Do đó, tất
cả các phân thức nằm vế trái của (3) đều mẫu thức dương. thế, hiệu VT biểu thức
nằm vế trái của (3), theo một hệ quả của bất đẳng thức Cauchy - Schwarz, ta
V T
(a + b + c)
2
2(a
2
+ b
2
+ c
2
)
.
(b + c a) + (c + a b) + (a + b c) = a + b + c
(b + c a)(b + c) + (c + a b)(c + a) + (a + b c)(a + b) = 2(a
2
+ b
2
+ c
2
).
nên (3) được chứng minh và thế (2) được chứng minh. Từ (1) và (2), hiển nhiên, suy ra bất
đẳng thức cần chứng minh theo yêu cầu bài toán. Từ điều kiện cần và đủ để dấu bằng xảy ra
các bất đẳng thức trên, dễ dàng suy ra dấu bằng bất đẳng thức đề bài xảy ra khi và chỉ khi a,
b, c độ dài ba cạnh của tam giác đều.
dụ 2.13. Cho a,b,c > 0 thỏa a + b + c = 2. Chứng minh rằng:
a
4a + 3bc
+
b
4b + 3ca
+
c
4c + 3ab
1.
Áp dụng bất đẳng thức Cauchy Schwarz ta có:
a
4a + 3bc
+
b
4b + 3ca
+
c
4c + 3ab
2
(a + b + c)
a
4a + 3bc
+
b
4b + 3ca
+
c
4c + 3ab
= 2
a
4a + 3bc
+
b
4b + 3ca
+
c
4c + 3ab
.
Ta chứng minh:
a
4a + 3bc
+
b
4b + 3ca
+
c
4c + 3ab
1
2
bc
4a + 3bc
+
ca
4b + 3ca
+
ab
4c + 3ab
1
3
(1)
Ta
V T (1)
(ab + bc + ca)
2
bc(4a + bc) + ca(4b + ca) + ab(4c + ab)
25
2. BẤT ĐẲNG THỨC CAUCHY - SCHWARZ
Do
bc(4a + bc) + ca(4b + ca) + ab(4c + ab) = 3(ab + bc + ca)
2
.
Nên ta có: V T (1)
1
3
(đpcm). Đẳng thức xảy ra khi và chỉ khi a = b = c =
2
3
.
dụ 2.14. Cho các số thực x,y,z > 0. Chứng minh rằng
x + y
p
x
2
+ y
2
+ zx + zy
+
y + z
p
y
2
+ z
2
+ xy + xz
+
z + x
p
z
2
+ x
2
+ yz + xy
3.
Áp dụng bất đẳng thức Cauchy Schwarz ta
V T
2
3
"
(x + y)
2
x
2
+ y
2
+ zx + yz
+
(y + z)
2
y
2
+ z
2
+ xy + xz
+
(z + x)
2
z
2
+ x
2
+ zy + yx
#
.
Mặt khác
(x + y)
2
x
2
+ y
2
+ zx + yz
=
(x + y)
2
x(x + z) + y(y + z)
x
2
x (x + z)
+
y
2
y (y + z)
=
x
x + z
+
y
y + z
Tương tự
(y + z)
2
y
2
+ z
2
+ xy + xz
y
y + x
+
z
z + x
và
(z + x)
2
z
2
+ x
2
+ zy + yx
z
z + y
+
x
x + y
Suy ra V T
2
9 V T 3, từ đây ta đpcm.
dụ 2.15. Cho a, b, c các số thực không âm và không hai số nào đồng thời bằng
0. Chứng minh rằng
a
2
(2a + b)(2a + c)
+
b
2
(2b + c)(2b + a)
+
c
2
(2c + a)(2c + b)
1
3
.
Áp dụng bất đẳng thức Cauchy-Schwarz, ta
9a
2
(2a + b)(2a + c)
=
(2a + a)
2
2a(a + b + c) + (2a
2
+ bc)
4a
2
2a(a + b + c)
+
a
2
2a
2
+ bc
=
2a
a + b + c
+
a
2
2a
2
+ bc
.
Do đó
9V T 2 +
a
2
2a
2
+ bc
+
b
2
2b
2
+ ca
+
c
2
2c
2
+ ab
.
Nên ta chứng minh
a
2
2a
2
+ bc
+
b
2
2b
2
+ ca
+
c
2
2c
2
+ ab
1
bc
2a
2
+ bc
+
ca
2b
2
+ ca
+
ab
2c
2
+ ab
1. (1)
Áp dụng bất đẳng thức Cauchy-Schwarz ta
V T (1)
(ab + bc + ca)
2
2abc(a + b + c) + a
2
b
2
+ b
2
c
2
+ c
2
a
2
=
(ab + bc + ca)
2
(ab + bc + ca)
2
= 1.
26
2. BẤT ĐẲNG THỨC CAUCHY - SCHWARZ
IV. Bài tập
Bài 2.1 (Bất đẳng thức Mincopski). Cho các 2n số thực a
1
,a
2
, ··· ,a
n
,b
1
,b
2
, ··· ,b
n
. Chứng
minh rằng
q
a
2
1
+ a
2
2
+ ··· + a
2
n
+
q
b
2
1
+ b
2
2
+ ··· + b
2
n
q
(a
1
+ b
1
)
2
+ (a
2
+ b
2
)
2
+ ··· + (a
n
+ b
n
)
2
.
Bài 2.2. Cho các số thực dương a,b,c. Chứng minh rằng
a
2
+ 1
b
2
+ 1
c
2
+ 1
(a + b) (b + c) (c + a) .
Bài 2.3. Cho các số thực dương a,b,c thỏa mãn a + b + c = 3. Chứng minh rằng
2
a
2
+ b
2
+ c
2
+ 3 9
1
a
2
+ 2
+
1
b
2
+ 2
+
1
c
2
+ 2
.
Bài 2.4. Cho a,b,c > 0 và a + b + c = 1. Chứng minh rằng
a
a
2
+ 8bc + b
b
2
+ 8ca + c
c
2
+ 8ab 1.
Bài 2.5. Cho các số thực dương a, b, c thỏa a
2
+ b
2
+ c
2
= 3. Chứng minh rằng:
a
3
b + 2c
+
b
3
c + 2a
+
c
3
a + 2b
1.
Bài 2.6. Cho a, b, c các số thực dương thỏa: a
2
+ b
2
+ c
2
3. Chứng minh rằng:
a
3
b
2
+ c
2
+ 7
+
b
3
c
2
+ a
2
+ 7
+
c
3
a
2
+ b
2
+ 7
1.
Bài 2.7. Cho các số thực dương a,b,c tổng bằng 3. Chứng minh rằng:
1
4a
2
+ b
2
+ c
2
+
1
a
2
+ 4b
2
+ c
2
+
1
a
2
+ b
2
+ 4c
2
1
2
.
Bài 2.8. Cho các số thực không âm a, b, c thỏa mãn a + b + c = 3. Chứng minh rằng
1
2a
2
+ 3
+
1
2b
2
+ 3
+
1
2c
2
+ 3
3
5
.
Bài 2.9. Cho các số thực không âm a, b, c thỏa mãn ab + bc + ca = 3. Chứng minh rằng
1
a
2
+ 1
+
1
b
2
+ 1
+
1
c
2
+ 1
3
2
.
Bài 2.10. Cho các số thực dương a, b, c thỏa mãn a
2
+ b
2
+ c
2
= 3. Chứng minh rằng
1
3 ab
+
1
3 bc
+
1
3 ca
3
2
.
27
2. BẤT ĐẲNG THỨC CAUCHY - SCHWARZ
Bài 2.11. Cho ba số thực dương x, y, z thỏa mãn x + y + z = 3. Chứng minh rằng:
4x + 5
x
3
+ xy
2
+ 3xyz
+
4y + 5
y
3
+ yz
2
+ 3xyz
+
4z + 5
z
3
+ zx
2
+ 3xyz
162
x
2
+ y
2
+ z
2
+ 27
.
Bài 2.12. Cho a, b, c > 0 thỏa mãn a + b + c = 1. Chứng minh rằng
a
2
(b + 2c)
2
(a + b)
+
b
2
(c + 2a)
2
(b + c)
+
c
2
(a + 2b)
2
(c + a)
1
2
.
Bài 2.13. Cho a,b,c (1; 2). Chứng minh rằng
b
a
4b
c c
a
+
c
b
4c
a a
b
+
a
c
4a
b b
c
1.
Bài 2.14. Cho a,b,c các số thực dương thỏa mãn a
2
b
2
+ b
2
c
2
+ c
2
a
2
a
2
b
2
c
2
. Chứng minh
rằng
a
2
b
2
c
3
(a
2
+ b
2
)
+
b
2
c
2
a
3
(b
2
+ c
2
)
+
c
2
a
2
b
3
(c
2
+ a
2
)
3
2
Bài 2.15 (IMO Shortlist-2007). Cho các số thực không âm a
1
, a
2
, . . . , a
100
thỏa mãn điều
kiện a
2
1
+ a
2
2
+ ··· + a
2
100
= 1. Chứng minh rằng
S = a
2
1
a
2
+ a
2
2
a
3
+ ··· + a
2
100
a
1
2
3
.
Bài 2.16. (China TST 2005) Cho các số thực không âm a,b,c thỏa ab + bc + ca =
1
3
. Chứng
minh rằng
1
a
2
bc + 1
+
1
b
2
ca + 1
+
1
c
2
ab + 1
3.
Bài 2.17. Cho các số thực dương a, b, c. Chứng minh rằng
2
ab
c
+
bc
a
+
ca
b
a
r
3 +
b
2
c
2
+ b
r
3 +
c
2
a
2
+ c
r
3 +
a
2
b
2
2(a + b + c).
Bài 2.18. Cho x,y,z > 1. Chứng minh rằng
1 + x
2
1 + y + z
2
+
1 + y
2
1 + z + x
2
+
1 + z
2
1 + x + y
2
2.
Bài 2.19 (P77, Tạp chí Pi, tháng 7 năm 2017). Cho a, b, c các số thực dương. Chứng
minh rằng
a
3
p
4(b
3
+ c
3
)
+
b
c + a
+
c
a + b
3
2
.
Hỏi đẳng thức xảy ra khi và chỉ khi nào?
Bài 2.20. Cho ba số thực không âm a, b, c thỏa mãn điều kiện (b + c)(c + 2a)(c + 4a) > 0.
Chứng minh rằng
a
b + c
+
b
c + 4a
+
2c
c + 2a
1.
Hỏi đẳng thức xảy ra khi và chỉ khi nào?
28
2. BẤT ĐẲNG THỨC CAUCHY - SCHWARZ
Bài 2.21. Cho a,b,c > 0 thỏa mãn a
2
+ b
2
+ c
2
= 3. Chứng minh rằng
1
2 a
+
1
2 b
+
1
2 c
3.
Bài 2.22. Cho bốn số thực a,b,c,d thỏa mãn a
2
+ b
2
+ c
2
+ d
2
= 1. Chứng minh rằng
1
1 ab
+
1
1 bc
+
1
1 cd
+
1
1 da
16
3
.
Bài 2.23. Cho x,y,z > 0 thỏa mãn xyz = 1. Chứng minh rằng
1
1 + x + x
2
+
1
1 + y + y
2
+
1
1 + z + z
2
1.
Bài 2.24. Cho x,y,z > 0 thỏa mãn xyz = 8. Chứng minh rằng
x
2
x
2
+ 2x + 4
+
y
2
y
2
+ 2y + 4
+
z
2
z
2
+ 2z + 4
1.
Bài 2.25 (IMO 2008). Cho các số thực x,y,z 6= 1 và xyz = 1. Chứng minh rằng
x
x 1
2
+
y
y 1
2
+
z
z 1
2
1.
Bài 2.26. Cho a, b, c các số thực dương. Chứng minh rằng
s
bc
a(3b + a)
+
r
ac
b(3c + b)
+
s
ab
c(3a + c)
3
2
.
Bài 2.27. Cho các số thực a, b, c tất cả không cùng dấu. Chứng minh rằng
(a
2
+ ab + b
2
)(b
2
+ bc + c
2
)(c
2
+ ca + a
2
) 3(ab + bc + ca)
3
.
Bài 2.28. Cho a, b, c 0 và không hai số nào đồng thời bằng 0. Chứng minh rằng
a
2
2b
2
bc + 2c
2
+
b
2
2c
2
ca + 2a
2
+
c
2
2a
2
ab + 2b
2
1.
Bài 2.29. Cho a, b, c các số thực thỏa mãn điều kiện 3a
2
+ 2b
2
+ c
2
= 6. Tìm giá trị lớn nhất
và giá trị nh nhất của biểu thức
P = 2(a + b + c) abc.
Bài 2.30. (Iran MO 2016 day 3) Cho các số thực dương a, b, c thỏa mãn abc = 1. Chứng
minh rằng
a + b
(a + b + 1)
2
+
b + c
(b + c + 1)
2
+
c + a
(c + a + 1)
2
2
a + b + c
.
29
2. BẤT ĐẲNG THỨC CAUCHY - SCHWARZ
Bài 2.31. Cho các số thực dương x, y, z thỏa mãn xyz 1. Chứng minh rằng
(x
4
+ y)(y
4
+ z)(z
4
+ x) (x + y
2
)(y + z
2
)(z + x
2
).
Bài 2.32. (Serbia TST 2016) Cho các số thực dương a, b, c thỏa mãn a + b + c = 3. Chứng
minh rằng
a
3a + b
+
b
3b + c
+
c
3c + a
3
2
.
Bài 2.33. (Hải Dương TST 2016)
Cho a, b, c [1,1] thỏa mãn: 1 + 2abc a
2
+ b
2
+ c
2
. Chứng minh rằng :
1 + 2a
3
b
3
c
3
a
6
+ b
6
+ c
6
.
Bài 2.34. Cho các số thực dương a, b, c thỏa mãn a
2
+ b
2
+ c
2
= 3. Chứng minh rằng
2
1
a
2
+
1
b
2
+
1
c
2
b + c
a
+
c + a
b
+
a + b
c
.
Bài 2.35. Cho n (n 1) x
1
, x
2
, . . . , x
n
thỏa mãn x
1
+ x
2
+ ··· + x
n
= 0. Chứng minh rằng
(n 2)x
2
1
+ 2x
1
(n 1)x
2
1
+ 1
+
(n 2)x
2
2
+ 2x
2
(n 1)x
2
2
+ 1
+ ··· +
(n 2)x
2
n
+ 2x
n
(n 1)x
2
n
+ 1
0.
Bài 2.36. Cho các số thực dương a,b,c thỏa mãn a
2
+ b
2
+ c
2
= 1. Chứng minh rằng
1
2a
2
+ bc
+
1
2b
2
+ ac
+
1
2c
2
+ ab
(a + b + c)
2
ab + bc + ac
.
Bài 2.37. Cho a, b, c các số thực dương và n N, n 2. Tìm giá trị nhỏ nhất của biểu thức
P =
n
s
a
2
+ bc
a(b + c)
+
n
s
b
2
+ ac
b(a + c)
+
n
s
c
2
+ ab
c(a + b)
.
Bài 2.38. Cho các số dương a, b, c thỏa mãn abc = 1. Chứng minh rằng:
a
3
+ 5
a
3
(b + c)
+
b
3
+ 5
b
3
(c + a)
+
c
3
+ 5
c
3
(a + b)
9.
Bài 2.39. Cho số nguyên dương n 3 và 2n số thực dương a
1
; a
2
; . . . ; a
n
; b
1
; b
2
; . . . ; b
n
thỏa
mãn:
n
X
k=1
a
k
= 1;
n
X
k=1
b
2
k
= 1.
Chứng minh rằng:
n
X
k=1
a
k
(b
k
+ a
k+1
) < 1 (với a
n+1
= a
1
).
30
3. MỘT SỐ BẤT ĐẲNG THỨC KHÁC
§3. Một số bất đẳng thức khác
I. Bất đẳng thức Schur
1. Bất đẳng thức Schur
Định 1. Cho các số thực không âm x,y,z số thực dương r. Khi đó, ta có bất đẳng thức sau
x
r
(x y)(x z) + y
r
(y x)(y z) + z
r
(z x)(z y) 0.
Đẳng thức xảy ra khi a = b = c hoặc c = 0,a = b các hoán vị.
Chứng minh. bất đẳng thức cần chứng minh đối xứng ba biến nên ta giả sử x y z,
khi đó z
r
(z x)(z y) 0 và
x
r
(x y)(x z) + y
r
(y x)(y z) (x y) (x
r
(y z) y
r
(y z))
= (x y)(y z)(x
r
y
r
) 0.
Từ hai bất đẳng thức trên ta suy ra đpcm.
2. Các trường hợp đặc biệt
Xét r = 1 ta các dạng sau
a) x
3
+ y
3
+ z
3
+ 3xyz xy(x + y) + yz(y + z) + zx(z + x)
b) 4(a
3
+ b
3
+ c
3
) + 15abc (a + b + c)
3
c) xyz (x + y z)(y + z x)(z + x y)
d) x
2
+ y
2
+ z
2
+
9xyz
x + y + z
2(xy + yz + zx)
e) (x + y + z)
3
+ 9xyz 4(x + y + z)(xy + yz + zx)
r = 2 ta các dạng sau
a) x
4
+ y
4
+ z
4
+ xyz(x + y + z) xy(x
2
+ y
2
) + yz(y
2
+ z
2
) + zx(z
2
+ x
2
)
b) 6xyz(x + y + z) [2(xy + yz + zx) (x
2
+ y
2
+ z
2
)] (x
2
+ y
2
+ z
2
+ xy + yz + zx).
3. Bất đẳng thức Schur mở rộng
Định 2. Cho các số thực dương a, b, c, x, y, z sao cho các b (a, b, c) (x, y, z) các b
đơn điệu. Khi đó, ta có bất đẳng thức
a(x y)(x z) + b(y z)(y x) + c(z x)(z y) 0.
Chứng minh. Việc chứng minh bất đẳng thức y tương tự như chứng minh bất đẳng thức
Schur trên.
4. Các dụ
dụ 3.1 (Đồng Nai TST 2016). Cho các số thực dương a, b, c thỏa mãn abc = 1.
Chứng minh rằng:
r
a
b + c
+
r
b
c + a
+
r
c
a + b
3
3
a
3
+ b
3
+ c
3
+ 3
.
31
3. MỘT SỐ BẤT ĐẲNG THỨC KHÁC
Với x, y, z > 0 ta có:
(x + y + z)
2
1
x
2
+
1
y
2
+
1
z
2
9
3
p
x
2
y
2
z
2
· 3
3
r
1
x
2
y
2
z
2
= 27.
Suy ra :
x + y + z
3
3
s
1
x
2
+
1
y
2
+
1
z
2
. ()
Áp dụng (*) với x =
r
a
b + c
, y =
r
b
c + a
, z =
r
c
a + b
ta
r
a
b + c
+
r
b
c + a
+
r
c
a + b
3
3
r
b + c
a
+
c + a
b
+
a + b
c
=
3
3
p
ab (a + b) + bc (b + c) + ca (c + a)
.
Mặt khác, theo bất đẳng thức Schur ta
a
3
+ b
3
+ c
3
+ 3abc ab (a + b) + bc (b + c) + ca (c + a) .
Nên ta :
r
a
b + c
+
r
b
c + a
+
r
c
a + b
3
3
a
3
+ b
3
+ c
3
+ 3
.
Đẳng thức a = b = c = 1.
dụ 3.2 (Nghệ an TST 2014, ngày 2). Cho các số thực x, y, z > 0 thỏa xyz = 1.
Chứng minh rằng
3
r
x + y
2z
+
3
r
y + z
2x
+
3
r
z + x
2y
5(x + y + z) + 9
8
.
Đặt x = a
3
, y = b
3
, z = c
3
bất đẳng thức cần chứng minh trở thành
3
r
a
3
+ b
3
2c
3
+
3
r
b
3
+ c
3
2a
3
+
3
r
c
3
+ a
3
2b
3
5(a
3
+ b
3
+ c
3
) + 9
8
.
Theo bất đẳng thức Schur ta
a
3
+ b
3
+ c
3
+ 3 = a
3
+ b
3
+ c
3
+ 3abc ab(a + b) + bc(b + c) + ca(c + a).
Do đó
5(a
3
+ b
3
+ c
3
) + 9 = 2(a
3
+ b
3
+ c
3
) + 3(a
3
+ b
3
+ c
3
+ 3)
2(a
3
+ b
b
+ c
3
) + 3ab(a + b) + 3bc(b + c) + 3ca(c + a)
= (a + b)
3
+ (b + c)
3
+ (c + a)
3
. (1)
Ta chứng minh
(a + b)
3
8
3
r
a
3
+ b
3
2c
3
. (2)
32
3. MỘT SỐ BẤT ĐẲNG THỨC KHÁC
Thật vy (1) tương đương với
(a + b)
3
8abc
1
c
3
r
a
3
+ b
3
2
(a + b)
9
8
3
a
3
b
3
a
3
+ b
3
2
. (3)
Ta
4 · 8
2
· a
3
b
3
(a
3
+ b
3
) = ab · ab · ab(a
2
ab + b
2
)(a + b)
4 · 8
2
ab + ab + ab + a
2
ab + b
2
4
4
(a + b) = (a + b)
9
.
Do đó (2) đúng. Tương tự ta
(b + c)
3
8
3
r
b
3
+ c
3
2a
3
,
(c + a)
3
8
3
r
c
3
+ a
3
b
3
. (4)
Công các bất đẳng thức (2), (4) và từ (1) ta đpcm.
dụ 3.3. (VMO 2014) Cho a, b, c 0. Chứng minh rằng
3(a
2
+ b
2
+ c
2
) P (a + b + c)
2
,
với P = (a + b + c)
ab +
bc +
ca
+ (a b)
2
+ (b c)
2
+ (c a)
2
.
Ta
3(a
2
+ b
2
+ c
2
) P a + b + c
ab +
bc +
ca.
Bất đẳng thức y kết quả quen thuộc.
Đặt x =
a, y =
b, z =
c. Khi đó, bất đẳng thức
P (a + b + c)
2
X
x
4
+ xyz
X
x +
X
xy(x
2
+ y
2
) 4
X
x
2
y
2
(1)
Sử dụng bất đẳng thức Schur (với trường hợp r = 2) ta
X
x
4
+ xyz
X
x
X
xy(x
2
+ y
2
)
do đó
V T (1) 2
X
xy(x
2
+ y
2
) 2.
X
xy.2xy = 4
X
x
2
y
2
.
Hay (1) được chứng minh.
dụ 3.4. Cho a,b,c > 0. Chứng minh rằng
a
2
+ bc
a
2
(b + c)
+
b
2
+ ca
b
2
(c + a)
+
c
2
+ ab
c
2
(a + b)
1
a
+
1
b
+
1
c
.
Ta
a
2
+ bc
a
2
(b + c)
1
a
=
a
2
+ bc a(b + c)
a
2
(b + c)
=
(a b)(a c)
a
2
(b + c)
.
Do đó, bất đẳng thức cần chứng minh tương đương với
x(a b)(a c) + y(b c)(b a) + z(c a)(c b) 0 (1).
Với x =
1
a
2
(b + c)
, y =
1
b
2
(c + a)
, z =
1
c
2
(a + b)
.
Giả sử a > b > c, ta
1
a
2
(b + c)
1
b
2
(c + a)
=
ab(b a) + c(b
2
a
2
)
a
2
b
2
(b + c)(c + a)
> 0 hay x < y.
Do đó, b (x,y,z) b đơn điệu giảm. Do đó, theo bất đẳng thức Schur suy rộng, ta (1)
đúng.
33
3. MỘT SỐ BẤT ĐẲNG THỨC KHÁC
II. Bất đẳng thức Holder
1. Bất đẳng thức Holder
Định 3. Cho mn số thực dương a
i
j
với i = 1,m j = 1,n. Khi đó ta có bất đẳng thức sau
m
Y
i=1
n
X
j=1
a
m
i
j
!
n
X
j=1
m
Y
i=1
a
i
j
!
m
.
2. Trường hợp đặc biệt
m = 2 ta bất đẳng thức Cauchy-Schwarz
m = 3 ta
(a
3
1
+ a
3
2
+ ··· + a
3
n
)(b
3
1
+ b
3
2
+ ··· + b
3
n
)(c
3
1
+ c
3
2
+ ··· + c
3
n
) (a
1
b
1
c
1
+ ··· + a
n
b
n
c
n
)
3
.
3. dụ minh họa
dụ 3.5. Cho các số thực dương a, b, c. Chứng minh rằng
(a
3
+ 2)(b
3
+ 2)(c
3
+ 2) (a + b + c)
3
.
Áp dụng bất đẳng thức Holder ta
(a
3
+ 2)(b
3
+ 2)(c
3
+ 2) = (a
3
+ 1 + 1)(1 + b
3
+ 1)(1 + 1 + c
3
)
(a + b + c)
3
.
dụ 3.6. Cho các số thực dương a, b, c. Chứng minh rằng
a + b
a + 2c
+
b + c
b + 2a
+
c + a
c + 2b
2
a + b + c.
Áp dụng bất đẳng thức Holder ta
a + b
a + 2c
+
b + c
b + 2a
+
c + a
c + 2b
2
X
(a + b)(a + 2c)
8(a + b + c)
3
.
Mặt khác
X
(a + b)(a + 2c) = (a + b + c)
2
+ 3(ab + bc + ca) 2(a + b + c)
2
.
Do đó
a + b
a + 2c
+
b + c
b + 2a
+
c + a
c + 2b
2
a + b + c.
dụ 3.7. Cho các số thực dương a, b, c. Chứng minh rằng
a
a
2
+ 8bc
+
b
b
2
+ 8ca
+
c
c
2
+ 8ab
1.
34
3. MỘT SỐ BẤT ĐẲNG THỨC KHÁC
Gọi P vế trái của bất đẳng thức. áp dụng bất đẳng thức Holder ta
P
2
X
a(a
2
+ 8bc)
(a + b + c)
3
.
X
a(a
2
+ 8bc) = a
3
+ b
3
+ c
3
+ 24abc (a + b + c)
3
,
nên ta P 1.
III. Bất đẳng thức Chebyshev
1. Bất đẳng thức Chebyshev
Định 4. a) Với hai dãy n số thực a
1
a
2
··· a
n
b
1
b
2
··· b
n
cùng tăng hoặc
cùng giảm, tức
(
a
1
a
2
··· a
n
b
1
b
2
··· b
n
hoặc
(
a
1
a
2
··· a
n
b
1
b
2
··· b
n
ta luôn có
a
1
b
1
+ a
2
b
2
+ ··· + a
n
b
n
n
a
1
+ a
2
+ ··· + a
n
n
···
b
1
+ b
2
+ ··· + b
n
n
.
b) Với hai dãy n số thực a
1
a
2
··· a
n
b
1
b
2
··· b
n
có một dãy tăng một
dãy giảm, tức
(
a
1
a
2
··· a
n
b
1
b
2
··· b
n
hoặc
(
a
1
a
2
··· a
n
b
1
b
2
··· b
n
ta luôn có
a
1
b
1
+ a
2
b
2
+ ··· + a
n
b
n
n
a
1
+ a
2
+ ··· + a
n
n
···
b
1
+ b
2
+ ··· + b
n
n
.
Chứng minh. Ta chứng minh cho trường hợp
(
a
1
a
2
··· a
n
b
1
b
2
··· b
n
.
Đặt a =
a
1
+ a
2
+ ··· + a
n
n
, khi đó tồn tại chỉ số k sao cho a
k
a a
k+1
, với k đó ta chọn số b
sao cho b
k
b b
k+1
. Khi đó ta
(a a
i
)(b b
i
) 0 ab ab
i
a
i
+ a
i
b
i
.
Cho i chạy từ 1 đến n và cộng n bất đẳng thức đó ta được
nab a
n
X
i=1
b
i
b
n
X
i=1
a
i
+
n
X
i=1
a
i
b
i
0.
b
n
P
i=1
a
i
= nab nên ta
n
X
i=1
a
i
b
i
a
n
X
i=1
b
i
=
1
n
n
X
i=1
a
i
!
n
X
i=1
b
i
!
.
Ta đpcm.
35
3. MỘT SỐ BẤT ĐẲNG THỨC KHÁC
2. dụ minh họa
dụ 3.8. Cho các số thực dương a, b, c thỏa a
2
+ b
2
+ c
2
1. Chứng minh rằng
a
3
b + c
+
b
3
c + a
+
c
3
a + b
1
2
.
Gải sử a b c, khi đó a
2
b
2
c
2
và
a
b + c
b
c + a
c
a + b
, nên áp dụng bất đẳng thức
Trebyshev ta
V T = a
2
a
b + c
+ b
2
b
c + a
+ c
2
c
a + b
1
3
(a
2
+ b
2
+ c
2
)
a
b + c
+
b
c + a
+
c
a + b
1
2
.
dụ 3.9. Cho các số thực không âm x, y, z thỏa mãn x + y + z = 1. Chứng minh rằng
1
1 + x
2
+
1
1 + y
2
+
1
z
2
+ 1
27
10
.
Bất đẳng thức cần chứng minh tương đương với
9
10
1
1 + x
2
+
9
10
1
1 + y
2
+
9
10
1
1 + z
2
0
(3x 1)
3x + 1
1 + x
2
+ (3y 1)
3y + 1
1 + y
2
+ (3z 1)
3z + 1
1 + z
2
0. (1)
Với 1 a b 0 ta
(3a + 1)(b
2
+ 1) (3b + 1)(a
2
+ 1) = (a b)(3ab a b + 3) 0
3a + 1
1 + a
2
3b + 1
1 + b
2
.
Giả sử x y z, ta 3x 1 3y 1 3z 1 và
3x + 1
1 + x
2
3y + 1
1 + z
2
3z + 1
1 + z
2
. Do đó áp dụng
bất đẳng thức Chebyshev ta
V T (1) (3x 1 + 3y 1 + 3z 1)
3x + 1
1 + x
2
+
3y + 1
1 + z
2
+
3z + 1
1 + z
2
= 0.
Vy bài toán được chứng minh.
IV. Bài tập
Bài 3.1. Cho các số thực dương a, b, c. Chứng minh rằng
2a
b + c
+
2b
c + a
+
2c
a + b
+
3
r
3abc
a
3
+ b
3
+ c
3
4.
Bài 3.2. Cho các số thực dương a, b, c thỏa mãn a + b + c = 1. Chứng minh rằng
2
ab
a + b
+
bc
b + c
+
ca
c + a
+ 1 6(ab + bc + ca).
36
3. MỘT SỐ BẤT ĐẲNG THỨC KHÁC
Bài 3.3. Cho các số thực dương a, b, c. Chứng minh rằng
X
cyc
3
s
a
2
+ bc
b
2
+ c
2
9
3
abc
a + b + c
.
Bài 3.4. (APMO 2004) Cho các số thực dương a,b,c. Chứng minh rằng
(a
2
+ 2)(b
2
+ 2)(c
2
+ 2) 9(ab + bc + ca).
Bài 3.5. Cho các số thực không âm a, b, c. Chứng minh rằng
s
(a + b)
3
ab(4a + 4b + c)
+
s
(b + c)
3
bc(4b + 4c + a)
+
s
(c + a)
3
ca(4c + 4a + b)
2
2.
Bài 3.6. (Hello IMO 2007- Trần Nam Dũng) Chứng minh rằng với mọi a, b, c 0,ta có:
2(a
2
+ b
2
+ c
2
) + abc + 8 5(a + b + c).
Bài 3.7. Cho các số thực dương a,b,c thỏa mãn abc = 1. Chứng minh rằng
1
a
2
+
1
b
2
+
1
c
2
+ 3 2(a + b + c).
Bài 3.8. Cho các số thực dương a, b, c thỏa mãn abc = 1. Chứng minh rằng
a
2
+ b
2
+ c
2
+ 3 2(ab + bc + ca).
Bài 3.9. Cho các số thực dương a, b, c thỏa a + b + c = 1. Chứng minh rằng
4a
3
+ 9b
3
+ 36c
3
1.
Bài 3.10. Cho các số thực dương a, b, c a + b + c = 1. Chứng minh rằng
a
3
a + 2b
+
b
3
b + 2c
+
c
3
c + 2a
1.
Bài 3.11. Cho các số thực dương x, y, z và a, b, c thỏa mãn
a
x
+
b
y
+
c
z
= 1. Chứng minh rằng
x
2
+ y
2
+ z
2
3
a
2
+
3
b
2
+
3
c
2
3
.
Bài 3.12. Cho a, b, c, x, y, z các số thực dương thỏa ax + by + cz = 1. Chứng minh rằng
x
n
+ y
n
+ z
n
n1
a
n
+
n1
b
n
+
n1
c
n
1n
.
Bài 3.13. Cho 3 số thực dương a,b,c. Chứng minh bất đẳng thức:
(a + b + c)
1
a
+
1
b
+
1
c
+ 4
2
ab + bc + ca
a
2
+ b
2
+ c
2
9 + 4
2.
37
4. PHƯƠNG PHÁP QUY NẠP
§4. Phương pháp quy nạp
I. thuyết
Quy nạp toán học một phương pháp mạnh để chứng minh các phát biểu phụ thuộc vào một
số tự nhiên.
Cho (P (n))
n0
một dãy các mệnh đề. Phương pháp quy nạp toán học đưc sử dụng để chứng
minh P (n) đúng với mọi n n
0
với n
0
một số tự nhiên.
Phương pháp quy nạp toán học (dạng yếu): Giả sử
P (n
0
) đúng.
Với mọi k n
0
P (k) đúng thì P (k + 1) đúng.
Khi đó P (n) đúng với mọi n n
0
.
Phương pháp quy nạp toán học (bước nhảy s): Cho s số nguyên dương. Giả sử
P (n
0
) , P (n
0
+ 1) ,...,P (n
0
+ s 1) đúng.
Với mọi k n
0
, P (k) đúng o theo P (k + s) đúng .
Khi đó P (n) đúng với mọi n n
0
.
Phương pháp quy nạp toán học (Dạng mạnh): Giả sử
P (n
0
) đúng
Với mọi k n
0
, P (m) đúng với mọi m n
0
m k o theo P (k + 1) đúng.
Khi đó P (n) đúng với mọi n n
0
.
II. dụ minh họa
dụ 4.1. Chứng minh rằng
n 1 +
1
2
+
1
3
+ ··· +
1
n
2
n (1.1)
với mọi số nguyên dương n.
Ta thấy (1.1) đúng với n = 1.
Giả sử (1.1) đúng với n = k 1, tức
k 1 +
1
2
+
1
3
+ ··· +
1
k
2
k, (1.2)
ta chứng minh (1.1) đúng với n = k + 1, tức
k + 1 1 +
1
2
+
1
3
+ ··· +
1
k + 1
2
k + 1. (1.3)
Thật vy, dựa vào (1.2) ta
k +
1
k + 1
1 +
1
2
+
1
3
+ ··· +
1
k + 1
2
k +
1
k + 1
.
Mặt khác
k +
1
k + 1
k + 1 =
1
k + 1
1
k +
k + 1
> 0,
38
4. PHƯƠNG PHÁP QUY NẠP
và
2
k + 1 2
k
1
k + 1
=
2
k +
k + 1
1
k + 1
> 0.
Từ đó ta đpcm.
dụ 4.2 (VMO 2011). Chứng minh rằng với n 1,x > 0 ta bất đẳng thức:
x
n
(x
n+1
+ 1)
x
n
+ 1
x + 1
2
2n+1
. (1.4)
Đẳng thức xảy ra khi nào?
Với n = 1 ta cần chứng minh:
x(x
2
+ 1)
x + 1
x + 1
2
3
8x(x
2
+ 1) (x + 1)
4
hay là:
x
4
4x
3
+ 6x
2
4x + 1 0 (x 1)
4
0 (đúng).
Suy ra (1.4) đúng với n = 1. Đẳng thức xảy ra khi x = 1.
Giả sử (1.4) đúng với n = k 1, tức là:
x
k
(x
k+1
+ 1)
x
k
+ 1
x + 1
2
2k+1
. (1.5)
Ta cần chứng minh:
x
k+1
(x
k+2
+ 1)
x
k+1
+ 1
x + 1
2
2k+3
. (1.6)
Thật vy, ta có:
x + 1
2
2k+3
=
x + 1
2
2
x + 1
2
2k+1
x + 1
2
2
x
k
(x
k+1
+ 1)
x
k
+ 1
.
Nên để chứng minh (1.6) ta chỉ cần chứng minh
x + 1
2
2
x
k
(x
k+1
+ 1)
x
k
+ 1
x
k+1
(x
k+2
+ 1)
x
k+1
+ 1
,
hay
x + 1
2
2
(x
k+1
+ 1)
2
x(x
k+2
+ 1)(x
k
+ 1). (1.7)
Khai triển (1.7), biến đổi và rút gọn ta thu được:
x
2k+2
(x 1)
2
2x
k+1
(x 1)
2
+ (x 1)
2
0 (x 1)
2
(x
k+1
1)
2
0,
bất đẳng thức cuối hiển nhiên đúng. Đẳng thức khi x = 1.
Vy bài toán được chứng minh.
39
4. PHƯƠNG PHÁP QUY NẠP
dụ 4.3. Cho n 2 số thực không âm a
1
a
2
··· a
n
tổng bằng 1. Chứng minh
rằng:
n
X
i=1
(i 1)a
2
i
n 1
2n
n
X
i=1
a
i
!
2
. (1.8)
Với n = 2 ta có: a
2
2
1
4
(a
1
+ a
2
)
2
đúng do a
2
a
1
, nên (1.8) đúng với n = 2.
Giả sử (1.8) đúng với n = k, tức là:
k
X
i=1
(i 1)a
2
i
k 1
2k
k
X
i=1
a
i
!
2
. (1.9)
Ta chứng minh (1.8) đúng với n = k + 1, tức là:
k+1
X
i=1
(i 1)a
2
i
k
2(k + 1)
k+1
X
i=1
a
i
!
2
k
X
i=1
(i 1)a
2
i
+ ka
2
k+1
k
2(k + 1)
k
X
i=1
a
i
+ a
k+1
!
2
. (1.10)
Đặt x =
1
k
k
P
i=1
x
i
,x x
k+1
. Sử dụng (1.9), ta chỉ cần chứng minh:
k 1
2k
k
X
i=1
a
i
!
2
+ ka
2
k+1
k
2(k + 1)
k
X
i=1
a
i
+ a
k+1
!
2
k(k 1)
2
x
2
+ kx
2
k+1
k
2(k + 1)
(kx + x
k+1
)
2
(k
2
1)x
2
+ 2(k + 1)x
k+1
(kx + x
k+1
)
2
x
2
+ 2kx · x
k+1
(2k + 1)x
2
k+1
0
(x x
k+1
) [x + (2k + 1)x
k+1
] 0 (đúng).
Vy (1.8) luôn đúng.
dụ 4.4. Cho các số nguyên dương phân biệt a
1
, a
2
, ··· , a
n
. Chứng minh rằng
a
3
1
+ a
3
2
+ ··· + a
3
n
(a
1
+ a
2
+ ··· + a
n
)
2
. (1.11)
đúng với mọi n 1.
Không mất tính tổng quát, ta giả sử a
1
< a
2
< ··· < a
n
.
Với n = 1 ta a
3
1
a
2
1
nên (1.11) đúng với n = 1.
Giả sử (1.11) đúng với n = k và a
1
< a
2
< ··· < a
k
< a
k+1
các số nguyên dương. Khi đó
a
k+1
a
k
+ 1, nên
(a
k+1
1) a
k+1
2
a
k
(a
k
+ 1)
2
= 1 + 2 + ··· + a
k
.
1 + 2 + ··· + a
k
tổng của a
k
số nguyên dương đầu tiên, nên
1 + 2 + ··· + a
k
a
1
+ a
2
+ ··· + a
k
.
40
4. PHƯƠNG PHÁP QUY NẠP
Suy ra
(a
k+1
1) a
k+1
2
a
1
+ a
2
+ ··· + a
k
.
Hay
a
3
k+1
2a
k+1
(a
1
+ a
2
+ ··· + a
k
) + a
2
k+1
.
Do đó
a
3
1
+ a
3
2
+ ··· + a
3
k
+ a
3
k+1
(a
1
+ a
2
+ ··· + a
k
)
2
+ 2a
k+1
(a
1
+ a
2
+ ··· + a
k
) + a
2
k+1
= (a
1
+ a
2
+ ··· + a
k+1
)
2
.
Suy ra (1.11) được chứng minh.
dụ 4.5. Cho 2n số thực dương a
1
, a
2
, ··· , a
n
và b
1
, b
2
, ··· , b
n
. Chứng minh rằng
n
X
k=1
(b
1
+ b
2
+ ··· + b
k
)b
k
a
1
+ a
2
+ ··· + a
k
< 2
n
X
i=1
b
2
i
a
i
. (1.12)
Ta thấy bất đẳng thức (1.12) đúng với n = 1.
Giả sử bất đẳng thức (1.12) đúng với n , tức
n
X
k=1
(b
1
+ b
2
+ ··· + b
k
)b
k
a
1
+ a
2
+ ··· + a
k
< 2
n
X
i=1
b
2
i
a
i
, (1.13)
ta chứng minh (1.12) đúng với n + 1, tức
n+1
X
k=1
(b
1
+ b
2
+ ··· + b
k
)b
k
a
1
+ a
2
+ ··· + a
k
< 2
n+1
X
i=1
b
2
i
a
i
. (1.14)
Áp dụng cho trường hợp 2n số
(a
1
+ a
2
, a
3
, a
4
. . . , a
n+1
, b
1
+ b
2
, b
3
, b
4
, . . . , b
n+1
)
ta
n+1
X
k=1
(b
1
+ b
2
+ ··· + b
k
)b
k
a
1
+ a
2
+ ··· + a
k
< 2
n+1
X
i=1
b
2
i
a
i
.
(b
1
+ b
2
)
2
a
1
+ a
2
+
n
X
k=3
(b
1
+ ··· + b
k
) b
k
a
1
+ ··· + a
k
3
2
(b
1
+ b
2
)
2
a
1
+ a
2
+ 2
n
X
k=3
b
2
k
a
k
.
Bất đẳng thức tương đương với
b
2
1
a
1
+
(b
1
+ b
2
) b
2
a
1
+ a
2
+
1
2
(b
1
+ b
2
)
2
a
1
+ a
2
3
2
b
2
1
a
1
+ 2
b
2
2
a
2
b
2
1
+ 4b
1
b
2
+ 3b
2
2
a
1
+ a
2
b
2
1
a
1
+ 4
b
2
2
a
2
b
2
1
+ 4b
1
b
2
+ 3b
2
2
b
2
1
+
a
2
a
1
b
2
1
+ 4
a
1
a
2
b
2
2
+ 4b
2
2
4b
1
b
2
a
2
a
1
b
2
1
+ 4
a
1
a
2
b
2
2
+ b
2
2
Bất đẳng thức cuối đúng, bằng cách áp dụng bất đẳng thức AM-GM
a
2
a
1
b
2
1
+ 4
a
1
a
2
b
2
2
2
r
a
2
a
1
b
2
1
· 4
a
1
a
2
b
2
2
= 4b
1
b
2
.
41
5. PHƯƠNG PHÁP PHÂN TÍCH BÌNH PHƯƠNG SOS
§5. Phương pháp phân tích bình phương SOS
I. thuyết
1. Một số tiêu chuẩn đánh giá
Xét biểu thức:
S = f (a, b, c) = S
a
(b c)
2
+ S
b
(c a)
2
+ S
c
(a b)
2
.
trong đó S
a
, S
b
, S
c
các biểu thức chứa a, b, c các số thực không âm.
Tính chất 1.
Nếu S
a
, S
b
, S
c
0 thì S 0.
Tính chất 2.
Nếu a b c và S
b
, S
b
+ S
c
0, S
b
+ S
a
0 thì S 0.
Thật vy:
a b c nên
(c a)
2
= [(a b) + (b c)]
2
(a b)
2
+ (b c)
2
.
Suy ra
S S
a
(b c)
2
+ S
b
(a b)
2
+ (b c)
2
+ S
c
(a b)
2
= (S
a
+ S
b
) (b c)
2
+ (S
b
+ S
c
) (a b)
2
0.
Tính chất 3.
Nếu a b c và S
b
0, S
a
+ 2S
b
0, S
c
+ 2S
b
0 thì S 0.
Thật vy, ta
(a c)
2
= [(a b) + (b c)]
2
2 (a b)
2
+ 2 (b c)
2
.
Suy ra
S (S
a
+ 2S
b
) (b c)
2
+ (S
c
+ 2S
b
) (a b)
2
0.
Tính chất 4.
Nếu a b c và S
b
0, S
c
0, a
2
S
b
+ b
2
S
a
0 thì S 0.
Ta a b c
a c
b c
b
a
. Suy ra
S
a
(b c)
2
+ S
b
(c a)
2
= (b c)
2
"
S
a
+ S
b
c a
b c
2
#
(b c)
2
S
a
+ S
b
b
2
a
2
0.
2. Một số biểu diễn sở
a
b
+
b
a
2 =
1
ab
(a b)
2
.
a
2
+ b
2
+ c
2
ab bc ca =
1
2
(a b)
2
+ (b c)
2
+ (c a)
2
.
p
2 (a
2
+ b
2
) (a + b) =
(a b)
2
a + b +
p
2 (a
2
+ b
2
)
.
a
3
+ b
3
+ c
3
3abc =
1
2
(a + b + c)
(a b)
2
+ (b c)
2
+ (c a)
2
.
a
3
+ b
3
ab (a + b) = (a + b) (a b)
2
.
42
5. PHƯƠNG PHÁP PHÂN TÍCH BÌNH PHƯƠNG SOS
a
3
+ b
3
+ c
3
a
2
b b
2
c c
2
a =
1
3
(2a + b) (a b)
2
+ (2b + c) (b c)
2
+ (2c + a) (c a)
2
.
(a + b) (b + c) (c + a) 8abc = a (b c)
2
+ b (c a)
2
+ c (a b)
2
.
a
2
b + b
2
c + c
2
a ab
2
bc
2
ca
2
=
1
3
(a b)
3
+ (b c)
3
+ (c a)
3
.
a
b + c
+
b
c + a
+
c
a + b
3
2
=
(a b)
2
2 (a + c) (b + c)
+
(b c)
2
2 (b + a) (c + a)
+
(c a)
2
2 (c + b) (a + c)
.
II. Các dụ
dụ 5.1 (IMO 2005). Cho x, y, z > 0 thỏa mãn xyz 1. Chứng minh rằng
x
5
x
2
x
5
+ y
2
+ z
2
+
y
5
y
2
y
5
+ z
2
+ x
2
+
z
5
z
2
z
5
+ x
2
+ y
2
0. (1)
Ta
x
5
x
2
x
5
+ y
2
+ z
2
x
5
x
2
xyz
x
5
+ (y
2
+ z
2
) xyz
=
x
4
x
2
yz
x
4
+ yz (y
2
+ z
2
)
2x
4
x
2
(y
2
+ z
2
)
2x
4
+ (y
2
+ z
2
)
2
=
2a
2
a (b + c)
2a
2
+ (b + c)
2
,
với a = x
2
, b = y
2
, c = z
2
.
Suy ra
V T (1)
X
a, b, c
2a
2
a (b + c)
2a
2
+ (b + c)
2
=
X
a, b, c
a (a b) + a (a c)
2a
2
+ (b + c)
2
=
X
(a b)
a
2a
2
+ (b + c)
2
b
2b
2
+ (c + a)
2
=
X
a, b, c
(a b)
2
c (2a + 2b + c) + a
2
ab + b
2
2a
2
+ (b + c)
2
(2b
2
+ (c + a)
2
)
0 (đúng).
dụ 5.2 (VMO 2015). Cho a, b, c 0. Chứng minh rằng
(a + b + c)
ab +
bc +
ca
+ (a b)
2
+ (b c)
2
+ (c a)
2
(a + b + c)
2
.
Bất đẳng thức cần chứng minh tương đương với
(a b)
2
+ (b c)
2
+ (c a)
2
(a + b + c)
a + b + c
ab
bc
ca
.
Đặt x =
a,y =
b,z =
c ta cần chứng minh
X
x
2
y
2
2
x
2
+ y
2
+ z
2
x
2
+ y
2
+ z
2
xy yz zx
. (2)
x
2
+ y
2
+ z
2
xy yz zx =
1
2
(x y)
2
+
1
2
(y z)
2
+
1
2
(z x)
2
.
43
5. PHƯƠNG PHÁP PHÂN TÍCH BÌNH PHƯƠNG SOS
Nên (2) trở thành
S
x
(y z)
2
+ S
y
(z x)
2
+ S
z
(x y)
2
0, (3)
với S
x
= y
2
+ z
2
+ 4yz x
2
,S
y
= z
2
+ x
2
+ 4zx y
2
, S
z
= x
2
+ y
2
+ 4xy z
2
.
Gải sử x y z , khi đó S
y
0,S
z
0 và S
x
+ S
y
= 2z
2
+ 4yz + 4zx 0 .
Lại (x z)
2
(y z)
2
nên
V T (3) (S
x
+ S
y
) (y z)
2
0 nên (3) đúng.
dụ 5.3. Cho a, b, c > 0. Chứng minh rằng:
a
2
+ b
2
+ c
2
ab + bc + ca
+
8abc
(a + b)(b + c)(c + a)
2.
Ta
a
2
+ b
2
+ c
2
(ab + bc + ca) =
1
2
(a b)
2
+ (b c)
2
+ (c a)
2
(a + b) (b + c) (c + a) 8abc = a (b c)
2
+ b (c a)
2
+ c (a b)
2
.
Bất đẳng thức cần chứng minh tương đương với
(a b)
2
+ (b c)
2
+ (c a)
2
ab + bc + ca
2a (b c)
2
+ 2b (c a)
2
+ 2c (a b)
2
(a + b) (b + c) (c + a)
X
(a b)
2
1
ab + bc + ca
2c
(a + b) (b + c) (c + a)
0
X
(a b)
2
S
c
0. (4)
Với
S
a
=
1
ab + bc + ca
2a
(a + b) (b + c) (c + a)
S
b
=
1
ab + bc + ca
2b
(a + b) (b + c) (c + a)
S
c
=
1
ab + bc + ca
2c
(a + b) (b + c) (c + a)
.
Giả sử a b c ta S
b
, S
c
0 và
S
a
+ S
b
=
2
ab + bc + ca
2
(a + c) (b + c)
0.
Suy ra (4) đúng.
dụ 5.4. Cho a, b, c > 0. Chứng minh rằng:
a
2
b
+
b
2
c
+
c
2
a
3(a
3
+ b
3
+ c
3
)
a
2
+ b
2
+ c
2
.
Ta giả sử b nằm giữa a và c. Ta
a
2
b
+
b
2
c
+
c
2
a
(a + b + c) =
(a b)
2
b
+
(b c)
2
c
+
(c a)
2
a
3(a
3
+ b
3
+ c
3
)
a
2
+ b
2
+ c
2
(a + b + c) =
(a + b) (a b)
2
+ (b + c) (b c)
2
+ (c + a) (c a)
2
a
2
+ b
2
+ c
2
.
44
5. PHƯƠNG PHÁP PHÂN TÍCH BÌNH PHƯƠNG SOS
Nên bất đẳng thức cần chứng minh tương đương với
S
a
(b c)
2
+ S
b
(c a)
2
+ S
c
(a b)
2
0. (5)
Với S
a
=
a
2
+ b
2
bc
c
; S
b
=
b
2
+ c
2
ac
a
; S
c
=
a
2
+ c
2
ab
b
.
Nếu a b c S
a
0,S
c
0 và
S
a
+ 2S
b
=
a
2
+ b
2
bc
c
+ 2
b
2
+ c
2
ca
a
> 0.
Ta
S
c
+ 2S
b
=
c
2
+ a
2
ab
b
+ 2
b
2
+ c
2
ca
a
c
2
b
+ a b + 2
b
2
+ c
2
ca
a
c
2
a
+ a b + 2
b
2
+ c
2
ca
a
,
hay
S
c
+ 2S
b
a
2
+ 2b
2
+ 3c
2
ab 2ca
a
.
Mặt khác
1
3
a
2
+ 3c
2
2ca
2
3
a
2
+ 2b
2
> ab
a
2
+ 2b
2
+ 3c
2
2ca + ab.
Do đó, ta S
c
+ 2S
b
> 0.
Nếu a b c S
b
0 thì ta
S
b
+ S
a
=
b
2
+ c
2
ca
a
+
a
2
+ b
2
bc
c
> 0
S
b
+ S
c
=
b
2
+ c
2
ca
a
+
c
2
+ a
2
ab
b
> 0.
dụ 5.5 (Iran 1996). Cho a, b, c số dương. Chứng minh rằng :
(ab + bc + ca)
1
(a + b)
2
+
1
(b + c)
2
+
1
(c + a)
2
9
4
.
Đặt x = a + b, y = b + c, z = c + a a =
x + z y
2
,b =
x + y z
2
,c =
z + y x
2
.
Bất đẳng thức cần chứng minh trở thành
2xy + 2yz + 2zx x
2
y
2
z
2
1
x
2
+
1
y
2
+
1
z
2
9
2
xy + yz + zx x
2
y
2
z
2
1
x
2
+
1
y
2
+
1
z
2
+
x
2
+ y
2
+ z
2
1
x
2
+
1
y
2
+
1
z
2
9 0
(x y)
2
+ (y z)
2
+ (z x)
2
1
x
2
+
1
y
2
+
1
z
2
+
x
2
y
2
+
y
2
x
2
2
+
y
2
z
2
+
z
2
y
2
2
+
+
z
2
x
2
+
x
2
z
2
2
0
S
z
(x y)
2
+ S
x
(y z)
2
+ S
y
(z x)
2
0; S
x
=
2
yz
1
x
2
, S
y
=
2
zx
1
y
2
, S
z
=
2
xy
1
z
2
.
45
5. PHƯƠNG PHÁP PHÂN TÍCH BÌNH PHƯƠNG SOS
Giả sử x y z suy ra S
x
0,S
x
S
y
S
z
.
Mặt khác
y + z x zx z (y + z) 2y
2
S
y
0
(x z)
2
y
2
z
2
(x y)
2
(y z) (y + z x) 0 (đúng).
Suy ra
S
z
(x y)
2
+ S
x
(y z)
2
+ S
y
(z x)
2
S
y
(z x)
2
+ S
z
(x y)
2
y
2
z
2
(x y)
2
S
y
+ S
z
(x y)
2
y
2
S
y
+ z
2
S
z
= y
2
2
zx
1
y
2
+ z
2
2
xy
1
z
2
=
2y
2
zx
+
2z
2
xy
2 0
y
3
+ z
3
xyz.
Do y + z x,y
2
+ z
2
yz yz suy ra y
2
S
y
+ z
2
S
z
0.
III. Bài tập
Bài 5.1 (Bất đẳng thức Schur). Cho a, b, c các không âm . Chứng minh rằng :
a
3
+ b
3
+ c
3
+ 3abc ab (a + b) + bc (b + c) + ca (c + a) .
Bài 5.2. Cho a, b, c 0 và không hai số nào đồng thời bằng 0. Chứng minh rằng
ab bc + ca
b
2
+ c
2
+
bc ca + ab
c
2
+ a
2
+
ca ab + bc
a
2
+ b
2
3
2
.
Bài 5.3. Cho a, b, c các không âm . Chứng minh rằng :
a
3
+ b
3
+ c
3
+ 3abc ab
p
2 (a
2
+ b
2
) + bc
p
2 (b
2
+ c
2
) + ca
p
2 (c
2
+ a
2
).
Bài 5.4. Cho a, b, c số không âm.Chứng minh rằng :
2a
2
+ bc
b
2
+ c
2
+
2b
2
+ ca
c
2
+ a
2
+
2c
2
+ ab
a
2
+ b
2
9
2
.
Bài 5.5. Cho a, b, c các số dương. Chứng minh rằng
a
3
+ b
3
+ c
3
+ 6
ab
2
+ bc
2
+ ca
2
3
a
2
b + b
2
c + c
2
a
+ 12abc.
Bài 5.6. (VN TST 2006) Chứng minh rằng với mọi số thực x,y,z [1; 2] , ta luôn bất đẳng
thức sau :
(x + y + z)
1
x
+
1
y
+
1
z
6
x
y + z
+
y
z + x
+
z
x + y
.
Hỏi đẳng thức xảy ra khi và chỉ khi nào ?
Bài 5.7. Bài 5.8. Cho các số thực a,b,c [0; 1] thỏa mãn a + b + c = 2.
Chứng minh rằng
2
3
(a
2
+ b
2
+ c
2
) a
3
+ b
3
+ c
3
4
3
(a
2
+ b
2
+ c
2
) 3abc.
46
5. PHƯƠNG PHÁP PHÂN TÍCH BÌNH PHƯƠNG SOS
Bài 5.9. Cho a, b, c > 0. Chứng minh rằng:
1
2
+
a
2
+ b
2
+ c
2
ab + bc + ca
a
b + c
+
b
c + a
+
c
a + b
.
Bài 5.10. Cho a, b, c > 0. Chứng minh rằng:
p
3(a
2
+ b
2
+ c
2
)
a
2
+ b
2
a + b
+
b
2
+ c
2
b + c
+
c
2
+ a
2
c + a
.
Bài 5.11. Cho a, b, c > 0. Chứng minh rằng:
3(a
2
+ b
2
+ c
2
)
a + b + c
a
2
+ b
2
a + b
+
b
2
+ c
2
b + c
+
c
2
+ a
2
c + a
.
Bài 5.12. Cho a, b, c > 0. Chứng minh rằng:
a
2
b + c
+
b
2
c + a
+
c
2
a + b
+ a + b + c
3
2
p
3(a
2
+ b
2
+ c
2
).
Bài 5.13. Cho a, b, c > 0. Chứng minh rằng:
a
b + c
+
b
c + a
+
c
a + b
1
2
4
ab + bc + ca
a
2
+ b
2
+ c
2
.
Bài 5.14. Cho a, b, c > 0. Chứng minh rằng:
2(a
3
+ b
3
+ c
3
)
abc
+
9(a + b + c)
2
a
2
+ b
2
+ c
2
33.
47
6. PHƯƠNG PHÁP DỒN BIẾN
§6. Phương pháp dồn biến
I. thuyết
Mục đích của phương pháp y làm giảm biến trong bất đẳng thức cần chứng minh.
II. dụ minh họa
dụ 6.1. Cho a, b, c 3 và a + b + c = 3. Chứng minh rằng
1
a
2
+
1
b
2
+
1
c
2
1
a
+
1
b
+
1
c
.
Giả sử a = min{a, b, c} và đặt f(a, b, c) =
P
cyc
1
a
2
1
a
.
Ta 3 a 1 và
f(a, b, c) f
a,
b + c
2
,
b + c
2
=
1
b
2
+
1
c
2
8
(b + c)
2
1
b
+
1
c
4
b + c
=
(b c)
2
(b
2
+ 4bc + c
2
bc(b + c))
b
2
c
2
(b + c)
2
=
(b c)
2
((3 a)
2
+ bc(a 1))
b
2
c
2
(b + c)
2
(b c)
2
(3 a)
2
+
(3 a)
2
(a 1)
4
b
2
c
2
(b + c)
2
=
(b c)
2
(3 a)
2
(a + 3)
4b
2
c
2
(b + c)
2
0.
Tiếp theo ta chứng minh f(a,t,t) 0 với t =
a + b
2
=
3 a
2
. Thật vy
f(a,t,t) =
1
a
2
+
2
t
2
1
a
2
t
=
t
2
+ 2a
2
at
2
2ta
2
a
2
t
2
=
(a + 3)(a 1)
2
4a
2
t
2
0
Vy bài toán được chứng minh.
dụ 6.2. Cho các số thực dương a, b, c tích bằng 1. Chứng minh rằng
1
a
+
1
b
+
1
c
+
6
a + b + c
5.
48
6. PHƯƠNG PHÁP DỒN BIẾN
Gải sử a = max{a, b, c}, suy ra bc 1 a.
Đặt f(a, b, c) =
1
a
+
1
b
+
1
c
+
6
a + b + c
và x =
bc. Ta
f(a, b, c) f(a,x,x) =
1
b
+
1
c
2
x
+
6
a + b + c
6
a + 2x
=
b
c
2
bc
6
b
c
2
(a + b + c)(a + 2x)
=
b
c
2
1
bc
6
(a + b + c)(a + 2x)
.
Ta
(a + b + c)(a + 2x) 6bc
1
x
2
+ 2x
1
x
2
+ 2x
6x
2
3 · 3 6x
2
> 0.
Suy ra
f(a, b, c) f(a,x,x) = f
1
x
2
,x,x
.
Ta chứng minh f
1
x
2
,x,x
5. (1)
Tuy nhiên (1) tương đương với
(x 1)
2
(2x
4
+ 4x
3
4x
2
x + 2) 0. (2)
Bất đẳng thức (2) đúng do 0 < x 1.
dụ 6.3. Cho các số thực dương a, b, c thỏa mãn a
2
+ b
2
+ c
2
= 3. Chứng minh rằng
a
3
(b + c) + b
3
(c + a) + c
3
(a + b) 6.
Giả sử a = min{a, b, c}. Đặt f(a, b, c) = a
3
(b + c) + b
3
(c + a) + c
3
(a + b) và t =
r
b
2
+ c
2
2
a.
Xét
P = f (a, b, c) f (a,t,t) = a
3
(b + c 2t) + a(b
3
+ c
3
2t
3
) + t
2
(2bc 2t
2
)
Ta
b
2
+ c
2
(b + c)
2
2
t
1
2
(b + c) b + c 2t 0,
b
3
+ c
3
2t
3
= (b + c)(b
2
+ c
2
bc) 2t
3
2t(b
2
+ c
2
bc) 2t
3
= t(b c)
2
2bc 2t
2
= (b c)
2
.
Suy ra
P 0 + ta(b c)
2
t
2
(b c)
2
= t(b c)
2
(a t) 0 f(a, b, c) f(a,t,t).
Mặt khác, ta a
2
+ 2t
2
= 3 nên
f(a,t,t) 6 a
3
t + t
3
a 3 t
4
at(a
2
+ t
2
) 3 t
4
a
2
t
2
(3 t
2
)
2
(3 t
4
)
2
(t
2
1)
2
(t
4
3t
2
+ 3) 0 (bđt y luôn đúng).
49
6. PHƯƠNG PHÁP DỒN BIẾN
dụ 6.4. Cho các số thực không âm a, b, c. Chứng minh rằng
(a
2
+ b
2
+ c
2
)
2
4(a + b + c)(a b)(b c)(c a).
hiệu (1) bất đẳng thức cần chứng minh Không mất tính tổng quát, ta giả sử a =
min {a, b, c}.
+) b > c thì (1) luôn đúng.
+) Xét c > b. Khi đó
V T (1) >
b
2
+ c
2
2
và c a 6 c
Ta chứng minh :
(a + b + c) (b a) 6 b (b + c) (2)
Thật vy
(2) ab a
2
+ b
2
ab + bc ca 6 b
2
+ bc a (c + a) > 0 (luôn đúng)
Từ đó, suy ra
V P (1) 6 bc (b + c) (c b) = bc
c
2
b
2
.
Để chứng minh (1), ta chứng minh:
b
2
+ c
2
2
> 4bc
b
2
c
2
(3).
Nếu b = 0 (3) đúng. Xét b 6= 0 ta đặt t =
c
b
> 1. Khi đó (3) trở thành
t
2
+ 1
2
> 4t
t
2
1
t
4
4t
3
+ 2t
2
+ 4t + 1 = 0
t
2
2t 1
2
> 0 (luôn đúng).
Suy ra (1) được chứng minh. Đẳng thức xảy khi
a = 0
c =
1 +
2
b
và các hoán vị.
dụ 6.5 (Hojoo Lee). Cho các số thực không âm a, b, c thỏa mãn ab + bc + ca = 1.
Chứng minh rằng
1
a + b
+
1
b + c
+
1
c + a
5
2
.
Giả sử c = max{a, b, c}. Đặt f(a, b, c) =
1
a + b
+
1
b + c
+
1
c + a
.
Nếu c = 0, ta ab = 1 nên a + b 2 và
f(a, b, c) =
1
a + b
+
1
a
+
1
b
=
1
a + b
+ a + b
=
1
a + b
+
a + b
4
+
3(a + b)
4
1 +
3
2
=
5
2
.
50
6. PHƯƠNG PHÁP DỒN BIẾN
Xét
P = f (a, b, c) f
a + b,
1
a + b
,0
=
1
a + b
+
1
b +
1 ab
a + b
+
1
c +
1 ab
a + b
1
a + b +
1
a + b
+ a + b +
1
a + b
= (a + b)
1
1 + a
2
+
1
1 + b
2
1
1
1 + (a + b)
2
= (a + b)
a (2 2ab ab(a + b)
2
)
(1 + a
2
)(1 + b
2
)(1 + (a + b)
2
)
.
Ta 2 2ab = 2(1 ab) = 2(bc + ac) = 2c(a + b) ab(a + b)
2
. Suy ra P 0, từ đó ta
f(a, b, c) f(t,
1
t
,0)
5
2
.
III. Bài tập
Bài 6.1. Xét các số thực dương a, b, c thỏa mãn a + b + c = 3.
a) Tìm giá trị nhỏ nhất của biểu thức P = abc +
12
ab + bc + ca
.
b) Chứng minh số nguyên k nhỏ nhất sao cho
abc +
k
ab + bc + ca
1 +
k
3
với mọi a, b, c thỏa mãn điều kiện trên k = 10.
Bài 6.2 (Nguyễn Văn Quý). Cho các số thực dương a,b,c thỏa mãn điều kiện a + b + c = 3.
Chứng minh rằng
3a
2
+ 4bc + 9 +
3b
2
+ 4ca + 9 +
3c
2
+ 4ab + 9 12.
Bài 6.3 (Liu Quan Bao). Cho a, b, c, d các số thực dương thỏa mãn điều kiện a + b + c = 3.
Chứng minh rằng
9 6ab + a
2
+ b
2
+
9 6bc + b
2
+ c
2
+
9 6ca + c
2
+ a
2
3
5.
Bài 6.4. (Yi Chang) Cho các số thực dương a,b,c thỏa mãn điều kiện a + b +c = 3. Chứng minh
rằng
r
1 + 3bc +
7
12
(b c)
2
+
r
1 + 3ca +
7
12
(c a)
2
+
r
1 + 3ab +
7
12
(a b)
2
6.
Bài 6.5. (Võ Quốc Cẩn) Cho các số thực dương a,b,c thỏa mãn điều kiện a + b + c = 3.
Chứng minh rằng
p
2(a
2
+ b
2
) + 21c +
p
2(b
2
+ c
2
) + 21a +
p
2(c
2
+ a
2
) + 21b 15.
Bài 6.6. (Phạm Thanh Tùng) Cho các số thực dương a,b,c có tổng bằng 3. Chứng minh rằng
3a
2
a + 1 +
3b
2
b + 1 +
3c
2
c + 1
p
6(a
2
+ b
2
+ c
2
) + 9
51
6. PHƯƠNG PHÁP DỒN BIẾN
Bài 6.7. (Phạm Kim Hùng) Cho các số thực a,b,c thỏa mãn điều kiện a + b + c = 2. Chứng minh
rằng
a + b 2ab +
b + c 2bc +
c + a 2ca 2.
Để kết thúc bài viết, chúng tôi xin giới thiệu một bài toán rất chặt và khó của Liu Quan Ban.
Ngoài lời giải bằng dồn biến chúng tôi biết và giới thiệu đây chúng tôi chưa thấy lời giải
nào khác.
Bài 6.8. (Liu Quan Bao) Cho các số thực a, b,c thỏa mãn điều kiện a + b + c = 3. Chứng minh
rằng
2a
2
a + 1 +
2b
2
b + 1 +
2c
2
c + 1
1
3
p
21(a
2
+ b
2
+ c
2
) + 99.
Bài 6.9. Cho các số thực dương a, b, c tích bằng 1. Chứng minh rằng
1
a
+
1
b
+
1
c
+
13
a + b + c
25
4
.
Bài 6.10. Cho các số thực dương a, b, c thỏa mãn a
2
+ b
2
+ c
2
= 9. Chứng minh rằng
2(a + b c) + abc 10.
Bài 6.11. Cho các số thực dương a, b, c thỏa a + b + c = 3. Chứng minh rằng
2(a
4
+ b
4
+ c
4
) + 36 7
a
3
+ b
3
+ c
3
+ 3abc
.
Bài 6.12. Cho các số thực a, b, c > 0. Chứng minh rằng
2(a
2
+ b
2
+ c
2
) + 3
3
a
2
b
2
c
2
(a + b + c)
2
.
Bài 6.13. Cho các số thực a, b, c 3 thỏa mãn a + b + c = 3. Chứng minh rằng
1
a
2
+
1
b
2
+
1
c
2
1
a
+
1
b
+
1
c
.
Bài 6.14. Cho các số thực a, b, c thỏa mãn a
2
+ b
2
+ c
2
= 9. Chứng minh rằng
2(a + b + c) abc 10.
Bài 6.15 (Iran 1996). Cho a, b, c số dương. Chứng minh rằng :
(ab + bc + ca)
1
(a + b)
2
+
1
(b + c)
2
+
1
(c + a)
2
9
4
.
52
Chương 2
Các phương pháp chứng minh bất đẳng
thức hiện đại
53
1. PHƯƠNG PHÁP P, Q, R
§1. Phương pháp p, q, r
I. thuyết
1. Bất đẳng thức Schur
Cho các số thực không âm x,y,z và số thực dương r. Khi đó, ta bất đẳng thức sau
x
r
(x y)(x z) + y
r
(y x)(y z) + z
r
(z x)(z y) 0.
Đẳng thức xảy ra khi x = y = z hoặc z = 0,x = y và các hoán vị.
Hệ quả 1.
a) Xét r = 1 ta các dạng sau
x
3
+ y
3
+ z
3
+ 3xyz xy(x + y) + yz(y + z) + zx(z + x)
4(x
3
+ y
3
+ z
3
) + 15xyz (x + y + z)
3
xyz (x + y z)(y + z x)(z + x y)
x
2
+ y
2
+ z
2
+
9xyz
x + y + z
2(xy + yz + zx)
(x + y + z)
3
+ 9xyz 4(x + y + z)(xy + yz + zx)
b) r = 2 ta các dạng sau
x
4
+ y
4
+ z
4
+ xyz(x + y + z) xy(x
2
+ y
2
) + yz(y
2
+ z
2
) + zx(z
2
+ x
2
)
6xyz(x + y + z) [2(xy + yz + zx) (x
2
+ y
2
+ z
2
)] (x
2
+ y
2
+ z
2
+ xy + yz + zx).
2. Một số biểu diễn đa thức đối xứng ba biến qua p, q, r
Cho các số thực a, b, c. Đặt p = a + b + c, p = ab + bc + ca và r = abc. Khi đó ta các biểu
diễn sau
ab(a + b) + bc(b + c) + ca(c + a) = pq 3r.
(a + b)(b + c)(c + a) = pq r.
ab(a
2
+ b
2
) + bc
(
b
2
+ c
2
) + ca(c
2
+ a
2
) = p
2
q 2q
2
pr.
(a + b)(a + c) + (b + c)(b + a) + (c + a)(c + b) = p
2
+ q.
a
2
+ b
2
+ c
2
= p
2
2q.
a
3
+ b
3
+ c
3
= p
3
3pq + 3r.
a
4
+ b
4
+ c
4
= p
4
4p
2
q + 2q
2
+ 4pr.
a
2
b
2
+ b
2
c
2
+ c
2
a
2
= q
2
2pr.
a
3
b
3
+ b
3
c
3
+ c
3
a
3
= q
3
3pqr + 3r
2
.
a
4
b
4
+ b
4
c
4
+ c
4
a
4
= q
4
4pq
2
r + 2p
2
r
2
+ 4qr
2
.
54
1. PHƯƠNG PHÁP P, Q, R
3. Một số đánh giá giữa p, q, r
Dựa vào các bất đẳng thức bản ba biến và bất đẳng thức Schur ta các đánh giá sau
(a + b + c)
2
3(ab + bc + ca) p
2
3q.
(a + b + c)
3
27abc p
3
27r.
(ab + bc + ca)
2
3abc(a + b + c) q
2
3pr.
(a + b + c)(ab + bc + ca) 9abc pq 9r.
p
3
+ 9r 4pq (BĐT Schur với r = 1).
p
4
+ 4q
2
+ 6pr 5p
2
q (BĐT Schur với r = 2).
r max
0,
p(4q p
2
)
9
(BĐT Schur với r = 1).
r max
0,
(4q p
2
)(p
2
q)
6p
(BĐT Schur với r = 2).
II. Một số dụ
dụ 1.1. Cho các số thực dương a, b, c thỏa mãn a
2
+ b
2
+ c
2
= 3. Chứng minh rằng
5(a + b + c) +
3
abc
18.
Ta p
2
2q = 3 và bất đẳng thức cần chứng minh trở thành
5p +
3
r
18. (1)
Ta
q
2
3rq
3
r
9p
q
2
=
36p
(p
2
3)
2
.
Do đó
V T (1) 5p +
36p
(p
2
3)
2
.
Nên ta đi chứng minh
5p +
36p
(p
2
3)
2
5p
5
18p
4
30p
3
+ 108p
2
+ 81p 162 0
(p 3)
2
(5p
3
+ 12p
2
3p 18) 0 (2)
Ta
3 < p
2
3(a
2
+ b
2
+ c
2
) = 9
3 < p 3 5p
3
+ 12p
2
3p 18 > 0,
nên (2) luôn đúng.
55
1. PHƯƠNG PHÁP P, Q, R
dụ 1.2. Cho các số thực dương a, b, c thỏa mãn ab + bc + ca = 1. Chứng minh rằng
1
a + b
+
1
b + c
+
1
c + a
5
2
.
Bất đẳng thức cần chứng minh tương đương với
(a + b)(b + c) + (b + c)(c + a) + (c + a)(a + b)
(a + b)(b + c)(c + a)
5
2
p
2
+ q
pq r
5
2
p
2
+ 1
p r
5
2
2p
2
5p + 5r + 2 0. (1)
Nếu p 2 thì
2p
2
5p + 2 + 5r = (p 2)(2p 1) + 5r 0,
suy ra (1) đúng.
Xét
3 p < 2 ta r
4pq p
3
9
=
4p p
3
9
. Nên để chứng minh (1) ta chứng minh
2p
2
5p + 2 + 5
4p p
3
9
0 (p 2)(5p
2
8p + 9) (luôn đúng).
dụ 1.3. Chứng minh rằng nếu x, y, z > 0 thì
(xy + yz + zx)(
1
(x + y)
2
+
1
(y + z)
2
+
1
(z + x)
2
)
9
4
.
Ta
(x + y)
2
(y + z)
2
+ (y + z)
2
(z + x)
2
+ (z + x)
2
(x + y)
2
= ((x + y)(y + z) + (y + z)(z + x) + (z + x)(x + y))
2
4(x + y)(y + z)(z + x)(x + y + z)
= (p
2
+ q)
2
4p(pq r).
Do đó bất đẳng thức cần chứng minh tương đương với
q(
(p
2
+ q)
2
4p(pq r)
(pq r)
2
)
9
4
4p
4
q 17p
2
q
2
+ 4q
3
+ 34pqr 9r
2
0
3pq(p
3
4pq + 9r) + q(p
4
5p
2
q + 4q
2
+ 6pr) + r(pq 9r) 0.
Bất đẳng thức cuối đúng nên ta đpcm.
III. Bài tập
Bài 1.1. Cho các số dương a, b, c thỏa abc = 1. Chứng minh rằng :
1 +
3
a + b + c
6
ab + bc + ca
.
56
1. PHƯƠNG PHÁP P, Q, R
Bài 1.2. Cho các số thực dương a, b, c thỏa mãn ab + bc + ca + 6abc = 9. Chứng minh rằng
a + b + c + 3abc 6.
Bài 1.3. Cho a, b, c các số thực không âm thỏa mãn ab + bc + ca = 3.Chứng minh rằng:
a
3
+ b
3
+ c
3
+ 7abc 10.
Bài 1.4. Cho a, b, c > 0 thỏa a + b + c = 3.Chứng minh rằng:
3 +
12
abc
5
1
a
+
1
b
+
1
c
.
Bài 1.5. Cho a, b, c các số thực dương thỏa mãn a
2
+ b
2
+ c
2
= 3.Chứng minh rằng:
1
2 a
+
1
2 b
+
1
2 c
3.
Bài 1.6. Cho a, b, c các số thực không âm thỏa mãn a + b + c = 3.Chứng minh rằng:
1
9 ab
+
1
9 bc
+
1
9 ca
3
8
.
Bài 1.7. Cho các số thực không âm x, y, z thỏa mãn xy + yz + zx + xyz = 4. Chứng minh rằng
x
2
+ y
2
+ z
2
+ 5xyz 8.
57
2. PHƯƠNG PHÁP SỬ DỤNG TIẾP TUYẾN VÀ T TUYẾN
§2. Phương pháp sử dụng tiếp tuyến và cát tuyến
I. thuyết
1. Hàm lồi - Dấu hiệu hàm lồi
Định nghĩa 1. Cho hàm số y = f (x) liên tục [a; b] và đồ thị (C). Khi đó ta hai điểm
A(a; f(a)), B(b; f(b)) nằm trên đồ thị (C).
i) Đồ thị (C) gọi lồi trên (a; b) nếu tiếp tuyến tại mọi điểm nằm trên cung AB luôn nằm
phía trên đồ thị (C).
ii) Đồ thị (C) gọi lõm trên (a; b) nếu tiếp tuyến tại mọi điểm nằm trên cung AB luôn nằm
phía dưới đồ thị (C).
x
y
O
M
A
B
Đồ thị hàm số lõm
x
y
O
M
A
B
Đồ thị hàm số lồi
Định 1 (Dấu hiệu hàm lồi, lõm). Cho hàm số y = f (x) có đạo hàm cấp hai liên tục trên
(a; b). Khi đó
Nếu f
00
(x) > 0 x (a; b) thì đồ thị hàm số lõm trên (a; b).
Nếu f
00
(x) < 0 x (a; b) thì đồ thị hàm số lồi trên (a; b).
2. Bất đẳng thức tiếp tuyến - Bất đẳng thức cát tuyến
Định 2 (Bất đẳng thức tiếp tuyến). Cho hàm số y = f(x) liên tục có đạo hàm đến
cấp hai trên [a;b]. Khi đó ta có
Nếu f
00
(x) 0 x [a; b] thì f(x) f
0
(x
0
)(x x
0
) + f(x
0
) x
0
[a; b]
Nếu f
00
(x) 0 x [a; b] thì f(x) f
0
(x
0
)(x x
0
) + f(x
0
) x
0
[a; b]
Đẳng thức trong hai bất đẳng thức trên xảy ra khi x = x
0
.
Chứng minh: Ta chứng minh trường hợp thứ nhất, trường hợp thứ hai chứng minh tương tự.
Xét hàm số g(x) = f (x) f
0
(x
0
)(x x
0
) f(x
0
), x [a; b].
Ta : g
0
(x) = f
0
(x) f
0
(x
0
) g
00
(x) = f
00
(x) 0 x [a; b].
Suy ra g
0
(x) = 0 x = x
0
và g
0
(x) đổi dấu từ sang + khi x qua x
0
nên ta :
g(x) g(x
0
) = 0 x [a; b].
Định 3 (Bất đẳng thức cát tuyến). Cho hàm số y = f(x) liên tục có đạo hàm đến cấp
hai trên [a; b]. Khi đó
58
2. PHƯƠNG PHÁP SỬ DỤNG TIẾP TUYẾN VÀ T TUYẾN
Nếu f
00
(x) 0 x [a; b] thì f(x)
f(a) f(b)
a b
(x a) + f(a) x
0
[a; b]
Nếu f
00
(x) 0 x [a; b] thì f(x)
f(a) f(b)
a b
(x a) + f(a) x
0
[a; b].
Đẳng thức trong các bất đẳng thức trên có khi chỉ khi x = a hoặc x = b.
II. Các dụ minh họa
dụ 2.1. Cho các số thực dương a,b,c thỏa a + b + c = 1. Chứng minh rằng
a
a
2
+ 1
+
b
b
2
+ 1
+
c
c
2
+ 1
3
10
.
Xét hàm số f(x) =
x
x
2
+ 1
với x (0; 1). Ta có:
f
0
(x) =
1
q
(x
2
+ 1)
3
f
00
(x) =
3x
q
(x
2
+ 1)
5
< 0 x (0; 1).
Nên ta có:
f(a) f
0
(
1
3
)(a
1
3
) + f(
1
3
)
f(b) f
0
(
1
3
)(b
1
3
) + f(
1
3
)
f(c) f
0
(
1
3
)(c
1
3
) + f(
1
3
).
Suy ra :
f(a) + f(b) + f(c) f
0
1
3
(a + b + c 1) + 3f(
1
3
) =
3
10
.
Đẳng thức xảy ra khi a = b = c =
1
3
.
dụ 2.2. Cho các số thực dương a, b, c thỏa : a
2
+ b
2
+ c
2
= 3. Chứng minh rằng
1
1 + 8a
+
1
1 + 8b
+
1
1 + 8b
1.
Xét hàm số : f(x) =
1
1 + 8a
, 0 < a
3. Ta :
f
0
(x) =
4
q
(1 + 8x)
3
f
00
(x) =
48
q
(1 + 8x)
5
> 0 x (
1
8
;
3].
Nên ta :
f(a) f
0
(1)(a 1) + f(1)
f(b) f
0
(1)(b 1) + f(1)
f(c) f
0
(1)(c 1) + f(1)
59
2. PHƯƠNG PHÁP SỬ DỤNG TIẾP TUYẾN VÀ T TUYẾN
Suy ra
f(a) + f(b) + f(c) f
0
(1)(a + b + c 3) + 3f(1). ()
Mặt khác
(a + b + c)
2
3(a
2
+ b
2
+ c
2
) = 9 3 a + b + c 3 a + b + c 3 0
và f
0
(1) =
4
27
< 0 nên từ (*) ta suy ra : f (a) + f (b) + f (c) 3f (1) = 1.
Nhận xét 1. Dấu hiệu giúp chúng ta nhận ra phương pháp trên bất đẳng thức cần chứng
minh dạng
f(a
1
) + f(a
2
) + ··· + f(a
n
) k
hoặc
f(a
1
) + f(a
2
) + ··· + f(a
n
) k,
trong đó a
i
(i = 1,..,n) các số thực cho trước. Trong một số trường hợp BĐT chưa dạng
trên, ta phải thực hiện một số phép biến đổi mới đưa về dạng trên.Chúng ta cần chú ý một số
dấu hiệu sau.
Nếu bất đẳng thức dạng f(a
1
) · f(a
2
) ···f(a
n
) k thì ta lấy ln hai vế
Nếu bất đẳng thức cần chứng minh đồng bậc thì ta thể chuẩn hóa. Tùy thuộc vào từng
bài toán ta lựa chọn cách chuẩn hóa phù hợp.
dụ 2.3. Cho các số thực dương a,b,c thỏa : a + b + c = 3. Tìm GTLN của biểu thức :
P =
a +
1 + a
2
b
b +
1 + b
2
c
c +
1 + c
2
a
.
Ta :
ln P = b ln(a +
1 + a
2
) + c ln
b +
1 + b
2
+ a ln
c +
1 + c
2
.
Xét hàm số : f(x) = ln
x +
1 + x
2
, 0 < x < 1. Ta :
f
0
(x) =
1
x
2
+ 1
f
00
(x) =
x
q
(1 + x
2
)
3
< 0, x (0; 1).
Suy ra :
f(a) f
0
(1) (a 1) + f(1) = f
0
(1)a + f(1) f
0
(1).
Do đó
bf(a) f
0
(1)ab + [f(1) f
0
(1)] b.
Tương tự
cf(b) f
0
(1)cb + [f(1) f
0
(1)] c và af (c) f
0
(1)ac + [f(1) f
0
(1)] a.
Công các bất đẳng thức theo vế ta được
ln P f
0
(1) (ab + bc + ca (a + b + c)) + f(1)(a + b + c) 3 ln(1 +
2).
(Do ab + bc + ca 3 = a + b + c)
Suy ra ln P 3 ln(1 +
2) P (1 +
2)
3
. Đẳng thức xảy ra a = b = c = 1.
Vy GTLN của P = (1 +
2)
3
.
60
2. PHƯƠNG PHÁP SỬ DỤNG TIẾP TUYẾN VÀ T TUYẾN
dụ 2.4. Cho x,y > 0 thỏa x+y+z = 1. Tìm GTNN của biểu thức P = x
y
+y
z
+z
x
.
Áp dụng bất đẳng thức AM-GM, ta :
P
3
3
x
y
.y
z
.z
x
.
Đặt A = x
y
.y
z
.z
x
ln A = y ln x + z ln y + x ln z.
hàm số f(t) = ln t f
00
(t) =
1
t
2
< 0. Suy ra
ln x f
0
1
3
x
1
3
+ f(
1
3
) = 3x 1 ln 3.
Do đó
ln A y(3x 1 ln 3) + z(3y 1 ln 3) + x(3z 1 ln 3)
= 3(xy + yz + zx) 1 3 ln 3
(x + y + z)
2
1 3 ln 3 = 3 ln 3.
Suy ra A
1
3
P 3
3
3. Đẳng thức xảy ra x = y = z =
1
3
.
Vy GTNN của P = 3
3
3.
dụ 2.5. Cho a,b,c
1
2
thỏa a + b + c = 2. Tìm GTNN của biểu thức
P = a
a
+ b
b
+ c
c
.
Xét hàm số f(t) = t
t
,
1
2
t 1. Ta : ln f (t) = t ln t lấy đạo hàm hai vế ta được
f
0
(t) = (1 + ln t)f (t) ln f
0
(t) = ln f (t) + ln (ln t + 1)
f
00
(t)
f
0
(t)
=
f
0
(t)
f(t)
+
1
t(ln t + 1)
= 1 + ln t +
1
t(ln t + 1)
f
00
(t) = (1 + ln t)f (t)
1 + ln t +
1
t(1 + ln t)
> 0 t [
1
2
; 1].
a,b,c
1
2
; 1
nên áp dụng bất đẳng thức tiếp tuyến, ta :
f(a) f
0
(
2
3
)(a
2
3
) + f(
2
3
)
f(b) f
0
(
2
3
)(b
2
3
) + f(
2
3
)
f(c) f
0
(
2
3
)(c
2
3
) + f(
2
3
).
Cộng ba bất đẳng thức trên ta :
f(a) + f(b) + f(c) f
0
(
2
3
) (a + b + c 2) + 3f(
2
3
) = 3
3
r
4
9
.
Vy GTNN của P = 3
3
r
4
9
đạt được a = b = c =
2
3
.
61
2. PHƯƠNG PHÁP SỬ DỤNG TIẾP TUYẾN VÀ T TUYẾN
dụ 2.6. Cho tam giác ABC một c không nhỏ hơn
2π
3
. Chứng minh rằng :
tan
A
2
+ tan
B
2
+ tan
C
2
4
3.
Không mất tính tổng quát, ta giả sử A
2π
3
> B C C
π
6
.
Hàm số f(x) = tan x, x
0;
π
3
f
00
(x) > 0 x
0;
π
3
.
Áp dụng BĐT tiếp tuyến, ta
f(
A
2
) f
0
(
π
3
)(
A
2
π
3
) + f(
π
3
)
f(
B
2
) f
0
(
π
12
)(
B
2
π
12
) + f(
π
12
)
f(
C
2
) f
0
(
π
12
)(
C
2
π
12
) + f(
π
12
).
Suy ra
f
A
2
+ f
B
2
+ f
C
2
h
f
0
(
π
3
) f
0
(
π
12
)
i
A
2
2π
3
+ f
0
(
π
12
)
A + B + C
2
π
2
+ f
π
3
+ 2f
π
12
.
Do f
0
π
3
f
0
π
12
> 0;
A
2
π
3
0 và
A + B + C
2
=
π
2
nên ta :
f
A
2
+ f
B
2
+ f
C
2
f
π
3
+ 2f
π
12
= 4
3.
Đẳng thức xảy ra A =
2π
3
; B = C =
π
6
và các hoán vị.
dụ 2.7. Cho các số thực không âm a,b,c thỏa max {a,b,c}
3
4
và a + b + c = 1. Tìm
GTNN của biểu thức :
P =
3
1 + 3a
2
+
3
1 + 3b
2
+
3
1 + 3c
2
.
Không mất tính tổng quát, ta giả sử a = max {a,b,c} a
3
4
,c
1
8
.
Xét hàm số f(x) =
3
1 + 3x
2
, x (0; 1)
f
0
(x) =
2x
3
q
(1 + 3x
2
)
2
f
00
(x) =
2 2x
2
3
q
(1 + 3x
2
)
5
> 0 x (0; 1).
62
2. PHƯƠNG PHÁP SỬ DỤNG TIẾP TUYẾN VÀ T TUYẾN
Áp dụng bất đẳng thức tiếp tuyến, ta :
f (a) f
0
3
4
a
3
4
+ f
3
4
f (b) f
0
1
8
b
1
8
+ f
1
8
f (c) f
0
1
8
c
1
8
+ f
1
8
Cộng các bất đẳng thức trên ta
f (a) + f (b) + f (c)
f
0
3
4
f
0
1
8

a
3
4
+ f
3
4
+ 2f
1
8
f
3
4
+ 2f
1
8
=
3
172 + 2
3
67
4
.
Đẳng thức xảy ra a =
3
4
; b = c =
1
8
và các hoán vị.
Vy min P =
3
172 + 2
3
67
4
.
Nhận xét 2. Trong một số trường hợp đồ thị hàm số y = f (x) khoảng lồi, lõm trên [a; b]
nhưng ta vẫn được đánh giá :
f(x) f
0
(x
0
)(x x
0
) + f(x
0
) ,x
0
(a; b).
dụ 2.8. Cho a,b,c R và a + b + c = 6. Chứng minh rằng :
a
4
+ b
4
+ c
4
2(a
3
+ b
3
+ c
3
).
Bất đẳng thức đã cho tương đương với
a
4
2a
3
+
b
4
2b
3
+
c
4
2c
3
0 f (a) + f (b) + f (c) 0.
Trong đó f(x) = x
4
2x
3
. Ta thấy f
00
(x) = 12x
2
12x nên đồ thị hàm số f khoảng lồi
và khoảng lõm do đó ta không thể áp dụng BĐT tiếp tuyến được. Tuy nhiên ta vẫn thể
đánh giá được f (x) qua tiếp tuyến của tại điểm hoành độ x = 2 (vì đẳng thức xảy ra khi
a = b = c = 2).
Ta tiếp tuyến của đồ thị hàm số tại y = f (x) điểm hoành độ x = 2 là: y = 8x 16.
f (x) (8x 16) = x
4
2x
3
8x + 16 = (x 2)
2
x
2
2x + 4
0 x R.
Suy ra
f (a) + f (b) + f (c) 8 (a + b + c) 48 = 0.
Vy bài toán được chứng minh.
dụ 2.9 (Ba Lan 1996). Cho a,b,c
3
4
và a + b + c = 1. Chứng minh rằng:
a
a
2
+ 1
+
b
b
2
+ 1
+
c
c
2
+ 1
9
10
.
63
2. PHƯƠNG PHÁP SỬ DỤNG TIẾP TUYẾN VÀ T TUYẾN
Ta thấy đẳng thức xảy ra khi a = b = c =
1
3
và bất đẳng thức đã cho dạng: f(a)+f(b)+f (c)
9
10
trong đó f(x) =
x
x
2
+ 1
với x [
3
4
;
5
2
] .
Tiếp tuyến của đồ thị hàm số y = f (x) tại điểm hoành độ x =
1
3
: y =
36x + 3
50
.
Ta có:
36x + 3
50
f(x) =
36x + 3
50
x
x
2
+ 1
=
(3x 1)
2
(4x + 3)
50(x
2
+ 1)
0 x [
3
4
;
5
2
].
Vy
a
a
2
+ 1
+
b
b
2
+ 1
+
c
c
2
+ 1
36(a + b + c) + 9
50
=
9
10
.
Bài toán được chứng minh.
dụ 2.10. Cho a,b,c độ dài ba cạnh tam giác. Chứng minh rằng :
1
a
+
1
b
+
1
c
+
9
a + b + c
4
1
a + b
+
1
b + c
+
1
c + a
.
Không làm mất tính tổng quát ta giả sử , khi đó Bđt đã cho trở thành
5a 1
a a
2
+
5a 1
b b
2
+
5c 1
c c
2
9.
a,b,c độ dài ba cạnh tam giác và a + b + c = 1 suy ra a,b,c (0;
1
2
). Ta
5a 1
a a
2
(18a 3) =
(3a 1)
2
(2a 1)
a a
2
0 a (0;
1
2
).
Suy ra
5a 1
a a
2
18a 3 a (0;
1
2
).
Ta cũng hai bất đẳng thức tương tự. Cộng các bất đẳng thức y lại với nhau ta
5a 1
a a
2
+
5a 1
b b
2
+
5c 1
c c
2
18(a + b + c) 9 = 9.
Bài toán được chứng minh. Đẳng thức xảy ra khia = b = c =
1
3
.
dụ 2.11 (Trung Quốc 2005). Cho a,b,c > 0 và a + b + c = 1. Chứng minh rằng:
10
a
3
+ b
3
+ c
3
) 9(a
5
+ b
5
+ c
5
1.
Giả sử a b c.
Xét hàm số f(x) = 10x
3
9x
4
, x (0; 1)
f
0
(x) = 30x
2
45x
4
f
00
(x) = 60x 180x
3
.
Suy ra f
00
(x) = 0 x = x
0
=
r
1
3
đồng thời
f
00
(x) > 0 x (0; x
0
) và f
00
(x) < 0 x (x
0
; 1).
64
2. PHƯƠNG PHÁP SỬ DỤNG TIẾP TUYẾN VÀ T TUYẾN
Nếu a < x
0
. Áp dụng bất đẳng thức tiếp tuyến ,ta có:
f(a) f
0
1
3
a
1
3
+ f
1
3
f(b) f
0
1
3
b
1
3
+ f
1
3
f(c) f
0
1
3
c
1
3
+ f
1
3
.
Suy ra
f(a) + f(b) + f(c) f
0
1
3
(a + b + c 1) + 3f
1
3
= 1.
Nếu a > x
0
. Áp dụng bất đẳng thức tiếp tuyến và cát tuyến ta có:
f(a)
f(1) f(x
0
)
1 x
0
(a 1) + f (1) > f(1) = 1
f(b) f
0
(0) (b 0) + f (0) = 0
f(c) f
0
(0) (c 0) + f (0) = 0.
Do đó f(a) + f(b) + f(c) > 1.
Vy bài toán được chứng minh.
dụ 2.12. Cho ABC nhọn. Tìm GTLN của biểu thức
F = sin A · sin
2
B ·sin
3
C.
Ta
ln F = ln sin A + 2 ln sin B + 3 ln sin C.
Xét hàm số f(x) = ln sin x, x (0;
π
2
), ta
f
0
(x) = cot x f
00
(x) =
1
sin
2
x
x
0;
π
2
.
Áp dụng bất đẳng thức tiếp tuyến với MNP nhọn, ta :
f(A) f
0
(M) (A M) + f (M ) = (A M ) cot M + ln sin M
f(B) f
0
(N) (B N ) + f (N ) = (B N) cot N + ln sin N
f(C) f
0
(P ) (C P ) + f (P ) = (C P ) cot P + ln sin P.
Suy ra
tan M ·f(A) + tan N ·f(B) + tan P ·f(C) tan M ·ln sin M + tan N ·ln sin N + tan P ·ln sin P.
Chọn ba c M, N, P sao cho :
tan M
1
=
tan N
2
=
tan P
3
= k tan M = k; tan N = 2k; tan P = 3k.
Mặt khác : tan M + tan N + tan P = tan M. tan N. tan P , suy ra
6k = 6k
3
k = 1 sin M =
tan M
1 + tan
2
M
=
1
2
; sin N =
2
5
; sin P =
3
10
.
65
2. PHƯƠNG PHÁP SỬ DỤNG TIẾP TUYẾN VÀ T TUYẾN
Do đó, ta
f(A) + f(B) + f(C) ln
1
2
+ 2 ln
2
5
+ 3 ln
3
10
= ln
27
25
5
,
hay F
27
25
5
. Đẳng thức xảy ra A = M; B = N; C = P .
Vy GTLN của F =
27
25
5
.
III. Bài tập
Bài 2.1 (Albania 2002). Cho a,b,c > 0. Chứng minh rằng :
1 +
3
3
3
(a
2
+ b
2
+ c
2
)(
1
a
+
1
b
+
1
c
) a + b + c +
a
2
+ b
2
+ c
2
.
Bài 2.2. Cho n số thực x
1
,x
2
, . . . ,x
n
thuộc khoảng (0;
π
2
) thỏa :
tan x
1
+ tan x
2
+ ··· + tan x
n
n.
Chứng minh rằng
sin x
1
· sin x
2
···sin x
n
1
2
n
.
Bài 2.3. Cho các số thực a,b,c > 0 thoả mãna + b + c = 1. Chứng minh :
a
1 + bc
+
b
1 + ac
+
c
1 + ab
9
10
.
Bài 2.4. Cho a,b,c > 0. Chứng minh rằng :
(b + c a)
2
(b + c)
2
+ a
2
+
(c + a b)
2
(c + a)
2
+ b
2
+
(a + b c)
2
(a + b)
2
+ c
2
3
5
.
Bài 2.5. Cho a, b, c > 1 và a + b + c = 1. Tìm giá trị nhỏ nhất của
S = a
3
+ b
3
+ c
3
+ 5(a
2
+ b
2
+ c
2
).
Bài 2.6. Cho a + b + c = 6 với a, b, c < 1. Tìm giá trị lớn nhất của
S =
a
a
2
+ a + 1
+
b
b
2
+ b + 1
+
c
c
2
+ c + 1
.
Bài 2.7. Cho a, b, c >
2
3
và 4(ab + bc + ca) + a + b + c 15. Tính giá trị nhỏ nhất của biểu
thức
S = a
3
+ b
3
+ c
3
+ 2(a
2
b + b
2
c + c
2
a).
66
2. PHƯƠNG PHÁP SỬ DỤNG TIẾP TUYẾN VÀ T TUYẾN
Bài 2.8. Cho a, b, c
0,
4
3
và 2(ab + bc + ca) + 3(a + b + c) = 3. Tìm giá trị nhỏ nhất
S = 2(a
3
+ b
3
+ c
3
) 3(a
2
b + b
2
c + c
2
a).
Bài 2.9. Cho các số dương a, b, c thỏa mãn ab + bc + ca = 3. Tìm giá trị nhỏ nhất của
S = 5(a
3
+ b
3
+ c
3
) + 2(a
2
b + b
2
c + c
2
a).
Bài 2.10. Cho a, b, c
1
2
,
2
và ab + bc + ca + 9 = 4(a + b + c). Tìm giá trị nhỏ nhất của
T =
1
a
2
+
1
b
2
+
1
c
2
b
a
+
a
c
+
c
b
.
Bài 2.11. Cho các số dương a, b, c thỏa mãn 3(ab + bc + ca) (a + b + c) 6. Tính giá trị nhỏ
nhất của
A = a
5
+ b
5
+ c
5
+ 3(a
4
b + b
4
c + c
4
a).
Bài 2.12. Cho a, b, c [0,4] thỏa mãn ab + bc + ca 3. Tìm giá trị nhỏ nhất của
B = (a
2
+ b
2
+ c
2
) + b
3a + 1 + c
3b + 1 + a
3c + 1.
Bài 2.13. Cho tam giác ABC nhọn. Tìm GTNN của biểu thức :
F = tan A + 2 tan B + 3 tan C.
Bài 2.14. Cho x, y, z > 0 thỏa x + y + z = 1. Tìm GTNN của :
P = x
3
+
p
1 + y
2
+
4
1 + z
4
.
67
Chương 3
Một số chuyên đề
§1. Ứng dụng điều kiện nghiệm của phương trình bậc
ba trong chứng minh bất đẳng thức
I. thuyết
1. Mở đầu
Định Vi-ét đảo đối với phương trình bậc hai được phát biểu như sau:
Định 1. Nếu hai số a,b có tổng S tích P thì hai số đó hai nghiệm của phương trình
x
2
Sx + P = 0. (3.1)
Phương trình (3.1) nghiệm khi và chỉ khi = S
2
4P 0 hay S
2
4P .
S = a + b,P = ab nên ta điều kiện để tồn tại hai số a,b (tức phương trình (3.1)
nghiệm) :
(a + b)
2
4ab.
Đây chính bất đẳng thức quen thuộc. Nếu a, b 0 thì ta thu được bất đẳng thức AM-GM.
Tương tự đối với định Vi-ét đảo của phương trình bậc ba như sau:
Đặt m = a + b + c,n = ab + bc + ca,p = abc . Khi đó, a, b, c nghiệm của phương trình
x
3
mx
2
+ nx p = 0. (3.2)
Ta đi tìm điều kiện để phương trình (3.2) ba nghiệm (có thể trùng nhau).
2. Một số kết quả
Đặt: x = y +
m
3
; α =
m
2
3
n; β =
9mn 2m
3
27p
27
. Từ (3.2) ta thu được phương trình
y
3
αy + β = 0. (3.3)
Số nghiệm của (3.3) chính số giao điểm của đồ thị (C) : f(y) = y
3
αy + β với trục hoành.
Ta có: f
0
(y) = 3y
2
α.
Nếu α < 0 thì f
0
(y) > 0, y nên phương trình (3.3) đúng 1 nghiệm.
Nếu α = 0 thì phương trình (3.3) nghiệm bội ba.
68
1. ỨNG DỤNG ĐIỀU KIỆN NGHIỆM CỦA PHƯƠNG TRÌNH BẬC BA TRONG
CHỨNG MINH BẤT ĐẲNG THỨC
Nếu α > 0 thì f
0
(y) = 0 hai nghiệm y
1
=
r
α
3
; y
2
=
r
α
3
, khi đó
f (y
1
) =
2α
3
r
α
3
+ β, f (y
2
) =
2α
3
r
α
3
+ β.
Suy ra
f (y
1
) .f (y
2
) = β
2
4α
3
27
=
27β
2
4α
3
27
.
Do đó, phương trình (3.3) ba nghiệm khi và chỉ khi:
f (y
1
) .f (y
2
) 0 4α
3
27β
2
0.
Hay là:
9mn 27p 2m
3
2
q
(m
2
3n)
3
.
Kết quả 1. Cho các số thực a, b, c. Đặt a + b + c = m, ab + bc + ca = n, abc = p. Khi đó, ta có
đánh giá sau:
9mn 27p 2m
3
2
q
(m
2
3n)
3
. (3.4)
Với a, b, c 0, đặt a + b + c = 3u, ab + bc + ca = 3v
2
và abc = w
3
.
(a + b + c)
2
3 (ab + bc + ca) 9
3
q
(abc)
2
nên ta u v w.
Khi đó (3.4) trở thành
9 · 3u · 3v
2
27w
3
2 · 27u
3
2
q
(9u
2
9v
2
)
3
.
Hay
3uv
2
w
3
2u
3
2
q
(u
2
v
2
)
3
. (3.5)
Chia hai vế của (3.5) cho u
3
ta
3
v
u
2
w
u
3
2
2
s
1
v
u
2
3
.
Hay
2
s
1
v
u
2
3
3
v
u
2
w
u
3
2 2
s
1
v
u
2
3
.
Suy ra
3
v
u
2
2
s
1
v
u
2
3
2
w
u
3
3
v
u
2
+ 2
s
1
v
u
2
3
2.
Kết quả 2: Cho các số thực dương a,b,c. Đặt a + b + c = 3u, ab + bc + ca = 3v
2
abc = w
3
với
u,v,w các số thực dương. Khi đó u v w
3
v
u
2
2
s
1
v
u
2
3
2
w
u
3
3
v
u
2
+ 2
s
1
v
u
2
3
2. (3.6)
69
1. ỨNG DỤNG ĐIỀU KIỆN NGHIỆM CỦA PHƯƠNG TRÌNH BẬC BA TRONG
CHỨNG MINH BẤT ĐẲNG THỨC
II. dụ minh họa
dụ 1.1. Cho các số thực a, b, c thỏa mãn a + b + c = 0. Chứng minh rằng
a
2
+ b
2
+ c
2
3
24
a
3
+ b
3
+ c
3
1
.
Ta m = 0 nên (3.4) trở thành
|27p| 2
q
(3n)
3
n
3
27
4
p
2
= 3
3
2
p 1
2
+ 9p 3 9p 3.
Hay
(ab + bc + ca)
3
9abc 3.
Mặt khác a + b + c = 0 nên ta a
3
+ b
3
+ c
3
= 3abc và
(ab + bc + ca) =
a
2
+ b
2
+ c
2
2
.
Do vy, ta
a
2
+ b
2
+ c
2
3
24
a
3
+ b
3
+ c
3
1
.
Vy bài toán được chứng minh.
dụ 1.2. Cho các số thực a, b, c tổng bằng 1. Tìm giá trị nhỏ nhất của biểu thức
P = 2abc + (ab + bc + ca)
2
.
Ta m = 1 nên áp dụng (3.4) ta
|9n + 27p 2| 2
q
(1 3n)
3
p
1
3
n +
2
27
2
27
q
(1 3n)
3
.
Do đó
27P 18n + 4 4
q
(1 3n)
3
+ 27n
2
= 3 (3n 1)
2
4
q
(1 3n)
3
+ 1.
Đặt t =
1 3n,t 0 ta
27P 3t
4
4t
3
+ 1 = (t 1)
2
3t
2
t + 1
0.
Do đó P 0. Đẳng thức xảy ra khi a = 1, b = c = 0 và các hoán vị.
Vy min P = 0.
dụ 1.3. Cho các số thực a,b,c thỏa mãn a
2
+ b
2
+ c
2
= 3.Chứng minh rằng
3 (abc 2) (a + b + c) (ab + bc + ca) 3 (abc + 2) .
Ta a
2
+ b
2
+ c
2
= 3 nên (a + b + c)
2
= 2 (ab + bc + ca) + 3 hay n =
m
2
3
2
.
Từ (3.4) ta suy ra
2
m
3
s
9 m
2
2
3
9mn 27p 2
m
3
+
s
9 m
2
2
3
.
70
1. ỨNG DỤNG ĐIỀU KIỆN NGHIỆM CỦA PHƯƠNG TRÌNH BẬC BA TRONG
CHỨNG MINH BẤT ĐẲNG THỨC
Ta chứng minh
m
3
+
s
9 m
2
2
3
27
9 m
2
2
3
27 m
3
2
.
Khai triển và biến đổi ta được
(m 3)
2
m
4
+ 6m
3
+ 24m
2
+ 42m + 63
0.
Bất đẳng thức y hiển nhiên đúng do
m
4
+ 6m
3
+ 24m
2
+ 42m + 63 =
m
2
+ 3m + 7
2
+ m
2
+ 14 > 0.
Chứng minh tương tự, ta
m
3
s
9 m
2
2
3
27.
Do vy ta được bất đẳng thức
54 9mn 27p 54 3p 6 mn 3p + 6.
Hay
3 (abc 2) (a + b + c) (ab + bc + ca) 3 (abc + 2) .
Bài toán được chứng minh.
dụ 1.4. Cho các số thực a, b, c thỏa mãn abc = 1. Tìm giá trị lớn nhất của biểu thức
P =
(a + b + c)
3
+ (ab + bc + ca)
3
(a + b + c)
2
(ab + bc + ca)
2
+ 27
.
Ta p = 1 nên từ (3.4) ta được
9mn 27 2m
3
2
q
(m
2
3n)
3
.
Bình phương hai vế và rút gọn ta thu được
(mn)
2
+ 18mn 4
m
3
+ n
3
+ 27.
Mặt khác 18mn (mn)
2
+ 81 nên ta
2 (mn)
2
+ 81 4
m
3
+ n
3
+ 27 m
2
n
2
+ 27 2
m
3
+ n
3
.
Do đó
P =
m
3
+ n
3
m
2
n
2
+ 27
1
2
.
Đẳng thức xảy ra khi
(
abc = 1
(ab + bc + ca) (a + b + c) = 9
, chẳng hạn a = b = c = 1.
Vy max P =
1
2
.
dụ 1.5. Cho các số thực a,b,c thỏa mãn ab + bc + ca = 3. Tìm giá trị lớn nhất của biểu
thức
P =
abc(a + b + c)
3
+ 27
(a + b + c + 3abc)
2
.
71
1. ỨNG DỤNG ĐIỀU KIỆN NGHIỆM CỦA PHƯƠNG TRÌNH BẬC BA TRONG
CHỨNG MINH BẤT ĐẲNG THỨC
Ta n = 3 nên từ (3.4), suy ra
27m 27p 2m
3
2
q
(m
2
9)
3
.
Bình phương hai vế và rút gọn ta thu được
27p
2
+ 4m
3
p + 108 54mp + 9m
2
,
hay
108p
2
+ 4m
3
p + 108 9 (m + 3p)
2
.
Suy ra
4m
3
p + 108 9 (m + 3p)
2
P =
m
3
p + 27
(m + 3p)
9
4
.
Đẳng thức xảy ra khi
abc = 0
a + b + c = ±2
3
ab + bc + ca = 3
, chẳng hạn ta chọn a = 0,b = c =
3.
Vy max P =
9
4
.
dụ 1.6. Cho các số thực a,b,c thoả
2
a
2
+ b
2
+ c
2
= 5 (ab + bc + ca) .
Chứng minh rằng:
(a + b + c)
2
+ 27.
3
abc + 1 0.
Ta
2
m
2
2n
= 5n n =
2
9
m
2
.
Khi đó (3.4) trở thành
|27p| 2
s
m
2
2
3
m
2
3
27
2
p
2
4
27
m
6
m
2
27
3
r
p
2
4
.
Mặt khác
p
2
+ 1
2
0 nên
p
2
4
(p + 1) ,
suy ra
m
2
27
3
p
p + 1
hay
(a + b + c)
2
+ 27
3
abc + 1 0.
Bài toán được chứng minh.
dụ 1.7. Cho các số thực a,b,c thoả a
2
+ b
2
+ c
2
= ab + bc + ca + 1. Chứng minh rằng:
(a + b + c)
2
4 + 3 (ab + bc + ca)
2
+ 18abc.
72
1. ỨNG DỤNG ĐIỀU KIỆN NGHIỆM CỦA PHƯƠNG TRÌNH BẬC BA TRONG
CHỨNG MINH BẤT ĐẲNG THỨC
Ta (a + b + c)
2
= 3 (ab + bc + ca) + 1 nên m
2
= 3n + 1. Khi đó (2) trở thành:
9m
m
2
1
3
27p 2m
3
2 27p m
3
3m 2.
Đặt T = (a + b + c)
2
3 (ab + bc + ca)
2
18abc, ta cần chứng minh T 4.
3T = 3m
2
9n
2
54p
3m
2
m
2
1
2
2
m
3
3m 2
= m
4
2m
3
+ 5m
2
+ 6m + 3
=
m
2
+ m 3
2
+ 12 12.
Suy ra T 4. Bài toán được chứng minh.
dụ 1.8 (Iran MO 2014, vòng 2). Cho các số thực không âm x,y,z thỏa mãn điều
kiện:
x
2
+ y
2
+ z
2
= 2(xy + yz + zx).
Chứng minh rằng:
x + y + z
3
3
2xyz.
Nếu x = y = z = 0 thì bất đẳng thức cần chứng minh hiển nhiên đúng. Ta xét x + y + z > 0.
Bất đẳng thức cần chứng minh tương đương với
u
3
2.w
w
u
3
1
2
.
Áp dụng (3.6) ta chỉ cần chứng minh
w
u
3
3
v
u
2
+ 2
s
1
v
u
2
3
2.
x
2
+ y
2
+ z
2
= 2 (xy + yz + zx) nên
9u
2
= 4.3v
2
v
u
2
=
3
4
.
Do đó
w
u
3
3
v
u
2
+ 2
s
1
v
u
2
3
2 =
1
2
.
Bài toán được chứng minh.
dụ 1.9. Cho các số thực không âm a,b,c. Chứng minh rằng
a
4
+ b
4
+ c
4
ab + bc + ca
+
3abc
a + b + c
2
3
a
2
+ b
2
+ c
2
.
Ta
a
4
+ b
4
+ c
4
= 81u
4
108u
2
v
2
+ 18v
4
+ 12uw
3
,
và
a
2
+ b
2
+ c
2
= 9u
2
6v
2
.
73
1. ỨNG DỤNG ĐIỀU KIỆN NGHIỆM CỦA PHƯƠNG TRÌNH BẬC BA TRONG
CHỨNG MINH BẤT ĐẲNG THỨC
Nên bất đẳng thức cần chứng minh trở thành
81u
4
108u
2
v
2
+ 18v
4
+ 12uw
3
3v
2
+
3w
3
3u
2
3
9u
2
6v
2
27 36x + 6x
2
+ 4y
x
+ y + 4x 6. (3.7)
Trong đó x =
v
u
2
và y =
w
u
3
.
Theo (3.6), ta
y 3x 2
q
(1 x)
3
2 vàx 1
nên
V T (3.7) =
27
x
36 + 10x + y
1 +
4
x
27
x
36 + 10x + 5y
27
x
36 + 10x + 5
3x 2
q
(1 x)
3
2
=
25x
2
52x + 27
x
10
q
(1 x)
2
+ 6
(1 x) (27 25x) 10
q
(1 x)
3
+ 6
(1 x) (1 + 25(1 x)) 10
q
(1 x)
3
+ 6
(1 x) 10
1 x 10
q
(1 x)
3
+ 6 = 6.
Bài toán được chứng minh. Đẳng thức xảy ra khi a = b = c.
III. Bài tập
Bài 1.1. Cho các số thực a,b,c không đồng thời bằng 0 thỏa a + b + c = 0. Tìm giá trị lớn nhất
của biểu thức:
P =
13a
2
b
2
c
2
2abc 2
(a
2
+ b
2
+ c
2
)
3
.
Bài 1.2. Cho các số thực a,b,c tổng bằng 0. Tìm giá trị nhỏ nhất của biểu thức
P =
a
2
+ b
2
+ c
2
5
32 (ab + bc + ca) a
2
b
2
c
2
8 |abc|.
Bài 1.3. Cho các số thực a,b,c thoả a
2
+ b
2
+ c
2
= 2(ab + bc + ca). Tìm giá trị nhỏ nhất của biểu
thức:
P = abc (a + b + c)
3
+
1
(abc)
4
.
Bài 1.4. Cho các số thực a,b,c thoả a
2
+ b
2
+ c
2
= ab + bc + ca + 4. Tìm giá trị nhỏ nhất của
biểu thức:
P = 18 (ab + bc + ca)
2
(ab + bc + ca) (a + b + c 48) + 9abc.
Bài 1.5. Cho các số thực dương a,b,c thoả (a + b + c)
3
= 32abc. Tìm giá trị lớn nhất, giá trị
nhỏ nhất của biểu thức:
P =
a
4
+ b
4
+ c
4
(a + b + c)
4
.
74
2. BÀI TOÁN TÌM HẰNG SỐ TỐT NHẤT TRONG BẤT ĐẲNG THỨC
§2. Bài toán tìm hằng số tốt nhất trong bất đẳng thức
I. thuyết
Trong chuyên đề nyaf ta đi giải quyết bài toán:
Tìm hằng số k lớn nhất (nhỏ nhất) để một BĐT luôn đúng với một giả thiết nào đó của các biến.
Để giải dạng toán y, ta thường giải quyết theo hai hướng sau:
Hướng 1:
Bước 1: Chọn giá trị đặc biệt của các biến hoặc đánh giá trực tiếp các biến để chỉ ra điều
kiện cần của k.
Bước 2: Chứng minh bất đẳng thức đã cho đúng với giá trị của k ( lớn nhất, nhỏ nhất)
vừa tìm được.
Hướng 2: Giả sử ta cần tìm k nhỏ nhất để bất đẳng thức
f(a
1
,a
2
, . . . ,a
n
) k
luôn đúng với mọi a
1
, a
2
, . . . , a
n
D. Ta đi tìm giá trị lớn nhất M của f(a
1
,a
2
, . . . ,a
n
) với
a
1
, a
2
, . . . , a
n
D. Khi đó k
min
= M.
II. dụ minh họa
dụ 2.1. Tìm hằng số k lớn nhất sao cho bất đẳng thức sau luôn đúng
q
a + k|b c|
α
+
q
b + k|c a|
α
+
q
c + k|a b|
α
2,
với mọi α 1 và a,b,c các số thực không âm thỏa mãn a + b + c = 1.
Cho a = b = 0,c = 1 = 1. Ta
1 +
k +
k 1 0 k
1
4
.
Ta chứng minh bất đẳng thức sau đúng
q
4a + |b c|
α
+
q
4b + |c a|
α
+
q
4c + |a b|
α
4. (3.8)
đúng với mọi α 1 và a,b,c 0 thỏa a + b + c = 1.
Không mất tính tổng quát ta giả sử a b c. Ta
0 |a b|, |b c|, |c a| 1, α 1
nên ta
V T (3.8)
p
4a + |b c| +
p
4b + |c a| +
p
4c + |a b|
=
1
2
p
4(4a + b c) +
4b + a c +
4c + a b
1
2
.
4 + 4a + b c
2
+
1 + 4b + a c
2
+
1 + 4c + a b
2
=
8a + 7b + 5c + 8
4
8 (a + b + c) + 8
4
= 4.
Vy k
max
=
1
4
.
75
2. BÀI TOÁN TÌM HẰNG SỐ TỐT NHẤT TRONG BẤT ĐẲNG THỨC
dụ 2.2. Tìm số k nhỏ nhất sao cho bất đẳng thức
a
3
+ b
3
+ c
3
+ kabc
k + 3
6
a
2
(b + c) + b
2
(c + a) + c
2
(a + b)
đúng với mọi a,b,c độ dài ba cạnh của tam giác.
Cho c = 1, a = b =
1
2
+
1
n
ta có: k
3
3 +
2
n
1 +
2
n
9. Ta chứng minh bất đẳng thức
a
3
+ b
3
+ c
3
+ 9abc 2
a
2
(b + c) + b
2
(c + a) + c
2
(a + b)
.
Giả sử a = max {a,b,c}, ta
a
3
+ b
3
+ c
3
+ 9abc 2
a
2
(b + c) + b
2
(c + a) + c
2
(a + b)
= (a b c) (a b) (a c) + (b c)
2
(b + c 3a) 0.
Vy k
max
= 9.
dụ 2.3. Cho a, b, c > 0. Tìm hằng số k lớn nhất sao cho bất đẳng thức sau đúng
a
b
+
b
c
+
c
a
3 k
a
2
+ b
2
+ c
2
ab + bc + ca
1
.
Cho a = 1, b = c
3
6= 1, ta
k
(c
3
+ c
2
+ 1) (c
3
+ 3c
2
+ 2c + 1)
c
2
(c
4
+ 2c
3
+ 2c
2
+ c + 1)
1.
Ta chứng minh bất đẳng thức
a
b
+
b
c
+
c
a
3
a
2
+ b
2
+ c
2
ab + bc + ca
1
,
hay
a
b
+
b
c
+
c
a
a
2
+ b
2
+ c
2
ab + bc + ca
+ 2
(ab + bc + ca)
a
b
+
b
c
+
c
a
(a + b + c)
2
.
Bất đẳng thức cuối dễ dàng chứng minh được bằng cách áp dụng bất đẳng thức Cauchy
Schwarz.
dụ 2.4. Tìm số thực dương k lớn nhất để bất đẳng thức sau đúng với mọi số thực
dương x,y,z thỏa mãn điều kiện xyz = 1 :
x
xy + 1
+
y
yz + 1
+
z
zx + 1
+
k
3
p
xy
2
+ yz
2
+ zx
2
3
2
+
k
3
3
.
76
2. BÀI TOÁN TÌM HẰNG SỐ TỐT NHẤT TRONG BẤT ĐẲNG THỨC
Do bất đẳng thức đã cho đúng với mọi x,y,z > 0 thỏa mãn xyz = 1 nên sẽ đúng khi
x = n; y = 1; z =
1
n
với mọi n > 0. Khi đó ta
2n
n + 1
+
1
2n
+
k
3
r
2n +
1
n
2
3
2
+
k
3
3
; n > 0.
Cho n + ta được
2
3
2
+
k
3
3
k
3
3
2
.
Ta sẽ chứng minh k =
3
3
2
thỏa mãn yêu cầu bài toán, tức
2x
xy + 1
+
2y
yz + 1
+
2z
zx + 1
+
3
r
3
xy
2
+ yz
2
+ zx
2
4.
Do x, y, z > 0 thỏa mãn xyz = 1 nên tồn tại a,b,c > 0 thỏa mãn x =
b
a
; y =
c
b
; z =
a
c
. Khi đó
bất đẳng thức cần chứng minh trở thành
2a
b + c
+
2b
c + a
+
2c
a + b
+
3
r
3abc
a
3
+ b
3
+ c
3
4.
Ta
2a
b + c
+
2b
c + a
+
2c
a + b
+
3
r
3abc
a
3
+ b
3
+ c
3
(a + b + c)
2
ab + bc + ca
+
9abc
3
3
q
(3abc)
2
(a
3
+ b
3
+ c
3
)
(a + b + c)
2
ab + bc + ca
+
9abc
a
3
+ b
3
+ c
3
+ 6abc
.
Ta chỉ cần chứng minh
(a + b + c)
2
ab + bc + ca
3 1
9abc
a
3
+ b
3
+ c
3
+ 6abc
1
ab + bc + ca
a + b + c
a
3
+ b
3
+ c
3
+ 6abc
.
Bất đẳng thức y hiển nhiên đúng theo Schur. Vy bất đẳng thức đã cho đúng khi k =
3
3
2
.
Đẳng thức xảy ra khi x = y = z = 1 hoặc x +; y = 1; z =
1
x
.
Tóm lại giá trị k tốt nhất cần tìm k =
3
3
2
.
dụ 2.5 (VN TST 2012). Chứng minh rằng C = 10
24 hằng số lớn nhất sao cho
nếu 17 số thực dương a
1
,a
2
, . . . ,a
17
thỏa các điều kiện
(
a
2
1
+ a
2
2
+ ··· + a
2
17
= 24
a
3
1
+ ··· + a
3
17
+ a
1
+ ··· + a
17
< C
.
thì với mọi 1 i j k 17 ta a
i
, a
j
, a
k
độ dài ba cạnh của một tam giác.
Trước hết ta b đề sau
77
2. BÀI TOÁN TÌM HẰNG SỐ TỐT NHẤT TRONG BẤT ĐẲNG THỨC
Bổ đề 1. Cho số nguyên n 3. Giả sử n số dương a
1
,a
2
, . . . ,a
n
thỏa mãn bất đẳng thức
(n 1)
a
4
1
+ a
4
2
+ ... + a
4
n
<
a
2
1
+ a
2
2
+ ··· + a
2
n
2
.
y chứng minh ba số bất a
i
, a
j
, a
k
(1 i < j < k n) độ dài các cạnh của một tam giác.
Chứng minh.
Với n = 3 thì ta
2
a
4
1
+ a
4
2
+ a
4
3
<
a
2
1
+ a
2
2
+ a
2
3
2
(a
1
+ a
2
+ a
3
)(a
2
+ a
3
a
1
)(a
3
+ a
1
a
2
)(a
1
+ a
2
a
3
) > 0,
suy ra a
1
,a
2
,a
3
độ dài 3 cạnh của một tam giác.
Với n > 3, không mất tính tổng quát ta chứng minh a
1
, a
2
, a
3
độ dài ba cạnh của một
tam giác. Ta
(n 1)
a
4
1
+ a
4
2
+ ··· + a
4
n
<
a
2
1
+ a
2
2
+ ··· + a
2
n
2
=
2
a
2
1
+ a
2
2
+ a
2
3
2
+ a
2
4
+ ··· + a
2
n
2
2 + 1 + ··· + 1
| {z }
n3
"
(a
2
1
+ a
2
2
+ a
2
3
)
2
2
+ a
4
4
+ ··· + a
4
n
#
= (n 1)
"
(a
2
1
+ a
2
2
+ a
2
3
)
2
2
+ a
4
4
+ ··· + a
4
n
#
.
Suy ra
2
a
4
1
+ a
4
2
+ a
4
3
<
a
2
1
+ a
2
2
+ a
2
3
2
,
do đó a
1
,a
2
,a
3
độ dài ba cạnh của một tam giác.
Bổ đề được chứng minh.
Trở lại bài toán.
Đặt x
i
=
a
i
24
,i = 1,17, khi đó các số dương x
1
,x
2
. . . , x
17
thỏa
(
x
2
1
+ x
2
2
+ ··· + x
2
17
= 1
24
x
3
1
+ x
3
2
+ ··· + x
3
17
+ x
1
+ ··· + x
17
< 10
Để chứng minh a
i
, a
j
, a
k
độ dài ba cạnh của một tam giác, ta chỉ cần chứng minh x
i
, x
j
, x
k
độ dài ba cạnh của một tam giác.
Ta chứng minh bài toán đúng với C = 10.
Ta đi tìm số thực dương a thỏa :
16x
4
+ (a 1)x
2
<
a
10
24x
3
+ x
, x (0; 1) . (1)
nếu bất đẳng thức (1) thì ta suy ra
16x
4
i
+ (a 1)x
2
i
<
a
10
24x
3
i
+ x
i
, i = 1,17.
78
2. BÀI TOÁN TÌM HẰNG SỐ TỐT NHẤT TRONG BẤT ĐẲNG THỨC
Do đó
16
17
X
i=1
x
4
i
+ (a 1)
17
X
i=1
x
2
i
<
a
10
17
X
i=1
24x
3
i
+ x
i
.
Hay
16
17
X
i=1
x
4
i
+ (a 1) < a 16
17
X
i=1
x
4
i
< 1 =
17
X
i=1
x
2
i
!
2
,
khi đó theo b đề ta đpcm.
Ta tìm a để (1) đúng? Ta viết lại (1) như sau
16x
3
+ (a 1)x
a
10
24x
2
+ 1
< 0. (2)
x (0; 1) nên ta chọn a sao cho vế trái của (2) thừa số x 1 hay
16 + a 1
5a
2
= 0 a = 10.
Khi đó (2) trở thành:
16x
3
24x
2
+ 9x 1 < 0 (x 1)
16x
2
8x + 1
< 0. (3)
ràng (3) đúng với x (0; 1). Từ đó, ta đpcm.
Ta chứng minh C = 10 số lớn nhất. Giả sử tồn tại số C
0
> 10 sao cho với 17 số thực
dương x
1
, x
2
, . . . , x
17
thỏa mãn
(
x
2
1
+ x
2
2
+ ··· + x
2
17
= 1
24
x
3
1
+ x
3
2
+ ··· + x
3
17
+ x
1
+ ··· + x
17
< C
0
thì x
i
, x
j
, x
k
độ dài ba cạnh của một tam giác với mọi 1 i < j < k 17.
Ta xét 17 số thực dương
x
1
=
1
4
, x
2
=
r
1
16
a, x
3
= a, x
4
= ··· = x
17
=
r
1
16
+
a a
2
14
, 0 < a <
1
16
.
Ta
x
2
1
+ x
2
2
+ ··· + x
2
17
= 1,
và
x
1
x
3
=
1
4
a =
s
1
4
a
2
<
r
1
16
a = x
2
x
1
> x
2
+ x
3
,
hay x
1
, x
2
, x
3
không độ dài ba cạnh của một tam giác.
Ta
S(a) = 24
17
X
i=1
x
3
i
+
17
X
i=1
x
i
= 24
1
4
3
+
s
1
16
a
3
+ a
3
+ 14
s
1
16
+
a a
2
14
3
+
1
4
+
r
1
16
a + a + 14
r
1
16
+
a a
2
14
24
1
4
3
+
s
1
16
3
+ 14
s
1
16
3
+
1
4
+
r
1
16
+ 14
r
1
16
= 10 khi a 0
+
.
79
2. BÀI TOÁN TÌM HẰNG SỐ TỐT NHẤT TRONG BẤT ĐẲNG THỨC
Do tính liên tục của S, nên tồn tại a
0
0;
1
16
sao cho S(a
0
) < C
0
nhưng b
1
4
,
r
1
16
a
0
, a
0
,
r
1
16
+
a
0
a
2
0
14
, . . . ,
r
1
16
+
a
0
a
2
0
14
!
thỏa mãn các điều kiện của bài toán, nhưng b (x
1
,x
2
,x
3
) không tạo thành 3 cạnh của một
tam giác.
Vy bài toán được chứng minh.
dụ 2.6. Tìm số thực k lớn nhất sao cho bất đẳng thức sau luôn đúng với mọi a,b
các số thực dương phân biệt thỏa mãn ab = 1
a
2
+ b
2
2
p
2(a + b) 2
k.
Bài toán chuyển về tìm GTNN của biểu thức
P =
a
2
+ b
2
2
p
2(a + b) 2
,
với a,b > 0 và ab = 1.
Đặt t =
p
2 (a + b) > 2, ta
P =
t
4
16
4 (t 2)
=
(t + 2) (t
2
+ 4)
4
> 8
khi t > 2.
Giả sử tồn tại k > 8 thỏa bài toán. Cho t = 2 +
1
n
, suy ra
k
1
4
4 +
1
n
"
2 +
1
n
2
+ 4
#
8 (vô lí).
Vy k
max
= 8.
dụ 2.7. Tìm hằng số dương k lớn nhất sao cho bất đẳng thức
a
2
b
2
b
2
c
2
c
2
a
2
k (a + b + c)
6
đúng với mọi a,b,c 0.
Giả sử a b c, ta
a
2
b
2
b
2
c
2
c
2
a
2
=
a
2
b
2
b
2
c
2
a
2
c
2
a
2
b
2
a
2
b
2
= (a + b)
q
(a b)
2
.ab.ab.ab.ab
(a + b)
v
u
u
t
(a b)
2
+ 4ab
5
!
5
=
(a + b)
6
25
5
1
25
5
(a + b + c)
6
.
Từ đó ta k
max
=
1
25
5
.
80
2. BÀI TOÁN TÌM HẰNG SỐ TỐT NHẤT TRONG BẤT ĐẲNG THỨC
dụ 2.8 (IMO 2006). Tìm hằng số M nhỏ nhất sao cho với mọi số thực a,b,c ta đều
ab
a
2
b
2
+ bc
b
2
c
2
+ ca
c
2
a
2
M
a
2
+ b
2
+ c
2
2
.
Bằng biến đổi đơn giản, ta
ab
a
2
b
2
+ bc
b
2
c
2
+ ca
c
2
a
2
= |(b c) (a b) (a c) (a + b + c)|
Bài toán trở thành: Tìm M nhỏ nhất để
|(b c) (a b) (a c) (a + b + c)| M
a
2
+ b
2
+ c
2
. (1)
Giả sử a = max {a,b,c}, ta
9
a
2
+ b
2
+ c
2
2
=
(a + b + c)
2
+ 2 (b c)
2
+ 2 (a b) (a c)
2
h
2
2 |(a + b + c) (b c)| + 2 |(a b) (a c)|
i
2
16
2 |(b c) (a b) (a c) (a + b + c)|.
Do đó, ta
|(b c) (a b) (a c) (a + b + c)|
9
2
32
a
2
+ b
2
+ c
2
2
.
thể chọn b = 1 thì a = 1 +
3
2
2
; c = 1
3
2
2
để đẳng thức xảy ra.
Vy min M =
9
2
32
.
dụ 2.9 (Tổng quát IMO 2004). Với mỗi số nguyên dương n 3, tìm hằng số dương
k = k(n) lớn nhất sao cho nếu n số thực dương t
1
, t
2
, . . . , t
n
thỏa mãn
(t
1
+ t
2
+ ··· + t
n
)
1
t
1
+
1
t
2
+ ··· +
1
t
n
< k
thì t
i
, t
j
, t
k
độ dài ba cạnh của một tam giác với mọi i, j, k thỏa mãn 1 i < j < k n.
Giả sử tồn tại ba số chẳng hạn t
1
, t
2
, t
3
thỏa mãn t
1
+ t
2
t
3
. Khi đó
(t
1
+ t
2
+ t
3
)
1
t
1
+
1
t
2
+
1
t
3
= 3 +
t
1
t
2
+
t
2
t
1
+
t
1
+ t
2
t
3
+ t
3
1
t
1
+
1
t
2
3 + 2 +
t
1
+ t
2
t
3
+
4t
3
t
1
+ t
2
5 + 5
5
s
t
1
+ t
2
t
3
t
3
t
1
+ t
2
4
10.
81
2. BÀI TOÁN TÌM HẰNG SỐ TỐT NHẤT TRONG BẤT ĐẲNG THỨC
Xét n 4, ta
n
X
i=1
t
i
!
n
X
i=1
1
t
i
!
= (t
1
+ t
2
+ t
3
)
1
t
1
+
1
t
2
+
1
t
3
+
+
n
X
i=4
(t
1
+ t
2
+ t
3
)
1
t
i
+
1
t
1
+
1
t
2
+
1
t
3
t
i
+
n
X
i=4
t
i
!
n
X
i=4
1
t
i
!
10 + 2(n 3)
s
(t
1
+ t
2
+ t
3
)
1
t
1
+
1
t
2
+
1
t
3
+ (n 3)
2
10 + 2
10(n 3) + (n 3)
2
=
n +
10 3
2
.
Đẳng thức xảy ra khi t
1
= t
2
, t
3
= 2t
1
, t
i
=
2
10t
1
5
, i = 4, . . . ,n.
Từ đó suy ra k
max
=
n +
10 3
2
.
Nhận xét 1.
a) Nội dung bài toán IMO 2004 như sau:
Cho số nguyên n 3. Giả sử t
1
,t
2
, . . . ,t
n
các số thực dương sao cho
(t
1
+ t
2
+ ··· + t
n
)
1
t
1
+
1
t
2
+ ··· +
1
t
n
< n
2
+ 1.
Chứng minh rằng t
i
, t
j
, t
k
độ dài ba cạnh của một tam giác với mọi 1 i < j < k n.
b) Ta thể tổng quát bài toán trên theo hướng khác như sau:
Cho các số nguyên dương n, k với n k 3 giả sử n số thực dương t
1
, t
2
, . . . , t
n
thỏa
mãn
(t
1
+ t
2
+ ··· + t
n
)
1
t
1
+
1
t
2
+ ··· +
1
t
n
<
n +
2k
2
4k + 4 k
2
.
Chứng minh rằng mỗi số trong k số t
i
1
, t
i
2
, . . . ,t
i
k
đều nhỏ hơn tổng của k 1 số còn lại
với 1 i
1
< ··· < i
k
III. Bài tập
Bài 2.1. Tìm giá trị lớn nhất của k để bất đẳng thức sau đúng với mọi giá trị a, b, c:
a
4
+ b
4
+ c
4
+ abc (a + b + c) k (ab + bc + ca)
2
.
Bài 2.2. Tìm số thực k lớn nhất sao cho với mọi số thực a, b, c không âm và thỏa mãn điều
kiện a + b + c = 1, ta luôn
a
1 + 9bc + k(b c)
2
+
b
1 + 9ca + k(c a)
2
+
c
1 + 9ab + k(a b)
2
1
2
.
Bài 2.3 (Turkey National Olympiad Second Round 2013). Tìm số thực M lớn nhất sao
cho với mọi a, b, c dương, ta luôn
a
3
+ b
3
+ c
3
3abc M
ab
2
+ bc
2
+ ca
2
3abc
.
82
2. BÀI TOÁN TÌM HẰNG SỐ TỐT NHẤT TRONG BẤT ĐẲNG THỨC
Bài 2.4. Cho các số dương x, y, z. Tìm số k lớn nhất sao cho:
x
y
+
y
z
+
z
x
+ 3k (k + 1) ·
x + y + z
3
xyz
.
Bài 2.5. Cho các số thực không âm a, b, c thỏa mãn ab + bc + ca = a + b + c. Tìm số thực k lớn
nhất sao cho ta luôn bất đẳng thức:
(a + b + c)
1
a + b
+
1
b + c
+
1
c + a
k · (a + b + c + 1) .
Bài 2.6 (Phạm Kim Hùng). Tìm số thực dương k lớn nhất sao cho bất đẳng thức sau đúng
với mọi số dương a, b, c:
(a + b + c)
1
a
+
1
b
+
1
c
+ k ·
(ab + bc + ca)
a
2
+ b
2
+ c
2
9 + k.
Bài 2.7 (Kiểm tra Trường Qúy Đôn năm 2014). Tìm hằng số k nhỏ nhất sao cho
với mọi số thực dương a, b, c thỏa mãn a + b + c = 3 thì ta luôn bất đẳng thức:
k
a
4
+ b
4
+ c
4
3
a
3
+ b
3
+ c
3
+ 3abc 6.
Bài 2.8. Với các số không âm a, b, c thỏa mãn a + b + c = ab + bc + ca. Tìm số thực k lớn nhất
sao cho
a + b + c + k · abc 3 + k.
Bài 2.9. Cho a, b, c các số thực dương. Tìm hằng số k lớn nhất sao cho bất đẳng thức sau
đúng:
a
3
+ b
3
+ c
3
(a + b) (b + c) (c + a)
+ k ·
ab + bc + ca
(a + b + c)
2
3
8
+
k
3
.
Bài 2.10 (Vietnamese Team Selection Test 2009). Tìm tất cả các hằng số k sao cho bất
đẳng thức sau đúng với mọi số dương a, b, c :
a
b + c
+ k
b
c + a
+ k
c
a + b
+ k
k +
1
2
3
.
Bài 2.11 (VN TST 2013). Tìm số nguyên dương k lớn nhất để bất đẳng thức sau đúng với
mọi số thực dương a, b, c thỏa mãn điều kiện abc = 1:
1
a
+
1
b
+
1
c
+
k
a + b + c + 1
3 +
k
4
.
Bài 2.12. Xét các số dương a, b, c thỏa mãn abc = 1. Tìm số k lớn nhất sao cho
a
2
+ b
2
+ c
2
+ 3k (k + 1)
1
a
+
1
b
+
1
c
.
83
2. BÀI TOÁN TÌM HẰNG SỐ TỐT NHẤT TRONG BẤT ĐẲNG THỨC
Bài 2.13. Xét các số dương a, b, c thỏa mãn a + b + c = 3. Tìm k lớn nhất sao cho
1
a
+
1
b
+
1
c
3 k
a
2
+ b
2
+ c
2
3
.
Bài 2.14 (Tạp chí Pi - P47 7/2017). Tìm số thực k bé nhất sao cho với mọi b ba số thực
a, b, c abc 0, ta luôn
abc + k ·
(a b)
2
+ (b c)
2
+ (c a)
2
+ 2 a + b + c.
Bài 2.15 (Tạp chí Pi - P68 9/2017). Tìm số thực k nhỏ nhất sao cho với mọi số thực
dương x, y, z min {xy,yz,zx} 1, ta luôn
3
p
(x
2
+ 1) (y
2
+ 1) (z
2
+ 1)
x + y + z
3
2
+ k.
Bài 2.16 (Tạp chí Pi - P61 9/2017). Tìm số thực k nhỏ nhất sao cho với mọi số thực a,
b, c độ dài ba cạnh của một tam giác, ta luôn
a
b + c
+
b
c + a
+
c
a + b
+ k ·
ab + bc + ca
a
2
+ b
2
+ c
2
3
2
+ k.
Bài 2.17 (P5 2/2017). Tìm số dương k nhỏ nhất sao cho với mọi số thực dương x, y, z thỏa
mãn điều kiện
xy +
yz +
zx = 1, ta luôn
1
x + y + k
+
1
y + z + k
+
1
z + x + k
9k + 3
2
.
Bài 2.18 (BMO 2012). Tìm số thực k lớn nhất để đối với mọi a,b,c khác 0 thỏa mãn
1
a
+
1
b
+
1
c
3, ta đều bất đẳng thức
a
2
+ 4
b
2
+ c
2

b
2
+ 4
a
2
+ c
2

c
2
+ 4
a
2
+ b
2

k.
Bài 2.19. Cho a
1
,a
2
, . . . ,a
5
các số thực bất tổng bằng 0. Tìm hằng số c = c(n) lớn nhất
sao cho bất đẳng thức sau đúng:
c
n
X
i=1
|a
i
|
X
1i<jn
|a
i
a
j
|.
Bài 2.20. Tìm hằng số M lớn nhất sao cho với mỗi n nguyên dương tồn tại các số a
1
,a
2
,...,a
n
> 0
và b
1
,b
2
,...,b
n
> 0 đồng thời thỏa mãn các điều kiện;
i)
P
n
k=1
b
k
= 1; 2b
k
b
k1
+ b
k+1
, k = 2,...,n;
ii) a
2
k
1 +
P
k
i=1
a
i
b
i
, k = 1,n;
iii) a
n
= M.
84
2. BÀI TOÁN TÌM HẰNG SỐ TỐT NHẤT TRONG BẤT ĐẲNG THỨC
ĐÁP SỐ VÀ HƯỚNG DẪN GIẢI
85
Chương 1
Các bất đẳng thức cổ điển
§1. Bất đẳng thức AM-GM
Câu 1.1.
a) Bất đẳng thức cần chứng minh tương đương với
3
s
1.1.1
(1 + a) (1 + b) (1 + c)
+
3
s
abc
(1 + a) (1 + b) (1 + c)
1.
Đặt :
T =
3
s
1.1.1
(1 + a) (1 + b) (1 + c)
+
3
s
abc
(1 + a) (1 + b) (1 + c)
T
1
3
1
1 + a
+
1
1 + b
+
1
1 + c
+
1
3
a
1 + a
+
b
1 + b
+
c
1 + c
T
1
3
a + 1
1 + a
+
b + 1
1 + b
+
c + 1
1 + c
=
1
3
.3 = 1
Dấu đẳng thức xảy ra khi a = b = c 0.
b) Ta
1 +
a
b
1 +
b
c
1 +
c
a
= 2 +
a
b
+
b
c
+
c
a
+
a
c
+
c
b
+
b
a
=
a + b
c
+ 1
+
b + c
a
+ 1
+
c + a
b
+ 1
1
= (a + b + c)
1
a
+
1
b
+
1
c
1
3(a + b + c)
3
abc
1
2(a + b + c)
3
abc
+ 2.
Câu 1.2. Bất đẳng thức cần chứng minh tương đương với
1
n
p
(1 + a
1
)(1 + a
2
) ···(1 + a
n
)
+
n
r
a
1
a
2
···a
n
(1 + a
1
)(1 + a
2
) ···(1 + a
n
)
1. (1)
Áp dụng bất đẳng thức AM-GM ta
V T (1)
1
n
1
X
i=1
1
1 + a
i
+
1
n
1
X
i=1
a
i
1 + a
i
= 1.
86
1. BẤT ĐẲNG THỨC AM-GM
Bài toán được chứng minh.
Câu 1.3. Ta abc
a + b + c
3
3
=
1
27
.
Khi đó
1 +
1
a
1 +
1
b
1 +
1
c
1 +
1
3
abc
3
64.
Suy ra (1 + a) (1 + b) (1 + c) 64abc.
Câu 1.4. Bất đẳng thức cần chứng minh tương đương với
n
r
a
1
a
2
···a
n
(a
1
+ b
1
)(a
2
+ b
2
) ···(a
n
+ b
n
)
+
n
s
b
1
b
2
···b
n
(a
1
+ b
1
)(a
2
+ b
2
) ···(a
n
+ b
n
)
1. (1)
Áp dụng bất đẳng thức AM-GM ta
n
r
a
1
a
2
···a
n
(a
1
+ b
1
)(a
2
+ b
2
) ···(a
n
+ b
n
)
1
n
a
1
a
1
+ b
1
+ ··· +
a
n
a
n
+ b
n
n
s
b
1
b
2
···b
n
(a
1
+ b
1
)(a
2
+ b
2
) ···(a
n
+ b
n
)
1
n
b
1
a
1
+ b
1
+ ··· +
b
n
a
n
+ b
n
Cộng hai bất đẳng thức trên theo vế ta đpcm.
Câu 1.5. α
i
các số hữu tỉ dương và
n
P
i=1
α
i
= 1 nên tồn tại các số nguyên dương N,k
1
,k
2
, ··· , k
n
sao cho α
i
=
k
i
N
. Áp dụng bất đẳng thức AM-GM cho N số, ta
n
X
i=1
α
i
· a
i
=
a
1
+ a
1
+ ··· + a
1
| {z }
k
1
số
+ ··· + a
n
+ a
n
+ ··· + a
n
| {z }
k
n
số
N
a
k
1
n
1
···a
k
n
n
n
= a
α
1
1
···a
α
n
n
.
Bất đẳng thức được chứng minh.
Câu 1.6. Chuẩn hóa a
1
+ a
2
+ ··· + a
n
= n, ta cần chứng minh
a
k
1
+ a
k
2
+ ···a
k
n
n. (1)
Áp dụng bất đẳng thức AM GM cho k số gồm k 1 số 1 và a
k
i
ta
a
k
i
+ k 1 ka
i
n
X
i=1
a
k
i
+ n(k 1) k
n
X
i=1
a
i
= kn
n
X
i=1
a
k
i
n.
Vy (1) đúng, hay bài toán được chứng minh.
Câu 1.7. Áp dụng bất đẳng thức
1
x
+
1
y
4
x + y
ta
1
a + 3b
+
1
b + 2c + a
4
(a + 3b) + (b + 2c + a)
=
2
a + 2b + c
1
b + 3c
+
1
2a + b + c
2
a + b + 2c
1
c + 3a
+
1
a + 2b + c
2
2a + b + c
.
Cộng các bất đẳng thức trên theo vế ta đpcm.
87
1. BẤT ĐẲNG THỨC AM-GM
Câu 1.8. Áp dụng bất đẳng thức si cho hai số ta
4
a +
4
b 2
8
ab
4
a +
4
b
4
16
ab và
1
a
+
1
b
2
ab
.
Suy ra
4
a +
4
b
4
1
a
+
1
b
16
ab.
2
ab
= 32.
Dẫn tới
1
4
a +
4
b
4
1
32
1
a
+
1
b
.
Tương tự:
1
4
b +
4
c
4
1
32
1
b
+
1
c
,
1
(
4
c +
4
a)
4
1
32
1
c
+
1
a
.
Cộng các bất đẳng thức trên ta
1
4
a +
4
b
4
+
1
4
b +
4
c
4
+
1
(
4
c +
4
a)
4
1
16
1
a
+
1
b
+
1
c
.
Mặt khác, theo giả thiết ta ab + bc + ca 3abc nên suy ra
1
a
+
1
b
+
1
c
3.
Suy ra
1
4
a +
4
b
4
+
1
4
b +
4
c
4
+
1
(
4
c +
4
a)
4
3
16
(đpcm).
Câu 1.9. Bất đẳng thức cần chứng minh tương đương với
2a
b + 2c
+
2b
c + 2a
+
2c
a + 2b
4
2a
b + 2a
2b
c + 2b
2c
a + 2c
a
1
b + 2c
+
1
b + 2a
+ b
1
c + 2a
+
1
c + 2b
+ c
1
a + 2b
+
1
a + 2c
2.
Áp dụng bất đẳng thức
1
x
+
1
y
4
x + y
ta
1
b + 2c
+
1
b + 2a
4
2a + 2b + 2c
=
2
a + b + c
.
Suy ra
a
1
b + 2c
+
1
b + 2a
2a
a + b + c
.
Tương tự:
b
1
c + 2a
+
1
c + 2b
2b
a + b + c
, c
1
a + 2b
+
1
a + 2c
2c
a + b + c
.
Cộng các bất đẳng thức trên ta
a
1
b + 2c
+
1
b + 2a
+ b
1
c + 2a
+
1
c + 2b
+ c
1
a + 2b
+
1
a + 2c
2 (đpcm).
88
1. BẤT ĐẲNG THỨC AM-GM
Câu 1.10. Áp dụng AM GM, ta
1 + x
3
= (1 + x)
1 x + x
2
(1 + x + 1 x + x
2
)
2
4
=
(2 + x
2
)
2
4
.
Tương tự
1
p
1 + y
3
2
2 + y
2
;
1
1 + z
3
2
2 + z
2
.
Vy
P =
1
1 + x
3
+
1
p
1 + y
3
+
1
1 + z
3
2
2 + x
2
+
2
2 + y
2
+
2
2 + z
2
.
Áp dụng Cauchy Swarzt, ta được:
P
18
x
2
+ y
2
+ z
2
+ 6
1.
Dấu ‘=’ xảy ra khi x = y = z = 2. Vậy GTNN của biểu thức P = 1.
Câu 1.11. Ta có:
a
1 + b
2
=
a (1 + b
2
) ab
2
1 + b
2
= a
ab
2
1 + b
2
a
ab
2
2b
= a
ab
2
.
Do đó:
a
1 + b
2
+
b
1 + c
2
+
c
1 + a
2
a + b + c
1
2
(ab + bc + ca) .
Mà:
ab + bc + ca
1
3
(a + b + c)
2
= 3.
Nên suy ra:
a
1 + b
2
+
b
1 + c
2
+
c
1 + a
2
3
3
2
=
3
2
.
Câu 1.12. Ta có:
a
2
a + 2b
2
=
a (a + 2b
2
) 2ab
2
a + 2b
2
= a
2ab
2
a + b
2
+ b
2
a
2ab
2
2
2ab
= a
1
2
b.
a.
Suy ra:
a
2
a + 2b
2
+
b
2
b + 2c
2
+
c
2
c + 2a
2
a + b + c
1
2
a.b +
b.c +
c.a
.
Mặt khác:
ab + bc + ca
1
3
(a + b + c)
2
=
3
4
.
Và
ab.
b +
bc.
c +
ca.
a
p
(ab + bc + ca) (a + b + c)
3
2
2
.
Vy:
a
2
a + 2b
2
+
b
2
b + 2c
2
+
c
2
c + 2a
2
3
2
3
4
=
3
4
.
89
1. BẤT ĐẲNG THỨC AM-GM
Câu 1.13. ax
2
+ ay
2
2axy. Đẳng thức xảy ra khi x = y.
by
2
+ cz
2
2
bcyz.Đẳng thức xảy ra khi by
2
= cz
2
.
cz
2
+ bx
2
2
cbzx. Đẳng thức xảy ra khi cz
2
= bx
2
.
y giờ ta chọn a, b, c sao cho :
a + b = 3
2c = 1
a =
bc
a = 1
b = 2
c =
1
2
Suy ra: x
2
+ y
2
2xy. Đẳng thức xảy ra khi x = y. .
2y
2
+
1
2
z
2
2yz.Đẳng thức xảy ra khi 2y
2
=
1
2
z
2
..
1
2
z
2
+ 2x
2
2zx. Đẳng thức xảy ra khi
1
2
z
2
= 2x
2
..
Cộng vế theo vế ta được :
3x
2
+ 3y
2
+ z
2
2 (xy + yz + zx) 3x
2
+ 3y
2
+ z
2
10 (đpcm).
Đẳng thức xảy ra khi :
x = y
2y
2
=
1
2
z
2
1
2
z
2
= 2x
2
xy + yz + zx = 5
(
x = y = 1
z = 2
.
Câu 1.14. Áp dụng bất đẳng thức si cho 3 số thực dương ta
a
3
(a + 2b) (b + 2c)
+
a + 2b
27
+
b + 2c
27
3
3
s
a
3
(a + 2b) (b + 2c)
.
a + 2b
27
.
b + 2c
27
=
1
3
a.
Tương tự:
b
3
(b + 2c) (c + 2a)
+
b + 2c
27
+
c + 2a
27
1
3
b,
và
c
3
(c + 2a) (a + 2b)
+
c + 2a
27
+
a + 2b
27
1
3
c.
Cộng ba bất đẳng thức trên ta
a
3
(a + 2b) (b + 2c)
+
b
3
(b + 2c) (c + 2a)
+
c
3
(c + 2a) (a + 2b)
+
2(a + b + c)
9
a + b + c
3
Suy ra
a
3
(a + 2b) (b + 2c)
+
b
3
(b + 2c) (c + 2a)
+
c
3
(c + 2a) (a + 2b)
a + b + c
9
.
Đẳng thức xảy ra khi a = b = c.
Câu 1.15. Ta thấy đẳng thức xảy ra khi a = b = c = 1.
Khi đó
a
4
b
2
(c + 2)
=
1
3
,b = 1,c + 2 = 3 nên áp dụng bất đẳng thức si cho 4 số ta được
a
4
b
2
(c + 2)
+
b
3
+
b
3
+
c + 2
9
4
4
s
a
4
b
2
(c + 2)
+
b
3
.
b
3
.
c + 2
9
=
4
3
a
90
1. BẤT ĐẲNG THỨC AM-GM
Tương tự:
b
4
c
2
(a + 2)
+
2b
3
+
a + 2
9
4
3
b,
c
4
a
2
(b + 2)
+
2a
3
+
b + 2
9
4
3
c.
Cộng các bất đẳng thức trên ta
a
4
b
2
(c + 2)
+
b
4
c
2
(a + 2)
+
c
4
a
2
(b + 2)
+
7(a + b + c) + 6
9
4 (a + b + c)
3
.
Hay
a
4
b
2
(c + 2)
+
b
4
c
2
(a + 2)
+
c
4
a
2
(b + 2)
5(a + b + c) 6
9
.
a + b + c 3
3
abc = 3 nên ta
a
4
b
2
(c + 2)
+
b
4
c
2
(a + 2)
+
c
4
a
2
(b + 2)
1 (đpcm).
Câu 1.16. Áp dụng bất đẳng thức
x +
y
p
2 (x + y), ta :
r
a + b
c
+
r
b + c
a
+
r
c + a
b
1
2
a
c
+
b
c
!
+
1
2
b
a
+
c
a
!
+
1
2
c
a
+
b
a
!
=
a
2
1
c
+
1
b
+
b
2
1
a
+
1
c
+
c
2
1
a
+
1
b
.
Áp dụng bất đẳng thức
1
x
+
1
y
4
x + y
, ta :
a
2
1
c
+
1
b
+
b
2
1
a
+
1
c
+
c
2
1
a
+
1
b
2
2a
b +
c
+
2
2b
a +
c
+
2
2c
a +
b
.
Áp dụng bất đẳng thức
x +
y
p
2 (x + y), ta :
2
2a
b +
c
+
2
2b
a +
c
+
2
2c
a +
b
2
2a
p
2 (b + c)
+
2
2b
p
2 (a + c)
+
2
2c
p
2 (a + b)
= 2
r
c
a + b
+
r
b
a + c
+
r
a
b + c
!
.
Câu 1.17. Áp dụng bất đẳng thức AM-GM ta
a
4
b + 2
+
b + 2
9
2
s
a
4
b + 2
.
b + 2
9
=
2
3
a
2
.
Tương tự:
b
4
c + 2
+
c + 2
9
2
3
b
2
,
c
4
a + 2
+
a + 2
9
2
3
c
2
.
Cộng ba bất đẳng thức trên ta
a
4
b + 2
+
b
4
c + 2
+
c
4
a + 2
+
a + b + c + 6
9
2
3
a
2
+ b
2
+ c
2
= 2.
Suy ra
a
4
b + 2
+
b
4
c + 2
+
c
4
a + 2
12 (a + b + c)
9
.
91
1. BẤT ĐẲNG THỨC AM-GM
Mặt khác:
a + b + c
p
3 (a
2
+ b
2
+ c
2
) = 3
nên suy ra
a
4
b + 2
+
b
4
c + 2
+
c
4
a + 2
12 3
9
= 1 (đpcm).
Đẳng thức xảy ra khi a = b = c = 1.
Câu 1.18. Bất đẳng thức cần chứng minh tương đương với
4
3 a
2
+ 1
4
3 b
2
+ 1
4
3 c
2
+ 1
3 (a + b + c)
2
.
Từ đề bài, ta suy ra a
2
,b
2
,c
2
< 3 .
Áp dụng bất đẳng thức si ta có:
4
3 a
2
+
3 a
2
2
r
4
3 a
2
(3 a
2
) = 4,
suy ra
4
3 a
2
+ 1 a
2
+ 2.
Tương tự:
4
3 b
2
+ 1 b
2
+ 2,
4
3 c
2
+ 1 c
2
+ 2.
Do đó
4
3 a
2
+ 1
4
3 b
2
+ 1
4
3 c
2
+ 1
a
2
+ 2
b
2
+ 2
c
2
+ 2
.
Mặt khác:
3 (a + b + c)
2
= 3
a + b
2
.
2 + 1.c
2
3
"
(a + b)
2
2
+ 1
#
c
2
+ 2
.
Ta chứng minh
3
"
(a + b)
2
2
+ 1
#
a
2
+ 2
b
2
+ 2
. ()
Khai triển và rút gọn, bất đẳng thức (*) trở thành
2a
2
b
2
+ a
2
+ b
2
+ 2 6ab.
Bất đẳng thức y hiển nhiên đúng vì:
2
a
2
b
2
+ 1
4ab, a
2
+ b
2
2ab.
Vy bài toán được chứng minh.
Câu 1.19. Ta có: b + c
p
2(b
2
+ c
2
) Suy ra
a
2
b + c
a
2
+ b
2
+ c
2
p
2(b
2
+ c
2
)
1
2
b
2
+ c
2
.
Tương tự:
b
2
c + a
a
2
+ b
2
+ c
2
p
2(c
2
+ a
2
)
1
2
c
2
+ a
2
;
c
2
a + b
a
2
+ b
2
+ c
2
p
2(a
2
+ b
2
)
1
2
a
2
+ b
2
.
92
1. BẤT ĐẲNG THỨC AM-GM
Suy ra
V T
a
2
+ b
2
+ c
2
2
1
a
2
+ b
2
+
1
b
2
+ c
2
+
1
c
2
+ a
2
1
2
a
2
+ b
2
+
b
2
+ c
2
+
c
2
+ a
2
.
Ta có:
1
a
2
+ b
2
+
1
b
2
+ c
2
+
1
c
2
+ a
2
9
a
2
+ b
2
+
b
2
+ c
2
+
c
2
+ a
2
và
a
2
+ b
2
+ c
2
1
6
a
2
+ b
2
+
b
2
+ c
2
+
c
2
+ a
2
2
.
Suy ra
V T
1
2
2
a
2
+ b
2
+
b
2
+ c
2
+
c
2
+ a
2
.
Đặt
t =
a
2
+ b
2
+
b
2
+ c
2
+
c
2
+ a
2
2
ab +
bc +
ca
3
2
3
abc.
Suy ra
1
54
2
t
3
abc nên từ giả thiết ta suy ra
t =
a
2
+ b
2
+
b
2
+ c
2
+
c
2
+ a
2
=
7 abc
2
7
2
1
108
t
3
t
3
+ 108t 378
2 0
t 3
2
t
2
+ 3
2t + 126
0 t 3
2.
Suy ra V T
3
2
2
2
=
3
2
.
Đẳng thức xảy ra khi và chỉ khi
a = b = c
3
2a =
7 a
3
2
(
a = b = c
a
3
+ 6a 7 = 0
a = b = c = 1.
Câu 1.20. Nhận thấy đẳng thức xảy ra khi a = b = c = 1 và 3 = 1 + 2 nên ta đánh giá
a
2
+ 2b
2
+ 3 =
a
2
+ b
2
+
b
2
+ 1
+ 2 2ab + 2b + 2.
Do đó:
1
a
2
+ 2b
2
+ 3
1
2
.
1
ab + b + 1
.
Suy ra:
1
a
2
+ 2b
2
+ 3
+
1
b
2
+ 2c
2
+ 3
+
1
c
2
+ 2a
2
+ 3
1
2
1
ab + b + 1
+
1
bc + c + 1
+
1
ca + a + 1
Vy ta cần chứng minh:
1
ab + b + 1
+
1
bc + c + 1
+
1
ca + a + 1
1.
Bất đẳng thức này hiển nhiên đúng đó đẳng thức. Để chứng minh ta thay c =
1
ab
vào vế
trái và biến đổi ta đpcm.
Câu 1.21. Ta
X
1
a
3
+ b
X
1
p
2
a
3
b
=
1
2
X
4
s
1
a
3
·
1
b
1
4
2
X
3
a
+
1
b
=
3
2
.
93
1. BẤT ĐẲNG THỨC AM-GM
Câu 1.22. Ta
ab + 1
2ab + a
2
+ b
2
+ c
2
+ ab + bc + ca
2
=
(a + b)
2
+ (c + a) (c + b)
2
.
Suy ra
V T
1
2
3 +
(a + c)(b + c)
(a + b)
2
+
(a + b)(a + c)
(b + c)
2
+
(b + a)(b + c)
(c + a)
2
3.
Câu 1.23. Áp dụng bất đẳng thức AM-GM ta
a + b + c = a +
b + c
2
+
b + c
2
3
3
s
a
(b + c)
2
4
,
suy ra
3
s
2a
b + c
2
3a
a + b + c
.
Chứng minh tương tự, ta cũng
3
s
2b
c + a
3
3b
a + b + c
và
3
s
2c
a + b
2
3c
a + b + c
.
Cộng ba bất đẳng thức trên theo vế ta đpcm.
Câu 1.24. Bài toán y thể chứng minh bằng cách sử dụng đánh giá sau:
3
r
a
3
+ b
3
2
a
2
+ b
2
a + b
.
Chú ý rằng:
a
2
+ b
2
a + b
= a + b
2ab
a + b
Như vy ta phải chứng minh:
2
ab
a + b
+
bc
b + c
+
ca
c + a
+ a + b + c 6.
Áp dụng bất đẳng thức AM-GM với abc = 1,ta ngay:
2ab
a + b
+
a + b
2
+
2bc
b + c
+
b + c
2
+
2ca
c + a
+
c + a
2
6.
Vy ta điều phải chứng minh.Đẳng thức xảy ra khi a = b = c = 1.
Câu 1.25. Ta
13a
2
b
2
c
2
2abc 2
(a
2
+ b
2
+ c
2
)
3
=
27a
2
b
2
c
2
(abc + 2)
2
2(a
2
+ b
2
+ c
2
)
3
27a
2
b
2
c
2
2(a
2
+ b
2
+ c
2
)
3
.
Ta chứng minh:
27a
2
b
2
c
2
2(a
2
+ b
2
+ c
2
)
3
1
4
a
2
b
2
c
2
(a
2
+ b
2
+ c
2
)
3
1
54
.
a + b + c = 0 nên trong ba số a,b,c hai số cùng dấu, ta giả sử hai số đó a,b. Khi đó ab 0
nên
a
2
+ b
2
+ c
2
3
=
a
2
+ b
2
+ (a + b)
2
3
=
"
a
2
+ b
2
+
(a + b)
2
2
+
(a + b)
2
2
#
3
27
4
a
2
+ b
2
(a + b)
2
(a + b)
2
27
4
.2ab.4ab.c
2
= 54a
2
b
2
c
2
.
Suy ra
a
2
b
2
c
2
(a
2
+ b
2
+ c
2
)
3
1
54
(đpcm).
94
1. BẤT ĐẲNG THỨC AM-GM
Câu 1.26. Không mất tính tổng quát, ta giả sử c = min {a,b,c}.
Nếu a b suy ra 6
3 (a b) (b c) (c a) 0 nên bất đẳng thức cần chứng minh luôn đúng.
Nếu a b, ta có:
6
3 (a b) (b c) (c a) = 6
3 (b a) (b c) (a c)
6
3 (b a) ba = 3
3.
q
(a b)
2
2ab.2ab
3
3
v
u
u
t
(a b)
2
+ 2ab + 2ab
3
!
3
=
q
(a + b)
3
q
(a + b + c)
3
(đpcm).
Đẳng thức xảy ra khi và chỉ khi
(
c = 0
(a b)
2
= 2ab
c = 0
a =
2 ±
2
b
.
Câu 1.27. Trước hết ta bất đẳng thức
1
x
+
1
y
2
2
p
x
2
+ y
2
.
Không mất tính tổng quát, ta giả sử a > b > c. Khi đó
P =
2
a b
+
2
b c
+
2
a c
+
5
ab + bc + ca
8
a b + b c
+
2
a c
+
5
ab + bc + ca
=
10
a c
+
10
2
ab + bc + ca
20
2
q
(a c)
2
+ 4(ab + bc + ca)
=
20
2
q
(a + c)
2
+ 4b (a + c)
=
20
2
p
(1 b) (1 + 3b)
.
Mặt khác
(1 b) (1 + 3b) =
1
3
(3 3b) (1 + 3b)
1
3
3 3b + 1 + 3b
2
2
=
4
3
.
Do đó P 10
6. Đẳng thức xảy ra khi
b =
1
3
a =
2
3
,c = 0
. Vy min P = 10
6.
Câu 1.28. Ta
X
a +
1
b
2
X
a +
1
b
b +
1
c
=
X
ab +
X
a
c
+
X
1
ab
+ 3.
Áp dụng
P
1
ab
=
P
a và
P
ab +
P
a
c
2
P
a. ta đpcm.
Cách 2: Ta
a
2
+
1
b
2
+
b
c
3
3
r
a
2
·
1
b
2
·
b
c
= 3
3
r
a
2
bc
= 3
3
r
a
2
· abc
bc
= 3a.
95
1. BẤT ĐẲNG THỨC AM-GM
Tương tự
b
2
+
1
c
2
+
c
a
3b,
và
c
2
+
1
a
2
+
a
b
3c.
Kết hợp với
a
b
+
b
c
+
c
a
3
3
r
a
b
·
b
c
·
c
a
= 3.
ta đpcm.
Câu 1.29. Ta b + 2a +
2
b + 1
5
2
+
3
2
a nên
a + 2b +
2
a + 1
b + 2a +
2
b + 1
5
2
+
3
2
a
5
2
+
3
2
b
và
5
2
+
3
2
a
5
2
+
3
2
b
=
25
4
+
15
4
(a + b) +
9
4
ab 16
nên ta đpcm.
Câu 1.30. Ta
2a + b + c = a + b + a + c 2
p
(a + b) (a + c).
Suy ra
1
(2a + b + c)
2
1
4
.
1
(a + b) (a + c)
.
Do đó
V T
1
2
.
a + b + c
(a + b) (b + c) (c + a)
.
Mặt khác ta có:
9 (a + b) (b + c) (c + a) 8(a + b + c)(ab + bc + ca),
và
(ab + bc + ca)
2
3abc (a + b + c) .
Từ giả thiết, ta suy ra ab + bc + ca = abc (a + b + c) nên ab + bc + ca 3. Suy ra
a + b + c
(a + b)(b + c)(c + a)
=
(a + b + c)(ab + bc + ca)
(a + b)(b + c)(c + a)
1
ab + bc + ca
9
8
.
1
3
=
3
8
.
Từ đó, suy ra đpcm.
Câu 1.31. Áp dụng bất đẳng thức Bunhia ta có:
a
2
+ b
2
2
=
a
a · a + b
b · b
2
a
3
+ b
3
(a + b)
a
3
+ b
3
p
2 (a
2
+ b
2
)
q
(a
2
+ b
2
)
3
2
a
3
+ b
3
a
2
+ b
2
3
q
2 (a
3
+ b
3
)
2
c
3
a
2
+ b
2
1
3
2
c
3
3
q
(a
3
+ b
3
)
2
=
1
3
2
a
3
+ b
3
+ c
3
3
q
(a
3
+ b
3
)
2
1
3
2
3
a
3
+ b
3
. (1)
96
1. BẤT ĐẲNG THỨC AM-GM
Ta cũng hai bất đẳng thức tương tự
b
3
c
2
+ a
2
1
3
2
a
3
+ b
3
+ c
3
3
p
(c
3
+ a
3
)
2
1
3
2
3
c
3
+ a
3
, (2)
a
3
b
2
+ c
2
1
3
2
a
3
+ b
3
+ c
3
3
p
(b
3
+ c
3
)
2
1
3
2
3
b
3
+ c
3
. (3)
Cộng ba bất đẳng thức (1), (2) và (3), ta được
P
1
3
2
a
3
+ b
3
+ c
3
1
3
q
(a
3
+ b
3
)
2
+
1
3
q
(a
3
+ b
3
)
2
+
1
3
q
(a
3
+ b
3
)
2
1
3
2
3
a
3
+ b
3
+
3
b
3
+ c
3
+
3
c
3
+ a
3
.
Đặt x =
3
a
3
+ b
3
, y =
3
b
3
+ c
3
, z =
3
c
3
+ a
3
. Suy ra:
P
1
3
2
x
3
+ y
3
+ z
3
1
x
2
+
1
y
2
+
1
z
2
(x + y + z)
.
Mặt khác áp dụng bất đẳng thức AM-GM ta được:
x
2
+ y
2
+ z
2
1
x
2
+
1
y
2
+
1
z
2
9
1
x
2
+
1
y
2
+
1
z
2
9
x
2
+ y
2
+ z
2
x
3
+ y
3
+ z
3
1
x
2
+
1
y
2
+
1
z
2
9
x
3
+ y
3
+ z
3
x
2
+ y
2
+ z
2
P
1
3
2
9
x
3
+ y
3
+ z
3
x
2
+ y
2
+ z
2
(x + y + z)
.
Áp dụng bất đẳng thức Cauchy-Schwarz ta có:
(x + y + z)
2
3
x
2
+ y
2
+ z
2
(x + y + z)
2
x
2
+ y
2
+ z
2
3
x
2
+ y
2
+ z
2
2
x
2
+ y
2
+ z
2
2
=
x
x.
x + y
y.
y + z
z.
z
2
(x + y + z)
x
3
+ y
3
+ z
3
(x + y + z)
2
x
2
+ y
2
+ z
2
3 (x + y + z)
x
3
+ y
3
+ z
3
x
3
+ y
3
+ z
3
x
2
+ y
2
+ z
2
1
3
(x + y + z).
Do đó
P
3
4 (x + y + z) =
3
4
3
a
3
+ b
3
+
3
b
3
+ c
3
+
3
c
3
+ a
3
t =
3
a
3
+ b
3
+
3
b
3
+ c
3
+
3
c
3
+ a
3
3
2
ab +
bc +
ca
3
3
2
3
abc
Suy ra abc
1
54
t
3
.
Nên từ giả thiết ta suy ra:
3 t +
1
54
t
3
t
3
+ 54t 162 0. ()
97
1. BẤT ĐẲNG THỨC AM-GM
hàm số f(t) = t
3
+ 54t 162 hàm đồng biến và f (2)f (3) < 0 nên f (t) nghiệm duy nhất
x = m (2; 3).
Suy ra () t m P
6
32.m. Đẳng thức xảy ra
(
a = b = c
a
3
+ 3
3
2a 3 = 0
a = b = c = α.
Trong đó α nghiệm duy nhất của phương trình X
3
+ 3
3
2X 3 = 0 và α > 0.
Vy min P =
6
32.m (đpcm).
Câu 1.32. Ta chứng minh a + bc 2 (1).
Do bc
b
2
+ c
2
2
< 2 nên 1 tương đương a
2
(2 bc)
2
. Do
a
2
3 (b
2
+ c
2
) và (2 bc)
2
(3 b
2
c
2
) = (bc 1)
2
+ (b c)
2
0
nên (1) đúng.
Ta chứng minh
p
(4 a
2
)(4 c
2
) ac + 2b (2).
Ta abc
a
2
+ b
2
+ c
2
3
3
2
1 nên
(4 a
2
)(4 c
2
) (ac + 2b)
2
= 16 4(a
2
+ b
2
+ c
2
+ abc) 0.
Suy ra (2) đúng.
Ta chứng minh a
2
+ b
2
+ c
2
a
b
+ b
2
c + c
2
a (3).
Ta a
2
+
1
4
(ab + c
2
)
2
a(ab + c
2
) = a
2
b + c
2
a (4) và
1
4
(4 a
2
)b
2
+ (4 c
2
)c
2
1
2
bc
p
(4 a
2
)(4 b
2
) bc
2
+
abc
2
2
(5)
Cộng (4) và (5) theo vế suy ra (3) được chứng minh.
Quay trở lại bài toán. Từ (1) ta a
2
b + ab
2
c 2ab, b
2
c + abc
2
2bc, c
2
a + a
2
bc 2ca. Suy ra
2(ab + bc + ca) a
2
b + b
2
c + c
2
a + abc(a + b + c)
(a + b + c)
2
a
2
+ b
2
+ c
2
+ abc(a + b + c) + a
2
b + b
2
c + c
2
a. (6)
Từ (3) và (6) ta
(a + b + c)
2
abc(a + b + c) + 2(a
2
b + b
2
c + c
2
a)
(a + b + c)(a + b + c abc) 2(a
2
b + b
2
c + c
2
a).
Câu 1.33. Không mất tính tổng quát, giả sử a = max{a; b; c}.
Khi đó b + c + 1 c + a + 1 và b + c + 1 a + b + 1
Suy ra
b
c + a + 1
b
b + c + 1
;
c
a + b + 1
c
b + c + 1
. Dấu bằng xảy ra khi a = b = c
Mặt khác theo BĐT AM-GM ta
(1 b) (1 c) (b + c + 1)
1 b + 1 c + b + c + 1
3
3
= 1
Suy ra
(1 a) (1 b) (1 c)
1 a
b + c + 1
Do đó
P
a
b + c + 1
+
b
b + c + 1
+
c
b + c + 1
+
1 a
b + c + 1
= 1
Dấu = xảy ra khi a = b = c = 1.
98
1. BẤT ĐẲNG THỨC AM-GM
Câu 1.34.
Cách 1: Theo BĐT AM-GM ta có: a + b 2
ab; b + c 2
bc; c + a 2
ca.
Ta có: P =
3a + 2b + c
(a + b)(b + c)(c + a)
3a + 2b + c
2
ab.2
bc.2
ca
=
1
8
1
ab
+
2
ac
+
3
bc
.
Lại có: 3bc + 4ac + 5ab 6abc
3
a
+
4
b
+
5
c
6. Đặt S =
3
a
+
4
b
+
5
c
.
Ta
S
2
=
9
a
2
+
16
b
2
+
25
c
2
+
24
ab
+
30
ac
+
40
bc
= 9
1
a
2
+
1
c
2
+ 16
1
b
2
+
1
c
2
+
24
ab
+
30
ac
+
40
bc
S
2
18
ac
+
32
bc
+
24
ab
+
30
ac
+
40
bc
=
24
ab
+
48
ac
+
72
bc
= 24
1
ab
+
2
ac
+
3
bc
1
ab
+
2
ac
+
3
bc
S
2
24
3
2
P
1
8
.
3
2
=
3
16
Dấu bằng xảy ra khi a = b = c = 2.
Vy giá trị lớn nhất của P bằng
3
6
khi a = b = c = 2.
Cách 2: 3bc + 4ac + 5ab 6abc
3
a
+
4
b
+
5
c
6. Đặt S =
3
a
+
4
b
+
5
c
.
Ta có:
S
2
=
9
a
2
+
16
b
2
+
25
c
2
+
24
ab
+
30
ac
+
40
bc
= 9
1
a
2
+
1
c
2
+ 16
1
b
2
+
1
c
2
+
24
ab
+
30
ac
+
40
bc
S
2
18
ac
+
32
bc
+
24
ab
+
30
ac
+
40
bc
=
24
ab
+
48
ac
+
72
bc
= 24
1
ab
+
2
ac
+
3
bc
.
1
ab
4
(a + b)
2
;
1
bc
4
(b + c)
2
;
1
ac
4
a + c)
2
S
2
24
4
(a + b)
2
+
8
(a + c)
2
+
12
(c + b)
2
= 96
1
(a + b)
2
+
2
(a + c)
2
+
3
(c + b)
2
S
2
96
1
(a + b)
2
+
1
(b + c)
2
+ 2
1
(a + c)
2
+
1
(b + c)
2

S
2
96
2
(a + b)(b + c)
+
4
(a + c)(b + c)
S
2
192
1
(a + b)(b + c)
+
2
(a + c)(b + c)
= 192
3a + 2b + c
(a + b)(b + c)(c + a)
P
S
2
192
=
36
192
=
3
16
Dấu bằng xảy ra khi a = b = c = 2.
Vy giá trị lớn nhất của P bằng
3
6
khi a = b = c = 2.
Câu 1.35. Biến đổi tương đương ta chứng minh được: a+b+c
p
3 (a
2
+ b
2
+ c
2
), a, b,c > 0.
Dấu = xảy ra khi và chỉ khi a = b = c.
Ta
V T
v
u
u
t
3.
1
p
x
3
+ 2y
3
+ 6
+
1
p
y
3
+ 2z
3
+ 6
+
1
z
3
+ 2x
3
+ 6
!
v
u
u
t
3.
s
3.
1
x
3
+ 2y
3
+ 6
+
1
y
3
+ 2z
3
+ 6
+
1
z
3
+ 2x
3
+ 6
99
1. BẤT ĐẲNG THỨC AM-GM
Theo bất đẳng thức TTBC-TBN ta có:
x
3
+ 2y
3
+ 6 = (x
3
+ y
3
+ 1) + (y
3
+ 1 + 1) + 3 3xy + 3y + 3 = 3(xy + y + 1)
Tương tự:
y
3
+ 2z
3
+ 6 3(yz + z + 1)
z
3
+ 2x
3
+ 6 3(xz + x + 1)
Do đó
V T
4
9
1
xy + y + 1
+
1
yz + z + 1
+
1
zx + x + 1
= 9
1
xy + y + 1
+
xy
xy(yz + z + 1)
+
1
zx + x + 1
= 9. (do xyz = 1)
Hay V T
3. Dấu = xảy ra khi và chỉ khi x = y = z = 1.
Câu 1.36. Ta P +
2
3 + 1
3
=
1
3x
+
1
3x
+
1
3x
+
1
3
x
3
+
1
y
2
+
y
2
3
+
1
z
3
+
z
9
+
z
9
+
z
9
Áp dụng Bất đẳng thức AM-GM, ta có:
1
3x
+
1
3x
+
1
3x
+
1
3
x
3
> 4
4
s
1
3x
3
.
x
3
3
=
4
3
. (1)
1
y
2
+
y
2
3
>
2
3
3
. (2)
1
z
3
+
z
9
+
z
9
+
z
9
>
4
3
9
(3)
Từ (1), (2) và (3) ta P +
2
3 + 1
3
>
4
3
+
2
3
3
+
4
3
9
hay P >
4
3 + 9
9
Dấu đẳng thức xảy ra khi và chỉ khi:
1
3x
=
x
3
3
1
y
2
=
y
2
3
1
z
3
=
z
9
x
3
+ y
2
+ z = 2
3 + 1.
x = 1
y =
4
3
z =
3
Vy min P =
4
3 + 9
9
khi (x,y,z) = (1,
4
3,
3)
Câu 1.37. Ta có: a
6
b
6
+ b
6
c
6
+ c
6
a
6
= a
5
b
5
.ab + b
5
c
5
.bc + c
5
a
5
.ca
a
5
b
5
a
5
+ b
5
+ 1 + 1 + 1
5
+ b
5
c
5
b
5
+ c
5
+ 1 + 1 + 1
5
+ c
5
a
5
c
5
+ a
5
+ 1 + 1 + 1
5
=
1
5
(a
5
b
5
(a
5
+ b
5
+ 3) + b
5
c
5
(b
5
+ c
5
+ 3) + c
5
a
5
(c
5
+ a
5
+ 3))
Đặt x = a
5
,y = b
5
,z = c
5
. Ta có: x + y + z = 3;với x,y,z > 0.
Cần chứng minh: xy(x + y + 3) + yz(y + z + 3) + zx(z + x + 3) 15
hay xy(x + y) + yz(y + z) + zx(z + x) + 3(xy + yz + zx) 15
(xy + yz + zx)(x + y + z + 3) 3xyz 15
2(xy + yz + zx) xyz 5 (1)
Mặt khác; xyz (3 2x)(3 2y)(3 2z) 9xyz 12(xy + yz + zx) 27
xyz
4
3
(xy + yz + zx) 3.
Ta có: VT(1) 2(xy + yz + zx)
4
3
(xy + yz + zx) + 3 =
2
3
(xy + yz + zx) + 3 5.
Vy bài toán được chứng minh.
100
1. BẤT ĐẲNG THỨC AM-GM
Câu 1.38. Với k = 1 thì b
1
2
;
1
2
; 2
không thỏa mãn.
Với k = 2 thì b
4
5
;
4
5
;
7
5
không thỏa mãn.
Ta chứng minh với k = 3 thì bất đẳng thức đúng hay x
3
y
3
z
3
(x
3
+ y
3
+ z
3
) 3.
Không mất tính tổng quá ta thể giả sử z nhỏ nhất suy ra z 1. Ta x
3
+ y
3
= (x + y)
3
3xy(x + y) = (3 z)
3
3xy(x + y). Khi đó bất đẳng thức cần chứng minh tương đương
(3 z)
3
+ z
3
3
x
3
y
3
z
3
+ 3xy(x + y) 3z
2
9z + 9
1
x
3
y
3
z
3
+ x
2
y + xy
2
Ta
1
x
3
y
3
z
3
+ x
2
y + xy
2
3
3
s
x
3
y
3
x
3
y
3
z
3
=
3
z
, như vy chỉ cần chứng minh
3z
2
9z + 9
3
z
3(z 1)
2
0
Câu 1.39. Theo bất đẳng thức AM-GM, ta có:
a + b +
p
2(a + c) = (a + b) +
r
a + c
2
+
r
a + c
2
3
3
v
u
u
t
(a + b).
r
a + c
2
!
2
= 3
3
r
(a + b)(a + c)
2
.
Suy ra
1
a + b +
p
2(a + c)
3
2
27(a + b)(a + c)
.
Tương tự với hai biểu thức còn lại. Do đó:
X
cyc
1
a + b +
p
2(a + c)
3
X
sym
2
27(a + b)(a + c)
=
4(a + b + c)
27(a + b)(b + c)(c + a)
.
Hơn nữa, ta thấy với mọi a, b, c dương:
9(a + b)(b + c)(c + a) 8(a + b + c)(ab + bc + ca) =
X
sym
a(b c)
2
0
(a + b)(b + c)(c + a)
8
9
(a + b + c)(ab + bc + ca).
Do đó:
X
cyc
1
a + b +
p
2(a + c)
3
1
6(ab + bc + ca)
. (1)
Mặt khác, ta cũng có: (ab + ca + ca)
2
3abc(a + b + c)nên theo giả thiết:
16(a + b + c)
1
a
+
1
b
+
1
c
=
ab + bc + ca
abc
3(a + b + c)
ab + bc + ca
ab + bc + ca
3
16
. (2)
Từ (1) và (2), suy ra:
1
(a + b +
p
2(a + c))
3
+
1
(b + c +
p
2(b + a))
3
+
1
(c + a +
p
2(c + b))
3
8
9
.
101
1. BẤT ĐẲNG THỨC AM-GM
Ta đpcm.
Đẳng thức xảy ra khi dấu bằng tất cả các bất đẳng thức trên xảy ra hay:
a,b,c > 0
a = b = c
16(a + b + c) =
1
a
+
1
b
+
1
c
a = b = c =
1
4
.
Câu 1.40. Gọi P vế trái của bất đẳng thức cần chứng minh. Không mất tính tổng quát, ta
giả sử a + b + c = 1.
Áp dụng bất đẳng thức AM-GM ta :
a
a
2
+ 8bc
+
a
a
2
+ 8bc
+ a
a
2
+ 8bc
3a
2a
a
2
+ 8bc
+ a
a
2
+ 8bc
3a.
Tương tự :
2b
b
2
+ 8ca
+ b
b
2
+ 8ca
3b ;
2c
c
2
+ 8ab
+ c
c
2
+ 8ab
3c
Cộng ba bất đẳng thức trên lại với nhau ta được :
2P + a
3
+ b
3
+ c
3
+ 24abc 3
Mặt khác ta lại :
1 = (a + b + c)
3
= a
3
+ b
3
+ c
3
+ 3 (a + b) (b + c) (c + a) a
3
+ b
3
+ c
3
+ 24abc.
Suy ra :
2P 3
a
3
+ b
3
+ c
3
+ 24abc
3 1 = 2 P 1 đpcm.
Câu 1.41. Áp dụng bất đẳng thức AM-GM ta
a
3
b + 2b = a
3
b + b + b 3ab, b
3
c + 2c 3bc, c
3
a + 2a 3ca.
Suy ra
a
3
b + b
3
c + c
3
a + 2(a + b + c) 3(ab + bc + ca),
hay
a
3
b + b
3
c + c
3
a + 6 3(ab + bc + ca).
Mặt khác
ab + bc + ca
(a + b + c)
2
3
= 3,
nên ta
a
3
b + b
3
c + c
3
a + 9 4(ab + bc + ca).
Câu 1.42. Áp dụng bất đẳng thức AM-GM ta
a =
4
r
a
4
abcd
1
4
a
b
+
a
b
+
b
c
+
a
d
=
1
4
2a
b
+
b
c
+
a
d
.
Tương tự:
b
1
4
2b
c
+
c
d
+
b
a
, c
1
4
2c
d
+
d
a
+
c
b
, d
1
4
2d
a
+
a
b
+
d
c
.
102
1. BẤT ĐẲNG THỨC AM-GM
Suy ra
a + b + c + d
3
4
a
b
+
b
c
+
c
d
+
d
a
+
1
4
b
a
+
c
b
+
d
c
+
a
d
3
4
(a + b + c + d) +
1
4
b
a
+
c
b
+
d
c
+
a
d
.
Nên
a + b + c + d
b
a
+
c
b
+
d
c
+
a
d
.
Câu 1.43. Ta giả sử a = min {a,b,c}.
Ta V T =
Q
(a + b) (a
2
ab + b
2
).
(a + b) (b + c) (c + a) = (a + b + c) (ab + bc + ca) 3abc
(a + b + c) (ab + bc + ca) = 2 (ab + bc + ca) .
Suy ra V T 2 (ab + bc + ca)
Q
(a
2
ab + b
2
).
Do a = min {a,b,c} nên a
2
ab + b
2
b
2
và c
2
ca + a
2
c
2
.
Tiếp theo ta chứng minh
b
2
c
2
(ab + bc + ca)
b
2
bc + c
2
1. ()
Áp dụng bất đẳng thức AM-GM ta
b
2
c
2
(ab + bc + ca)
b
2
bc + c
2
bc + bc + ab + bc + ca + b
2
bc + c
2
4
4
=
1
4
4
b
2
+ c
2
+ 2bc + ab + ca
4
.
Lại
b
2
+ c
2
+ 2bc + ca + ab = (b + c)
2
+ a (b + c) = (2 a)
2
+ a (2 a) = 2a + 4 4.
Từ đó, ta đpcm.
Câu 1.44. Xét x
2
yz, y
2
zx, z
2
xy các số thực dương. Áp dụng bất đẳng thức AM-GM
ta
P
x
2
+ y
2
+ z
2
(xy + yz + zx)
3
3
Mặt khác
x
2
+ y
2
+ z
2
+ 2(xy + yz + zx) = (x + y + z)
2
0,
suy ra
xy + yz + zx
1
2
.
Do đó
P
1
8
.
Xét x
2
yz > 0 > y
2
zx, z
2
xy,khi đó áp dụng bất đẳng thức AM-GM ta
P = (x
2
yz)(xz y
2
)(xy z
2
)
xy + xz + x
2
yz y
2
z
2
3
3
103
1. BẤT ĐẲNG THỨC AM-GM
Mặt khác
x
2
+ xy + xz yz y
2
z
2
= x
2
+ xy + xz + yz (y + z)
2
x
2
+ xy + xz + yz
=
3
2
(x
2
+ y
2
+ z
2
) (x 2y)
2
(x 2z)
2
3
2
(x
2
+ y
2
+ z
2
) =
3
2
.
Suy ra P
1
8
. Đẳng thức xảy ra chẳng hạn khi x =
2
2
, y =
2
2
, z = 0.
Vy GTLN của P bằng:
1
8
.
Câu 1.45. Ta
(x + y)
2
4xy
x
2
y
2
1 z
=
x
2
y
2
x + y
1
4
xy(x + y).
Do đó, V T
1
4
(xy(x + y) + yz(y + z) + zx(z + x)) + 3xyz. Tiếp theo ta chứng minh
xy(x + y) + yz(y + z) + zx(z + x) + 12xyz
2
3
=
2(x + y + z)
3
3
. (1)
Ta (1) tương đương với
2(x
3
+ y
3
+ z
3
) + 3 (xy(x + y) + yz(y + z) + zx(z + x)) 24xyz. (2)
2(x
3
+ y
3
+ z
3
) 2xy
xy + 2yz
yz + 2zx
zx,
nên
V T (2) 8(xy
xy + yz
zy + zx
zx) 24
xyz 24xyz.
(Do xyz 1 nên
xyz xyz.).
Câu 1.46. Áp dụng bất đẳng thức AM GM, ta
a
2
b + 2c
+
(b + 2c)a
2
9
2
s
a
2
b + 2c
(b + 2c)a
2
9
=
2a
2
3
.
Tương tự
b
2
c + 2a
+
(c + 2a)b
2
9
2b
2
3
,
c
2
a + 2b
+
(a + 2b)c
2
9
2c
2
3
.
Suy ra:
F =
a
2
b + 2c
+
b
2
c + 2a
+
c
2
a + 2b
2
3
a
2
+ b
2
+ c
2
1
9
a
2
(b + 2c) + b
2
(c + 2a) + c
2
(a + 2b)
. (1)
Lại áp dụng AM GM, ta
a
2
c + b
2
a + c
2
b
a
3
+ a
3
+ c
3
3
+
b
3
+ b
3
+ a
3
3
+
c
3
+ c
3
+ b
3
3
= a
3
+ b
3
+ c
3
. (2)
Từ (1) và (2) suy ra:
F
2
3
a
2
+ b
2
+ c
2
1
9
(a + b + c) (a
2
+ b
2
+ c
2
)
2
3
a
2
+ b
2
+ c
2
1
9
a
2
+ b
2
+ c
2
p
3 (a
2
+ b
2
+ c
2
).
104
1. BẤT ĐẲNG THỨC AM-GM
Đặt t =
p
3 (a
2
+ b
2
+ c
2
) , từ giả thiết ta có:
25
a
2
+ b
2
+ c
2
48 = 9
a
4
+ b
4
+ c
4
3
a
2
+ b
2
+ c
2
2
3
a
2
+ b
2
+ c
2
2
25
a
2
+ b
2
+ c
2
+ 48 0
3 a
2
+ b
2
+ c
2
16
3
.
Do đó
F
2
9
t
2
1
27
t
3
= f(t).
với t [3; 4] (3) min
t[3;4]
f(t) = f (3) = 1 (4). Từ (3) và (4) suy ra F 1.
Vy min F = 1 xảy ra khi a = b = c = 1.
Câu 1.47. Bất đẳng thức cần chứng minh tương đương với
X
5a
2
+ 4bc 2
bc
>
p
3(a
2
+ b
2
+ c
2
)
X
a
2
p
3(a
2
+ b
2
+ c
2
)
5a
2
+ 4bc + 2
bc
1
5
. (1)
Áp dụng đt AM-GM ta
p
3(a
2
+ b
2
+ c
2
)(5a
4
bc)
8a
2
+ 3b
2
+ 3c
2
+ 4bc
2
p
3(a
2
+ b
2
+ c
2
).2
bc a
2
+ b
2
+ c
2
+ 3bc.
Do đó
p
3(a
2
+ b
2
+ c
2
)
5a
2
+ 4bc + 2
bc
10a
2
+ 5(b + c)
2
2
5(a
2
+ b
2
+ c
2
).
Suy ra
V T (1)
X
a
2
5(a
2
+ b
2
+ c
2
)
=
1
5
.
Câu 1.48. Gọi P vết trái. Ta
4a
2
b
1 + a + b
=
a
2
b (1 + a + b + c)
1 + a + b
= a
2
b +
a
2
bc
1 + a + b
.
Tương tự
4b
2
c
1 + b + c
= b
2
c +
ab
2
c
1 + b + c
,
4c
2
a
1 + c + a
= c
2
a +
abc
2
1 + c + a
.
Do đó
4P = 4
X
a
2
b
1 + a + b
=
X
a
2
b + abc
X
a
1 + a + b
.
Mặt khác
4
X
a
1 + a + b
=
X
a (1 + a + b + c)
1 + a + b
= 3 +
X
ac
1 + a + b
.
Lại
ac
1 + a + b
=
3ac
2 (2a + b) + (2b + c)
1
3
2ac
2a + b
+
ac
2b + c
.
Nên
X
ac
1 + a + b
1
3
(a + b + c) = 1.
Suy ra
4P a
2
b + b
2
c + c
2
a + abc 4 P 1.
105
1. BẤT ĐẲNG THỨC AM-GM
Câu 1.49. hiệu P vế trái của bất đẳng thức cần chứng minh theo yêu cầu đề bài.
Tiếp theo, thể giải bài đã ra theo một trong các cách sau:
Cách 1 a, b, c > 0 nên ta có:
P =
a
2
+ bc
p
a(b + c).(a
2
+ bc)
+
b
2
+ ca
p
b(c + a).(b
2
+ ca)
+
c
2
+ ab
p
c(a + b).(c
2
+ ab)
.
Áp dụng bất đẳng thức Cauchy cho hai số thực dương a(b + c) và (a
2
+ bc), ta được:
p
a(b + c). (a
2
+ bc)
ab + ac + a
2
+ bc
2
=
(a + b)(a + c)
2
.
Suy ra
a
2
+ bc
p
a(b + c). (a
2
+ bc)
2 (a
2
+ bc)
(a + b)(a + c)
. (1.1)
Chứng minh tương tự, ta được:
b
2
+ ac
p
b(c + a). (b
2
+ ca)
2 (b
2
+ ca)
(b + c)(a + c)
. (1.2)
c
2
+ ab
p
c(a + b). (c
2
+ ab)
2 (c
2
+ ab)
(c + a)(c + b)
. (1.3)
Cộng vế theo vế, các bất đẳng thức (1.1), (1.2) và (1.3), ta được:
P 2
a
2
+ bc
(a + b)(a + c)
+
b
2
+ ca
(b + c)(b + a)
+
c
2
+ ab
(c + a)(c + b)
. (1.4)
Ta sẽ chứng minh:
a
2
+ bc
(a + b)(a + c)
+
b
2
+ ca
(b + c)(b + a)
+
c
2
+ ab
(c + a)(c + b)
3
2
. (1.5)
Thật vy, a, b, c > 0 nên
(1.5) 2

a
2
+ bc
(b + c) +
b
2
+ ca
(a + c) +
c
2
+ ab
(a + b)
3(a + b)(b + c)(c + a)
4
a
2
b + ab
2
+ b
2
c + bc
2
+ c
2
a + ca
2
3
a
2
b + ab
2
+ b
2
c + bc
2
+ c
2
a + ca
2
+ 2abc
a
2
b + ab
2
+ b
2
c + bc
2
+ c
2
a + ca
2
6abc
a
b
2
+ c
2
2abc
+ b
c
2
+ a
2
2ac
+ c
a
2
+ b
2
2ab
0
a(b c)
2
+ b(c a)
2
+ c(a b)
2
0. (1.6)
Hiển nhiên, (1.6) bất đẳng thức đúng. thế, (1.5) được chứng minh. Từ (1.4) và (1.5), suy
ra P 3. Từ các chứng minh trên ta thấy, đẳng thức xảy ra khi và chỉ khi dấu bằng xảy ra đồng
thời (1.1), (1.2), (1.3) và (1.6). Dễ thấy, điều cuối cùng được khi và chỉ khi a = b = c.
Cách 2 a, b, c > 0 nên ta có:
P =
p
(a
2
+ bc) (ab + ac)
a(b + c)
+
p
(b
2
+ ca) (bc + ba)
b(c + a)
+
p
(c
2
+ ab) (ca + cb)
c(a + b)
. (2.1)
Áp dụng bất đẳng thức Bunhiacopxki cho hai b 2 số
a,
bc
và
ab,
ac
, ta được:
a
2
+ bc
(ab + ac)
a.
ab +
bc.
ac
2
= ab. (a + c)
2
.
106
1. BẤT ĐẲNG THỨC AM-GM
Suy ra
q
(a
2
+ bc) (ab + ac)
ab.(a + c). (2.2)
Bằng cách tương tự, ta cũng chứng minh được:
q
(b
2
+ ac) (bc + ba)
bc.(b + a) (2.3)
q
(c
2
+ ab) (ca + cb)
ca.(c + b). (2.4)
Từ (2.1), (2.2), (2.3) và (2.4), suy ra
P
ab(a + c)
a(b + c)
+
bc(b + a)
b(c + a)
+
ca(c + b)
c(a + b)
. (2.5)
hiệu Q vế phải của (2.5). Theo bất đẳng thức Cauchy cho 3 số thực dương, ta có:
Q
3
s
ab(a + c)
a(b + c)
+
bc(b + a)
b(c + a)
+
ca(c + b)
c(a + b)
= 3. (2.6)
Từ (2.5) và (2.6), suy ra P 3. Từ các chứng minh trên ta thấy, đẳng thức xảy ra khi và chỉ
khi dấu bằng xảy ra đồng thời (2.2),(2.3), (2.4) và (2.6). Dễ thấy, điều cuối cùng được khi
và chỉ khi a = b = c.
Cách 3 Theo bất đẳng thức Cauchy cho 3 số thực dương, ta có:
P 3
6
s
a
2
+ bc
a(b + c)
.
b
2
+ ca
b(c + a)
.
c
2
+ ab
c(a + b)
. (3.1)
Tiếp theo, ta sẽ chứng minh:
a
2
+ bc
a(b + c)
.
b
2
+ ca
b(c + a)
.
c
2
+ ab
c(a + b)
1. (3.2)
Thật vy, do a, b, c > 0 nên
(3.2)
a
2
+ bc
b
2
+ ca
c
2
+ ab
abc(a + b)(b + c)(c + a). (3.3)
Áp dụng bất đẳng thức Bunhiacopxki cho hai b 2 số
a,
bc
và
b,
bc
, ta được:
a
2
+ bc
b
2
+ bc
(ab + bc)
2
.
Hay
b
a
2
+ bc
(b + c) b
2
(a + c)
2
. (3.4)
Chứng minh tương tự, ta cũng có:
c
b
2
+ ca
(c + a) c
2
(b + a)
2
. (3.5)
a
c
2
+ ab
(a + b) a
2
(c + b)
2
. (3.6)
các vế của (3.4), (3.5) và (3.6) đều dương nên nhân các bất đẳng thức đó với nhau, vế theo
vế, rồi chia cả 2 vế của bất đẳng thức thu được cho abc(a + b)(b + c)(c + a), ta sẽ nhận được bất
đẳng thức (3.3). thế, (3.2) được chứng minh. Từ (3.1) và (3.2), hiển nhiên suy ra P 3. Từ
các chứng minh trên ta thấy, đẳng thức xảy ra khi và chỉ khi dấu bằng xảy ra đồng thời (3.1),
(3.4), (3.5) và (3.6). Dễ thấy, điều cuối cùng đưc khi và chỉ khi a = b = c.
107
1. BẤT ĐẲNG THỨC AM-GM
Câu 1.50. Áp dụng Cauchy ta có:
(a
1
+ a
2
+ ··· + a
2018
)
2
= (a
1
+ a
2
+ ··· + a
2018
)
1
a
1
+
1
a
2
+ ··· +
1
a
2018
2018
2018
a
1
a
2
···a
2018
·
2018
2018
a
1
a
2
···a
2018
= 2018
2
.
Hay a
1
+ a
2
+ ··· + a
2018
2018. (1)
Ta P = a
1
+
a
2
2
+ 1
2
+
a
3
3
+ 2
3
+ ··· +
a
2018
2018
+ 2017
2018
1
2
+
2
3
+ ··· +
2017
2018
. (2)
Tiếp tục sử dụng Cauchy và sử dụng (1) ta thu được
a
1
+
a
2
2
+ 1
2
+
a
3
3
+ 2
3
+ ··· +
a
2018
2018
+ 2017
2018
a
1
+ a
2
+ ··· + a
2018
2018. (3)
Dấu đẳng thức xảy ra khi a
1
= a
2
= ··· = a
2018
= 1.
Từ (2) và (3) suy ra P 2018
1
2
+
2
3
+ ··· +
2017
2018
= 1 +
1
2
+
1
3
+ ··· +
1
2018
.
108
2. BẤT ĐẲNG THỨC CAUCHY-SCHWARZ
§2. Bất đẳng thức Cauchy-Schwarz
Câu 2.1. Bình phương hai vế và rút gọn, ta
q
(a
2
1
+ a
2
2
+ ··· + a
2
n
) (b
2
1
+ b
2
2
+ ··· + b
2
n
) (a
1
b
1
+ a
2
b
2
+ ··· + a
n
b
n
) .
Đây chính bất đẳng thức Cauchy-Schwarz dạng đa thức. Đẳng thức xảy ra khi a
i
= kb
i
.
Câu 2.2. Áp dụng bất đẳng thức Cauchy Schwarz ta
a
2
+ 1
b
2
+ 1
=
a
2
+ 1
1 + b
2
(a + b)
2
.
Tương tự, ta cũng
(b
2
+ 1) (c
2
+ 1) (b + c)
2
, (c
2
+ 1) (a
2
+ 1) (a + c)
2
.
Nhân ba bất đẳng thức trên theo vế ta được
a
2
+ 1
b
2
+ 1
c
2
+ 1
(a + b) (b + c) (c + a) .
Đẳng thức xảy ra khi a = b = c = 1.
Câu 2.3. Áp dụng bất đẳng thức Cauchy Schwarz ta
a
2
+ b
2
+ 1
1 + 1 + c
2
(a + b + c)
2
= 9
hay
a
2
+ b
2
+ 1
9
c
2
+ 2
.
Tương tự
b
2
+ c
2
+ 1
9
a
2
+ 2
và c
2
+ a
2
+ 1
9
b
2
+ 2
.
Cộng các bất đẳng thức trên theo vế, ta được
2
a
2
+ b
2
+ c
2
+ 3 9
1
a
2
+ 2
+
1
b
2
+ 2
+
1
c
2
+ 2
.
Đẳng thức xảy ra khi a = b = c = 1.
Câu 2.4. Áp dụng bất đẳng thức Cauchy Schwarz ta
V T =
a ·
a
3
+ 8abc +
b ·
b
3
+ 8abc +
c ·
c
3
+ 8abc
p
(a + b + c) (a
3
+ b
3
+ c
3
+ 24abc).
Mặt khác
(a + b + c)
3
= a
3
+ b
3
+ c
3
+ 3 (a + b) (b + c) (c + a) a
3
+ b
3
+ c
3
+ 24abc
Suy ra
V T
q
(a + b + c) (a + b + c)
3
= (a + b + c)
2
= 1.
Bài toán được chứng minh. Đẳng thức xảy ra khi a = b = c =
1
3
.
109
2. BẤT ĐẲNG THỨC CAUCHY-SCHWARZ
Câu 2.5. Áp dụng bất đẳng thức Cauchy-Schwarz ta
V T =
X
a
4
a(b + 2c)
(a
2
+ b
2
+ c
2
)
a(b + 2c) + b(c + 2a) + c(a + 2b)
=
(a
2
+ b
2
+ c
2
)
2
3(ab + bc + ca)
a
2
+ b
2
+ c
2
3
= 1.
Câu 2.6. Áp dụng bất đẳng thức Cauchy-Schwarz ta
V T =
X
a
4
a
b
2
+ c
2
+ 7
(a
2
+ b
2
+ c
2
)
2
a
b
2
+ c
2
+ 7 + b
c
2
+ a
2
+ 7 + c
a
2
+ b
2
+ 7
(a
2
+ b
2
+ c
2
)
2
p
(a
2
+ b
2
+ c
2
)(2a
2
+ 2b
2
+ 2c
2
+ 21)
=
t
t
2t + 21
, t = a
2
+ b
2
+ c
2
3.
Ta chứng minh
t
t
2t + 21
1 t
3
2t + 21 (t 3)(t
2
+ 3t + 9) 0 (đúng).
Câu 2.7. Áp dụng bất đẳng thức Cauchy-Schwarz ta có:
9
4a
2
+ b
2
+ c
2
=
(a + b + c)
2
2a
2
+ (a
2
+ b
2
) + (a
2
+ c
2
)
a
2
2a
2
+
b
2
a
2
+ b
2
+
c
2
a
2
+ c
2
=
1
2
+
b
2
a
2
+ b
2
+
c
2
a
2
+ c
2
.
Tương tự:
9
a
2
+ 4b
2
+ c
2
a
2
a
2
+ b
2
+
c
2
c
2
+ b
2
+
1
2
và
9
a
2
+ b
2
+ 4c
2
a
2
a
2
+ c
2
+
b
2
b
2
+ c
2
+
1
2
.
Cộng 3 bất đẳng thức trên theo vế ta đpcm.
Câu 2.8. Bất đẳng thức cần chứng minh tương đương với
1
3
2a
2
+ 3
+
1
3
2b
2
+ 3
+
1
3
2c
2
+ 3
6
5
a
2
2a
2
+ 3
+
b
2
2b
2
+ 3
+
c
2
2c
2
+ 3
3
5
. (1)
Áp dụng bất đẳng thức Cauchy-Schwarz ta
25
3(2a
2
+ 3)
=
25
6a
2
+ (a + b + c)
2
=
(2 + 2 + 1)
2
2(2a
2
+ bc) + 2a(a + b + c) + a
2
+ b
2
+ c
2
2
2a
2
+ bc
+
2
a(a + b + c)
+
1
a
2
+ b
2
+ c
2
.
110
2. BẤT ĐẲNG THỨC CAUCHY-SCHWARZ
Suy ra
a
2
2a
2
+ 3
3
25
2a
2
2a
2
+ bc
+
2a
a + b + c
+
a
2
a
2
+ b
2
+ c
2
.
Suy ra
V T (1)
3
25
2a
2
2a
2
+ bc
+
2b
2
2b
2
+ ca
+
2c
2
2c
2
+ ab
+ 3
.
Ta chứng minh
a
2
2a
2
+ bc
+
b
2
2b
2
+ ca
+
c
2
2c
2
+ ab
1
bc
2a
2
+ bc
+
ca
2b
2
+ ca
+
ab
2c
2
+ ab
1. (2)
Áp dụng bất đẳng thức Cauchy-Schwarz ta
V T (2)
(ab + bc + ca)
2
2abc(a + b + c) + a
2
b
2
+ b
2
c
2
+ c
2
a
2
=
(ab + bc + ca)
2
(ab + bc + ca)
2
= 1.
Vy bài toán được chứng minh.
Câu 2.9. Bất đẳng thức cần chứng minh tương đương với
1
1
a
2
+ 1
+
1
1
b
2
+ 1
+
1
1
c
2
+ 1
3
2
a
2
a
2
+ 1
+
b
2
b
2
+ 1
+
c
2
c
2
+ 1
3
2
4a
2
3a
2
+ ab + bc + ca
+
4b
2
3b
2
+ ab + bc + ca
+
4c
2
3c
2
+ ab + bc + ca
2. (1)
Ta
4a
2
3a
2
+ ab + bc + ca
=
(a + a)
2
(2a
2
+ bc) + a(a + b + c)
a
2a
2
+ bc
+
a
a + b + c
.
Suy ra
V T (1)
a
2
2a
2
+ bc
+
b
2
2b
2
+ ca
+
c
2
2c
2
+ ab
+ 1 1 + 1 = 2.
Câu 2.10. Ta
3
3 ab
= 1 +
ab
a
2
+ b
2
+ c
2
ab
=
2ab
a
2
+ b
2
+ 2c
2
+ (a b)
2
1 +
(a + b)
2
2(a
2
+ b
2
+ 2c
2
)
1 +
1
2
a
2
a
2
+ c
2
+
b
2
b
2
+ c
2
.
Tương tự
3
3 bc
1 +
1
2
b
2
b
2
+ a
2
+
c
2
c
2
+ a
2
và
3
3 ca
1 +
1
2
c
2
c
2
+ b
2
+
a
2
a
2
+ b
2
.
Công ba bất đẳng thức trên theo vế ta đpcm.
111
2. BẤT ĐẲNG THỨC CAUCHY-SCHWARZ
Câu 2.11. Ta
V T =
4
x
2
+ y
2
+ 3yz
+
4
y
2
+ z
2
+ 3zx
+
4
z
2
+ x
2
+ 3xy
+
5
x
x
2
+ y
2
+ 3yz
+
5
y
y
2
+ z
2
+ 3zx
+
5
z
z
2
+ x
2
+ 3xy
Áp dụng bất đẳng thức Schwarz ta được:
V T
4.9
2(x
2
+ y
2
+ z
2
) + 3(xy + yz + zx)
+
5
1
x
+
1
y
+
1
z
2
2(x
2
+ y
2
+ z
2
) + 3(xy + yz + zx)
36
2(x
2
+ y
2
+ z
2
) + 3(xy + yz + zx)
+
45
2(x
2
+ y
2
+ z
2
) + 3(xy + yz + zx)
=
84
2(x
2
+ y
2
+ z
2
) + 3(xy + yz + zx)
162
x
2
+ y
2
+ z
2
+ 27
.
Vy bài toán được chứng minh.
Câu 2.12. Áp dụng bất đẳng thức Cauchy-Schwarz ta
a
b + 2c
+
b
c + 2a
+
c
a + 2b
=
a
2
a(b + 2c)
+
b
2
b(c + 2a)
+
c
2
c(a + 2b)
(a + b + c)
2
a(b + 2c) + b(c + 2a) + c(a + 2b)
=
(a + b + c)
2
3(ab + bc + ca)
1.
Do đó
V T
a
b + 2c
+
b
c + 2a
+
c
a + 2b
2
2(a + b + c)
1
2
.
Câu 2.13. Đặt x =
ab, y =
bc, z =
ca, ta
b
a
4b
c c
a
=
ba
4b
ca ca
=
x
2
4yz z
2
,
c
b
4c
a a
b
=
y
2
4xz x
2
,
a
c
4a
b b
c
=
z
2
4xy y
2
.
Áp dụng bất đẳng thức Cauchy-Schwarz ta
V T
(x + y + z)
2
4(xy + yz + zx) (x
2
+ y
2
+ z
2
)
3(xy + yz + zx)
4(xy + yz + zx) (xy + yz + zx)
= 1.
Câu 2.14. Đặt
1
a
= x,
1
b
= y,
1
c
= z ta x
2
+ y
2
+ z
2
1 và ta chứng minh bất đẳng thức
x
3
y
2
+ z
2
+
x
3
y
2
+ z
2
+
x
3
y
2
+ z
2
3
2
Ta có:
x
3
y
2
+ z
2
+
x
3
y
2
+ z
2
+
x
3
y
2
+ z
2
=
x
4
x (y
2
+ z
2
)
+
y
4
y (z
2
+ x
2
)
+
z
4
z (x
2
+ y
2
)
(x
2
+ y
2
+ z
2
)
2
z (y
2
+ z
2
) + y (z
2
+ x
2
) + z (x
2
+ y
2
)
112
2. BẤT ĐẲNG THỨC CAUCHY-SCHWARZ
Áp dụng bất đẳng thức AM-GM, ta
x
y
2
+ z
2
=
1
2
p
2x
2
(y
2
+ z
2
) (y
2
+ z
2
)
2
3
9
x
2
+ y
2
+ z
2
p
x
2
+ y
2
+ z
2
y
z
2
+ x
2
=
1
2
p
2y
2
(z
2
+ x
2
) (z
2
+ x
2
)
2
3
9
x
2
+ y
2
+ z
2
p
x
2
+ y
2
+ z
2
z
x
2
+ y
2
=
1
2
p
2z
2
(x
2
+ y
2
) (x
2
+ y
2
)
2
3
9
x
2
+ y
2
+ z
2
p
x
2
+ y
2
+ z
2
Suy ra
x
3
y
2
+ z
2
+
y
3
z
2
+ x
2
+
z
3
x
2
+ y
2
3
2
p
x
2
+ y
2
+ z
2
3
2
Vy bất đẳng thức được chứng minh. Đẳng thức xảy ra khi và chi chỉ a = b = c =
3.
Câu 2.15. Áp dụng bất đẳng thức Cauchy-Schwarz ta
(3S)
2
=
100
X
k=1
a
k+1
(a
2
k
+ 2a
k+1
a
k+2
)
!
2
100
X
k=1
a
2
k+1
!
100
X
k=1
a
2
k
+ 2a
k+1
a
k+2
2
!
= 1.
100
X
k=1
a
2
k
+ 2a
k+1
a
k+2
2
!
=
100
X
k=1
a
4
k
+ 4a
2
k
a
k+1
a
k+2
+ 4a
2
k+1
a
2
k+2
100
X
k=1
a
4
k
+ 2a
2
k
(a
2
k+1
+ a
2
k+2
) + 4a
2
k+1
a
2
k+2
!
=
100
X
k=1
a
4
k
+ 6a
2
k
a
2
k+1
+ 2a
2
k
a
2
k+2
.
Mặt khác
100
X
k=1
a
4
k
+ 2a
2
k
a
2
k+1
+ 2a
2
k
a
2
k+2
100
X
k=1
a
2
k
!
2
và
100
X
k=1
a
2
k
a
2
k+1
50
X
i=1
a
2
2i1
!
50
X
j=1
a
2
2j
!
,
nên ta
(3S)
2
100
X
k=1
a
2
k
!
2
+ 4
50
X
i=1
a
2
2i1
!
50
X
j=1
a
2
2j
!
1 +
50
X
i=1
a
2
2i1
+
50
X
j=1
a
2
2j
!
2
= 2.
Suy ra S
2
3
.
Câu 2.16. Bất đẳng thức cần chứng minh tương đương với
X
cyc
1
a(a + b + c) + 2(ab + bc + ca)
1
ab + bc + ca
X
cyc
2(ab + bc + ca)
a(a + b + c) + 2(ab + bc + ca)
2
113
2. BẤT ĐẲNG THỨC CAUCHY-SCHWARZ
Hay
X
cyc
a(a + b + c)
a(a + b + c) + 2(ab + bc + ca)
1. (4)
Ta
V T (4) = (a + b + c)
X
cyc
a
2
a
2
(a + b + c) + 2a(ab + bc + ca)
(a + b + c)
3
P
cyc
[a
2
(a + b + c) + 2a(ab + bc + ca)]
= 1
Vy bài toán được chứng minh.
Câu 2.17. Đặt S = a
r
3 +
b
2
c
2
+ b
r
3 +
c
2
a
2
+ c
r
3 +
a
2
b
2
.
Áp dụng bất đẳng thức Cauchy-Schwarz ta
(3 + 1)
3 +
b
2
c
2
3 +
b
c
2
Suy ra
a
r
3 +
b
2
c
2
1
2
3a +
ab
c
.
Do đó
S
1
2
3(a + b + c) +
ab
c
+
bc
a
+
ca
b
.
Lại
ab
c
+
bc
a
+
ca
b
a + b + c.
Nên ta S 2(a + b + c).
Đặt x = ab, y = bc, z = ca, khi đó bất đẳng thức
2
ab
c
+
bc
a
+
ca
b
S
2(x
2
+ y
2
+ z
2
) x
x
2
+ 3z
2
+ y
p
y
2
+ 3x
2
+ z
p
z
2
+ 3y
2
(4).
Áp dụng AM-GM ta
x
x
2
+ 3z
2
=
1
2
(2x)
x
2
+ 3z
2
1
4
(4x
2
+ x
2
+ 3z
2
) =
1
4
(5x
2
+ 3z
2
).
Tương tự cho hai bđt còn lại và công 3 bđt đó theo vế ta đpcm.
Câu 2.18. Đặt a = 1 + x
2
, b = 1 + y
2
, c = 1 + z
2
. Ta
x
x
2
+ 1
2
=
a
2
, y
b
2
, z
c
2
.
Do đó
V T 2
a
b + 2c
+
b
c + 2a
+
c
a + 2b
2
(a + b + c)
2
a(b + 2c) + b(c + 2a) + c(a + 2b)
1.
114
2. BẤT ĐẲNG THỨC CAUCHY-SCHWARZ
Câu 2.19. Theo bất đẳng thức Cauchy-Shwarz, ta
a
3
p
4(b
3
+ c
3
)
+
b
c + a
+
c
a + b
=
a
2
a
3
p
4(b
3
+ c
3
)
+
b
2
bc + ab
+
c
2
bc + ca
(a + b + c)
2
a
3
p
4(b
3
+ c
3
) + 2bc + a(b + c)
.
Từ đó, để chứng minh bất đẳng thức của bài ra, ta sẽ chứng minh
(a + b + c)
2
a
3
p
4(b
3
+ c
3
) + 2bc + a(b + c)
3
2
(1)
với mọi a, b, c > 0.
a, b, c > 0 nên
(1) 2a
2
+ a(b + c) + 2(b
2
bc + c
2
) 3a
3
p
4(b
3
+ c
3
). (2)
b
2
bc + c
2
> 0 nên áp dụng bất đẳng thức Cauchy cho 3 số thực dương 2a
2
, a(b + c) và
b
2
bc + c
2
, ta được
2a
2
+ a(b + c) + 2(b
2
bc + c
2
) 3
3
p
2a
2
· a(b + c) · 2(b
2
bc + c
2
) = 3a
3
p
4(b
3
+ c
3
).
Vy (2) được chứng minh; do đó (1) được chứng minh và thế bất đẳng thức của bài toán được
chứng minh. Dễ thấy, đẳng thức xảy khi và chỉ khi a = b = c.
Câu 2.20. hiệu P vế trái của bất đẳng thức cần chứng minh theo yêu cầu đề bài.
Nếu a = 0 thì từ các ràng buộc đối với a, b, c suy ra c > 0. Do đó P =
b
c
+ 2 > 1.
Nếu b = 0 thì từ các ràng buộc đối với a, b, c suy ra c > 0. Do đó
P =
a
c
+
2c
c + 2a
>
a
a + c
+
2c
2c + 2a
= 1.
Nếu c = 0 thì từ các ràng buộc đối với a, b, c suy ra a,b > 0.
Do đó P =
a
b
+
b
4a
2 ·
r
a
b
·
b
4a
= 1. Dấu bằng xảy ra khi và chỉ khi b = 2a.
Xét a,b,c > 0. Khi đó, theo bất đẳng thức Cauchy-Schwarz, ta
P =
4a
2
4ab + 4ac
+
b
2
bc + 4ab
+
4c
2
2c
2
+ 4ac
(2a + b + 2c)
2
8ab + 8ac + bc + 2c
2
. (1.1)
Tiếp theo, ta sẽ chứng minh
(2a + b + 2c)
2
8ab + 8ac + bc + 2c
2
> 1 (1.2)
Thật vy, ta
(1.2) 4a
2
+ b
2
+ 4c
2
+ 4ab + 8ac + abc > 8ab + 8ac + bc + 2c
2
(2a b)
2
+ (3b + 2c)c > 0.
Bất đẳng thức cuối cùng hiển nhiên đúng và thế (1.2) được chứng minh.
Từ (1.1) và (1.2), hiển nhiên suy ra P > 1.
Vy, tóm lại, bất đẳng thức đề bài được chứng minh. Dấu đẳng thức xảy ra khi và chỉ khi b = 2a
và c = 0.
Câu 2.21. Ta a + b + c
p
3(a
2
+ b
2
+ c
2
) = 3 Bất đẳng thức cần chứng minh tương đương
với
a
2 a
+
b
2 b
+
c
2 c
3. (1)
115
2. BẤT ĐẲNG THỨC CAUCHY-SCHWARZ
Ta
V T (1) =
a
2
a(2 a)
+
b
2
b(2 b)
+
c
2
c(2 c)
(a + b + c)
2
2(a + b + c) 3
.
Ta chứng minh
(a + b + c)
2
2(a + b + c) 3
3 (a + b + c 3)
2
0 (bđt luôn đúng). Đẳng thức xảy ra
khi và chỉ khi a = b = c = 1.
Câu 2.22. Để ý rằng
1
1 ab
= 2
1 2ab
1 ab
. Do đó bất đẳng thức cần chứng minh tương đương
với
1 2ab
1 ab
+
1 2bc
1 bc
+
1 2cd
1 cd
+
1 2da
1 da
8
3
.
1 2ab = (a b)
2
+ c
2
+ d
2
0 nên ta thể áp dụng bất đẳng thức Cauchy-Schwarz như
sau
1 2ab
1 ab
+
1 2bc
1 bc
+
1 2cd
1 cd
+
1 2da
1 da
[(1 2ab) + (1 2bc) + (1 2cd) + (1 2da)]
2
(1 2ab)(1 ab) + (1 2bc)(1 bc) + (1 2cd)(1 cd) + (1 2da)(1 da)
=
4[2 (a + c)(b + d)]
2
4 3(a + c)(b + d) + 2(a
2
+ c
2
)(b
2
+ d
2
)
.
Bài toán được quy v chứng minh
3 [2 (a + c)(b + d)]
2
2
4 3(a + c)(b + d) + 2(a
2
+ c
2
)(b
2
+ d
2
)
.
Bất đẳng thức y tương đương với
4 6(a + c)(b + d) + 3(a + c)
2
(b + d)
2
4(a
2
+ c
2
)(b
2
+ d
2
) 0,
hay
3 [1 (a + c)(b + d)]
2
+ 1 4(a
2
+ c
2
)(b
2
+ d
2
) 0.
Ta
1 4(a
2
+ c
2
)(b
2
+ d
2
) = (a
2
+ b
2
+ c
2
+ d
2
)
2
4(a
2
+ c
2
)(b
2
+ d
2
)
= (a
2
+ c
2
b
2
d
2
)
2
0.
Nên bất đẳng thức cuối đúng. Bài toán được chứng minh.
Câu 2.23. Từ giả thiết, ta thể đặt x =
2a
2
bc
,y =
2b
2
ca
và z =
2c
2
ab
với a,b,c > 0. Thay vào bất
đẳng thức đã cho, ta thể viết dưới dạng sau
a
4
a
4
+ a
2
bc + b
2
c
2
+
b
4
b
4
+ b
2
ca + c
2
a
2
+
c
4
c
4
+ c
2
ab + a
2
b
2
1.
Sử dụng bất đẳng thức Cauchy- Schwarz, ta dễ thấy
V T
(a
2
+ b
2
+ c
2
)
2
(a
4
+ b
4
+ c
4
) + abc(a + b + c) + (a
2
b
2
+ b
2
c
2
+ c
2
a
2
)
,
suy ra ta chỉ cần chứng minh bất đẳng thức sau nữa đủ
a
2
+ b
2
+ c
2
2
a
4
+ b
4
+ c
4
+ abc (a + b + c) + a
2
b
2
+ b
2
c
2
+ c
2
a
2
a
2
b
2
+ b
2
c
2
+ c
2
a
2
abc(a + b + c)
(ab ac)
2
+ (bc ba)
2
+ (ca cb)
2
0.
Bài toán được chứng minh xong.
116
2. BẤT ĐẲNG THỨC CAUCHY-SCHWARZ
Câu 2.24. Từ giả thiết, ta thể đặt x =
2a
2
bc
,y =
2b
2
ca
và z =
2c
2
ab
với a,b,c > 0. Thay vào bất
đẳng thức đã cho, ta thể viết dưới dạng sau
a
4
a
4
+ a
2
bc + b
2
c
2
+
b
4
b
4
+ b
2
ca + c
2
a
2
+
c
4
c
4
+ c
2
ab + a
2
b
2
1.
Sử dụng bất đẳng thức Cauchy Schwarz, ta dễ thấy
V T
(a
2
+ b
2
+ c
2
)
2
(a
4
+ b
4
+ c
4
) + abc(a + b + c) + (a
2
b
2
+ b
2
c
2
+ c
2
a
2
)
,
suy ra ta chỉ cần chứng minh bất đẳng thức sau nữa đủ
a
2
+ b
2
+ c
2
2
a
4
+ b
4
+ c
4
+ abc (a + b + c) + a
2
b
2
+ b
2
c
2
+ c
2
a
2
,
hay
a
2
b
2
+ b
2
c
2
+ c
2
a
2
abc(a + b + c) (ab ac)
2
+ (bc ba)
2
+ (ca cb)
2
0.
Bài toán được chứng minh xong.
Câu 2.25. Từ xyz = 1 x =
a
2
bc
,y =
b
2
ca
,z =
c
2
ab
. Bất đẳng thức cần chứng minh tương đương
với
a
4
(a
2
bc)
2
+
b
4
(b
2
ca)
2
+
c
4
(c
2
ab)
2
1. (1)
Áp dụng bất đẳng thức Schwarz ta có:
V T (1)
(a
2
+ b
2
+ c
2
)
2
(a
2
bc)
2
+ (b
2
ca)
2
+ (c
2
ab)
2
.
Ta chứng minh:
(a
2
+ b
2
+ c
2
)
2
(a
2
bc)
2
+
b
2
ca
2
+
c
2
ab
2
(ab + bc + ca)
2
0.
Vy bài toán được chứng minh.
Câu 2.26. Đặt a =
1
x
; b =
1
y
; c =
1
z
x ; y ; z > 0 . Bất đẳng thức cần chứng minh tương
đương với bất đẳng thức sau
x
p
z (3x + y)
+
y
p
x (3y + z)
+
z
p
y (3z + x)
3
2
.
Đặt vế trái P , sử dụng bất đẳng thức C - S ta
P =
x
2
x.
p
z (3x + y)
+
y
2
y.
p
x (3y + z)
+
z
2
z.
p
y (3z + x)
(x + y + z)
2
x
p
z (3x + y) + y
p
x (3y + z) + z
p
y (3z + x)
Đặt Q = x
p
z (3x + y) + y
p
x (3y + z) + z
p
y (3z + x) Sử dụng bất đẳng thức Cauchy - Schwarz
ta
Q =
x.
p
xz (3x + y) +
y.
p
yx (3y + z) +
z.
p
zy (3z + x)
p
(x + y + z) [xz (3x + y) + xy (3y + z) + zy (3z + x)]
=
p
3 (x + y + z) (x
2
z + y
2
x + z
2
y + xyz)
117
2. BẤT ĐẲNG THỨC CAUCHY-SCHWARZ
Mặt khác, ta chứng minh được: x
2
z + y
2
x + z
2
y + xyz
4
27
(x + y + z)
3
Thật vy, không mất
tính tổng quát ta giả sử y số nằm giữa x và z. Khi đó
(y x) (y z) 0 y
2
+ xz xy + yz y
2
x + x
2
z x
2
y + xyz
x
2
z + y
2
x + z
2
y + xyz x
2
y + z
2
y + 2xyz = y (x + z)
2
= 4.y.
x + z
2
.
x + z
2
4.
y +
x + z
2
+
x + z
2
3
3
=
4
27
(x + y + z)
3
Do đó Q
2
3
(x + y + z)
2
P
3
2
(đpcm). Đẳng thức xảy ra khi x = y = z.
Câu 2.27. Khi thay a, b, c bởi a, b, c thì bất đẳng thức không đổi, do đó ta giả sử
a 0 b, c. Khi đó ta thay a bởi a ta cần chứng minh BĐT
(a
2
ab + b
2
)(b
2
+ bc + c
2
)(c
2
ca + a
2
) 3(bc ab ac)
3
. (1)
với a, b, c 0.
Ta chỉ xét bc ca ab 0. Do
b
2
+ bc + c
2
(bc ac ab) = b
2
+ c
2
+ a(b + c) 0,
nên ta chứng minh
(a
2
ab + b
2
)(c
2
ca + a
2
) 3(bc ab ac)
2
. (2)
Ta
4(a
2
ab + b
2
) · 4(c
2
ca + a
2
) =
(a + b)
2
+ 3(b c)
2
·
(c + a)
2
+ 3(c a)
2
[(a + b)(a + c) + 3(a c)(a b)]
2
.
Ta chứng minh
(a + b)(a + c) + 3(a b)(a c) 4(bc ab ac).
Bất đẳng thức y đúng tương đương với
a(2a + b + c) 0.
Câu 2.28. Áp dụng bất đẳng thức Cauchy-Schwarz ta
V T =
X
a
4
a
2
(2b
2
bc + 2c
2
)
(a
2
+ b
2
+ c
2
)
2
P
a
2
(2b
2
bc + 2c
2
)
.
Ta chứng minh
(a
2
+ b
2
+ c
2
)
2
X
a
2
(2b
2
bc + 2c
2
),
hay
X
a
4
+ abc
X
a 2
X
a
2
b
2
. (1)
Áp dụng bất đẳng thức Schur ta
X
a
4
+ abc
X
a
X
ab(a
2
+ b
2
) 2
X
a
2
b
2
,
nên (1) đúng.
118
2. BẤT ĐẲNG THỨC CAUCHY-SCHWARZ
Câu 2.29. Áp dụng bất đẳng thức Cauchy-Schwarz, ta
P
2
=
a(2 bc) +
2.
2(b + c)
2
(a
2
+ 2)
(2 bc)
2
+ 2(b + c)
2
= (a
2
+ 2)(b
2
+ 2)(c
2
+ 2).
Lại áp dụng bất đẳng thức AM-GM, ta
(a
2
+ 2)(b
2
+ 2)(c
2
+ 2) =
1
6
3(a
2
+ 2).2(b
2
+ 2).(c
2
+ 2)
1
6
3(a
2
+ 2) + 2(b
2
+ 2) + (c
2
+ 2)
3
3
= 36.
Từ đó suy ra P
2
36 . Suy ra 6 P 6. Mặt khác với a = 0, b = 1, c = 2 thì P = 6;
a = 0, b = 1, c = 2 thì P = 6.
Vy P
max
= 6 và P
min
= 6.
Câu 2.30. Đặt P =
a + b
(a + b + 1)
2
+
b + c
(b + c + 1)
2
+
c + a
(c + a + 1)
2
. Áp dụng bất đẳng thức Cauchy
- Schwarz ta
2(a + b + c)P
a + b
a + b + 1
+
b + c
b + c + 1
+
c + a
c + a + 1
2
.
Tiếp theo ta chứng minh
a + b
a + b + 1
+
b + c
b + c + 1
+
c + a
c + a + 1
2,
hay
1
a + b + 1
+
1
b + c + 1
+
1
c + a + 1
1.
Bất đẳng thức y đúng do
1
a
3
+ b
3
+ abc
+
1
b
3
+ c
3
+ abc
+
1
c
3
+ a
3
+ abc
1
abc
.
Câu 2.31. Áp dụng bất đẳng thức Cauchy-Schwarz ta
(x
4
+ y)(
z
2
y
+ 1) (x
2
+ z)
2
(x
4
+ y)(z
2
+ y) y(x
2
+ z)
2
.
Tương tự:
(y
4
+ z)(x
2
+ z) z(y
2
+ x)
2
(x
4
+ y)(z
2
+ y) y(x
2
+ z)
2
(z
4
+ x)(y
2
+ x) x(z
2
+ y)
2
.
Nhân các bất đẳng thức trên theo vế ta được
(x
4
+ y)(y
4
+ z)(z
4
+ x) xyz(x + y
2
)(y + z
2
)(z + x
2
) (x + y
2
)(y + z
2
)(z + x
2
).
Câu 2.32. Áp dụng bất đẳng thức Cauchy-Schwarz ta
V T
s
a
3a + b
+
b
3b + c
+
c
3c + a
· (a + b + c).
119
2. BẤT ĐẲNG THỨC CAUCHY-SCHWARZ
Ta chứng minh
a
3a + b
+
b
3b + c
+
c
3c + a
1
2
X
cyc
b
3a + b
3
4
.
Ta
X
cyc
b
3a + b
=
X
cyc
b
2
3ab + b
2
(a + b + c)
2
a
2
+ b
2
+ c
2
+ 3(ab + bc + ca)
.
Do đó, ta chứng minh
(a + b + c)
2
a
2
+ b
2
+ c
2
+ 3(ab + bc + ca)
3
4
a
2
+ b
2
+ c
2
ab + bc + ca (đúng).
Câu 2.33. Từ giả thiết
(1 b
2
)(1 c
2
) (a bc)
2
.
Bất đẳng thức cần chứng minh tương đương với
(1 b
6
)(1 c
6
) (a
3
b
3
c
3
)
2
,
hay
(1 b
2
)(1 c
2
)(1 + b
2
+ b
4
)(1 + c
2
+ c
4
) (a bc)
2
(a
2
+ abc + b
2
c
2
)
2
.
Như vy cần chứng minh
(1 + b
2
+ b
4
)(1 + c
2
+ c
4
) (a
2
+ abc + b
2
c
2
)
2
Áp dụng bất đẳng thức Cauchy-Schwarz, ta
(1 + b
2
+ b
4
)(1 + c
2
+ c
4
) (1 + |bc| + b
2
c
2
)
2
(a
2
+ |abc| + b
2
c
2
)
2
(a
2
+ abc + b
2
c
2
)
2
.
Do 1 a 1. Suy ra điều phải chứng minh. Dấu bằng xảy ra khi a = b = c = 1.
Câu 2.34. Ta
(a
2
+ b
2
+ c
2
)
1
a
2
+
1
b
2
+
1
c
2
1
2
a
b
+
b
c
+
c
a
2
+
b
a
+
c
b
+
a
c
2
!
1
4
a
b
+
b
c
+
c
a
+
b
a
+
c
b
+
a
c
2
6
4
a
b
+
b
c
+
c
a
+
b
a
+
c
b
+
a
c
=
3
2
b + c
a
+
c + a
b
+
a + b
c
Suy ra
2
1
a
2
+
1
b
2
+
1
c
2
b + c
a
+
c + a
b
+
a + b
c
.
120
2. BẤT ĐẲNG THỨC CAUCHY-SCHWARZ
Câu 2.35. Bất đẳng thức cần chứng minh tương đương với
n1
X
k=1
(n 2)x
2
1
+ 2x
1
(n 1)x
2
1
+ 1
+
1
n 1
1
(n 2)
n1
P
k=1
x
k
2
2
n1
P
k=1
x
k
(n 1)
n1
P
k=1
x
k
2
+ 1
n1
X
k=1
((n 1)x
k
+ 1)
2
(n 1)((n 1)x
2
k
+ 1)
n1
P
k=1
x
k
+ 1
2
(n 1)
n1
P
k=1
x
k
2
+ 1
.
Áp dụng bất đẳng thức Cauchy-Schwarz ta
n1
X
k=1
((n 1)x
k
+ 1)
2
(n 1)((n 1)x
2
k
+ 1)
n1
P
k=1
x
k
+ 1
2
n1
P
k=1
x
2
k
+ 1
.
Nên ta chứng minh
n1
P
k=1
x
k
+ 1
2
n1
P
k=1
x
2
k
+ 1
n1
P
k=1
x
k
+ 1
2
(n 1)
n1
P
k=1
x
k
2
+ 1
,
hay
(n 1)
n1
X
k=1
x
k
!
2
n1
X
k=1
x
2
k
. (1)
Câu 2.36. Áp dụng bđt Cauchy-Schwarz ta
1
2a
2
+ bc
+
1
2b
2
+ ac
+
1
2c
2
+ ab
(a + b + c)
2
ab + bc + ac
4(a + b + c)
2
P
(2a
2
+ bc)(b
2
+ c
2
+ 2bc)
.
Ta chứng minh
X
(2a
2
+ bc)(b
2
+ c
2
+ 2bc) 4(a
2
+ b
2
+ c
2
)(ab + bc + ca),
hay
X
ab(a
2
+ b
2
) 2
X
a
2
b
2
.
Bất đẳng thức y đúng theo AM-GM
Câu 2.37. Áp dụng bất đẳng thức AM GM ta P 3 ·
3n
s
a
2
+ bc
a(b + c)
·
b
2
+ ac
b(a + c)
·
c
2
+ ab
c(a + b)
.
(1)
Ta chứng minh
a
2
+ bc
a(b + c)
·
b
2
+ ac
b(a + c)
·
c
2
+ ab
c(a + b)
1. (2)
Thật vy: Do a, b, c dương nên bất đẳng thức (1) đưa v
(a
2
+ bc)(b
2
+ ac)(c
2
+ ab) abc(a + b)(b + c)(c + a) (3).
121
2. BẤT ĐẲNG THỨC CAUCHY-SCHWARZ
Áp dụng bất đẳng thức Cauchy - Schwarz cho 2 b 2 số
a,
bc
và
b,
bc
ta được
(a
2
+ bc)(b
2
+ bc) (ab + bc)
2
hay b(a
2
+ bc)(b + c) b
2
(a + c)
2
. (4)
Chứng minh tương tự ta c(b
2
+ ac)(a + c) c
2
(a + b)
2
(5)
a(c
2
+ ab)(a + b) a
2
(c + b)
2
(6)
các vế của (4), (5), (6) đều dương. Nhân vế với vế các bất đẳng thức đó với nhau, rồi chia cả
2 vế của bất đẳng thức thu được cho abc(a + b)(b + c)(c + a) ta được (3), do đó (2), suy ra
P 3.
Dấu bằng xảy ra khi và chỉ khi dấu bằng xảy ra đồng thời (1), (4), (5) và (6), khi đó a = b = c.
Vy giá trị nhỏ nhất của P 3.
Câu 2.38. a
3
+ 5 = (a
3
+ 1 + 1) + 3 3
3
a
3
· 1 · 1 + 3 = 3a + 3.
Tương tự: b
3
+ 5 3b + 3; c
3
+ 5 3c + 3.
a
3
+ 5
a
3
(b + c)
+
b
3
+ 5
b
3
(c + a)
+
c
3
+ 5
c
3
(a + b)
3a + 3
a
3
(b + c)
+
3b + 3
b
3
(c + a)
+
3c + 3
c
3
(a + b)
Ta có:
3a + 3
a
3
(b + c)
+
3b + 3
b
3
(c + a)
+
3c + 3
c
3
(a + b)
=
3a(abc) + 3(abc)
2
a
3
(b + c)
+
3b(abc) + 3(abc)
2
b
3
(c + a)
+
3c(abc) + 3(abc)
2
c
3
(a + b)
=
3(bc + b
2
c
2
)
ab + bc
+
3(ca + c
2
a
2
)
bc + ca
+
3(ab + a
2
b
2
)
ca + ab
Đặt x = bc, y = ca, z = ab; x,y,z > 0, xyz = 1.
3a + 3
a
3
(b + c)
+
3b + 3
b
3
(c + a)
+
3c + 3
c
3
(a + b)
3
x + x
2
y + z
+
y + y
2
z + x
+
z + z
2
x + y
= 3
x
y + z
+
y
z + x
+
z
x + y
+ 3
x
2
y + z
+
y
2
z + x
+
z
2
z + y
x
y + z
+
y
z + x
+
z
x + y
3
2
x
2
y + z
+
y
2
z + x
+
z
2
z + y
x + y + z
2
3
3
xyz
2
=
3
2
.
Vy
a
3
+ 5
a
3
(b + c)
+
b
3
+ 5
b
3
(c + a)
+
c
3
+ 5
c
3
(a + b)
9.
Dấu = xảy ra khi a = b = c = 1.
Câu 2.39. Biến đổi và áp dụng bất đẳng thức Cauchy - Schwartz:
n
X
k=1
a
k
(b
k
+ a
k+1
) =
n
X
k=1
a
k
b
k
+
n
X
k=1
a
k
a
k+1
v
u
u
t
n
X
k=1
a
2
k
!
n
X
k=1
b
2
k
!
+
n
X
k=1
a
k
a
k+1
=
v
u
u
t
n
X
k=1
a
2
k
+
n
X
k=1
a
k
a
k+1
=
v
u
u
t
n
X
k=1
a
k
!
2
2
X
1i<jn
a
i
a
j
+
n
X
k=1
a
k
a
k+1
122
2. BẤT ĐẲNG THỨC CAUCHY-SCHWARZ
=
s
1 2
X
1i<jn
a
i
a
j
+
n
X
k=1
a
k
a
k+1
. (1)
Đặt x =
X
1i<jn
a
i
a
j
và y =
n
X
k=1
a
k
a
k+1
ta ngay x > y. (2)
Đồng thời cũng
1 =
n
X
k=1
a
k
!
2
=
n
X
k=1
a
2
k
+ 2x > y y < 1.
Do (1) nên ta cần chứng minh:
1 2x + y < 1 1 2x < 1 2y + y
2
2(y x) < y
2
(hiển nhiên đúng).
123
3. MỘT SỐ BẤT ĐẲNG THỨC KHÁC
§3. Một số bất đẳng thức khác
Câu 3.1. Ta
2a
b + c
+
2b
c + a
+
2c
a + b
+
3
r
3abc
a
3
+ b
3
+ c
3
(a + b + c)
2
ab + bc + ca
+
9abc
3
3
q
(3abc)
2
(a
3
+ b
3
+ c
3
)
(a + b + c)
2
ab + bc + ca
+
9abc
a
3
+ b
3
+ c
3
+ 6abc
.
Ta chỉ cần chứng minh
(a + b + c)
2
ab + bc + ca
3 1
9abc
a
3
+ b
3
+ c
3
+ 6abc
1
ab + bc + ca
a + b + c
a
3
+ b
3
+ c
3
+ 6abc
.
Bất đẳng thức y hiển nhiên đúng theo Schur.
Câu 3.2. a + b + c = 1, nên ta
ab
a + b
+
bc
b + c
+
ca
c + a
= (a + b + c)
ab
a + b
+
bc
b + c
+
ca
c + a
= ab + bc + ca + abc
1
a + b
+
1
b + c
+
1
c + a
ab + bc + ca +
9abc
2(a + b + c)
= ab + bc + ca +
9
2
abc.
Suy ra
2
ab
a + b
+
bc
b + c
+
ca
c + a
+ 1 2(ab + bc + ca) + 9abc + 1.
Do đó, ta chứng minh
9abc + 1 4(ab + bc + ca). (1)
Áp dụng bất đẳng thức Schur ta
a
2
+ b
2
+ c
2
+
9abc
a + b + c
2(ab + bc + ca).
Suy ra
1 + 9abc 4(ab + bc + ca),
hay (1) được chứng minh.
Câu 3.3. Bất đẳng thức cần chứng minh tương đương với
(a + b + c)
X
cyc
3
p
(a
2
+ bc)(a
2
+ b
2
)(a
2
+ c
2
) 9
3
p
(a
2
+ b
2
)(a
2
+ c
2
)(b
2
+ c
2
)abc.
Áp dụng bất đẳng thức Cauchy-Schwarz ta
(a
2
+ b
2
)(a
2
+ c
2
) (a
2
+ bc)
2
,
124
3. MỘT SỐ BẤT ĐẲNG THỨC KHÁC
và theo AM-GM
9
3
p
(a
2
+ b
2
)(a
2
+ c
2
)(b
2
+ c
2
)abc 3
X
cyc
c(a
2
+ b
2
).
Do đó, ta chứng minh
(a + b + c)
X
cyc
(a
2
+ bc) 3
X
cyc
c(a
2
+ b
2
),
Đây chính bđt Schur.
Câu 3.4. Ta
V T = a
2
b
2
c
2
+ 2(a
2
b
2
+ b
2
c
2
+ c
2
a
2
) + 4(a
2
+ b
2
+ c
2
) + 8.
Mặt khác
a
2
b
2
+ b
2
c
2
+ c
2
a
2
+ 3 = a
2
b
2
+ 1 + b
2
c
2
+ 1 + c
2
a
2
+ 1 2(ab + bc + ca)
và
a
2
b
2
c
2
+ 2 = a
2
b
2
c
2
+ 1 + 1 3
3
a
2
b
2
c
2
=
3abc
3
abc
9abc
a + b + c
2(ab + bc + ca) (a
2
+ b
2
+ c
2
).
Suy ra
V T 2(ab + bc + ca) (a
2
+ b
2
+ c
2
) + 2.2(ab + bc + ca) + 4(a
2
+ b
2
+ c
2
)
= 6(ab + bc + ca) + 3(a
2
+ b
2
+ c
2
) 6(ab + bc + ca) + 3(ab + bc + ca) 9(ab + bc + ca).
Bài toán được chứng minh.
Câu 3.5. Gọi P vế trái của bất đẳng thức cần chứng minh. Không mất tính tổng quát, ta
giả sử a + b + c = 3. Áp dụng bất đẳng thức AM-GM ta
s
(a + b)
3
8ab(4a + 4b + c)
+
s
(a + b)
3
8ab(4a + 4b + c)
+
ab(4a + 4b + c)
27
1
2
(a + b)
Suy ra
s
(a + b)
3
8ab(4a + 4b + c)
+
ab(4a + 4b + c)
54
1
4
(a + b).
Tương tự
s
(b + c)
3
8bc(4b + 4c + a)
+
bc(4b + 4c + a)
54
1
4
(b + c)
và
s
(c + a)
3
8ca(4c + 4a + b)
+
ca(4c + 4a + b)
54
1
4
(c + a).
Cộng ba bất đẳng thức trên ta
1
2
2
P + A B.
Với
A =
1
54
[ab(4a + 4b + c) + bc(4b + 4c + a) + ca(4c + 4a + b)]
=
1
54
[4ab(a + b) + 4bc(b + c) + 4ca(c + a) + 3abc]
=
1
54
[4(a + b + c)(ab + bc + ca) 9abc]
1
54
(a + b + c)
3
=
1
2
.
125
3. MỘT SỐ BẤT ĐẲNG THỨC KHÁC
và
B =
1
4
.2 (a + b + c) =
3
2
.
Suy ra
1
2
2
P
3
2
1
2
= 1 P 2
2.
Bài toán được chứng minh.
Câu 3.6. Sử dụng bất đẳng thức AM-GM,ta có:
12(a
2
+ b
2
+ c
2
) + 6abc + 48 30(a + b + c)
= 12(a
2
+ b
2
+ c
2
) + 3(2abc + 1) + 45 5.2.3(a + b + c)
12(a
2
+ b
2
+ c
2
) + 9
3
a
2
b
2
c
2
+ 45 5.((a + b + c)
2
+ 9)
= 7(a
2
+ b
2
+ c
2
) +
9abc
3
abc
10(ab + bc + ca)
7(a
2
+ b
2
+ c
2
) +
27
a + b + c
10(ab + bc + ca)
Mặt khác sử dụng bất đẳng thức Schur,
9
a + b + c
4(ab + bc + ca) (a + b + c)
2
= 2(ab + bc + ca) (a
2
+ b
2
+ c
2
)
Do đó
7(a
2
+ b
2
+ c
2
) +
27
a + b + c
10(ab + bc + ca)
7(a
2
+ b
2
+ c
2
) + 6(ab + bc + ca) 3(a
2
+ b
2
+ c
2
) 10(ab + bc + ca)
= 4(a
2
+ b
2
+ c
2
ab bc ca) 0.
Bất đẳng thức được chứng minh.
Câu 3.7. Đặt x =
1
a
, y =
1
b
, z =
1
c
. Bất đẳng thức cần chứng minh trở thành
x
2
+ y
2
+ z
2
+ 3 2(xy + yz + zx)
(x + y + z)(x
2
+ y
2
+ z
2
+ 3) 2(x + y + z)(xy + yz + zx)
Hay
x
3
+ y
3
+ z
3
+ 3(x + y + z) x
2
(y + z) + y
2
(z + x) + z
2
(x + y) + 6 (1).
Ta
x
3
+ y
3
+ z
3
+ 3(x + y + z) x
3
+ y
3
+ z
3
+ 9 = x
3
+ y
3
+ z
3
+ 3xyz + 6 V P (1).
Vy bài toán được chứng minh.
Câu 3.8. Đặt x =
3
a
2
, y =
3
b
2
, z =
3
c
2
, ta xyz = 1 và bất đẳng thức cần chứng minh trở
thành
x
3
+ y
3
+ z
3
+ 3xyz 2
xy
xy + yz
yz + zx
zx
. (1)
Áp dụng bất đăng thức Schur ta
x
3
+ y
3
+ z
3
+ 3xyz xy(x + y) + yz(y + z) + zx(z + x)
2xy
xy + 2yz
yz + 2zx
zx.
Suy ra (1) đúng, hay bài toán được chứng minh.
126
3. MỘT SỐ BẤT ĐẲNG THỨC KHÁC
Câu 3.9. Áp dụng bất đẳng thức Holder với chú ý
1
2
+
1
3
+
1
6
= 1 ta
4a
3
+ 9b
3
+ 36c
3
1
2
+
1
3
+
1
6
2
(a + b + c)
3
= 1.
Từ đó ta đpcm.
Câu 3.10. Đặt P vế trái của bất đẳng thức. Áp dụng bất đẳng thức Holder ta có:
P
3
X
a(a + 2b)
(a + b + c)
4
.
Mặt khác
X
a(a + 2b) = (a + b + c)
2
,
nên ta
P
3
(a + b + c)
2
= 1 P 1.
Câu 3.11. Áp dụng bất đẳng thức Holder ta
(x
2
+ y
2
+ z
2
)
a
x
+
b
y
+
c
z
2
3
a
2
+
3
b
2
+
3
c
2
3
.
Từ đó ta đpcm.
Câu 3.12. Áp dụng bất đẳng thức Holder ta
(x
n
+ y
n
+ z
n
)
n1
a
n
+
n1
b
n
+
n1
c
n
n1
(ax + by + cz)
n
= 1.
Suy ra
x
n
+ y
n
+ z
n
n1
a
n
+
n1
b
n
+
n1
c
n
1n
.
Câu 3.13.
(a + b + c)
1
a
+
1
b
+
1
c
+ 4
2
ab + bc + ca
a
2
+ b
2
+ c
2
9 + 4
2
(a + b + c)
1
a
+
1
b
+
1
c
9 4
2 4
2
ab + bc + ca
a
2
+ b
2
+ c
2
(a b)
2
ab
+
(b c)
2
bc
+
(c a)
2
ca
2
2
(a b)
2
+ (a c)
2
+ (b c)
2
a
2
+ b
2
+ c
2
. (*)
Không mất tính tổng quát giả sử a b c
(a b)
2
b
=
(a c)
2
c
Áp dụng bất đẳng thức
Chebyshev ta có:
(a b)
2
b
+
(a c)
2
c
(b + c) 2
(a b)
2
+ (a c)
2
(a b)
2
ab
+
(a c)
2
ac
2 [(a b)
2
+ (a c)
2
]
a(b + c)
. (1)
Đẳng thức xảy ra khi và chỉ khi b = c.
Mặt khác theo bất đẳng thức AM GM:
2(a
2
+ b
2
+ c
2
) 2a
2
+ (b + c)
2
2
2a(b + c)
127
3. MỘT SỐ BẤT ĐẲNG THỨC KHÁC
2
a(b + c)
2
2
a
2
+ b
2
+ c
2
2 [(a b)
2
+ (a c)
2
]
a(b + c)
2
2
(a b)
2
+ (a c)
2
a
2
+ b
2
+ c
2
. (2)
Đẳng thức xảy ra khi và chỉ khi a = b = c hoặc a
2 = b + c.
a
2
+ b
2
+ c
2
3bc > 2
2bc (do a b c)
(b c)
2
bc
2
2
(b c)
2
a
2
+ b
2
+ c
2
. (3)
Từ (1),(2),(3) suy ra bất đẳng thức (*) đúng. Suy ra điều phải chứng minh.
128
Chương 2
Một số phương pháp chứng minh bất
đẳng thức
§1. Phương pháp quy nạp
129
2. PHƯƠNG PHÁP PHÂN TÍCH BÌNH PHƯƠNG SOS
§2. Phương pháp phân tích bình phương SOS
130
2. PHƯƠNG PHÁP PHÂN TÍCH BÌNH PHƯƠNG SOS
Câu 5.1. Bất đẳng thức cần chứng minh tương đương với
2
a
3
+ b
3
+ c
3
ab (a + b) bc (b + c) ca (c + a)
a
3
+ b
3
+ c
3
3abc
0
(a + b) (a b)
2
+ (b + c) (b c)
2
+ (c + a) (c a)
2
1
2
(a + b + c)
(a b)
2
+ (b c)
2
+ (c a)
2
0
(a + b c) (a b)
2
+ (b + c a) (b c)
2
+ (c + a b) (c a)
2
0
S
c
(a b)
2
+ S
a
(b c)
2
+ S
b
(c a)
2
0.
Không mất tính tổng quát giả sử a b c S
b
0,S
c
0 Ta S
a
+ S
b
= 2c 0
Câu 5.2. Bất đẳng thức cần chứng minh tương đương với
ab bc + ca
b
2
+ c
2
1
2
+
bc ca + ab
c
2
+ a
2
1
2
+
ca ab + bc
a
2
+ b
2
1
2
0
X
(b + c)(2a b c)
b
2
+ c
2
0
X
(b + c)(a b)
b
2
+ c
2
+
X
(b + c)(a c)
b
2
+ c
2
0
X
(b + c)(a b)
b
2
+ c
2
+
X
(c + a)(b a)
c
2
+ a
2
0
X
(a b)
2
(ab + bc + ca c
2
)
(b
2
+ c
2
)(c
2
+ a
2
)
0. (1)
Mặt khác
ab + bc + ca c
2
= (b c)(c a) + 2bc (b c)(c a),
nên để chứng minh (1) ta chứng minh
X
(a b)
2
(b c)(c a)
(b
2
+ c
2
)(c
2
+ a
2
)
0
X
(a
2
+ b
2
)(a b)
2
(b c)(c a) 0
(a b)
2
(b c)
2
(c a)
2
0.
Bất đẳng thức cuối hiển nhiên đúng.
Câu 5.3. BĐT cần chứng minh tương đương với
2
a
3
+ b
3
+ c
3
ab (a + b) bc (b + c) ca (c + a)
a
3
+ b
3
+ c
3
3abc
ab
p
2 (a
2
+ b
2
) a b
+ bc
p
2 (b
2
+ c
2
) b c
+ ca
p
2 (c
2
+ a
2
) c a
S
c
(a b)
2
+ S
a
(b c)
2
+ S
b
(c a)
2
0,
với
S
c
= a + b c
2ab
p
2 (a
2
+ b
2
) + a + b
,
S
a
= b + c a
2bc
p
2 (b
2
+ c
2
) + b + c
,
S
b
= c + a b
2ca
p
2 (c
2
+ a
2
) + c + a
.
Do a, b, c bình đẳng, không mất tính tổng quát giả sử a b c.
Ta
S
c
a + b c
2ab
4b
=
a
2
+ b c 0, S
b
c + a b
ca
a + c
0
131
2. PHƯƠNG PHÁP PHÂN TÍCH BÌNH PHƯƠNG SOS
Ta cần chứng minh
S
b
+ S
a
= 2c
1
a
p
2 (c
2
+ a
2
) + c + a
b
p
2 (b
2
+ c
2
) + b + c
!
2b
1
a
2 (a + c)
b
2 (b + c)
=
bc
a + c
+
bc
b + c
0.
Câu 5.4. Bất đẳng thức cần chứng minh tương đương với
4a
2
+ 2bc 3 (b
2
+ c
2
)
b
2
+ c
2
+
4b
2
+ 2ca 3 (c
2
+ a
2
)
c
2
+ a
2
+
4c
2
+ 2ab 3 (a
2
+ b
2
)
a
2
+ b
2
0
2 (a
2
b
2
) + 2 (a
2
c
2
) (b c)
2
b
2
+ c
2
+
2 (b
2
c
2
) + 2 (b
2
a
2
) (c a)
2
c
2
+ a
2
+
2 (c
2
a
2
) + 2 (c
2
b
2
) (a b)
2
a
2
+ b
2
0
S
c
(a b)
2
+ S
a
(b c)
2
+ S
b
(c a)
2
0,
với
S
c
=
2(a + b)
2
(b
2
+ c
2
) (c
2
+ a
2
)
1
a
2
+ b
2
,
S
b
=
2(c + a)
2
(b
2
+ c
2
) (b
2
+ a
2
)
1
c
2
+ a
2
,
S
a
=
2(b + c)
2
(a
2
+ b
2
) (a
2
+ c
2
)
1
b
2
+ c
2
.
Giả sử a b c suy ra S
b
0,S
c
0, S
c
S
b
S
a
Ta cần chứng minh S
a
+ S
b
0, hay
2 (c + a)
2
c
2
+ a
2
+ 2 (b + c)
2
b
2
+ c
2
b
2
+ c
2
b
2
+ a
2
+
a
2
+ b
2
a
2
+ c
2
Ta thấy hệ số c
2
vế trái 2 (c + a)
2
+ 2 (b + c)
2
lớn hơn hoặc bằng vế phải 2 (a
2
+ b
2
) nên
ta chỉ cần chứng minh khi c = 0.
Hay
2
a
4
+ b
4
a
2
+ b
2
2
a
2
b
2
2
0 (đúng).
Câu 5.5. Ta : BĐT cần chứng minh ơng đương với
4
a
3
+ b
3
+ c
3
3abc
+ 6
ab
2
+ bc
2
+ ca
2
a
3
b
3
c
3
3
a
2
b + b
2
c + c
2
a a
3
b
3
c
3
2 (a + b + c)
(a b)
2
+ (b c)
2
+ (c a)
2
2
(2b + a) (a b)
2
+ (2c + b) (b c)
2
+ (2a + c) (c a)
2
+ (2a + b) (a b)
2
+ (2b + c) (b c)
2
+ (2c + a) (c a)
2
0
(2a b + 2c) (a b)
2
+ (2b c + 2a) (b c)
2
+ (2c a + 2b) (c a)
2
0
S
c
(a b)
2
+ S
a
(b c)
2
+ S
b
(c a)
2
0.
Nếu a b c S
a
0,S
c
0, mặt khác
S
a
+ 2S
b
0,S
c
+ 2S
b
0.
Theo tiêu chuẩn 3 suy ra ĐPCM.
Nếu a b c S
b
0,S
c
0.
Theo tiêu chuẩn 2 ta cần chứng minh S
a
+ S
b
0 ( hiển nhiên ) Suy ra điều phải chứng minh.
132
2. PHƯƠNG PHÁP PHÂN TÍCH BÌNH PHƯƠNG SOS
Câu 5.6. Trước hết ta thy rằng :
(x + y + z)
1
x
+
1
y
+
1
z
9 =
X
(x y)
2
xy
,
6
x
y + z
+
y
z + x
+
z
x + y
9 =
X
(x y)
2
(y + z)(z + x)
.
Ta cần chứng minh :
(x + y + z)
1
x
+
1
y
+
1
z
6
x
y + z
+
y
z + x
+
z
x + y
X
(x y)
2
xy
X
3(x y)
2
(y + z)(z + x)
.
với mọi số thực x, y, z thuộc đoạn [1; 2]. Đặt S
x
=
1
yz
3
(x + y)(x + z)
, S
y
=
1
zx
3
(y + x)(y + z)
,
S
z
=
1
xy
3
(z + x)(z + y)
.
Bất đẳng thức đã cho viết dưới dạng tương đương là:
S
x
(y z)
2
+ S
y
(z x)
2
+ S
z
(x y)
2
0.
Không mất tính tổng quát, ta giả sử 2 x y z 1 . Ta sẽ chứng minh rằng S
x
, S
y
0 .
Thật vy:
S
x
0 x
2
+ xy + xz 2yz 0, (đúng).
S
y
0 y
2
+ yx + yz 2zx x(y z) + z(z + y x) 0
(do x,y,z [1; 2] nên y + z x 0).
- NếuS
z
0 , ta đpcm.
- Nếu S
z
< 0, ta chứng minh được rằng S
x
+ 2S
z
0, S
y
+ 2S
z
0 .
Khi đó dễ dàng thấy rằng vì: (x y)
2
2
(y z)
2
+ (z x)
2
và S
z
< 0 nên
S
x
(y z)
2
+ S
y
(z x)
2
+ S
z
(x y)
2
(S
x
+ 2S
z
)(y z)
2
+ (S
y
+ 2S
z
)(z x)
2
0.
Vy trong mọi trường hợp, ta luôn đpcm.
Đẳng thức xảy ra khi x = y = z hoặc y = z = 1, x = 2 và các hoán vị của chúng.
Câu 5.8.
a) Chứng minh
2
3
(a
2
+ b
2
+ c
2
) a
3
+ b
3
+ c
3
, (1).
(1) (a + b + c)
a
2
+ b
2
+ c
2
3
a
3
+ b
3
+ c
3
a
2
b + a
2
c + b
2
c + b
2
a + c
2
a + c
2
b 2
a
3
+ b
3
+ c
3
(b + c) (b c)
2
+ (c + a) (c a)
2
+ (a + b) (a b)
2
0
Bất đẳng thức trên đúng nên (1) đúng. Đẳng thức xảy ra khi a = b = c =
2
3
.
b) Chứng minh a
3
+ b
3
+ c
3
4
3
(a
2
+ b
2
+ c
2
) 3abc, (2)
(2) 3
a
3
+ b
3
+ c
3
2 (a + b + c)
a
2
+ b
2
+ c
2
9abc
a
3
+ b
3
+ c
3
+ 9abc 2
a
2
b + a
2
c + b
2
c + b
2
a + c
2
a + c
2
b
S
a
(b c)
2
+ S
b
(c a)
2
+ S
c
(a b)
2
0, (3) .
trong đó S
a
= 3a b c; S
b
= 3b a c; S
c
= 3c a b.
Không mất tính tổng quát giả sử 1 a b c.
133
2. PHƯƠNG PHÁP PHÂN TÍCH BÌNH PHƯƠNG SOS
Ta V T (3) = S
a
(b c)
2
+ S
b
(c a)
2
(a b)
2
+ (S
b
+ S
c
) (a b)
2
S
a
0; S
b
= 2(1 a) + 2 (b c) 0;
(c a)
2
(a b)
2
0;S
b
+ S
c
= 4 (1 a) 0.
Suy ra V T (3) 0 (2) đúng.
Đẳng thức xảy ra khi a = b = c =
2
3
; a = 1,b = c =
1
2
và các hoán vị.
134
3. PHƯƠNG PHÁP DỒN BIẾN
§3. Phương pháp dồn biến
Câu 6.1.
a) Ta bất đẳng thức sau với mọi a, b, c > 0
abc (a + b c)(b + c a)(c + a b).
Thật vy, giả sử a b c, ta
abc (a + b c)(b + c a)(c + a b)
= (a + b c)(a b)
2
+ c(a c)(b c) 0.
Do đó
abc (3 2c)(3 2a)(3 2b)
abc 27 18(a + b + c) + 12(ab + bc + ca) 8abc
3abc 4(ab + bc + ca) 9
abc
4
3
(ab + bc + ca) 3.
Suy ra
P = abc +
12
ab + bc + ca
4
3
(ab + bc + ca) +
12
ab + bc + ca
3 8 3 = 5.
Dấu “=” xảy ra khi a = b = c = 1.
Vy giá trị nhỏ nhất của P 5.
b) Do bất đẳng thức đã cho đúng với mọi a, b, c > 0 thỏa mãn a + b + c = 3 nên cũng đúng
với b số a = b = x, c = 3 2x với mọi x
0;
3
2
. Khi đó
k
1
x
2
+ 2x(3 2x)
1
3
1 x
2
(3 2x), x
0;
3
2
k 3x(2 x)(2x + 1), x
0;
3
2
.
Với x =
4
3
ta k
88
9
> 9. Do k nguyên nên k 10.
Với k = 10, ta cần chứng minh f(a,b,c) := abc +
10
ab + bc + ca
13
3
.
Không mất tính tổng quát, giả sử a b c, khi đó 0 < c 1.
Ta sẽ chứng minh f (a, b, c) f
a + b
2
,
a + b
2
, c
. Thật vy, điều này tương đương với
10
ab + bc + ca
10
(a + b)
2
4
+ bc + ca
c
(a + b)
2
4
ab
10(a b)
2
(ab + bc + ca)
(a + b)
2
4
+ c(a + b)
c(a b)
2
.
Điều y đúng do c 1, ab + bc + ca 3 và
3(a + b)(a + b + 4c)
(4a + 4b + 4c)
2
4
= 36.
135
3. PHƯƠNG PHÁP DỒN BIẾN
Ta sẽ chứng minh f
a + b
2
,
a + b
2
, c
13
3
. Thật vy, điều này tương đương với
c(3 c)
2
4
+
40
(3 c)
2
+ 4c(3 c)
13
3
c(3 c)
2
4
4
+
40 10(1 + c)(3 c)
3(1 + c)(3 c)
0
(c 1)
2
c 4
4
+
10
3(1 + c)(3 c)
0.
Điều y đúng do
3(4 c)(1 + c)(3 c) 40
=3c
3
18c
2
+ 15c 4
=3c(1 c)(5 c) 4 < 3 ·
1
4
· 5 4 < 0, c (0, 1].
Vy số nguyên k nhỏ nhất cần tìm k = 10.
Câu 6.2. Từ điều kiện a + b + c = 3 ta sẽ nghĩ đến việc dồn biến về trung bình cộng. Bài toán
y ngoài trường hợp dấu bằng xảy ra khi a = b = c = 1. Còn xảy ra tại a = b = 0, c = 3
hoặc các hoán vị của b số (0,0,3). Không mất tính tổng quát, giả sử c = max{a,b,c}, suy ra
a c, b c và 1 c 3, 0 a + b 2.
Đặt
f(a,b,c) =
3a
2
+ 4bc + 9 +
3b
2
+ 4ca + 9 +
3c
2
+ 4ab + 9.
Ta sẽ chứng minh f (a,b,c) f
a + b
2
,
a + b
2
,c
.
Bước 1: Ta sẽ chứng minh rằng
3a
2
+ 4bc + 9 +
3b
2
+ 4ca + 9
r
3(a + b)
2
+ 8(a + b)c + 36 +
5(a b)
2
2
.
Bình phương hai vế và sử dụng (x + y)
2
= 2(x
2
+ y
2
) (x y)
2
, ta được
2(3a
2
+ 4bc + 9 + 3b
2
+ 4ca + 9
3a
2
+ 4bc + 9
3b
2
+ 4ca + 9
2
3(a + b)
2
+ 8(a + b)c + 36 +
5(a b)
2
2
.
Tương đương với
(a b)
2
2
3a
2
+ 4bc + 9
3b
2
+ 4ca + 9
2
.
Ta
3a
2
+ 4bc + 9
3b
2
+ 4ca + 9 =
(a b) [3(a + b) 4c]
3a
2
+ 4bc + 9 +
3b
2
+ 4ca + 9
.
Do đó, ta chỉ cần chứng minh
1
2
[3(a + b) 4c]
2
3a
2
+ 4bc + 9 +
3b
2
+ 4ca + 9
2
.
Suy ra
3a
2
+ 4bc + 9 +
3b
2
+ 4ca + 9
2
2 [3(a + b) 4c]
2
. (1)
136
3. PHƯƠNG PHÁP DỒN BIẾN
Kết hợp với c = max{a,b,c} và bất đẳng thức Minkowski, ta
3a
2
+ 4bc + 9 +
3b
2
+ 4ca + 9
3a
2
+ 4b
2
+ 9 +
3b
2
+ 4a
2
+ 9
=
r
3a
2
+ (2b)
2
+ 3
2
+
r
3b
2
+ (2a)
2
+ 3
2
p
3(a + b)
2
+ 4(a + b)
2
+ 36
=
p
7(a + b)
2
+ 36.
Từ 1 c 3, ta
[3(a + b) 4c]
2
[3(a + b) 4]
2
= 9(a + b)
2
24(a + b) + 16.
Kết hợp với (1), ta chỉ cần chứng minh
7(a + b)
2
+ 36 2
9(a + b)
2
24(a + b) + 16
.
Tương đương với
11(a + b) [3 (a + b)] + 15(a + b) + 4 0.
Bất đẳng thức cuối đúng, bởi 0 a + b 2. Bước 1 được giải quyết hoàn toàn.
Bước 2. Để hoàn thành bước dồn biến, ta sẽ chứng minh
r
3(a + b)
2
+ 8(a + b)c + 36 +
5(a b)
2
2
+
3c
2
+ 4ab + 9
p
3(a + b)
2
+ 8(a + b)c + 36 +
p
3c
2
+ (a + b)
2
+ 9.
Bất đẳng thức trên tương đương với
r
3(a + b)
2
+ 8(a + b)c + 36 +
5(a b)
2
2
p
3(a + b)
2
+ 8(a + b)c + 36
p
3c
2
+ (a + b)
2
+ 9
3c
2
+ 4ab + 9.
Nhân liên hợp và phân tích thành tổng bình phương, ta đưa v chứng minh
5
p
3c
2
+ (a + b)
2
+ 9 + 5
3c
2
+ 4ab + 9
2
r
3(a + b)
2
+ 8(a + b)c + 36 +
5(a b)
2
2
+ 2
p
3(a + b)
2
+ 8(a + b)c + 36. (2)
(i) Ta sẽ chứng minh
5
p
3c
2
+ (a + b)
2
+ 9 2
r
3(a + b)
2
+ 8(a + b)c + 36 +
5(a b)
2
2
.
Thật vy, ta a c, b c và 1 c 3. Do đó, ta
2
r
3(a + b)
2
+ 8(a + b)c + 36 +
5(a b)
2
2
=
p
12(a
2
+ 2ab + b
2
) + 32(a + b)c + 144 + 10(a
2
2ab + b
2
)
=
p
22(a
2
+ b
2
) + 2ab + 32(a + b)c + 144
p
22(c
2
+ c
2
) + 2c
2
+ 32(a + b)c + 144
=
p
46c
2
+ 32(a + b)c + 144.
137
3. PHƯƠNG PHÁP DỒN BIẾN
Do đó, ta chỉ cần chứng minh rằng
5
p
3c
2
+ (a + b)
2
+ 9
p
46c
2
+ 32(a + b)c + 144.
Bình phương hai vế và rút gọn, ta được
29c
2
+ 25(a + b)
2
+ 81 32(a + b)c 0.
Từ a + b + c = 3, ta
29c
2
+ 25(a + b)
2
+ 9(a + b + c)
2
32(a + b)c 0.
hay
10c
2
+ 34(a + b)
2
+ 14c [2c (a + b)] 0.
Bất đẳng thức cuối đúng
2c (a + b) = 2c (3 c) = 3c 3 = 3(c 1) 0, 1 c 3.
(ii) Tiếp theo, ta sẽ chứng minh
5
3c
2
+ 4ab + 9 2
p
3(a + b)
2
+ 8(a + b)c + 36.
Thật vy, từ c = max{a,b,c}, ta
p
3(a + b)
2
+ 8(a + b)c + 36
p
3(c + c)
2
+ 8(a + b)c + 36
=
p
12c
2
+ 8(a + b)c + 36.
Ta chỉ cần chứng minh
5
3c
2
+ 4ab + 9 2
p
12c
2
+ 8(a + b)c + 36.
Bình phương hai vế, ta được
25(3c
2
+ 4ab + 9) 4
12c
2
+ 8(a + b)c + 36
.
Chú ý rằng a + b + c = 3 nên
8c
2
+ 100ab + 9(a + b)
2
+ 14c [2c (a + b)] 0.
Bất đẳng thức cuối đúng tương tự trường hợp trên.
Do đó, bất đẳng thức (2) được chứng minh. Bước 2 được giải quyết hoàn toàn.
Bước 3. Để hoàn thành lời giải, ta chỉ cần chứng minh
f
a + b
2
,
a + b
2
,c
=
p
3(a + b)
2
+ 8(a + b)c + 36 +
p
3c
2
+ (a + b)
2
+ 9 12.
Từ a + b + c = 3, ta
p
3(3 c)
2
+ 8(3 c)c + 36 +
p
3c
2
+ (3 c)
2
+ 9 12.
Suy ra
63 + 6c 5c
2
+
4c
2
6c + 18 12.
Bình phương hai vế và khử căn hai lần, ta đưa v được
(c 1)
2
(3 c)(3c + 7) 0.
Bất đẳng thức cuối đúng bởi ta 1 c 3. Bước 3 được giải quyết hoàn toàn. Vậy bài
toán 1 được giải quyết hoàn toàn.
Đẳng thức xảy ra khi và chỉ khi a = b = c = 1 hoặc a = 3,b = c = 0 hoặc b = 3,c = a = 0 hoặc
c = 3,a = b = 0.
138
3. PHƯƠNG PHÁP DỒN BIẾN
Câu 6.3. Bài toán này ngoài trường hợp dấu bằng xảy ra khi a = b = c = 1. Còn xảy ra tại
a = b =
3
2
, c = 0 hoặc các hoán vị của các b số
3
2
,
3
2
,0
. Không mất tính tổng quát giả sử
rằng c = min{a,b,c}. Từ đây suy ra 0 c 1.
Đặt
f(a,b,c) =
9 6ab + a
2
+ b
2
+
9 6bc + b
2
+ c
2
+
9 6ca + c
2
+ a
2
.
Ta sẽ chứng minh rằng
f(a,b,c) f
a + b
2
,
a + b
2
,c
.
Bước 1. Ta sẽ chứng minh rằng
9 6bc + b
2
+ c
2
+
9 6ca + c
2
+ a
2
r
36 12(a + b)c + (a + b)
2
+ 4c
2
(a b)
2
5
.
Bình phương hai vế và biến đổi, ta thu được
6(a b)
2
5
9 6bc + b
2
+ c
2
9 6ca + c
2
+ a
2
2
.
Nhân liên hợp và phân tích tổng bình phương, ta đưa v chứng minh rằng
6
5
[6c (a + b)]
2
9 6bc + b
2
+ c
2
+
9 6ca + c
2
+ a
2
2
.
hay
6
9 6bc + b
2
+ c
2
+
9 6ca + c
2
+ a
2
2
5 [6c (a + b)]
2
. (3)
Từ c = min{a,b,c} và a + b + c = 3, ta chứng minh được rằng
9 6bc + b
2
+ c
2
p
b
2
+ (c + a)
2
+ (b c)
2
.
và
9 6ca + c
2
+ a
2
p
a
2
+ (b + c)
2
+ (c a)
2
.
Cộng hai đánh giá trên lại và áp dụng thêm bất đẳng thức Minkowski, ta
p
b
2
+ (c + a)
2
+ (b c)
2
+
p
a
2
+ (b + c)
2
+ (c a)
2
p
(a + b)
2
+ (a + b + 2c)
2
+ (a b)
2
p
(a + b)
2
+ (a + b + 2c)
2
=
18 + 2c
2
.
Ngoài ra, [6c (a + b)]
2
= [6c (3 c)]
2
= 49c
2
42c + 9. Kết hợp với (3), ta đưa v
6(18 + 2c
2
) 5(49c
2
42c + 9).
Bất đẳng thức cuối tương đương với 233c
2
+ 210c + 63 = (1 c)(233c + 23) + 40 0 đúng
c 1. Bước 1 được giải quyết hoàn toàn.
Bước 2. Để hoàn thành bước dồn biến, ta cần chứng minh
9 6ab + a
2
+ b
2
+
r
36 12(a + b)c + (a + b)
2
+ 4c
2
(a b)
2
5
p
9 (a + b)
2
+
p
36 12(a + b)c + (a + b)
2
+ 4c
2
.
139
3. PHƯƠNG PHÁP DỒN BIẾN
Thật vy, bất đẳng thức trên tương đương với
9 6ab + a
2
+ b
2
p
9 (a + b)
2
p
36 12(a + b)c + (a + b)
2
+ 4c
2
r
36 12(a + b)c + (a + b)
2
+ 4c
2
(a b)
2
5
.
Tiếp tục nhân liên hợp và phân tích tổng bình phương, ta chỉ cần chứng minh
10
p
36 12(a + b)c + (a + b)
2
+ 4c
2
+ 10
r
36 12(a + b)c + (a + b)
2
+ 4c
2
(a b)
2
5
9 6ab + a
2
+ b
2
+
p
9 (a + b)
2
. (4)
(i) Ta sẽ chứng minh
10
p
36 12(a + b)c + (a + b)
2
+ 4c
2
9 6ab + a
2
+ b
2
.
Thật vy, từ a + b + c = 3 và c = min{a,b,c}, suy ra a c, b c, 0 c 1 và
a
2
+ b
2
6ab = (a + b)
2
2ab 6ab = (a + b)
2
8ab (a + b)
2
8c
2
.
Suy ra
9 6ab + a
2
+ b
2
p
9 + (a + b)
2
8c
2
.
Do đó, ta chỉ cần chứng minh
10
p
36 12(a + b)c + (a + b)
2
+ 4c
2
p
9 + (a + b)
2
8c
2
.
Từ a + b + c = 3, ta
10
p
36 12(3 c)c + (3 c)
2
+ 4c
2
p
9 + (3 c)
2
8c
2
.
Khai triển và thu gọn, ta được
1707c
2
+ 4194(1 c) + 288 0.
Bất đẳng thức cuối cùng đúng 0 c 1.
(ii) Tiếp theo, ta sẽ chứng minh
10
r
36 12(a + b)c + (a + b)
2
+ 4c
2
(a b)
2
5
p
9 (a + b)
2
.
Ta có:
10
r
36 12(a + b)c + (a + b)
2
+ 4c
2
(a b)
2
5
=
p
3600 1200(a + b)c + 80(a + b)
2
+ 400c
2
+ 80ab.
Ta cần chứng minh
p
3600 1200(a + b)c + 80(a + b)
2
+ 400c
2
+ 80ab
p
9 (a + b)
2
.
Bình phương hai vế và biến đổi, ta thu được
4086(1 c) + 234 + 1681c
2
+ 80ab 0.
Bất đẳng thức cuối đúng 0 c 1 và a,b các số thực dương.
Do đó, bất đẳng thức (4) được chứng minh. Bước 2 được giải quyết hoàn toàn.
Bước 3. Để hoàn thành lời giải, ta chỉ cần chứng minh
f
a + b
2
,
a + b
2
,c
=
p
9 (a + b)
2
+
p
36 12(a + b)c + (a + b)
2
+ 4c
2
3
5.
140
3. PHƯƠNG PHÁP DỒN BIẾN
Từ a + b + c = 3, ta
p
9 (3 c)
2
+
p
36 12(3 c)c + (3 c)
2
+ 4c
2
3
5.
Suy ra
6c c
2
+
17c
2
42c + 45 3
5.
Lại bình phương hai vế và phân tích, ta thu được
c(c 1)
2
(10 3c) 0.
Bất đẳng thức cuối đúng 0 c 1.
Dấu bằng xảy ra khi và chỉ khi a = b = c = 1 hoặc a = b =
3
2
, c = 0 hoặc b = 0, c = a =
3
2
.
Tương tự bài toán 1 trên, đánh giá bước 1 chính chìa khóa để giải quyết bài toán y.
Câu 6.4. Bài toán này ngoài trường hợp dấu bằng xảy ra khi a = b = c = 1, còn xảy ra
tại a = b = 0, c = 3 hoặc các hoán vị của b số (0; 0; 3). Không mất tính tổng quát giả sử
c = max{a,b,c}, suy ra a c, b c. Từ a + b + c = 3, suy ra 1 c 3 kéo theo 0 a + b 2.
Bất đẳng thức được viết lại như sau
p
12 + 22ab + 7(a
2
+ b
2
) +
p
12 + 22bc + 7(b
2
+ c
2
) +
p
12 + 22ca + 7(c
2
+ a
2
) 12
3.
Ta đặt
f(a; b; c) =
p
12 + 22ab + 7(a
2
+ b
2
) +
p
12 + 22bc + 7(b
2
+ c
2
) +
p
12 + 22ca + 7(c
2
+ a
2
).
Ta sẽ chứng minh f (a; b; c) f
a + b
2
;
a + b
2
; c
.
Bước 1. Ta sẽ chứng minh rằng
p
12 + 22bc + 7(b
2
+ c
2
) +
p
12 + 22ca + 7(c
2
+ a
2
)
p
48 + 44(a + b)c + 7(a + b)
2
+ 28c
2
+ 2(a b)
2
.
Bình phương hai vế và biến đổi, ta thu được
5(a b)
2
h
p
12 + 22bc + 7(b
2
+ c
2
)
p
12 + 22ca + 7(c
2
+ a
2
)
i
2
.
hay
5
h
p
12 + 22bc + 7(b
2
+ c
2
) +
p
12 + 22ca + 7(c
2
+ a
2
)
i
2
[7(a + b) + 22c]
2
.(5)
Áp dụng bất đẳng thức Cauchy-Schwar, ta
h
p
12 + 22bc + 7(b
2
+ c
2
) +
p
12 + 22ca + 7(c
2
+ a
2
)
i
2
(1 + 1)
12 + 22bc + 7(b
2
+ c
2
) + 12 + 22ca + 7(c
2
+ a
2
)
=48 + 44(a + b)c + 28c
2
+ 14(a
2
+ b
2
).
Kết hợp với (5), ta chỉ cần chứng minh
5
48 + 44(a + b)c + 28c
2
+ 14(a
2
+ b
2
)
[7(a + b) + 22c]
2
.
Tương đương với
(a + b)[88c 21(a + b)] + 240(c
2
1) + 104c
2
+ 140ab 0.
141
3. PHƯƠNG PHÁP DỒN BIẾN
Bất đẳng thức cuối cùng đúng 0 a + b 2 và 1 c 3. Bước 1 được giải quyết hoàn toàn.
Bước 2. Để hoàn thành bước dồn biến ta cần chứng minh
p
12 + 22ab + 7(a
2
+ b
2
) +
p
48 + 44(a + b)c + 7(a + b)
2
+ 28c
2
+ 2(a b)
2
p
12 + 9(a + b)
2
+
p
48 + 44(a + b)c + 7(a + b)
2
+ 28c
2
.
Thật vy bất đẳng thức trên tương đương với
p
48 + 44(a + b)c + 7(a + b)
2
+ 28c
2
+ 2(a b)
2
p
48 + 44(a + b)c + 7(a + b)
2
+ 28c
2
p
12 + 9(a + b)
2
p
12 + 22ab + 7(a
2
+ b
2
).(6)
Bằng cách nhân liên hợp và phân tích tổng bình phương, ta đưa v chứng minh
p
48 + 44(a + b)c + 7(a + b)
2
+ 28c
2
+ 2(a b)
2
+
p
48 + 44(a + b)c + 7(a + b)
2
+ 28c
2
p
12 + 9(a + b)
2
+
p
12 + 22ab + 7(a
2
+ b
2
).(7)
Đầu tiên ta chứng minh
p
48 + 44(a + b)c + 7(a + b)
2
+ 28c
2
+ 2(a b)
2
p
12 + 9(a + b)
2
.
Bình phương hai vế và thu gọn ta thu được
36 + 2(a + b)[22c (a + b)] + 28c
2
+ 2(a b)
2
0.
Bất đẳng thức cuối đúng 0 a + b 2 và 1 c 3.
Tiếp theo ta chứng minh
p
48 + 44(a + b)c + 7(a + b)
2
+ 28c
2
p
12 + 22ab + 7(a
2
+ b
2
).
Bình phương hai vế và thu gọn ta thu được
36 + 44(a + b)c + 28c
2
8ab 0.
c = max{a,b,c} nên 28c
2
8ab 0 và bất đẳng thức trên đúng. Do đó, bất đẳng thức (7)
được chứng minh. Bước 2 được giải quyết hoàn toàn.
Bước 3. Để hoàn thành lời giải ta cần chứng minh
f
a + b
2
;
a + b
2
; c
=
p
12 + 9(a + b)
2
+
p
48 + 44(a + b)c + 7(a + b)
2
+ 28c
2
12
3.
Từ a + b + c = 3 ta được điều phải chứng minh trở thành
93 54c + 9c
2
+
111 + 90c 9c
2
12
3
Bình phương hai vế và thu gọn ta được
(c 1)
2
(c 3)(c 11) 0.
Bất đẳng thức cuối cùng đúng do 1 c 3. Bước 3 được giải quyết hoàn toàn hay bài toán
được chứng minh xong.
Dấu bằng xảy ra khi và chỉ khi a = b = c = 1 hoặc a = 3,b = c = 0 hoặc b = 3,a = c = 0 hoặc
c = 3,a = b = 0.
Bước đánh giá bước 1 mấu chốt để ta giải quyết bài toán.
142
3. PHƯƠNG PHÁP DỒN BIẾN
Câu 6.5. Bài toán y ngoài trường hợp dấu bằng xảy ra khi a = b = c = 1 còn xảy ra tại
a = b =
3
2
,c = 0 hoặc các hoán vị của b số
3
2
;
3
2
; 0
. Không mất tính tổng quát giả sử
c = min{a,b,c}, khi đó 0 c 1.
Đặt f (a; b; c) =
p
2(a
2
+ b
2
) + 21c +
p
2(b
2
+ c
2
) + 21a +
p
2(c
2
+ a
2
) + 21b. Ta sẽ chứng minh
rằng
f(a; b; c) f
a + b
2
;
a + b
2
; c
.
Bước 1. Ta sẽ chứng minh
p
2(b
2
+ c
2
) + 21a +
p
2(c
2
+ a
2
) + 21b
p
2(a + b)
2
+ 8c
2
+ 42(a + b) 2(a b)
2
.
Bình phương hai vế và khai triển ta
4(a b)
2
h
p
2(b
2
+ c
2
) + 21a
p
2(c
2
+ a
2
) + 21b
i
2
.
Nhân liên hợp cho bất đẳng thức trên ta đưa v
4
h
p
2(b
2
+ c
2
) + 21a +
p
2(c
2
+ a
2
) + 21b
i
2
[21 2(a + b)]
2
.(8)
Áp dụng tính chất 2(x
2
+ y
2
) (x + y)
2
và bất đẳng thức Minkowski, ta
p
2(b
2
+ c
2
) + 21a +
p
2(c
2
+ a
2
) + 21b
p
(b + c)
2
+ 21a +
p
(c + a)
2
+ 21b
=
q
(b + c)
2
+ (
21a)
2
+
q
(c + a)
2
+ (
21b)
2
q
(a + b + 2c)
2
+ (
21a +
21b)
2
.
Từ a + b + c = 3 và c = min{a; b; c}, suy ra
q
(a + b + 2c)
2
+ (
21a +
21b)
2
=
q
(3 c + 2c)
2
+ 21(a + b + 2
ab)
p
(3 + c)
2
+ 21(3 c + 2c) =
c
2
+ 27c + 72.
Ngoài ra [21 2(a + b)]
2
= [21 2(3 c)]
2
= 225 + 60c + 4c
2
,kết hợp với (8) ta cần
4(c
2
+ 27c + 72) 225 + 60c + 4c
2
16c + 21 0.
Bất đẳng thức y đúng 0 c 1. Bước 1 được giải quyết hoàn toàn.
Bước 2. Để hoàn thành bước dồn biến, ta cần chứng minh
p
2(a
2
+ b
2
) + 21c +
p
2(a + b)
2
+ 8c
2
+ 42(a + b) 2(a b)
2
p
(a + b)
2
+ 21c +
p
2(a + b)
2
+ 8c
2
+ 42(a + b).
Bất đẳng thức tương đương với
p
2(a + b)
2
+ 8c
2
+ 42(a + b)
p
2(a + b)
2
+ 8c
2
+ 42(a + b) 2(a b)
2
p
2(a
2
+ b
2
) + 21c
p
(a + b)
2
+ 21c.
143
3. PHƯƠNG PHÁP DỒN BIẾN
Nhân lượng liên hợp ta được
p
2(a + b)
2
+ 8c
2
+ 42(a + b) +
p
2(a + b)
2
+ 8c
2
+ 42(a + b) 2(a b)
2
2
p
2(a
2
+ b
2
) + 21c + 2
p
(a + b)
2
+ 21c.(9)
Đầu tiên ta chứng minh
p
2(a + b)
2
+ 8c
2
+ 42(a + b) 2
p
2(a
2
+ b
2
) + 21c.
Thật vậy ta c = min{a,b,c} suy ra 2(a
2
+ b
2
) = 2(a + b)
2
4ab 2(a + b)
2
4c
2
. Khi đó ta
thu được
2
p
2(a
2
+ b
2
) + 21c 2
p
2(a + b)
2
4c
2
+ 21c.
Do đó ta cần chứng minh
p
2(a + b)
2
+ 8c
2
+ 42(a + b) 2
p
2(a + b)
2
4c
2
+ 21c.
Từ a + b + c = 3, bình phương hai vế và biến đổi ta cần chứng minh
2(3 c)
2
+ 8c
2
+ 42(3 c) 8(3 c)
2
16c
2
+ 84c (1 c)(4 c) 0.
Bất đẳng thức y đúng do 0 c 1.
Tiếp theo ta sẽ chứng minh
p
2(a + b)
2
+ 8c
2
+ 42(a + b) 2(a b)
2
2
p
(a + b)
2
+ 21c.
Để ý rằng
p
2(a + b)
2
+ 8c
2
+ 42(a + b) 2(a b)
2
=
p
8c
2
+ 42(a + b) + 8ab nên ta cần chứng
minh
p
8c
2
+ 42(a + b) + 8ab 2
p
(a + b)
2
+ 21c.
Từ a + b + c = 3 nên điều y tương đương với
8c
2
+ 42(3 c) + 8ab 4[(3 c)
2
+ 21c].
Từ c = min{a,b,c} ta 8ab 8c
2
, do đó ta chỉ cần chứng minh
8c
2
+ 42(3 c) + 8c
2
4[(3 c)
2
+ 21c] (1 c)(15 2c) 0.
Bất đẳng thức cuối cùng đúng do 0 c 1. Đến đây thì bất đẳng thức (9) đã được chứng minh
hay bước 2 được giải quyết hoàn toàn.
Bước 3. Ta chỉ cần chứng minh
f
a + b
2
;
a + b
2
; c
=
p
(a + b)
2
+ 21c +
p
2(a + b)
2
+ 8c
2
+ 42(a + b) 15.
Từ a + b + c = 3 thì ta chỉ cần chứng minh
p
(3 c)
2
+ 21c +
p
2(3 c)
2
+ 8c
2
+ 42(3 c) 15.
Bình phương khử căn ta đưa v
c(c 1)
2
(40 3c) 0.
Bất đẳng thức cuối đúng 0 c 1. Bước 3 được giải quyết hoàn toàn. Vy bài toán đã được
giải quyết.
Dấu bằng xảy ra khi và chỉ khi a = b = c = 1 hoặc a = b =
3
2
,c = 0 hoặc b = c =
3
2
,a = 0 hoặc
a = c =
3
2
,b = 0.
144
3. PHƯƠNG PHÁP DỒN BIẾN
Câu 6.6. Bài toán này ngoài trường hợp dấu bằng xảy ra a = b = c = 1. Còn xảy ra tại a =
b =
3
2
,c = 0 hoặc các hoán vị b
3
2
,
3
2
,0
. Không mất tính tổng quát giả sử rằng c = min {a,b,c}.
Từ đây suy ra 0 c 1.
Đặt f (a,b,c) =
3a
2
a + 1 +
3b
2
b + 1 +
3c
2
c + 1
p
6(a
2
+ b
2
+ c
2
) + 9.
Bước 1. Ta dễ chứng minh được
3a
2
a + 1 +
3b
2
b + 1 +
3c
2
c + 1
p
6(a
2
+ b
2
2(a + b) + 4 (a + b)
2
.
Bước 2. Để hoàn thành bước dồn biến ta cần chứng minh
p
6(a
2
+ b
2
2(a + b) + 4 (a + b)
2
p
6 (a
2
+ b
2
+ c
2
) + 9
p
3(a + b)
2
2(a + b) + 4 +
p
3(a + b)
2
+ 6c
2
+ 9.
Nhân liên hợp và phân tích tổng bình phương, ta đưa v
3
q
6 (a
2
+ b
2
) 2 (a + b) + 4 (a b)
2
+
q
3(a + b)
2
2 (a + b) + 4
2
p
6 (a
2
+ b
2
+ c
2
) + 9 +
q
3(a + b)
2
+ 6c
2
+ 9
.
Ta dễ chứng minh được
3
q
3(a + b)
2
2(a + b) + 4 2
q
3(a + b)
2
+ 6c
2
+ 9.
Ta dễ chứng minh được
3
q
6 (a
2
+ b
2
) 2(a + b) + 4 (a b)
2
2
p
6 (a
2
+ b
2
+ c
2
) + 9.
Bước 3. Ta chỉ cần chứng minh
q
3(a + b)
2
2 (a + b) + 4 +
3c
2
c + 1
q
3(a + b)
2
+ 6c
2
+ 9.
Từ a + b + c = 3 ta suy ra
3c
2
16c + 25 +
3c
2
c + 1
p
9 (c
2
2c + 4).
Bình phương hai vế và rút gọn, ta được
9c(c 1)
2
(16 3c) 0.
Bất đẳng thức cuối đúng 0 c 1. Bước 3 được giải quyết hoàn toàn. Vậy bài toán được giải
quyết.
Đẳng thức xảy ra khi a = b = c = 1 hoặc a = b =
3
2
,c = 0, hoặc a = 0,b = c =
3
2
, hoặc
b = 0,c = a =
3
2
.
Câu 6.7. Bài toán y ngoài trường hợp dấu bằng xảy ra a = b = c =
2
3
. Còn xảy ra tại
a = b = 1 hoặc các hoán vị b (1,1,0). Không mất tính tổng quát giả sử rằng c = min {a,b,c}.
Từ đây suy ra 0 c
2
3
.
Đặt f (a,b,c) =
a + b 2ab +
b + c 2bc +
c + a 2ca.
Bước 1. Ta chứng minh được
b + c 2bc +
c + a 2ca
r
2(a + b) + 4c 4(a + b)c
1
2
(a b)
2
.
145
3. PHƯƠNG PHÁP DỒN BIẾN
Bước 2. Để hoàn thành bước dồn biến, ta cần chứng minh
a + b 2ab +
r
2(a + b) + 4c 4(a + b)c
1
2
(a b)
2
r
a + b +
(a + b)
2
2
+
p
2(a + b) + 4c 4c(a + b).
Nhân lượng liên hợp, ta được
p
2(a + b) + 4c 4c(a + b) +
r
2(a + b) + 4c 4(a + b)c
1
2
(a b)
2
a + b 2ab +
r
a + b +
(a + b)
2
2
.
(i) Ta chứng minh được
p
2(a + b) + 4c 4c(a + b)
a + b 2ab.
(ii) Ta chứng minh được
r
2(a + b) + 4c 4c(a + b)
1
2
(a b)
2
r
a + b
(a + b)
2
2
.
Bước 3. Ta chỉ cần chứng minh
r
2c c
2
2
+
4 6c + 4c
2
2.
Bình phương hai vế và rút gọn, ta được
c(3c 2)
2
(16 9c) 0.
Bất đẳng thức cuối đúng 0 c
2
3
. Bước 3 được giải quyết hoàn toàn. Vậy bài toán đã cho
được giải quyết.
Đẳng thức xảy ra khi và chỉ khi a = b = c =
2
3
, hoặc a = b = 1,c = 0, hoặc a = 0,b = c = 1, hoặc
b = 0,c = a = 1.
Câu 6.8. Bài toán ngày ngoài trường hợp dấu bằng xảy ra khi a = b = c = 1. Còn xảy ra tại
a = b =
2
3
,c =
5
3
, hoặc các hoán vị của b số
2
3
,
2
3
,
5
3
. Không mất tính tổng quát giả sử rằng
c = min {a,b,c}. Từ đây suy ra 1 c 3. Đặt
f(a,b,c) =
2a
2
a + 1 +
2b
2
b + 1 +
2c
2
c + 1
1
3
p
21(a
2
+ b
2
+ c
2
) + 99.
Bước 1. Dễ dàng chứng minh được
2a
2
a + 1 +
2b
2
b + 1+
r
2(a + b)
2
2(a + b) + 4 +
7
16
(a b)
2
.
Bước 2. Để hoàn thành bước dồn biến, ta cần chứng minh
r
2(a + b)
2
2(a + b) + 4 +
7
16
(a b)
2
1
3
p
21(a
2
+ b
2
+ c
2
+ 99
146
3. PHƯƠNG PHÁP DỒN BIẾN
p
2(a + b)
2
2(a + b) + 4
1
3
r
21c
2
+
21
2
(a + b)
2
+ 99.
Nhân lượng liên hợp và biến đổi ta đưa v
p
21(a
2
+ b
2
+ c
2
) + 99 +
r
21c
2
+
21
2
(a + b)
2
+ 99
8
r
2(a + b)
2
2(a + b) + 4 +
7
16
(a b)
2
+ 8
p
2(a + b)
2
2(a + b) + 4.
Ta dễ dàng chứng minh được các kết quả sau:
r
21c
2
+
21
2
(a + b)
2
+ 99 8
p
2(a + b)
2
2(a + b) + 4.
và
p
21(a
2
+ b
2
+ c
2
+ 99 8
r
2(a + b)
2
2(a + b) + 4 +
7
16
(a b)
2
.
Bước 3. Ta chỉ cần chứng minh
p
2(a + b)
2
2(a + b) + 4 +
2c
2
c + 1
1
3
r
21c
2
+
21
2
(a + b)
2
+ 99 0.
Từ a + b + c = 3, ta suy ra
2c
2
10c + 16 +
2c
2
c + 1
r
7c
2
14c + 43
2
.
Bình phương hai vế và rút gọn, ta được
7(3c 5)
2
(c 1)
2
0.
Bất đẳng thức cuối cùng đúng. Bước 3 giải quyết hoàn toàn và cũng kết thúc bài toán.
Đẳng thức xảy ra khi và chỉ khi a = b = c = 1 hoặc a = b =
2
3
,c =
5
3
hoặc a =
5
3
,bc =
2
3
hoặc
b =
5
3
,c = a =
2
3
.
Câu 6.15. Chuẩn hóa ab + bc + ca = 3. Bất đẳng thức cần chứng minh trở thành
1
(a + b)
2
+
1
(b + c)
2
+
1
(c + a)
2
3
4
.
Đặt f(a, b, c) =
1
(a + b)
2
+
1
(b + c)
2
+
1
(c + a)
2
và t số thực dương thỏa mãn 2at + t
2
= 3. Xét
P = f (a, b, c) f (a,t,t) =
1
(a + b)
2
+
1
(a + c)
2
2
(a + t)
2
+
1
(b + c)
2
1
4t
2
.
Ta
2at + t
2
= ab + bc + ca a(b + c 2t) = t
2
bc. (1)
Nếu b + c < 2t thì V T (1) < 0 và t
2
>
a + b
2
2
ab nên V P (1) 0, dẫn đến mâu thuẫn với
(1). Từ đó, suy ra b + c 2t (b + c)
2
4t
2
1
(a + b)
2
147
4. PHƯƠNG PHÁP P, Q, R
§4. Phương pháp p, q, r
Câu 1.1. Bất đẳng thức cần chứng minh tương đương với
1 +
3
p
6
q
pq + 3q 6p 0 q
6p
p + 3
. (1)
Ta p
2
3pr = 3p, nên q
3p. Do đó, để chứng minh (1) ta chứng minh
p
3p
6p
p + 3
p(p + 3)
2
12p
2
(p 3)
2
0 (đúng).
Câu 1.2. Ta q + 6r = 9 và bất đẳng thức cần chứng minh trở thành
p + 3r 6 2p q + 3. (2)
Theo BĐT Schur ta
9r 4pq p
3
9 ·
9 q
6
4pq p
3
q
27 + 2p
3
8p + 3
.
Do đó để chứng minh (2) ta chứng minh
2p
27 + 2p
3
8p + 3
+ 3 p
3
2p
2
+ 9p + 18 0 (p + 1)(p 3)(p 6) 0. (3)
Ta 9 = q + 6r
p
2
3
+
6p
3
27
p 3 nên (3) đúng khi p 6. Nếu p > 6 thì a + b + c + 3abc > 6.
Câu 1.3. Nếu a + b + c > 4 thì ta a
3
+ b
3
+ c
3
3
a + b + c
3
3
>
64
9
Áp dụng BĐT Schur,
ta có:
a
3
+ b
3
+ c
3
+ 3abc ab(a + b) + bc(b + c) + ca(c + a)
a
3
+ b
3
+ c
3
+ 6abc (ab + bc + ca)(a + b + c) = pq = 3p
và
r
p(4q p
2
)
9
=
p(12 p
2
)
9
.
Ta cần chứng minh:
3p +
p(12 p
2
)
9
10
(p 3)[(16 p
2
) + 3(4 p) + 2]
9
0.
Bất đẳng thức cuối hiển nhiên đúng nên ta đpcm.
Đẳng thức xảy ra khi a = b = c = 1.
Câu 1.4. Bât đẳng thức cần chứng minh được viết lại như sau:
3r + 12 5q. (1)
Mặt khác,theo BDT Schur,ta có:
3r
3p(4q p
2
)
9
= 4q 9.
Nên ta chứng minh
4q 9 + 12 5q q 3 (đúng).
148
4. PHƯƠNG PHÁP P, Q, R
Câu 1.5. Bất đẳng thức cần chứng minh tương đương với
8p + 3r 12 + 5q. (1)
Áp dụng BDT Schur,ta có:
3r
p(4q p
2
)
3
=
p(2q 3)
3
.
Từ giả thiết p
2
2q = 3 q =
p
2
3
2
.
Nên ta chỉ cần chứng minh:
8p +
p(p
2
6)
3
12 +
5(p
2
3)
2
(2p 3)(p 3)
2
0.
Bất đẳng thức cuối đúng nên ta đpcm.
Câu 1.6. Biến đổi bất đẳng thức cần chứng minh và chuyển về dạng p,q,r,ta có:
8(243 18p + 3r) 3(729 81q + 27r r
2
) 243 99q + 57r 3r
2
0.
Theo BDT AM-GM thì 3 = 3(
a + b + c
3
)
6
3(abc)
2
= r
2
.
Theo BDT Schur,ta có:
r
p(4q p
2
)
3
=
4q 9
3
57r 19(4q 9).
Nên ta cần chứng minh:
72 23q 3r
2
0 3(1 r
2
) + 23(3 q) 0.
Vy BDT được chứng minh.
Câu 1.7. Đưa bất đẳng thức v dạng p,q,r,từ giả thiết,ta q + r = 4.
và lúc đó,bất đẳng thức trở thành
p
2
2q + 5r 8 p
2
7q + 12 0.
Nếu 4 p,sử dụng BDT Schur,ta có:
r
p(4q p
2
)
9
,
suy ra
4 q +
p(4q p
2
)
9
q
p
3
+ 36
4p + 9
p
2
7(p
3
+ 36)
4p + 9
+ 12 0 (p 3)(p
2
16) 0.
điều y đúng 4 p
3q 3.
Nếu p 4, ta p
2
16 4q và
p
2
2q + 5r p
2
2q
p
2
2
8.
Vy BDT được chứng minh.
Đẳng thức xảy ra khi x = y = z = 1 hoặc x = y = 2,z = 0 và các hoán vị
149
5. PHƯƠNG PHÁP TIẾP TUYẾN VÀ T TUYẾN
§5. Phương pháp tiếp tuyến và cát tuyến
Câu 2.1. BĐT đã cho thuần nhất nên ta chỉ cần chứng minh Bđt đúng với mọi số thực dương
a,b,c thỏa mãn a
2
+b
2
+c
2
= 1, khi đó bđt cần chứng minh trở thành: f(a)+f (b)+ f (c) 1trong
đó: f(x) =
1 +
3
3
3
.
1
x
x với 0 < x < 1. Dễ thấy hàm số f(x) f
00
(x) > 0 x (0; 1).
Theo BĐT tiếp tuyến ta :
f(a) + f(b) + f(c) f
0
1
3
(a + b + c
3) + 3f
1
3
.
Do
f
0
1
3
< 0
a + b + c
p
3(a
2
+ b
2
+ c
2
) =
3
f(a) + f (b) + f (c) 3f
1
3
= 1.
Câu 2.2. Đặt a
i
= tan x
i
(i = 1,2, . . . ,n) a
i
> 0 i = 1,2, . . . ,n và
n
P
i=1
a
i
n.
Ta cần chứng minh :
n
Y
i=1
a
i
p
1 + a
2
i
1
2
n
.
Xét hàm số f(x) =
x
1 + x
2
, x > 0
f
0
(x) =
1
q
(1 + x
2
)
3
f
00
(x) < 0 x > 0.
Do đó
f(x) f
0
(1)(x 1) + f(1) =
1
2
3
(x 1) +
1
2
=
1
2
2
(x + 1).
Suy ra
n
Y
i=1
a
i
p
1 + a
2
i
=
n
Y
i=1
f(a
i
)
1
8
n
n
Y
i=1
(a
i
+ 1)
1
8
n
n
P
i=1
(a
i
+ 1)
n
n
2
n
8
n
=
1
2
n
.
Đẳng thức xảy ra khi a
1
= a
2
= ··· = a
n
= 1 hay tan x
1
= tan x
2
= ··· = tan x
n
= 1 x
1
=
x
2
= ··· = x
n
=
π
4
.
Câu 2.3. Ta :
bc (
b + c
2
)
2
= (
1 a
2
)
2
; ca (
a + c
2
)
2
= (
1 b
2
)
2
; ab (
b + a
2
)
2
= (
1 c
2
)
2
nên
a
1 + bc
+
b
1 + ac
+
c
1 + ab
4a
a
2
2a + 5
+
4b
b
2
2b + 5
+
4c
c
2
2c + 5
= f(a) + f(b) + f(c).
Ta thấy đẳng thức xảy ra khi a = b = c =
1
3
và tiếp tuyến của đồ thị hàm số f(x) =
4x
x
2
2x + 5
tại điểm hoành độ x =
1
3
:y =
99x 3
100
, nên ta xét
4x
x
2
2x + 5
99x 3
100
=
(3x 1)
2
(15 11x)
100(x
2
2x + 5)
0 x (0; 1).
150
5. PHƯƠNG PHÁP TIẾP TUYẾN VÀ T TUYẾN
Suy ra
4a
a
2
2a + 5
+
4b
b
2
2b + 5
+
4c
c
2
2c + 5
99(a + b + c) 9
100
=
9
10
.
Bài toán được chứng minh.
Câu 2.4. Bđt cần chứng minh thuần nhất nên ta chỉ cần chứng minh Bđt đúng với mọi
số thực dương a,b,c thỏa mãn a + b + c = 1. Khi đó Bđt đã cho trở thành:
(1 2a)
2
(1 a)
2
+ a
2
+
(1 2b)
2
(1 b)
2
+ b
2
+
(1 2c)
2
(1 c)
2
+ c
2
3
5
4a
2
4a + 1
2a
2
2a + 1
+
4b
2
4b + 1
2b
2
2b + 1
+
4c
2
4c + 1
2c
2
2c + 1
3
5
1
2a
2
2a + 1
+
1
2b
2
2b + 1
+
1
2c
2
2c + 1
27
5
f(a) + f(b) + f(c)
27
5
.
Trong đó f(x) =
1
2x
2
2x + 1
với x (0; 1).
Tiếp tuyến của đồ thị hàm số y = f (x) tại điểm hoành độ x =
1
3
y =
54x + 27
25
.
Ta
54x + 27
25
f(x) =
2(54x
3
27x
2
+ 1)
25(2x
2
2x + 1)
=
2(3x 1)
2
(6x + 1)
25(2x
2
2x + 1)
0 x (0; 1).
Suy ra
f(a) + f(b) + f(c)
54(a + b + c) + 81
25
=
27
5
. (đpcm).
Câu 2.5. Phân tích. Với điều kiện a + b + c = 1 ta chọn điểm rơi tại a = b = c =
1
3
= x
0
và
đặt f(a) = a
3
+ 5a
2
. Khi đó f
0
(a) = 3a
2
+ 10a, f
0
1
3
=
11
3
, f
1
3
=
16
27
.
Ta chứng minh
a
3
+ 5a
2
11
3
a
1
3
+
16
27
a
3
+ 5a
2
11
3
a +
17
27
0
a
1
3
2
a +
17
3
0 (BĐT cuối luôn đúng với điều kiện a > 1).
Chứng minh tương tự với các biểu thức còn lại:
b
3
+ 5b
2
11
3
b
1
3
+
16
27
,
c
3
+ 5c
2
11
3
c
1
3
+
16
27
.
Cộng vế với vế, ta được
S
11
3
(a + b + c 1) +
16
9
=
16
9
.
Khi a = b = c =
1
3
thì S =
16
9
.
Vy min S =
16
9
tại a = b = c =
1
3
.
151
5. PHƯƠNG PHÁP TIẾP TUYẾN VÀ T TUYẾN
Câu 2.6. Phân tích. Chọn điểm rơi a = b = c = 2 và đặt f(a) =
a
a
2
+ a + 1
. Tính được
f
0
(2) =
1
3
, f(2) =
2
3
. Ghép vào công thức (∗∗).
Ta sẽ chứng minh
a
a
2
+ a + 1
1
3
(a + 2)
2
3
a
3
5a
2
8a 4 0
(a + 2)
2
(a + 1) 0 (luôn đúng với a < 1).
Chứng minh tương tự với các biến b, c, ta
b
b
2
+ b + 1
1
3
(b + 2)
2
3
,
c
c
2
+ c + 1
1
3
(c + 2)
2
3
.
Cộng vế với vế được S
1
3
(a + b + c) 4 = 2 4 = 2.
Với a = b = c = 2 thì S = 2.
Vy max S = 2 tại a = b = c = 2.
Câu 2.7. Phân tích. Lần này 3 biến không còn đối xứng (chỉ hoán vị vòng quanh) và cũng
không nhóm dạng phân ly biến được. Ta sẽ nhóm riêng (a
3
+2a
2
b) để đánh giá và coi b tham số
(hoặc thể nhóm a
3
+ 2c
2
a). Từ điều kiện ta thể nhẩm ra dấu bằng xảy ra khi a = b = c = 1
và đặt f (a) = a
3
+ 2a
2
b f
0
(a) = 3a
2
+ 4ab, f
0
(1) = 3 + 4b, f (1) = 1 + 2b.
Ta sẽ chứng minh
a
3
+ 2a
2
b (3 + 4b)(a 1) + (1 + 2b)
(a
3
1) + (2a
2
b 2b) (3 + 4b)(a 1) 0
(a 1)
2
(a + 2b + 2) 0 (BĐT cuối luôn đúng với điều kiện ban đầu).
Chứng minh tương tự, ta
b
3
+ 2b
2
c (3 + 4c)(b 1) + (1 + 2c),
c
3
+ 2c
2
a (3 + 4a)(c 1) + (1 + 2a).
Cộng vế với vế được S 4(ab + bc + ca) + a + b + c 6 15 6 = 9.
Với a = b = c = 1 thì S = 9.
Vy min S = 9 tại a = b = c = 1.
Câu 2.8. Phân tích. Từ điều kiện đề bài nhẩm ra a = b = c = 1. Đặt f (a) = 2a
3
3a
2
b suy
ra f
0
(1) = 6 6b, f (1) = 2 3b.
Ta sẽ chứng minh
2a
3
3a
2
b (6 6b)(a 1) + 2 3b
(a 1)
2
(2a 3b + 4) 0 (BĐT cuối luôn đúng bởi điều kiện xác định).
Chứng minh tương tự, ta
2b
3
3b
2
c (6 6c)(b 1) + 2 3c,
2c
3
3c
2
a (6 6a)(c 1) + 2 3a.
Cộng vế với vế ta được S 9(a + b + c) 6(ab + bc + ac) 12 3 · 3 12 = 3.
Với a = b = c = 1 thì S = 3.
Vy min S = 3 tại a = b = c = 1.
152
5. PHƯƠNG PHÁP TIẾP TUYẾN VÀ T TUYẾN
Câu 2.9. Phân tích. Đặt f (a) = 5a
3
+ 2a
2
b, tính được f
0
(1) = 15 + 4b, f (1) = 5 + 2b.
Ta
5a
3
+ 2a
2
b (15 + 4b)(a 1) + 5 + 2b
5(a 1)(a
2
+ a + 1) + 2b(a 1)(a + 1) (15 + 4b)(a 1) 0
(a 1)[5a
2
+ 5a 10 + 2b(a 1)] 0
(a 1)
2
(5a + 2b + 10) 0 (BĐT cuối luôn đúng điều kiện xác định).
Chứng minh tương tự rồi cộng vế với vế được S 13(a + b + c) + 4(ab + bc + ca) 30.
Lại a + b + c
p
3(ab + bc + ca) = 3.
Suy ra S 13 · 3 + 4 · 3 30 = 21.
Với a = b = c = 1 thì S = 21.
Vy min S = 21 tại a = b = c = 1.
Câu 2.10. Phân tích. Đặt f (a) =
1
a
2
b
a
, tính được f
0
(1) = 2 + b, f (1) = 1 b.
Ta
1
a
2
b
a
(2 + b)(a 1) + 1 b
(1 a)
1 + a
a
2
+ b
a 1
a
+ (2 + b)(1 a) 0
(1 a)
1 + a
a
2
2 + b
b
a
0
(1 a)
2
2a + 1
a
2
b
a
0 (BĐT cuối luôn đúng điều kiện xác định).
Chứng minh tương tự rồi cộng vế với vế được T 4(a + b + c) + ab + bc + ca + 9 = 0.
Với a = b = c = 1 thì T = 0.
Vy min T = 0 tại a = b = c = 1.
Câu 2.11. Phân tích. Đặt f (a) = a
5
+ 3a
4
b, tính được f
0
(1) = 5 + 12b, f (1) = 1 + 3b.
Ta
a
5
+ 3a
4
b (5 + 12b)(a 1) + 1 + 3b
(a 1)(a
4
+ a
3
+ a
2
+ a + 1) + 3b(a 1)(a
3
+ a
2
+ a + 1) (5 + 12b)(a 1) 0
(a 1)[3b(a
3
+ a
2
+ a 3) + a
4
+ a
3
+ a
2
+ a 4] 0
(a 1)
2
[3b(a
2
+ 2a + 3) + a
3
+ 2a
2
+ 3a + 4] 0 (BĐT cuối luôn đúng điều kiện xác định).
Chứng minh tương tự rồi cộng vế với vế được A 12(ab+bc+ca)4(a+b+c)12 2412 = 12.
Với a = b = c = 1 thì A = 12.
Vy min A = 12 tại a = b = c = 1.
Câu 2.12. Phân tích. Đặt f(a) = a
2
+ b
3a + 1, tính được f
0
(1) = 2 +
3
4
b, f(1) = 1 + 2b.
Ghép vào công thức.
Ta
a
2
+ b
3a + 1
2 +
3
4
b
(a 1) + 1 + 2b
(a 1)
a + 1 +
3b
3a + 1 + 2
2
3
4
b
0
(a 1)
2
1
9b
4(
3a + 1 + 2)
2
0 (BĐT cuối luôn đúng điều kiện xác định).
153
5. PHƯƠNG PHÁP TIẾP TUYẾN VÀ T TUYẾN
Chứng minh tương tự rồi cộng vế với vế được B
3
4
(ab + bc + ca) +
13
4
(a + b + c) 3 9.
Với a = b = c = 1 thì B = 9.
Vy min B = 9 tại a = b = c = 1.
Câu 2.13. Xét hàm số f(x) = tan x, x
0;
π
2
, f
0
(x) = 1 + tan
2
x, suy ra f
00
(x) =
2 tan x(1 + tan
2
x) > 0, x
0;
π
2
.
Áp dụng BĐT tiếp tuyến với MNP nhọn, ta :
f(A) f
0
(M)(A M) + f (M ) =
1
cos
2
M
(A M) + tan M,
suy ra
cos
2
M.f(A)
1
2
sin 2M + A M.
Tương tự :
cos
2
N.f(B)
1
2
sin 2N + B N ; cos
2
P.f(C)
1
2
sin 2P + C P.
Suy ra
cos
2
M.f(A) + cos
2
N.f(B) + cos
2
P.f(C)
sin 2M + sin 2N + sin 2P
2
.
Ta chọn các c M, N, P sao cho
cos M = k > 0; cos N =
2k; cos P =
3k.
M, N, P ba c của tam giác nên ta đẳng thức :
cos
2
M + cos
2
N + cos
2
P + 2 cos M. cos N. cos P = 1,
nên ta (1 +
2 +
3)k + 2
6k
3
= 1, do đó k nghiệm dương của phương trình :
2
6x
3
+ (1 +
2 +
3)x 1 = 0. (1)
Suy ra
sin 2M = 2
1 cos
2
M · cos M = 2k
1 k
2
;
sin 2N = 2k
p
2(1 2k
2
); sin 2P = 2k
p
3(1 3k
2
).
Dẫn tới
F
sin 2M + sin 2N + sin 2P
2k
2
=
1 k
2
+
p
2(1 2k
2
) +
p
3(1 3k
2
)
k
.
Vy min F =
1 k
2
+
p
2(1 2k
2
) +
p
3(1 3k
2
)
k
đạt được khi A = M; B = N; C = P với
M, N, P ba c của tam giác nhọn được xác định bởi
cos M = k > 0; cos N =
2k; cos P =
3k,
trong đó k nghiệm dương duy nhất của (1).
154
5. PHƯƠNG PHÁP TIẾP TUYẾN VÀ T TUYẾN
Câu 2.14. Ta các hàm số f (t) = t
3
; g(t) =
1 + t
2
; h(t) =
4
1 + t
4
, t (0; 1) những
hàm số đạo hàm cấp hai dương trên khoảng (0; 1). Nên với a,b,c > 0 thỏa a + b + c = 1 áp
dụng BĐT tiếp tuyến, ta có:
f(x) f
0
(a)(x a) + f(a), h(y) h
0
(b)(y b) + h(b), g(z) g
0
(c)(z c) + g(c).
Ta chọn a,b,c sao cho
f
0
(a) = g
0
(b) = h
0
(c) = k
3a
2
= k
b
1 + b
2
= k
c
3
4
q
(1 + c
4
)
3
= k
a =
r
k
3
b =
k
1 k
2
c =
3
k
4
p
1 k
3
k
(1)
Do a + b + c = 1 nên ta
r
k
3
+
k
1 k
2
+
3
k
4
p
1 k
3
k
= 1. (2)
Dễ thấy phương trình (2) luôn nghiệm trong khoảng (0; 1). Suy ra
P = f (x) + g(y) + h(z) f(a) + h(b) + g(c) =
k
3k
9
+
1
1 k
2
+
1
4
p
1 k
3
k
.
Đẳng thức xảy ra x = a; y = b; z = c.
Vy min P =
k
3k
9
+
1
1 k
2
+
1
4
p
1 k
3
k
với k nghiệm nằm trong (0; 1) của (2).
155
Chương 3
Một số chuyên đề
§1. Ứng dụng đều kiện nghiệm của phương trình bậc
ba
Câu 1.1. Đặt n = ab + bc + ca, p = abc Suy ra a, b, c ba nghiệm của phương trình :
x
3
mx + n = 0 (4) Ta có:
p
2
4
27
n
3
n
3
27
4
p
2
.
Do đó:
13p
2
+ 2p 2 + 2n
3
13p
2
+ 2p 2
27
2
p
2
=
1
2
p 1
2
0.
Suy ra:
13p
2
+ 2p 2 2n
3
13a
2
b
2
c
2
2ab 2 2 (ab + bc + ca)
3
.
Mà:
(a + b + c)
2
= 0 ab + bc + ca =
1
2
a
2
+ b
2
+ c
2
,
dẫn tới:
13a
2
b
2
c
2
2abc 2
1
4
a
2
+ b
2
+ c
2
3
P
1
4
.
Đẳng thức xảy ra
(
n = 2
m = 3
a,b,c ba nghiệm của phương trình
x
3
3x + 2 = 0 (x 1)
2
(x + 2) = 0 x = 1,x = 2.
Vy max P =
1
4
đạt được khi (a,b,c) = (1,1, 2) và các hoán vị.
Câu 1.2. Đặt n = ab + bc + ca, p = abc Ta có:
p
2
4
27
n
3
n
3
27
4
p
2
n
3
27
4
p
2
.
a + b + c = 0 a
2
+ b
2
+ c
2
= 2(ab + bc + ca) = 2n n 0.
Do đó:
P = 32n
5
32np
2
8 |p| = 32
(n)
5
+ (n)p
2
8 |p|
64
n
3
|p| 8 |p| 8
54|p|
3
|p|
.
156
1. ỨNG DỤNG ĐỀU KIỆN NGHIỆM CỦA PHƯƠNG TRÌNH BẬC BA
Xét hàm số f(t) = 54t
3
t,t 0 ta có:
f
0
(t) = 162t
2
1,f
0
(t) = 0 t =
2
18
.
Lập bảng biến thiên ta min
t0
f(t) = f
2
18
!
=
2
27
.
Suy ra P
8
2
27
. Đẳng thức xảy ra khi
p =
2
18
n =
1
3
24
hay a,b,c nghiệm của phương trình
t
3
1
3
24
t +
2
18
= 0
t
1
p
6
3
3
!
2
t +
2
3
9
!
= 0 t =
1
p
6
3
3
,t =
2
3
9
.
Vy min P =
8
2
27
. Đạt được khi a = b =
1
p
6
3
3
,c =
2
3
9
và các hoán vị.
Câu 1.3. Đặt m = (a + b + c), n = ab + bc + ca, p = abc. Từ giả thiết ta suy ra:
(a + b + c)
2
= 4 (ab + bc + ca) n =
m
2
4
.
Suy ra
27p
m
3
4
|m
3
|
4
108p m
3
m
3
p(54p m
3
) 0 pm
3
54p
2
.
Do đó:
P = pm
3
+
1
p
4
54p
2
+
1
p
4
= 27p
2
+ 27p
2
+
1
p
4
3
3
r
27p
2
.27p
2
1
p
4
= 27 (đpcm).
Đẳng thức xảy ra
n =
m
2
4
27p
2
=
1
p
4
54p = m
3
, chẳng hạn ta chọn
p =
1
3
m =
3
q
18
3
n =
3
972
4
hay a,b,c nghiệm của
phương trình:
t
3
+
3
q
18
3t
2
+
3
972
4
t +
1
3
= 0
t +
3
p
18
3
6
!
2
t +
3
4
3
!
= 0.
Vy min P = 27 đạt được khi a = b =
3
p
18
3
6
,c =
3
4
3
và các hoán vị.
Câu 1.4. Đặt m = (a + b + c), n = ab + bc + ca, p = abc. Từ giả thiết ta suy ra:
(a + b + c)
2
= 3 (ab + bc + ca) + 4 m
2
= 3n + 4.
Mặt khác :
27p + 2m
3
9mn
2
q
(m
2
3n)
3
.
157
1. ỨNG DỤNG ĐỀU KIỆN NGHIỆM CỦA PHƯƠNG TRÌNH BẬC BA
Suy ra
|27p + 2m (3n + 4) 9mn| 18
|27p 3mn + 8m| 16
mn 9p
8m 16
3
.
Mặt khác:
P = 18 (ab + bc + ca)
2
+ 48 (ab + bc + ca) (ab + bc + ca) (a + b + c) + 9abc
= 2 [3 (ab + bc + ca) + 4]
2
(ab + bc + ca) (a + b + c) + 9abc 16
= 2 (a + b + c)
2
(ab + bc + ca) (a + b + c) + 9abc 16
= 2m
4
+ mn 9p 16 2m
4
+
8m 16
3
16 =
1
3
6m
4
+ 8m 64
.
Xét hàm số f(m) = 6m
4
+ 8m 64, ta có:
f
0
(m) = 24m
3
+ 8 f
0
(m) = 0 m =
1
3
3
.
Suy ra
f (m) f
1
3
3
=
6
3
3
64.
Nên P
1
3
2
3
3
+ 64
.
Đẳng thức xảy ra khi và chỉ khi
m =
1
3
3
n =
1
3
1
3
9
4
p =
1
9
4
3
3
+
47
9
, suy ra a,b,c nghiệm của phương trình
: t
3
1
3
3
t
2
+
1
3
1
3
9
4
t +
1
9
4
3
3
+
47
9
= 0.
Vy min P =
1
3
2
3
3
+ 64
.
Câu 1.5. Chuẩn hoá abc = 2 a + b + c = 4. Đặt n = ab + bc + ca, suy ra
|18n 91|
q
(16 3n)
3
3n
3
12n
2
108n + 465 0
(n 5)(3n
2
+ 3n 93) 0 5 n
1 + 5
5
2
.
Mặt khác:
a
4
+ b
4
+ c
4
=
a
2
+ b
2
+ c
2
2
2
a
2
b
2
+ b
2
c
2
+ c
2
a
2
= (16 2n)
2
2
n
2
16
= 2n
2
64n + 288.
Nên
P =
1
256
a
4
+ b
4
+ c
4
=
1
128
n
2
32n + 144
hàm f(n) = n
2
32n + 144 nghịch biến trên
"
5;
5
5 1
2
#
nên ta suy ra
max P =
1
128
f(5) =
9
128
và min P =
1
128
f
5
5 1
2
!
=
383 165
5
2
.
158
2. BÀI TOÁN TÌM HẰNG SỐ TỐT NHẤT
§2. Bài toán tìm hằng số tốt nhất
Câu 2.1. bất đẳng thức đúng với mọi giá trị a,b,cnên phải đúng với a = b = c = 1 k
2
3
.
Ta chứng minh k =
2
3
giá trị lớn nhất.
Xét k =
2
3
bất đẳng thức trở thành
a
4
+ b
4
+ c
4
+ abc (a + b + c)
2
3
(ab + bc + ca)
2
,
hay
3
a
4
+ b
4
+ c
4
2
a
2
b
2
+ b
2
c
2
+ c
2
a
2
+ abc (a + b + c) . (1)
Áp dụng bđt AM GM ta
a
4
+ b
4
+
b
4
+ c
4
+
b
4
+ c
4
2a
2
b
2
+ 2b
2
c
2
+ 2c
2
a
2
.
Suy ra
3
a
4
+ b
4
+ c
4
3
a
2
b
2
+ b
2
c
2
+ c
2
a
2
. (2)
Mặt khác
a
2
b
2
+ b
2
c
2
+ c
2
a
2
abc (a + b + c) =
1
2
(ab bc)
2
+
1
2
(bc ca)
2
+
1
2
(ca ab)
2
0. (3)
Từ (2) và (3) suy ra (1) được chứng minh .
Vy số k lớn nhất k =
2
3
.
Câu 2.2. Không mất tính tổng quát, ta giả sử a = min {a; b; c}. Chọn b = c. Từ điều kiện, ta
a + 2b = 1
1
3
b
1
2
.
Bất đẳng thức trở thành:
a
1 + 9b
2
+
2b
1 + 9ab + k(a b)
2
1
2
1 2b
1 + 9b
2
1
6
+
2b
1 + 9b (1 2b) + k(1 3b)
2
1
3
0
(3b + 5) (1 3b)
6 (1 + 9b
2
)
+
(3b 1) (6b + 1) k(3b 1)
2
3
9b 18b
2
+ 1 + k(1 3b)
2
0
6b + 1 k (3b 1)
9b 18b
2
+ 1 + k(1 3b)
2
3b + 5
6 (1 + 9b
2
)
54b
3
+ 27b
2
+ 12b 1
9b
2
+ 4b 1
k
18b
2
+ 15b + 1
9b
2
+ 4b 1
k
7b + 3
9b
2
+ 4b 1
k 2.
Ta xét hàm số f (b) =
7b + 3
9b
2
+ 4b 1
trên
1
3
;
1
2
.
Ta dễ dàng thấy đưc f nghịch biến trên đoạn
1
3
;
1
2
nên suy ra f (b) f
1
2
= 2.
159
2. BÀI TOÁN TÌM HẰNG SỐ TỐT NHẤT
Do đó k 2 2 k 4.
Ta chứng minh k = 4 giá trị lớn nhất cần tìm. Tức ta cần chứng minh:
a
1 + 9bc + 4(b c)
2
+
b
1 + 9ca + 4(c a)
2
+
c
1 + 9ab + 4(a b)
2
1
2
. (1)
Áp dụng Bất đẳng thức Cauchy Schwarz, ta có:
VT
(a + b + c)
2
(a + b + c) + 27abc + 4a(b c)
2
+ 4b(c a)
2
+ 4c(a b)
2
=
1
1 + 3abc + 4ab (a + b) + 4bc (b + c) + 4ca (c + a)
.
Do đó để chứng minh (1) ta cần chứng minh:
1 3abc + 4ab (a + b) + 4bc (b + c) + 4ca (c + a)
(a + b + c)
3
3abc + 4ab (a + b) + 4bc (b + c) + 4ca (c + a)
a
3
+ b
3
+ c
3
+ 3abc ab (a + b) + bc (b + c) + ca (c + a)
(điều y đúng, do đây bất đẳng thức Schur).
Do đó (1) đúng với mọi a, b, c không âm và thỏa a + b + c = 1 và với k
max
= 4.
Vy k = 4 số cần tìm.
Câu 2.3. Thử chọn b = c rồi xét hàm theo biến
a
b
, ta không dẫn đến kết quả cần tìm. Như vậy
ta sẽ thử chọn một biến bằng 0, đây một biến tiến dần đến 0.
Trong bất đẳng thức, ta cho c 0, khi đó a
3
+ b
3
Mab
2
. Nhưng lại theo bất đẳng thức AM
GM, ta
a
3
+ b
3
= a
3
+
b
3
2
+
b
3
2
3
3
4
· ab
2
.
Như vy ta thấy M
3
3
4
. Do đó ta chứng minh M =
3
3
4
giá trị lớn nhất cần tìm như sau:
Không giảm tính tổng quát, giả sử c = min {a,b,c}. Đặt a = u + c, b = v + c với u, v 0.
Ta chứng minh:
(u + c)
3
+ (v + c)
3
+ c
3
3c (u + c) (v + c)
M
(u + c) (v + c)
2
+ (v + c) c
2
+ c(u + c)
2
3c (u + c) (v + c)
(3 M )
u
2
uv + v
2
+ u
3
+ v
3
Muv
2
0.
Bất đẳng thức cuối cùng luôn đúng vì, 3 M > 0, u
2
uv + v
2
uv 0 và
u
3
+ v
3
= u
3
+
v
3
2
+
v
3
2
3
3
4
uv
2
= Muv
2
.
Từ đó ta hoàn tất phần chứng minh.
Vy giá trị lớn nhất cần tìm là: M =
3
3
4
.
Câu 2.4. tính thuần nhất của bất đẳng thức nên ta chuẩn hóa xyz = 1. Từ đó ta biến đổi:
x =
a
b
, y =
b
c
, z =
c
a
. Khi đó bất đẳng thức trở thành:
a
2
bc
+
b
2
ca
+
c
2
ab
+ 3k (k + 1)
a
b
+
b
c
+
c
a
a
3
+ b
3
+ c
3
+ 3k · abc (k + 1)
ab
2
+ bc
2
+ ca
2
a
3
+ b
3
+ c
3
3abc (k + 1)
ab
2
+ bc
2
+ ca
2
3abc
.
160
2. BÀI TOÁN TÌM HẰNG SỐ TỐT NHẤT
Đây bất đẳng thức bài trên.
Vy k + 1 =
3
3
4
k = 1 +
3
3
4
.
Câu 2.5. Ta chọn b = c thì giả thiết trở thành:
a + 2b = b
2
+ 2ab a =
2b b
2
2b 1
.
a > 0 nên
2b b
2
2b 1
> 0
1
2
< b 2.
Bất đẳng thức trở thành:
(a + 2b)
1
2b
+
2
a + b
k (a + 2b + 1)
2b b
2
2b 1
+ 2b
1
2b
+
2
2b b
2
2b 1
+ b
k
2b b
2
2b 1
+ 2b + 1
3b
2
b
3b
3
+ 5b
2
+ b 1
2k
9
.
Xét hàm số f (b) =
3b
2
b
3b
3
+ 5b
2
+ b 1
trên
1
2
; 2
.
Ta
f
0
(b) = 0
(b
2
1) (3b 1)
2
(3b
3
+ 5b
2
+ b 1)
2
= 0 b = 1.
Bằng cách lập bảng biến thiên ta tìm được min
1
2
;2
f (b) =
2
9
khi b = 2. Từ đó tìm được k 1.
Ta chứng minh k = 1 giá trị lớn nhất cần tìm. Thực vậy ta cần chứng minh:
(a + b + c)
1
a + b
+
1
b + c
+
1
c + a
a + b + c + 1
(ab + bc + ca)
1
a + b
+
1
b + c
+
1
c + a
a + b + c + 1
ab
a + b
+
bc
b + c
+
ca
c + a
1.
Nhận thấy rằng dấu = đạt được tại một biến bằng 0 và hai biến bằng 2 nên nếu đánh giá bằng
những bất đẳng thức thông thường dấu = tại tâm sẽ không dẫn đến kết quả. Để đảm bảo
dấu = ta sẽ loại b dấu = tại biên này bằng cách sử dụng abc 0 như sau:
Theo Cauchy - Schwarz:
ab
a + b
+
bc
b + c
+
ca
c + a
(ab + bc + ca)
2
ab (a + b) + bc (b + c) + ca (c + a)
=
(ab + bc + ca)
2
(a + b + c) (ab + bc + ca) 3abc
(ab + bc + ca)
2
(a + b + c) (ab + bc + ca)
= 1.
Chứng minh hoàn thành. Ta kết luận số k lớn nhất cần tìm k = 1.
161
2. BÀI TOÁN TÌM HẰNG SỐ TỐT NHẤT
Câu 2.6. Cho b = c, ta bất đẳng thức trở thành:
(a + 2b)
1
a
+
2
b
9 k
1
2ab + b
2
a
2
+ 2b
2
2(a b)
2
ab
k ·
(a b)
2
a
2
+ 2b
2
a
2
+ 2b
2
ab
k
2
.
Theo bất đẳng thức AM GM, ta có:
a
2
+ 2b
2
ab
2
2.
Suy ra k 4
2.
Ta chứng minh đây giá trị k lớn nhất cần tìm. Sử dụng bất đẳng thức Schur SOS như sau:
(a + b + c)
1
a
+
1
b
+
1
c
9 4
2
1
ab + bc + ca
a
2
+ b
2
+ c
2
(a + b + c) (ab + bc + ca) 9abc
abc
4
2 ·
a
2
+ b
2
+ c
2
ab bc ca
a
2
+ b
2
+ c
2
2c(a b)
2
+ (a + b) (a c) (b c)
abc
4
2 ·
(a b)
2
+ (a c) (b c)
a
2
+ b
2
+ c
2
M · (a b)
2
+ N · (a c) (b c) 0.
Trong đó: M =
2c
abc
4
2
a
2
+ b
2
+ c
2
, N =
a + b
abc
4
2
a
2
+ b
2
+ c
2
.
Giả sử c = max {a,b,c} thì mọi việc hoàn tất nếu chỉ ra N 0, hay
(a + b)
a
2
+ b
2
+ c
2
4
2abc.
Do tính thuần nhất nên ta chuẩn hóa a + b + c = 1. Ta đi chứng minh kết quả mạnh hơn sau:
a
2
+ b
2
+ c
2
c (a + b)
2 1 2c (a + b) c (a + b)
2 + 2ab.
Và mạnh hơn nữa:
1 2 · c (a + b)
2 · c (a + b) +
(a + b)
2
2
1 2 · t (1 t)
2 · t (1 t) +
t
2
2
, với t = a + b
3 2
2
2
t
2
2 +
2
t + 1 0
2 +
2
t
2
1
!
2
0.
Điều y đúng. Như vậy chứng minh hoàn tất.
Vy số k lớn nhất thỏa đề bài k = 4
2.
Câu 2.7. Ta chọn a = b = t, a = b = t, c = 3 2t. Khi đó, ta có:
k
2t
4
+ (3 2t)
4
3
2t
3
+ (3 2t)
3
+ 3t
2
(3 2t) 6
k
12t
3
+ 45t
2
54t + 21
18t
4
96t
3
+ 216t
2
216t + 78
=
3(t 1)
2
(7 4t)
(t 1)
2
(18t
2
60t + 78)
=
7 4t
2 (3t
2
10t + 13)
= f (t) .
162
2. BÀI TOÁN TÌM HẰNG SỐ TỐT NHẤT
Ta có:
f
0
(t) =
12 (2t
2
7t + 3)
4(3t
2
10t + 13)
2
= 0 t
3;
1
2
.
Khoảng giá trị của t (0; 3), từ đó ta thấy min
(0;3)
f (t) =
2
7
khi t =
1
2
.
Vy trở lại bài toán, ta cho a = b =
1
2
, c = 2 thì ta được k
2
7
, ta chứng minh giá trị nhỏ nhất
của k
2
7
bằng cách chứng minh:
2
7
a
4
+ b
4
+ c
4
3
a
3
+ b
3
+ c
3
+ 3abc 6.
Đặt f (a,b,c) = V T V P . Ta chứng minh:
f (a,b,c) f
a + b
2
,
a + b
2
,c
2
7
"
a
4
+ b
4
(a + b)
4
8
#
"
a
3
+ b
3
(a + b)
3
4
#
3c
"
ab
(a + b)
2
4
#
0
(a b)
4
+ 6(a b)
2
(a + b)
2
28
3 (a + b) (a b)
2
4
+ 3c ·
(a b)
2
4
0
7a
2
+ 7b
2
+ 10ab
28
3
4
(a + b) +
3c
4
0
7
a
2
+ b
2
+ 10ab 21 (a + b) + 21c 0
7
a
2
+ b
2
+ 10ab 21 (a + b) + 21 (3 a b) 0
5 (a + b)
2
+ 2
a
2
+ b
2
42 (a + b) + 63 0.
Do 2 (a
2
+ b
2
) (a + b)
2
nên ta sẽ chứng minh:
6(a + b)
2
42(a + b) + 63 0.
Và điều y sẽ đúng nếu ta giả sử c = max {a,b,c} khi đó a + b 2.
Vy ta chỉ cần chỉ ra:
f
a + b
2
,
a + b
2
,c
0 f (t; t; 3 t) 0,t =
a + b
2
2
7
2t
4
+ (3 2t)
4
3
2t
3
+ (3 2t)
3
+ 3t
2
(3 2t) 6.
Bất đẳng thức y đúng theo lập luận chặn t phía trên.
Vy k =
2
7
giá trị nhỏ nhất cần tìm.
Câu 2.8. Cho b = c giả thiết đã cho viết thành:
a + 2b = b
2
+ 2ab a =
b
2
2b
1 2b
0
1
2
b 2.
Lưu ý tìm số thực k lớn nhất nên xét k > 0.
Thay vào bất đẳng thức, ta được:
2b +
b
2
2b
1 2b
+ k
b
2
(b
2
2b)
1 2b
k + 3
2b (1 2b) +
b
2
2b
+ kb
2
b
2
2b
k (1 2b) + 3 (1 2b)
3 (b 1)
2
k
b
2
1
(b 1)
2
3
k
b
2
1.
163
2. BÀI TOÁN TÌM HẰNG SỐ TỐT NHẤT
b
2
1 2
2
1 = 3 suy ra k 1. Ta chứng minh k = 1 giá trị lớn nhất cần tìm.
Với giả thiết a + b + c = ab + bc + ca, ta chỉ cần chứng tỏ
ab + bc + ca + abc 4.
Ta phản chứng ab + bc + ca + abc = 4 và đi chứng minh a + b + c ab + bc + ca.
Đây một kết quả quen thuộc của thi VMO 1996.
Câu 2.9. Trong bất đẳng thức ban đầu, ta cho b = c, ta được:
a
3
+ 2b
3
2b (a + b)
+ k ·
b
2
+ 2ab
(a + 2b)
2
3
8
+
k
3
1
2
a
3
+ 2b
3
b(a + b)
2
1
4
k ·
1
3
b
2
+ 2ab
(a + 2b)
2
(a b)
2
(4a + 5b)
8b(a + b)
2
k ·
(a b)
2
3(a + 2b)
2
8k
3
(a + 2b)
2
(4a + 5b)
b(a + b)
2
.
Do tính thuần nhất nên ta thể chọn b = 1. Khi đó, ta có:
8k
3
(a + 2)
2
(4a + 5)
(a + 1)
2
.
Ta xét hàm f (a) =
(a + 2)
2
(4a + 5)
(a + 1)
2
, a > 0. Ta có:
f
0
(a) =
2 (a + 2) (2a
2
+ 2a 1)
(a + 1)
3
= 0 a =
1 +
3
2
(do a > 0).
Lập bảng biến thiên ta được f (a) f
1 +
3
2
!
= 9 + 6
3. Suy ra: k
3
9 + 6
3
8
.
Ta chứng minh giá trị k lớn nhất cần tìm
3
9 + 6
3
8
.
Ta sử dụng thuật Schur SOS trong phần chứng minh. Bất đẳng thức cần chứng minh viết
dưới dạng:
8 (a
3
+ b
3
+ c
3
3abc) 3 ((a + b) (b + c) (c + a) 8abc)
8 (a + b) (b + c) (c + a)
k ·
a
2
+ b
2
+ c
2
ab bc ca
3(a + b + c)
2
.
Ta sử dụng các khai triển:
a
3
+ b
3
+ c
3
3abc = (a + b + c)
(a b)
2
+ (a c) (b c)
(a + b) (b + c) (c + a) 8abc = 2c (a b)
2
+ (a + b) (a c) (b c)
a
2
+ b
2
+ c
2
ab bc ca = (a b)
2
+ (a c) (b c) .
Từ đó ta nhóm được điều cần chứng minh thành:
M (a b)
2
+ N (a c) (b c) 0,
trong đó:
M =
8a + 8b + 2c
8 (a + b) (b + c) (c + a)
k
3(a + b + c)
2
N =
5a + 5b + 8c
8 (a + b) (b + c) (c + a)
k
3(a + b + c)
2
.
164
2. BÀI TOÁN TÌM HẰNG SỐ TỐT NHẤT
y giờ, không giảm tổng quát ta giả sử c = min {a,b,c} nên a + b 2c.
Từ đó 8a + 8b + 2c 5a + 5b + 8c.
Từ đây ta thấy ngay M N . Hơn nữa cũng (a c) (b c) 0.
Hơn nữa cũng (a c) (b c) 0.
Như vy bất đẳng thức chứng minh hoàn tất nếu ta chỉ ra được N 0. Tức là:
3 (5a + 5b + 8c) (a + b + c)
2
8k (a + b) (b + c) (c + a) .
Đổi biến (a + b,b + c,c + a) (X,Y,Z). Khi đó cần chứng minh:
3 (X + 4Y + 4Z) (X + Y + Z)
2
32k · XY Z.
Ta chứng minh kết quả mạnh hơn là:
3 (X + 4Y + 4Z) (X + Y + Z)
2
8k · X (Y + Z)
2
=
27 + 18
3
X (Y + Z)
2
.
tính thuần nhất nên ta thể chuẩn hóa X + Y + Z = 1. Từ đó ta cần chứng minh:
3 (4 3X)
27 + 18
3
X (1 X)
2
g (X) =
4 3X
X(1 X)
2
9 + 6
3,
trong đó 0 < X < 1.
Khảo sát hàm g trên khoảng (0; 1) ta được g (X) 9 + 6
3.
Như vy ta hoàn tất chứng minh.
Vy k lớn nhất bằng
3
9 + 6
3
8
.
Câu 2.10. Cho b = c ta được:
a
2b
+ k
b
b + a
+ k
2
k +
1
2
3
.
Cho a 0 ta được:
8k (k + 1)
2
(2k + 1)
3
4k
2
+ 2k 1
k
1
5
4
k
1 +
5
4
.
Ta chứng minh đây toàn b giá trị k cần tìm. Đặt x =
2a
b + c
, y =
2b
c + a
, z =
2c
a + b
thì
xy + yz + zx + xyz = 4.
Ta cần chứng minh:
(x + 2k) (y + 2k) (z + 2k) (2k + 1)
3
xyz + 2k (xy + yz + zx) + 4k
2
(x + y + z) + 8k
3
(2k + 1)
3
.
Theo kết quả VMO 1996, ta được:
x + y + z xy + yz + zx.
Từ đó:
xyz + 2k (xy + yz + zx) + 4k
2
(x + y + z) + 8k
3
xyz + 2k (xy + yz + zx) + 4k
2
(xy + yz + zx) + 8k
3
= (xy + yz + zx + xyz) +
4k
2
+ 2k 1
(xy + yz + zx) + 8k
3
= 8k
3
+ 4 +
4k
2
+ 2k 1
(xy + yz + zx) .
165
2. BÀI TOÁN TÌM HẰNG SỐ TỐT NHẤT
Hơn nữa ta có:
xy + yz + zx 3
3
q
(xyz)
2
xyz
q
(xy + yz + zx)
3
3
3
.
Từ đó:
4 = xy + yz + zx + xyz xy + yz + zx +
q
(xy + yz + zx)
3
3
3
= t
2
+
t
3
3
3
,
với t =
xy + yz + zx. Suy ra: t
3 xy + yz + zx 3. Suy ra:
xyz + 2k (xy + yz + zx) + 4k
2
(x + y + z) + 8k
3
8k
3
+ 4 +
4k
2
+ 2k 1
(xy + yz + zx)
8k
3
+ 4 + 3
4k
2
+ 2k 1
= (2k + 1)
3
.
Bài toán hoàn tất.
Câu 2.11. Trước hết ta đi tìm điều kiện cần cho k.
Thay a = b = x, c =
1
x
2
với x > 0, x 6= 1, bất đẳng thức đã cho thể viết lại thành
x
2
+
2
x
+
k
2x +
1
x
2
+ 1
3 +
k
4
x
2
+
2
x
3 k
1
4
x
2
2x
3
+ x
2
+ 1
(x 1)
2
(x + 2)
x
k(x 1)
2
(2x + 1)
4 (2x
3
+ x
2
+ 1)
k
4
(x + 2) (2x
3
+ x
2
+ 1)
x (2x + 1)
k
4
x
2
+ 2x +
2
x
3
2x + 1
, x > 0, x 6= 1.
y giờ, ta sẽ cho x =
3 + 1
4
và tính giá trị của biểu thức
f (x) = x
2
+ 2x +
2
x
3
2x + 1
bên vế phải. Để ý rằng x nghiệm của phương trình, do ta x
2
=
4x + 1
8
,
1
x
= 8x 4,
3
2x + 1
= 4 4x. Suy ra
f (x) =
4x + 1
8
+ 2x + 2 (8x 4) (4 4x)
=
45
2
x
95
8
=
45
2
·
3 + 1
4
95
8
=
45
3 50
8
.
Với kết quả này, ta thu được
k
45
3 50
2
= 45 ×
3
2
25 45 × 0.866 25 13.97.
166
2. BÀI TOÁN TÌM HẰNG SỐ TỐT NHẤT
Mặt khác, k số nguyên nên từ đây k 13. Tiếp theo, ta sẽ chứng minh k = 13 thỏa mãn
yêu cầu bài toán, tức
1
a
+
1
b
+
1
c
+
13
a + b + c + 1
25
4
.
Đặt
f (a,b,c) =
1
a
+
1
b
+
1
c
+
13
a + b + c + 1
.
Không mất tính tổng quát, giả sử a = max{a,b,c}, ta
f (a,b,c) f
a,
bc,
bc
=
1
b
+
1
c
2
bc
+ 13
1
a + b + c + 1
1
a + 2
bc + 1
=
b
c
2
1
bc
13
(a + b + c + 1)
a + 2
bc + 1
.
Do a = max{a; b; c} và giả thiết abc = 1 nên ta bc 1, suy ra
1
bc
1. Mặt khác, sử dụng bất
đẳng thức AM - GM, ta lại
13
(a + b + c + 1)
a + 2
bc + 1
13
3
3
abc + 1
3
3
abc + 1
=
13
16
< 1.
Từ đây ta đưa bài toán v chứng minh
f
1
x
2
,x,x
25
4
với x =
bc, 0 < x 1.
Nếu x = 1 thì bất đẳng thức trên trở thành đẳng thức. Trong trường hợp 0 < x < 1 bằng cách
sử dụng lại biến đổi đã thực hiện trong quá trình tìm điều kiện cho k, ta thấy bất đẳng thức
tương đương với
(x + 2) (2x
3
+ x
2
+ 1)
x (2x + 1)
13
4
4 (x + 2)
2x
3
+ x
2
+ 1
13x (2x + 1)
4
2x
4
+ 5x
3
+ 2x
2
+ x + 2
26x
2
+ 13x
8x
4
+ 20x
3
18x
2
9x + 8 0.
Ta
8x
4
+ 20x
3
18x
2
9x + 8 =
8x
4
8x
2
+ 2
+
20x
3
20x
2
+ 5x
+
10x
2
14x + 6
= 2
2x
2
1
2
+ 5x (2x 1)
2
+ 2
5x
2
7x + 3
> 0.
Do 2 (2x
2
1)
2
0,5 · x (2x 1)
2
0 và 5x
2
7x + 3 > 0 (tam thức bất hai hệ số cao nhất
dương và biệt thức = 11 < 0).
Như vy, bất đẳng thức cuối cùng hiển nhiên đúng. Vy k = 13 giá trị cần tìm.
167
2. BÀI TOÁN TÌM HẰNG SỐ TỐT NHẤT
Câu 2.12. Cho a = b = t > 0, c =
1
t
2
ta được
2t
2
+
1
t
4
+ 3k (k + 1)
2
t
+ t
2
2t
6
+ 1
t
4
+ 3k (k + 1) ·
t
3
+ 2
t
k
3
t
3
+ 2
t
t
3
+ 2
t
2t
6
+ 1
t
4
k
3t t
3
2
t
t
6
+ 2t
3
1
t
4
k (t 1)
2
(t + 2)
(t
3
1)
2
t
3
k (t + 2)
(t
2
+ t + 1)
2
t
3
k
(t
2
+ t + 1)
2
t
4
+ 2t
3
k
1 +
1
t
+
1
t
2
2
1 +
2
t
.
Đặt z =
1
t
. Khi đó
k
(z
2
+ z + 1)
2
2z + 1
.
Khi t + thì z 0. Như vy
k
max
= min
[0;+)
f (z) ,
với f (z) =
(z
2
+ z + 1)
2
2z + 1
, với z 0. Ta
f
0
(z) =
2 (z
2
+ z + 1) (2z + 1)
2
2 (z
2
+ z + 1)
(2z + 1)
2
=
2 (z
2
+ z + 1)
(2z + 1)
2
1
(2z + 1)
2
0, z 0.
Suy ra hàm số f đồng biến trên [0; +), do đó với z 0 thì f (z) f (0) = 1. Do đó k 1.
Từ đó k
max
= 1 nếu chỉ ra được
a
2
+ b
2
+ c
2
+ 3 2
1
a
+
1
b
+
1
c
a
2
+ b
2
+ c
2
+ 3 2 (ab + bc + ca) (do abc = 1)
a
2
+ b
2
+ c
2
+ 2abc + 1 2 (ab + bc + ca) .
Đây một kết quả quen thuộc được chứng minh bằng nguyên Diricle (ta cũng thể dùng
dồn biến để chứng minh).
Dấu đẳng thức xảy ra khi (a,b,c) = (1; 1; 1); a = b +, c 0 và các hoán vị.
Câu 2.13. Thông thường trong những dạng toán này, đẳng thức đạt được ngoài giá trị tâm
(1; 1; 1) thì còn đạt được khi hai biến bằng nhau. Như vy để chặn k, ta chọn a = b = t, c = 32t.
Ta có:
2
t
+
1
3 2t
3 k
2t
2
+ (3 2t)
2
3
6 4t + t 3 (3t 2t
2
)
3t 2t
2
k
6t
2
12t + 6
k
6t
2
12t + 6
(3t 2t
2
) (6t
2
12t + 6)
k
1
t (3 2t)
.
168
2. BÀI TOÁN TÌM HẰNG SỐ TỐT NHẤT
Như vy giá trị c = 3 2t lớn nhất cần tìm chính giá trị lớn nhất của hàm số
f (t) =
1
t (3 2t)
, t
0;
3
2
.
Theo bất đẳng thức AM - GM, ta
t (3 2t) =
1
2
· 2t (3 2t)
1
2
·
(2t + 3 2t)
2
4
=
9
8
.
Từ đó kết luận được k
max
=
8
9
nếu chứng tỏ được
1
a
+
1
b
+
1
c
3
9
8
a
2
+ b
2
+ c
2
3
. (1)
Đặt f (a,b,c) =
1
a
+
1
b
+
1
c
3
9
8
(a
2
+ b
2
+ c
2
3). Xét hiệu
f (a,b,c) f
a + b
2
,
a + b
2
,c
=
1
a
+
1
b
+
1
c
3
8
9
a
2
+ b
2
+ c
2
3
4
a + b
1
c
+ 3 +
8
9
(a + b)
2
2
+ c
2
3
!
=
1
a
+
1
b
4
a + b
8
9
a
2
+ b
2
(a + b)
2
2
!
=
(a + b)
2
4ab
ab (a + b)
8
9
·
a
2
+ b
2
2ab
2
= (a b)
2
1
ab (a + b)
4
9
.
Ta giả sử c = max {a,b,c}. Khi đó a + b 2, suy ra
ab (a + b)
(a + b)
3
4
8
4
<
9
4
.
Như vy
f (a,b,c) f
a + b
2
,
a + b
2
,c
.
Ta chỉ cần chứng minh
f
a + b
2
,
a + b
2
,c
0 f (t,t,3 2t) 0, với t =
a + b
2
.
Tức chứng minh
2
t
+
1
3 2t
3
8
9
2t
2
+ (3 2t)
2
3
8
9
1
t (3 2t)
24t 16t
2
9
t (3 2t)
0
16t
2
24t + 9
t (3 2t)
0
(4t 3)
2
t (3 2t)
0 (đúng).
Vy () được chứng minh. Từ đó kết luận k
max
=
8
9
.
169
2. BÀI TOÁN TÌM HẰNG SỐ TỐT NHẤT
Câu 2.14. hiệu (1) bất đẳng thức đã nêu trong đề bài. Giả sử k số thực sao cho bất
đẳng thức (1) đúng với mọi b ba số thực a, b, c, abc 0.
Trong (1) thay a = 0, b = c = 2, ta được 2 + 8k 4. Suy ra k
1
4
.
Ta sẽ chứng minh k =
1
4
giá trị nhỏ nhất cần tìm, tức chứng minh với mọi b ba số thực
không âm a, b, c, ta luôn
abc +
1
4
(a b)
2
+ (b c)
2
+ (c a)
2
+ 2 a + b + c.
hay
a
2
+ b
2
+ c
2
+ 2abc + 4 ab + bc + ca + 2 (a + b + c) . (2)
Xét 3 số (b 1) (c 1), (c 1) (a 1), (a 1) (b 1), ta
a (b 1) (c 1) · b (c 1) (a 1) · c (a 1) (b 1) = abc (a 1)
2
(b 1)
2
(c 1) 0.
Suy ra ít nhất một số trong 3 số nêu trên không âm. Không mất tính tổng quát, giả sử
a (b 1) (c 1) 0. Khi đó, ta abc a (b + c 1).
Do vy, bất đẳng thức (2) sẽ được chứng minh, nếu ta chứng minh được
a
2
+ b
2
+ c
2
+ 2a (b + c 1) + 4 ab + bc + ca + 2 (a + b + c) ,
hay
a
2
(4 b c) a + b
2
+ c
2
bc 2 (b + c) + 4 0, (3)
với mọi a, b, c, abc 0.
Ta xem vế trái của (3) một tam thức bậc hai theo ẩn a, ta có:
= (b + c 4)
2
4b
2
4c
2
+ 4bc + 8 (b + c) 16 = 3b
2
3c
2
+ 6bc = 3 (b c)
2
0,
với mọi b, c nên (3) đúng với mọi a, b, c, abc 0.
Vy k =
1
4
giá trị cần tìm, theo yêu cầu của đề bài.
Câu 2.15. ràng, các giá trị x = y = z = 1 thỏa mãn ràng buộc nêu trong đề bài. thế,
trong bất đẳng thức của đề bài, cho x = y = z = 1 ta được k + 1 2. Suy ra k 1.
Tiếp theo, ta chứng minh với k = 1, bất đẳng thức của đề bài một bất đẳng thức đúng; tức
ta sẽ chứng minh
3
p
(x
2
+ 1) (y
2
+ 1) (z
2
+ 1)
x + y + z
3
2
+ 1, (1)
với mọi số thực dương x, y, z min {xy,yz,zx} 1.
Thật vy, trước hết, ta chứng minh nhận xét sau: Nhận xét: Với a, b hai số thực dương thỏa
mãn ab 1, ta luôn có:
a
2
+ 1
b
2
+ 1
a + b
2
2
+ 1
!
.
Chứng minh: ab 1 nên
a + b
2
2
1 ab 1 0. Do đó
a
2
+ 1
b
2
+ 1
= (ab 1)
2
+ (a + b)
2
a + b
2
2
1
!
2
+ (a + b)
2
=
a + b
2
2
+ 1
!
2
.
170
2. BÀI TOÁN TÌM HẰNG SỐ TỐT NHẤT
Nhận xét được chứng minh.
Không mất tổng quát, giả sử x y z. Khi đó, từ ràng buộc của đề bài, suy ra x 1.
Đặt t =
x + y + z
3
. Ta
xt =
x (x + y + z)
3
=
x
2
+ xy + xz
3
1 + 1 + 1
3
= 1.
Do đó, áp dụng nhận xét lần lượt cho cặp (x,t) và cặp (y,z), ta được
x
2
+ 1
t
2
+ 1
x + t
2
2
+ 1
!
2
, (2)
và
y
2
+ 1
z
2
+ 1
y + z
2
2
+ 1
!
2
. (3)
Nhân (2) và (3), vế theo vế, ta được
x
2
+ 1
y
2
+ 1
z
2
+ 1
t
2
+ 1
x + t
2
2
+ 1
!
2
y + z
2
2
+ 1
!
2
. (4)
Nhận thấy
x + t
2
·
y + z
2
xt ·
yz 1. Do đó, theo nhận xét, ta
x + t
2
2
+ 1
!
2
y + z
2
2
+ 1
!
2
x + y + z + t
4
2
+ 1
!
4
. (5)
Từ (4) và (5), suy ra
x
2
+ 1
y
2
+ 1
z
2
+ 1
t
2
+ 1
t
2
+ 1
4
.
Do đó
3
p
(x
2
+ 1) (y
2
+ 1) (z
2
+ 1) (t
2
+ 1)
x + y + z
3
2
+ 1.
Suy ra (1) được chứng minh và thế, giá trị k nhỏ nhất cần tìm theo yêu cầu bài k = 1.
Câu 2.16. hiệu (1) bất đẳng thức đã nêu trong đề bài. Giả sử k số thực sao cho bất
đẳng thức (1) đúng với mọi b ba số thực a, b, c độ dài ba cạnh của một tam giác.
Trong (1) thay b = c > 0 ta được
a
2b
+
2b
a + b
+ k ·
2ab + b
2
a
2
+ 2b
2
3
2
+ k.
Cố định b và cho a tiến tới 0
+
ta được 2 +
k
2
3
2
+ k. Suy ra k 1.
Ta sẽ chứng minh k = 1 giá trị nhỏ nhất cần tìm, tức chứng minh với mọi b ba số thực a, b,
c độ dài ba cạnh của một tam giác, ta luôn
a
b + c
+
b
c + a
+
c
a + b
+
ab + bc + ca
a
2
+ b
2
+ c
2
5
2
. (2)
171
2. BÀI TOÁN TÌM HẰNG SỐ TỐT NHẤT
Thật vy, ta có:
(2)
1
a
b + c
+
1
b
c + a
+
1
c
a + b
1
2
+
ab + bc + ca
a
2
+ b
2
+ c
2
b + c a
b + c
+
c + a b
c + a
+
a + b c
a + b
(a + b + c)
2
2 (a
2
+ b
2
+ c
2
)
(b + c a)
2
(b + c) (b + c a)
+
(c + a b)
2
(c + a) (c + a b)
+
(a + b c)
2
(a + b) (a + b c)
(a + b + c)
2
2 (a
2
+ b
2
+ c
2
)
. (3)
Do a, b, c độ dài ba cạnh của một tam giác nên b + c a > 0, c + a b > 0, a + b c > 0.
Do đó tất cả các phân thức nằm vế trái của (3) đều mẫu thức dương. thế, hiệu V T
biểu thức nằm vế trái của (3), theo bất đẳng thức Cauchy Schwarz dạng Engel, ta có:
V T
(a + b + c)
2
2 (a
2
+ b
2
+ c
2
)
.
(b + c a) + (c + a b) + (a + b c) = a + b + c
và
(b + c a) (b + c) + (c + a b) (c + a) + (a + b c) (a + b) = 2
a
2
+ b
2
+ c
2
.
Nên (3) được chứng minh và thế (2) được chứng minh.
Vy k = 1 giá trị cần tìm, theo yêu cầu của đề bài.
Câu 2.17. ràng, các giá trị x = y = z =
1
3
thỏa mãn ràng buộc nêu trong đề bài. thế,
trong bất đẳng thức của đề bài, cho x = y = z =
1
3
ta được
9
3k + 2
9k + 3
2
, k > 0 nên suy
ra k
1
3
.
Tiếp theo, ta chứng minh với k =
1
3
, bất đẳng thức của đề bài một bất đẳng thức đúng; tức
ta sẽ chứng minh
1
x + y +
1
3
+
1
y + z +
1
3
+
1
z + x +
1
3
3, (1)
với mọi số thực dương x, y, z
xy +
yz +
zx = 1.
Thật vy, đặt
T =
x +
y +
z
2
= x + y + z + 2.
Áp dụng bất đẳng thức Cauchy-Schwarz cho hai b 3 số dương, ta
x + y +
1
3
1
3
+
1
3
+ z
x ·
1
3
+
y ·
1
3
+
1
3
·
z
2
=
T
3
hay
1
x + y +
1
3
3z + 2
T
.
Tương tự ta có:
1
y + z +
1
3
3x + 2
T
;
1
z + x +
1
3
3y + 2
T
.
172
2. BÀI TOÁN TÌM HẰNG SỐ TỐT NHẤT
Cộng 3 bất đẳng thức vừa nêu trên, vế với vế, với lưu ý T = x + y + z + 2, ta được
1
x + y +
1
3
+
1
y + z +
1
3
+
1
z + x +
1
3
3z + 2 + 3x + 2 + 3y + 2
T
= 3.
(1) được chứng minh.
Vy k =
1
3
giá trị cần tìm, theo yêu cầu của đề bài.
Câu 2.18. hiệu (1) bất đẳng thức cần chứng minh.
Trong (1) cho a = b = c = 1, suy ra k 729. Ta chứng minh (1) đúng với k = 729. Từ giả thiết
ta
3 |abc| |ab|+ |bc| + |ca| 3
3
q
(abc)
2
abc 1.
Do đó
a
2
+ 4
b
2
+ c
2

b
2
+ 4
a
2
+ c
2

c
2
+ 4
a
2
+ b
2

a
2
+ 8 |bc|
b
2
+ 8 |ac|
c
2
+ 8 |ab|
9
3
(abc)
2
729.
Sử dụng bất đẳng thức Cauchy cho chín số a
2
và 8 số |bc|. Vậy max k = 729.
Câu 2.19. Gọi (1) bất đẳng thức cần tìm. Trong (1) cho a
1
= 1; a
2
= ··· = a
n
=
1
n 1
ta
được c
n
2
.
Ta chứng minh BĐT (1) đúng với c =
n
2
. Tức cần chứng minh
X
1i<jn
|a
i
a
j
|
n
2
n
X
i=1
|a
i
| (2)
Ta thể giả sử a
1
a
2
··· a
n
khi đó tồn tại k với (1 k n 1) sao cho a
1
a
2
···
a
k
0 a
k+1
··· a
n
. Khi đó ta a
1
+ a
2
+ ··· + a
k
= (a
k+1
+ ··· + a
n
) và
X
1i<jn
|a
i
a
j
| =
X
1i<jn
(a
i
a
j
) =
n
X
i=1
(n + 1 2i) a
i
;
n
X
i=1
|a
i
| = 2
n
X
i=1
a
i
.
BĐT (2) trở thành
n
X
i=1
(n + 1 2i) a
i
n
k
X
i=1
a
i
k
X
i=1
(n + 1 2i) a
i
+
n
X
i=k+1
(n + 1 2i) a
i
n
k
X
i=1
a
i
(3)
Ta a
1
a
2
··· a
k
và n + 1 2.1 ··· n + 1 2.k.
Áp dụng bất đẳng thức Chebychev ta được
k
X
i=1
(n + 1 2i) a
i
1
k
"
k
X
i=1
(n + 1 2i)
!
k
X
i=1
a
i
!#
= (n k)
k
X
i=1
a
i
!
. (4)
Tương tự thì a
k+1
··· a
n
và n + 1 2(k + 1) ··· n + 1 2.n nên
n
X
i=k+1
(n + 1 2i) a
i
1
n k
"
n
X
i=k+1
(n + 1 2i)
!
n
X
i=k+1
a
i
!#
= k
n
X
i=k+1
a
i
!
= k
k
X
i=1
a
i
!
. (5)
Từ (4) và (5) suy ra BĐT (3) đúng.
Vy max c
n
=
n
2
.
173
2. BÀI TOÁN TÌM HẰNG SỐ TỐT NHẤT
Câu 2.20. Ta chứng minh max a
k
k=1,n
< 2 và max b
k
k=1,n
<
2
n1
.
Thật vy, đặt L = max a
k
k=1,n
, từ (ii) suy ra L
2
1 + L L < 2.
Đặt b
m
= max b
k
k=1,n
, từ (i) dễ thấy
b
k
(k 1) b
m
+ (m k) b
1
m 1
, nếu 1 k m và
b
k
(k m) b
n
+ (n k) b
m
n m
, nếu m k n.
Suy ra b
k
>
(k 1) b
m
m 1
nếu 1 k m và b
k
(n k) b
m
n m
nếu m k n.
Do đó
1 =
n
X
k=1
b
k
=
m
X
k=1
b
k
+
n
X
k=m+1
b
k
>
1
m 1
m
X
k=1
(k 1)
!
b
m
+
1
n m
n
X
k=m+1
(n k)
!
b
m
=
n 1
2
.b
m
b
m
<
2
n 1
.
Đặt x
0
= 1; x
k
= 1+
P
k
i=1
a
i
b
i
, k = 1,n thì x
k
x
k1
= a
k
b
k
. Từ (ii) suy ra a
2
k
x
k
a
k
x
k
.
Từ max a
k
k=1,n
< 2 suy ra x
k
x
k1
< 2b
k
. Do đó với mọi k = 1,n thì
x
k
x
k1
< b
k
.
x
k
x
k
+
x
k1
= b
k
1
2
+
x
k
x
k1
2
x
k
+
x
k1
2
!
< b
k
1
2
+
b
k
4
< b
k
1
2
+
1
2 (n 1)
.
Lấy tổng k từ 1 đến n ta được
a
n
x
n
<
x
0
+
n
X
k=1
b
k
1
2
+
1
2 (n 1)
=
3
2
+
1
2 (n 1)
.
Cho n + ta được M = a
n
3
2
.
Cuối cùng chọn b a
k
= 1 +
k
2n
; b
k
=
1
n
thỏa các điều kiện đề bài. Vy max M =
3
2
.
174
| 1/174

Preview text:

Mục lục 1
Các bất đẳng thức cổ điển 3 1
Bất đẳng thức AM - GM . . . . . . . . . . . . . . . . . . . . . . . . . . . . . . . . 3 I.
Bất đẳng thức AM - GM . . . . . . . . . . . . . . . . . . . . . . . . . . . . 3 II.
Một số ví dụ áp dụng . . . . . . . . . . . . . . . . . . . . . . . . . . . . . . 5 III. Bài tập
. . . . . . . . . . . . . . . . . . . . . . . . . . . . . . . . . . . . . 13 2
Bất đẳng thức Cauchy - Schwarz
. . . . . . . . . . . . . . . . . . . . . . . . . . . 19 I.
Bất đẳng thức Cauchy-Schwarz dạng đa thức
. . . . . . . . . . . . . . . . 19 II.
Bất đẳng thức Cauchy-Schwarz dạng phân thức . . . . . . . . . . . . . . . 19 III.
Các ví dụ minh họa . . . . . . . . . . . . . . . . . . . . . . . . . . . . . . . 19 IV. Bài tập
. . . . . . . . . . . . . . . . . . . . . . . . . . . . . . . . . . . . . 27 3
Một số bất đẳng thức khác . . . . . . . . . . . . . . . . . . . . . . . . . . . . . . . 31 I.
Bất đẳng thức Schur . . . . . . . . . . . . . . . . . . . . . . . . . . . . . . 31 1.
Bất đẳng thức Schur . . . . . . . . . . . . . . . . . . . . . . . . . 31 2.
Các trường hợp đặc biệt . . . . . . . . . . . . . . . . . . . . . . . 31 3.
Bất đẳng thức Schur mở rộng . . . . . . . . . . . . . . . . . . . . 31 4.
Các ví dụ . . . . . . . . . . . . . . . . . . . . . . . . . . . . . . . 31 II. Bất đẳng thức Holder
. . . . . . . . . . . . . . . . . . . . . . . . . . . . . 34 1.
Bất đẳng thức Holder . . . . . . . . . . . . . . . . . . . . . . . . 34 2.
Trường hợp đặc biệt . . . . . . . . . . . . . . . . . . . . . . . . . 34 3.
Ví dụ minh họa . . . . . . . . . . . . . . . . . . . . . . . . . . . . 34 III.
Bất đẳng thức Chebyshev . . . . . . . . . . . . . . . . . . . . . . . . . . . 35 1.
Bất đẳng thức Chebyshev . . . . . . . . . . . . . . . . . . . . . . 35 2.
Ví dụ minh họa . . . . . . . . . . . . . . . . . . . . . . . . . . . . 36 IV. Bài tập
. . . . . . . . . . . . . . . . . . . . . . . . . . . . . . . . . . . . . 36 4 Phương pháp quy nạp
. . . . . . . . . . . . . . . . . . . . . . . . . . . . . . . . . 38 I.
Lý thuyết . . . . . . . . . . . . . . . . . . . . . . . . . . . . . . . . . . . . 38 II.
Ví dụ minh họa . . . . . . . . . . . . . . . . . . . . . . . . . . . . . . . . . 38 5
Phương pháp phân tích bình phương SOS
. . . . . . . . . . . . . . . . . . . . . . 42 I.
Lý thuyết . . . . . . . . . . . . . . . . . . . . . . . . . . . . . . . . . . . . 42 1.
Một số tiêu chuẩn đánh giá . . . . . . . . . . . . . . . . . . . . . 42 2.
Một số biểu diễn cơ sở . . . . . . . . . . . . . . . . . . . . . . . . 42 II.
Các ví dụ . . . . . . . . . . . . . . . . . . . . . . . . . . . . . . . . . . . . 43 III. Bài tập
. . . . . . . . . . . . . . . . . . . . . . . . . . . . . . . . . . . . . 46 6
Phương pháp dồn biến . . . . . . . . . . . . . . . . . . . . . . . . . . . . . . . . . 48 I.
Lý thuyết . . . . . . . . . . . . . . . . . . . . . . . . . . . . . . . . . . . . 48 II.
Ví dụ minh họa . . . . . . . . . . . . . . . . . . . . . . . . . . . . . . . . . 48 III. Bài tập
. . . . . . . . . . . . . . . . . . . . . . . . . . . . . . . . . . . . . 51 2
Các phương pháp chứng minh bất đẳng thức hiện đại 53 1 Phương pháp p, q, r
. . . . . . . . . . . . . . . . . . . . . . . . . . . . . . . . . . 54 I.
Lý thuyết . . . . . . . . . . . . . . . . . . . . . . . . . . . . . . . . . . . . 54 1.
Bất đẳng thức Schur . . . . . . . . . . . . . . . . . . . . . . . . . 54 1 MỤC LỤC 2.
Một số biểu diễn đa thức đối xứng ba biến qua p, q, r . . . . . . 54 3.
Một số đánh giá giữa p, q, r . . . . . . . . . . . . . . . . . . . . . 55 II.
Một số ví dụ . . . . . . . . . . . . . . . . . . . . . . . . . . . . . . . . . . . 55 III. Bài tập
. . . . . . . . . . . . . . . . . . . . . . . . . . . . . . . . . . . . . 56 2
Phương pháp sử dụng tiếp tuyến và cát tuyến . . . . . . . . . . . . . . . . . . . . 58 I.
Lý thuyết . . . . . . . . . . . . . . . . . . . . . . . . . . . . . . . . . . . . 58 1.
Hàm lồi - Dấu hiệu hàm lồi . . . . . . . . . . . . . . . . . . . . . 58 2.
Bất đẳng thức tiếp tuyến - Bất đẳng thức cát tuyến . . . . . . . 58 II.
Các ví dụ minh họa . . . . . . . . . . . . . . . . . . . . . . . . . . . . . . . 59 III. Bài tập
. . . . . . . . . . . . . . . . . . . . . . . . . . . . . . . . . . . . . 66 3 Một số chuyên đề 68 1
Ứng dụng điều kiện có nghiệm của phương trình bậc ba trong chứng minh bất
đẳng thức . . . . . . . . . . . . . . . . . . . . . . . . . . . . . . . . . . . . . . . . 68 I.
Lý thuyết . . . . . . . . . . . . . . . . . . . . . . . . . . . . . . . . . . . . 68 1.
Mở đầu . . . . . . . . . . . . . . . . . . . . . . . . . . . . . . . . 68 2.
Một số kết quả . . . . . . . . . . . . . . . . . . . . . . . . . . . . 68 II.
Ví dụ minh họa . . . . . . . . . . . . . . . . . . . . . . . . . . . . . . . . . 70 III. Bài tập
. . . . . . . . . . . . . . . . . . . . . . . . . . . . . . . . . . . . . 74 2
Bài toán tìm hằng số tốt nhất trong bất đẳng thức . . . . . . . . . . . . . . . . . 75 I.
Lý thuyết . . . . . . . . . . . . . . . . . . . . . . . . . . . . . . . . . . . . 75 II.
Ví dụ minh họa . . . . . . . . . . . . . . . . . . . . . . . . . . . . . . . . . 75 III. Bài tập
. . . . . . . . . . . . . . . . . . . . . . . . . . . . . . . . . . . . . 82 1
Các bất đẳng thức cổ điển 86 1
Bất đẳng thức AM-GM . . . . . . . . . . . . . . . . . . . . . . . . . . . . . . . . . 86 2
Bất đẳng thức Cauchy-Schwarz . . . . . . . . . . . . . . . . . . . . . . . . . . . . 109 3
Một số bất đẳng thức khác . . . . . . . . . . . . . . . . . . . . . . . . . . . . . . . 124 2
Một số phương pháp chứng minh bất đẳng thức 129 1
Phương pháp quy nạp . . . . . . . . . . . . . . . . . . . . . . . . . . . . . . . . . 129 2
Phương pháp phân tích bình phương SOS
. . . . . . . . . . . . . . . . . . . . . . 130 3
Phương pháp dồn biến . . . . . . . . . . . . . . . . . . . . . . . . . . . . . . . . . 135 4 Phương pháp p, q, r
. . . . . . . . . . . . . . . . . . . . . . . . . . . . . . . . . . 148 5
Phương pháp tiếp tuyến và cát tuyến . . . . . . . . . . . . . . . . . . . . . . . . . 150 3 Một số chuyên đề 156 1
Ứng dụng đều kiện có nghiệm của phương trình bậc ba . . . . . . . . . . . . . . . 156 2
Bài toán tìm hằng số tốt nhất . . . . . . . . . . . . . . . . . . . . . . . . . . . . . 159 2 Chương 1
Các bất đẳng thức cổ điển
§1. Bất đẳng thức AM - GM
Bất đẳng thức AM − GM là bất đẳng thức cổ điển được sử dụng nhiều trong các bài toán chứng a1 + a2 + · · · + an
minh bất đẳng thức. Ta biết trung bình cộng của nsố thực a1,a2, · · · ,an là số n √ √
và trung bình nhân của n số đó là n a1a2 · · · an (với điều kiện là n a1a2 · · · an tồn tại). Bất đẳng
thức AM − GM cho chúng ta đánh giá giữa trung bình cộng của các số thực không âm và trung
bình nhân của chúng. Cụ thể như sau: I. Bất đẳng thức AM - GM
Định lí 1. Cho n số thực không âm a1, a2, · · · , an. ta có a1 + a2 + · · · + an √ ≥ n a1 · a2 · · · an. n
Đẳng thức xảy ra khi a1 = a2 = · · · = an.
Chứng minh. Có nhiều cách đề chứng minh bất đẳng thức AM − GM , dưới đây ta sẽ chứng
minh bất đẳng thức AM − GM bằng phương pháp quy nạp.
Trước hết ta chứng minh bất đẳng thức AM − GM cho trường hợp n = 2. Tức là, cần chứng minh a1 + a2 √ ≥ a1 · a2 2
Bất đẳng thức này tương đương với √ √ √
a1 + a2 ≥ 2 a1a2 ⇔ ( a1 − a2)2 ≥ 0.
Bất đẳng thức cuối hiển nhiên đúng. Đẳng thức xảy ra khi a1 = a2.
Tiếp theo ta chứng minh cho trường hợp n = 4. Tức là cần chứng minh a1 + a2 + a3 + a4 √ ≥ 4 a1 · a2 · a3 · a4. 4
Áp dụng trường hợp n = 2 ta có a1 + a2 √ ≥ a1 · a2 2 và a3 + a4 √ ≥ a3 · a4. 2 Do đó a1 + a2 a3 + a4 √ √ a + 1 + a2 + a3 + a4 a1a2 + a3a4 √ = 2 2 ≥ ≥ 4 a1a2a3a4. 4 2 2 3
1. BẤT ĐẲNG THỨC AM - GM
Nên trường hợp n = 4 được chứng minh.
Tiếp đến ta chứng minh trường hợp n = 3, tức là chứng minh a1 + a2 + a3 √ ≥ 3 a1 · a2 · a3 3 a1 + a2 + a3 Đặt a4 =
. Áp dụng cho trường hợp n = 4 ta có 3 a1 + a2 + a3 + a4 √ ≥ 4 a1 · a2 · a3 · a4, 4 hay a a 1 + a2 + a3 1 + a2 + a3 + r 3 a ≥ 4 a 1 + a2 + a3 1 · a2 · a3 · 4 3 Suy ra a1 + a2 + a3 √ ≥ 3 a1 · a2 · a3 (đpcm). 3
Để chứng minh cho trường hợp tổng quát ta chứng minh theo hai bước sau:
Bước 1: Ta chứng minh bất đẳng thức đúng với n = 2m
+) Với m = 1, ta có n = 2nên bất đẳng thức đúng với m = 1
+) Giả sử bất đẳng thức đúng với n = 2m−1, ta chứng minh bất đẳng thức đúng với n = 2m. Tức là a1 + a2 + · · · + a √
2m−1 + · · · + an ≥ n a1a2 · · · an. (1) n Đặt a a x =
1 + a2 + · · · + a2m−1 , y = 2m−1+1 + a2m−1+2 + · · · + a2m 2m−1 2m−1
Theo giả thiết quy nạp ta có √ √
x ≥ 2m−1 a1a2 · · · a2m−1,y ≥ 2m−1 a2m−1+1 · · · an.
Áp dụng cho trường hợp n = 2 ta có: x + y √ ≥ xy 2 hay a1 + a2 + · · · + a √
2m−1 + a2m−1+1 + · · · + an ≥ 2m a1a2 · · · an 2m Hay (1) được chứng minh.
Bước 2: Ta chứng minh nếu bất đẳng thức đúng với n ≥ 2 thì cũng đúng với n − 1 Gải sử a1 + a2 + · · · + an √ ≥ n a1a2 · · · an n Ta chứng minh a1 + a2 + · · · + an−1 √
≥ n−1 a1 · a2 · · · an−1. n − 1 a1 + a2 + · · · + an−1 Thật vậy: Đặt an =
. ÁP dụng bất đẳng thức AM-GM cho n số ta có n − 1 a1 + a2 + · · · + an √ ≥ n a1a2 · · · an, n hay a a 1 + a2 + · · · + an−1 1 + a2 + · · · + r n − 1 a ≥ n a 1 + a2 + · · · + an−1 1a2 · · · an−1 · . n n − 1 4
1. BẤT ĐẲNG THỨC AM - GM Suy ra a1 + a2 + · · · + an−1 √
≥ n−1 a1 · a2 · · · an−1 (đpcm). n − 1
Từ hai bước trên ta có bất đẳng thức AM − GM được chứng minh.
Hệ quả 1. Cho các số thực dương a1,a2, · · · ,an. Ta có 1 1 1 n2 + + · · · + ≥ . a1 a2 an a1 + a2 + · · · + an
Đẳng thức xảy ra khi a1 = a2 = · · · = an. II. Một số ví dụ áp dụng
Ví dụ 1.1. Cho a,b,c > 0 thỏa a2 + b2 + c2 = 3. Chứng minh rằng a5 + b5 + c5 ≥ 3.
Áp dụng bất đẳng thức AM-GM ta có
a5 + a5 + 1 + 1 + 1 ≥ 3a2 hay 2a5 + 3 ≥ 3a2. Tương tự
2b5 + 3 ≥ 3b2 và 2c5 + 3 ≥ 3c2.
Cộng ba bất đẳng thức trên ta có đpcm.
Nhận xét 1. Ta có bài toán tổng quát như sau:
Cho a,b,c > 0 thỏa mãn a + b + c = 3 (hoặc abc = 1) và m,n ∈ N,m ≥ n. Khi đó am + bm + cm ≥ an + bn + cn (1).
Bất đẳng thức (1) còn đúng khi m,n là các số hữu tỉ dương. Và ta có thể tổng quát 3 biến thành k biến.
Ví dụ 1.2. Cho a,b,c > 0 thỏa a + 4b + 9c = 6.Chứng minh rằng 1 a3 + b3 + c3 ≥ . 6
Xét x, y, z là các số thực dương. Áp dụng bất đẳng thức AM-GM ta có
a3 + 2x3 = a3 + x3 + x3 ≥ 3x2a,
đẳng thức xảy ra khi a = x. Tương tự ta cũng có:
b3 + 2y3 ≥ 3y2b, c3 + 2z3 ≥ 3y2c.
Đẳng thức xảy ra khi b = y, c = z. Cộng các bất đẳng thức trên theo vế ta được
a3 + b3 + c3 ≥ 3(x2a + y2b + z2c) − 2(x3 + y3 + z3). 5
1. BẤT ĐẲNG THỨC AM - GM Ta chọn x, y, z sao cho  1 x = 
x + 4y + 9z = a + 4b + 9c = 6  6     1 x2 y2 z2 ⇒ y = . 3  = = = t2  1 4 9   1   z = 2 Do đó 1
a3 + b3 + c3 ≥ 3t2(a + 4b + 9c) − 2(x3 + y3 + z3) = . 6
Ví dụ 1.3. Cho a, b, c > 0 thỏa ab + bc + ca = 3. Chứng minh rằng a3 + b3 + c3 ≥ 3.
Áp dụng bất đẳng thức AM-GM ta có a3 + b3 + 1 ≥ 3ab b3 + c3 + 1 ≥ 3bc c3 + a3 + 1 ≥ 3ca.
Cộng ba bất đẳng thức trên ta có đpcm.
Ví dụ 1.4. Cho các số thực dương a, b, c có tổng bình phương bằng 3. Chứng minh rằng ab bc ca + + ≥ 3. c a b
Gọi P là vế trái của bất đẳng thức cần chứng minh, ta có ab bc ca 2 P 2 = + + c a b a2b2 c2b2 c2a2 = + + + 2(a2 + b2 + c2) c2 a2 b2 1 a2b2 c2b2 1 c2b2 c2a2 1 a2b2 c2a2 = + + + + + + 6 2 c2 a2 2 a2 b2 2 c2 b2 ≥ b2 + c2 + a2 + 6 = 9.
Suy ra P ≥ 3. Đẳng thức xảy ra khi a = b = c = 1.
Ví dụ 1.5. Cho a, b, c > 0 và a + b + c = abc. Chứng minh rằng : a b c + + ≥ 1. b3 c3 a3
Ta có bất đẳng thức cần chứng minh tương đương với: a b c abc + + ≥ a + b + c. b3 c3 a3 6
1. BẤT ĐẲNG THỨC AM - GM Hay a2c b2a c2b + + ≥ a + b + c. (1) b2 c2 a2
Áp dụng bất đẳng thức Cô si cho ba số ta được : r a2c b2a 3 a2c b2a + + c ≥ 3. . .c = 3a. b2 c2 b2 c2 Tương tự : b2a c2b c2b a2c + + a ≥ 3b ; + + b ≥ 3c. c2 a2 a2 b2
Cộng ba bất đẳng thức trên ta có được bất đẳng thức (1). 1
Bài toán được chứng minh. Đẳng thức xảy ra ⇔ a = b = c = √ . 3
Ví dụ 1.6. Cho a, b, c > 0. Chứng minh rằng : a5 b5 c5 + + ≥ a3 + b3 + c3. b2 c2 a2
Áp dụng bất đẳng thức Cô si : r a5 a5 + ab2 ≥ 2 ab2 = 2a3. b2 b2 Tương tự : b5 c5 + bc2 ≥ 2b3; + ca2 ≥ 2c3. c2 a2
Công 3 bất đẳng thức trên lại với nhau ta được : a5 b5 c5 + +
≥ a3 + b3 + c3 + a3 + b3 + c3 − ab2 − bc2 − ca2 . b2 c2 a2 Nên ta cần chứng minh :
a3 + b3 + c3 − ab2 − bc2 − ca2 ≥ 0 ⇔ a3 + b3 + c3 ≥ ab2 + bc2 + ca2. (1)
Áp dụng bất đẳng thức Cô si : √
a3 + b3 + b3 ≥ 3 3 a3b3b3 = 3ab2 ⇒ a3 + 2b3 ≥ 3ab2 Tương tự :
b3 + 2c3 ≥ 3bc2; c3 + 2a3 ≥ 3ca2.
Công 3 bất đẳng thức trên lại với nhau ta có (1).
Vậy bài toán được chứng minh.
Ví dụ 1.7. Cho các số thực dương a,b,c. Chứng minh rằng a4 b4 c4 a + b + c + + ≥ . b2 (c + a) c2 (a + b) a2 (b + c) 2 7
1. BẤT ĐẲNG THỨC AM - GM
Áp dụng bất đẳng thức AM-GM ta có a4 b b c + a + + + ≥ 2a b2 (c + a) 2 2 4 hay a4 c + a + b + ≥ 2a. b2 (c + a) 4 Tương tự, ta cũng có b4 a + b c4 b + c + c + ≥ 2b và + a + ≥ 2c. c2 (a + b) 4 a2 (b + c) 4
Cộng ba bất đẳng thức trên theo vế ta có đpcm.
Ví dụ 1.8 (BĐT Nesbit cho 3 số). Cho a, b, c > 0. Chứng minh rằng a b c 3 + + ≥ . b + c c + a a + b 2
Bất đẳng thức cần chứng minh tương đương với a b c 9 + 1 + + 1 + + 1 ≥ b + c c + a a + b 2 Hay 1 1 1 9 (a + b + c) + + ≥ (1). a + b b + c c + a 2 Ta có 1 1 1 9 9 + + ≥ = a + b b + c c + a a + b + b + c + c + a 2 (a + b + c) Nên (1) đúng.
Ví dụ 1.9. Cho các số thực dương a, b, c thỏa a + b + c = 1. Chứng minh rằng 1 1 1 1 + + + ≥ 30. a2 + b2 + c2 ab bc ca Ta có: (a + b + c)2 1 ab + bc + ca ≤ = 3 3 1 1 1 9 + + ≥ ab bc ca ab + bc + ca 1 1 1 9 + + ≥ = 9. a2 + b2 + c2 ab + bc + ca ab + bc + ca (a + b + c)2 Do đó 1 9 V T ≥ + a2 + b2 + c2 ab + bc + ca 1 1 1 7 7 = + + + ≥ 9 + = 30. a2 + b2 + c2 ab + bc + ca ab + bc + ca ab + bc + ca 1 3
Ta có điều phải chứng minh. 8
1. BẤT ĐẲNG THỨC AM - GM
Ví dụ 1.10. Cho các số thực dương x,y,z thỏa mãn : xy + yz + zx = 3.Chứng minh rằng: 1 4 3 + ≥ . xyz (x + y)(y + z)(z + x) 2 Ta có:
x (y + z) + y (z + x) + z (x + y) 3
pxyz (x + y) (y + z) (z + x) ≤ = 2. 3 Suy ra 4 xyz ≥ (x + y) (y + z) (z + x) 2 Do đó 1 xyz 1 xyz 1 1 3 V T ≥ + ≥ + + ≥ 1 + = . xyz 2 2xyz 2 2xyz 2 2
Bài toán được chứng minh.
Ví dụ 1.11. (IMO 2012) Cho n ≥ 3 và các số thực dương a2, a3, . . . , an thỏa mãn
a2a3 · · · an = 1. Chứng minh rằng
(1 + a2)2 (1 + a3)3 · · · (1 + an)n > nn.
Áp dụng bất đẳng thức AM-GM ta có 1 1 1 k kka (1 + a k k )k = + + · · · + + ak ≥ . k − 1 k − 1 k − 1 (k − 1)k−1 Suy ra 22 33 44 nn
(1 + a2)2 . (1 + a3)3 · · · (1 + an)n ≥ · · · · · 11 22 33
(n − 1)n a1a2 · · · an = nn.
Ta thấy không có đẳng thức xảy ra. Vậy bài toán được chứng minh.
Ví dụ 1.12. Cho các số thực dương a, b, c có tích bằng 1. Chứng minh rằng 3 6 1 + ≥ . a + b + c ab + bc + ca
Bất đẳng thức cần chứng minh tương đương với 3(ab + bc + ca) ab + bc + ca + ≥ 6. (1) a + b + c
Áp dụng bất đẳng thức AM-GM ta có s 3(ab + bc + ca) 3(ab + bc + ca)2 ab + bc + ca + ≥ 2 . a + b + c a + b + c 9
1. BẤT ĐẲNG THỨC AM - GM Mặt khác
(ab + bc + ca)2 ≥ 3(ab · bc + bc · ca + ca · ab) = 3abc(a + b + c) = 3(a + b + c). Suy ra 3(ab + bc + ca) ab + bc + ca + ≥ 6. a + b + c
Vậy bài toán được chứng minh.
Ví dụ 1.13. (Moldova TST 2014) Cho các số thực dương a,b,c thỏa mãn abc = 1. Chứng minh rằng ab bc ca 9 a3 + b3 + c3 + + + ≥ . a2 + b2 b2 + c2 c2 + a2 2
Bất đẳng thức cần chứng minh tương đương với 2ab 2bc 2ca 2 a3 + b3 + c3 + + + ≥ 9 (1). a2 + b2 b2 + c2 c2 + a2
Ta có x3 + y3 ≥ x2y + y2x với mọi x,y > 0 nên c (a2 + b2) b (c2 + a2) a (b2 + c2) a3 + b3 + c3 ≥ + + 2 2 2 Suy ra c (a2 + b2) 2ab b (c2 + a2) 2bc a (b2 + c2) 2ca V T (1) ≥ + + + + + +3abc ≥ 9. 2 a2 + b2 2 b2 + c2 2 c2 + a2
Bài toán được chứng minh.
Ví dụ 1.14. Chứng minh rằng mỗi số thực dương a,b,c ta luôn có: ab bc ca a + b + c + + ≤ . a + 3b + 2c b + 3c + 2a c + 3a + 2b 6 Ta có : ab ab ab 1 1 1 = ≤ . + + . a + 3b + 2c (a + c) + (b + c) + 2b 9 a + c b + c 2b Tương tự : bc bc 1 1 1 ac ac 1 1 1 ≤ + + , ≤ + + . b + 3c + 2a 9 a + b a + c 2c c + 3a + 2b 9 b + c a + b 2a
Cộng vế theo vế ta được ab bc ca 1 bc + ac bc + ab ab + ac 1 + + ≤ + + + (a + b + c) . a + 3b + 2c b + 3c + 2a c + 3a + 2b 9 a + b a + c b + c 18 Hay ab bc ca 1 1 a + b + c + + ≤ (a + b + c) + (a + b + c) = . a + 3b + 2c b + 3c + 2a c + 3a + 2b 9 18 6 10
1. BẤT ĐẲNG THỨC AM - GM
Ví dụ 1.15. Cho các số thực dương a,b,c thỏa a + b + c = 3. Chứng minh rằng ab bc ca 3 √ + √ + √ ≤ . c2 + 3 a2 + 3 b2 + 3 2 Ta có
3 (ab + bc + ca) ≤ (a + b + c)2 = 9 ⇒ ab + bc + ca ≤ 3 Suy ra 1 1 1 1 1 1 √ ≤ √ = ≤ + . c2 + 3 c2 + ab + bc + ca p(a + c)(b + c) 2 a + c b + c Do đó: ab 1 ab ab √ ≤ + c2 + 3 2 a + c b + c Tương tự: bc 1 bc bc ca 1 ca ca √ ≤ + và √ ≤ + a2 + 3 2 a + b a + b b2 + 3 2 b + a b + c
Cộng ba bất đẳng thức trên theo vế ta có: ab bc ca 1 3 √ + √ + √ ≤ (a + b + c) = . c2 + 3 a2 + 3 b2 + 3 2 2
Ví dụ 1.16. (IMO 2005) Cho các số thực dương x,y,z thỏa xyz ≥ 1. Chứng minh rằng x5 − x2 y5 − y2 z5 − z2 + + ≥ 0. x5 + y2 + z2 y5 + z2 + x2 z5 + x2 + y2
Bất đẳng thức cần chứng minh tương đương với x5 − x2 y5 − y2 z5 − z2 1 − + 1 − + 1 − ≤ 3 x5 + y2 + z2 y5 + z2 + x2 z5 + x2 + y2 1 1 1 3 ⇔ + + ≤ . (1) x5 + y2 + z2 y5 + z2 + x2 z5 + x2 + y2 x2 + y2 + z2 Ta có 2 2 x4 2x4 + (y2 + z2) 2 (x2 + y2 + z2) x5 + y2 + z2 ≥ + y2 + z2 ≥ ≥ . . yz y2 + z2 3 y2 + z2 Do đó 1 3 y2 + z2 ≤ . x5 + y2 + z2 2 (x2 + y2 + z2)2 Chứng minh tương tự 1 3 z2 + x2 1 3 x2 + y2 ≤ và ≤ . y5 + z2 + x2 2 (x2 + y2 + z2)2 z5 + x2 + y2 2 x2 + y2 + z2 Suy ra 1 1 1 3 + + ≤ x5 + y2 + z2 y5 + z2 + x2 z5 + x2 + y2 x2 + y2 + z2 Hay (1) đúng. 11
1. BẤT ĐẲNG THỨC AM - GM
Ví dụ 1.17. (IMO Shortlist 2009) Cho các số thực dương a,b,c thỏa ab+bc+ca ≤ 3abc. Chứng minh rằng s s s a2 + b2 b2 + c2 c2 + a2 √ √ √ √ + + + 3 ≤ 2 a + b + b + c + c + a . a + b b + c c + a Ta có s s s r p 2 (a + b)2 a2 + b2 2ab a2 + b2 2ab 2(a + b) = = 2 + ≥ + . a + b a + b a + b a + b a + b Suy ra s s s r r 2ab 2bc r 2ca a2 + b2 b2 + c2 c2 + a2 V P ≥ + + + + + . a + b b + c c + a a + b b + c c + a
Mặt khác áp dụng bất đẳng thức 1 1 1 + + (x + y + z)2 ≥ 27 x2 y2 z2 ta suy ra √ v u 1 x + y + z ≥ 3 3u t 1 1 1 + + x2 y2 z2 Do đó r r v 2ab 2bc r 2ca √ u 1 + + ≥ 3 3u a + b b + c c + a u r !2 r !2 2 r u a + b b + c c + a t + + 2ab 2bc 2ca r 3abc = 3 = 3. ab + bc + ca Từ đó, ta có đpcm.
Ví dụ 1.18. Cho các số thực dương a,b,c. Chứng minh rằng a3 b3 c3 a + b + c + + ≥ . a2 + b2 b2 + c2 c2 + a2 2 Ta có a3 a (a2 + b2) − ab2 ab2 b = = a − ≥ a − . a2 + b2 a2 + b2 a2 + b2 2 Tương tự b3 c c3 a ≥ b − và ≥ c − . b2 + c2 2 c2 + a2 2
Cộng các bất đẳng thức trên theo vế ta có đpcm. 12
1. BẤT ĐẲNG THỨC AM - GM
Ví dụ 1.19. Cho các số thực a,b,c thỏa abc < 0 và a + b + c = 0. Chứng minh rằng: 1 1 1 12abc − 8 + + (1 − ab − bc − ca) + ≥ 16. a b c ab + bc + ca
Gọi P là vế trái của bất đẳng thức.
Đặt m = − (ab + bc + ca) ,n = −abc
Do a + b + c = 0 ⇒ 2(ab + bc + ca) = − (a2 + b2 + c2) < 0 ⇒ m,n > 0 Khi đó: m(1 + m) 12n + 8 P = + n m
Áp dụng bất đẳng thức Cô sita có:
m3 + 8n2 + 8n ≥ 12mn và m2 + 4n2 ≥ 4mn Suy ra m3 + m2 + 12n2 + 8n ≥ 16mn Do đó: m(1 + m) 12n + 8 P = + ≥ 16 n m
Đẳng thức xảy ra khi và chỉ khi m = 2,n = 1, tức là a,b,c là ba nghiệm của phương trình √ −1 ± 5
x3 − 2x + 1 = 0 ⇔ (x − 1)(x2 + x − 1) = 0 ⇔ x = 1,x = . 2 III. Bài tập
Bài 1.1. Cho các số thực dương a,b,c. Chứng minh rằng √ 3
a) (1 + a) (1 + b) (1 + c) ≥ 1 + 3 abc . a b c a + b + c b) 1 + 1 + 1 + ≥ 2 1 + √ . b c a 3 abc
Bài 1.2. Cho các số thực dương a1, a2, · · · , an. Chứng minh rằng √
(1 + a1)(1 + a2) · · · (1 + an) ≥ (1 + n a1 · a2 · · · an)n .
Bài 1.3. Cho các số thực a, b, c thỏa mãn a + b + c = 1. Chứng minh rằng
(1 + a) (1 + b) (1 + c) ≥ 64abc.
Bài 1.4. Cho 2n số thực dương a1,a2, . . . ,an,b1,b2, . . . ,bn. Chứng minh rằng √ n
p(a1 + b1) (a2 + b2) · · · (an + bn) ≥ n a1a2 · · · an + npb1b2 · · · bn. 13
1. BẤT ĐẲNG THỨC AM - GM
Bài 1.5. (BĐT AM-GM suy rộng) Cho ai ≥ 0 (i = 1,n) và các số hữu tỉ dương αi thỏa mãn n
P αi = 1. Chứng minh rằng: i=1 n
X αiai ≥ aα1 · aα2 · · · aαn. 1 2 n i=1
Bài 1.6. Cho n số thực dương a1, a2, · · · , an và số nguyên dương k. Chứng minh rằng ak + ak + · · · + ak a k 1 2 n ≥ 1 + a2 + · · · + an . n n
Bài 1.7. Cho a, b, c > 0. Chứng minh rằng 1 1 1 1 1 1 + + ≥ + + . a + 3b b + 3c c + 3a a + 2b + c b + 2c + a c + 2a + b
Bài 1.8. Cho các số thực a,b,c > 0 thỏa ab + bc + ca ≤ 3abc. Chứng minh rằng 1 1 1 3 + + √ √ ≤ . √ √ 4 √ √ 4 4 16 4 a + 4 b 4 b + 4 c ( 4 c + 4 a)
Bài 1.9. Cho a,b,c > 0. Chứng minh rằng a b c b c a 2 + + ≥ 1 + + + . b + 2c c + 2a a + 2b b + 2a c + 2b a + 2c
Bài 1.10. Cho x, y, z là các số thực dương thoả mãn x2 + y2 + z2 = 12.Tìm giá trị nhỏ nhất của biểu thức 1 1 1 P = √ + + √ . 1 + x3 p1 + y3 1 + z3
Bài 1.11. Cho 3 số thực dương a,b,c thoả mãn a + b + c = 3 . Chứng minh rằng : a b c 3 + + ≥ . 1 + b2 1 + c2 1 + a2 2 3
Bài 1.12. Cho các số thực dương a,b,c thỏa a + b + c = . Chứng minh rằng: 2 a2 b2 c2 3 + + ≥ . a + 2b2 b + 2c2 c + 2a2 4
Bài 1.13. Chứng minh rằng nếu xy + yz + zx = 5 thì 3x2 + 3y2 + z2 ≥ 10
Bài 1.14. Cho a,b,c > 0. Chứng minh bất đẳng thức a3 b3 c3 a + b + c + + ≥ . (a + 2b) (b + 2c) (b + 2c) (c + 2a) (c + 2a) (a + 2b) 9 14
1. BẤT ĐẲNG THỨC AM - GM
Bài 1.15. Cho các số thực dương a,b,c > 0 thỏa abc = 1. Chứng minh rằng a4 b4 c4 + + ≥ 1. b2 (c + 2) c2 (a + 2) a2 (b + 2)
Bài 1.16. Chứng minh rằng nếu a, b, c > 0 thì : r r r ! a + b b + c r c + a r c r a b + + ≥ 2 + + . c a b a + b b + c a + c
Bài 1.17. Cho các số thực a,b,cthỏa a2 + b2 + c2 = 3. Chứng minh rằng a4 b4 c4 + + ≥ 1. b + 2 c + 2 a + 2
Bài 1.18. Cho các số thực dương a,b,c thỏa a2 + b2 + c2 = 3 . Chứng minh rằng 4 4 4 + 1 + 1 + 1 ≥ 3 (a + b + c)2 . a2 + b2 b2 + c2 c2 + a2
Bài 1.19. Cho các số thực dương a,b,c thỏa: √ √ √ 7 − abc a2 + b2 + b2 + c2 + c2 + a2 = √ . 2 a2 b2 c2 3 Chứng minh rằng: + + ≥ . b + c c + a a + b 2
Bài 1.20. Chứng minh rằng nếu a,b,c > 0 và thỏa mãn a.b.c = 1 thì 1 1 1 1 + + ≤ . a2 + 2b2 + 3 b2 + 2c2 + 3 c2 + 2a2 + 3 2 1 1 1
Bài 1.21. (Baltic Way 2014) Cho các số thực dương a,b,c thỏa + + = 3. Chứng minh a b c rằng 1 1 1 3 √ + √ + √ ≤ √ . a3 + b b3 + c c3 + a 2
Bài 1.22. (USA 2011) Với a, b, c là các số thực dương thỏa a2 + b2 + c2 + (a + b + c)2 ≤ 4, chứng minh rằng ab + 1 bc + 1 ca + 1 + + ≥ 3. (a + b)2 (b + c)2 (c + a)2
Bài 1.23. Cho a, b, c > 0. Chứng minh rằng s s s 2a 2 2b 2 2c 2 3 + 3 + 3 ≥ 3. b + c c + a a + b 15
1. BẤT ĐẲNG THỨC AM - GM
Bài 1.24. Cho các số thực dương a, b, c thỏa abc = 1. Chứng minh rằng r r r 3 a3 + b3 3 b3 + c3 3 c3 + a3 + + + 6 ≤ 3 (a + b + c) . 2 2 2
Bài 1.25. Cho các số thực a,b,c thỏa a + b + c = 0. Chứng minh rằng 13a2b2c2 − 2abc − 2 1 ≤ . (a2 + b2 + c2)3 4
Bài 1.26. Cho các số thực không âm a,b,c. Chứng minh rằng: q √
(a + b + c)3 ≥ 6 3 (a − b) (b − c) (c − a) .
Bài 1.27. Cho các số thực a,b,c phân biệt thỏa a + b + c = 1 và ab + bc + ca > 0. Tìm giá trị nhỏ nhất 2 2 2 5 P = + + + √ . |a − b| |b − c| |c − a| ab + bc + ca
Bài 1.28. (JBMO 2014) Cho các số thực dương a, b, c thỏa mãn abc = 1. Chứng minh rằng 1 2 1 2 1 2 a + + b + + c + ≥ 3(a + b + c + 1). b c a
Bài 1.29. Cho các số thực dương a, b thỏa mãn ab ≥ 1. Chứng minh rằng 2 2 a + 2b + b + 2a + ≥ 16. a + 1 b + 1 1 1 1
Bài 1.30. (IMO Shortlist 2009) Cho các số thực dương a,b,c thỏa a + b + c = + + . a b c Chứng minh rằng 1 1 1 3 + + ≤ . (2a + b + c)2 (2b + c + a)2 (2c + a + b)2 16
Bài 1.31. Cho a, b, c là các số thực dương thỏa mãn: √ √ √
3 a3 + b3 + 3 b3 + c3 + 3 c3 + a3 + abc = 3.
Chứng minh rằng giá trị nhỏ nhất của biểu thức: a3 b3 c3 P = + + b2 + c2 c2 + a2 a2 + b2 √
bằng 6 32m, trong đó m là nghiệm của phương trình t3 + 54t − 162 = 0.
Bài 1.32 (Đề thi chọn đội tuyển, vòng 1, Hà Tĩnh, năm học 2017-20178). Cho các số
thực không âm a, b, c thoả mãn điều kiện a2 + b2 + c2 ≤ 3. Chứng minh rằng
(a + b + c)(a + b + c − abc) ≥ 2(a2b + b2c + c2a). 16
1. BẤT ĐẲNG THỨC AM - GM
Bài 1.33 (Đề thi chọn đội tuyển, vòng 2, Nam Định, năm học 2017-2018). Xét các số
thực a,b,c ∈ [0; 1]. Tìm giá trị lớn nhất của biểu thức a b c P = + +
+ (1 − a) (1 − b) (1 − c) b + c + 1 c + a + 1 a + b + 1
Bài 1.34 (Đề thi chọn đội tuyển, vòng 2, Quảng Ngãi, năm học 2017-2018). Cho a, b, c
là các số thực dương thỏa mãn 3bc + 4ac + 5ab ≤ 6abc . Tìm giá trị lớn nhất của biểu thức 3a + 2b + c P = . (a + b)(b + c)(c + a)
Bài 1.35 (ĐỀ THI HSG TỈNH TÂY NINH,VÒNG 1, 2017-2018). Cho ba số thực dương
x, y, z thỏa mãn xyz = 1. Chứng minh rằng: 1 1 1 √ + + √ ≤ 3. 4 px3 + 2y3 + 6 4 py3 + 2z3 + 6 4 z3 + 2x3 + 6
Bài 1.36 (Đề thi chọn đội tuyển, Lâm Đồng, năm học 2017-2018). Cho x,y,z là các số √
thực dương thỏa mãn điều kiện x3 + y2 + z = 2 3 + 1. 1 1 1
Tìm giá trị nhỏ nhất của biểu thức P = + + . x y2 z3
Bài 1.37 (Đề thi chọn đội tuyển, vòng 1, Hà Tĩnh, năm học 2016-2017). Cho các số thực
a,b,c dương và thỏa a5 + b5 + c5 = 3. Chứng minh rằng: a6b6 + b6c6 + c6a6 ≤ 3.
Bài 1.38. Tìm số nguyên dương k nhỏ nhất sao cho bất đẳng thức xkykzk(x3 + y3 + z3) ≤ 3
đúng với mọi số thực dương x, y, z thỏa mãn điều kiện x + y + z = 3. 1 1 1
Bài 1.39. (VN TST 2010) Cho các số thực dương a, b, c thỏa mãn 16 (a + b + c) ≥ + + . a b c Chứng minh rằng 1 1 1 8 + + ≤ . 3 3 3 9 a + b + p2 (a + c) b + c + p2 (b + a) c + a + p2 (c + b)
Bài 1.40. (IMO 2001) Cho các số thực dương a, b, c. Chứng minh rằng a2 b2 c2 √ + √ + √ ≥ 1. a2 + 8bc b2 + 8ca c2 + 8ab
Bài 1.41 (Turkey TST 2017). Cho các số thực dương a, b, c thỏa mãn a + b + c = 3. Chứng minh rằng
a3b + b3c + c3a + 9 ≥ 4(ab + bc + ca).
Bài 1.42 (IMO Shortlits A5-2008). Cho các số thực dương a,b,c,d thỏa mãn abcd = 1 và a b c d a + b + c + d ≥ + + + . b c d a b c d a
Chứng minh rằng a + b + c + d ≤ + + + . a b c d 17
1. BẤT ĐẲNG THỨC AM - GM
Bài 1.43. Cho các số thực không âm a, b, c thỏa mãn a + b + c = 2. Chứng minh rằng a3 + b3 b3 + c3 c3 + a3 ≤ 2.
Bài 1.44. (Hàn Quốc MO 2016) Cho các số thực x, y, z thỏa mãn x2 + y2 + z2 = 1. Tìm GTLN của biểu thức
P = (x2 − yz)(y2 − zx)(z2 − xy).
Bài 1.45. Cho các số thực dương x, y, z thỏa mãn x + y + z = 1. Chứng minh rằng x2y2 y2z2 z2x2 1 + + + 3xyz ≤ 1 − z 1 − x 1 − y 6
Bài 1.46. Cho các số thực dương a,b,c thỏa mãn:
9 a4 + b4 + c4 − 25 a2 + b2 + c2 + 48 = 0.
Tìm giá trị nhỏ nhất của biểu thức: a2 b2 c2 F = + + . b + 2c c + 2a a + 2b
Bài 1.47. (TST Quảng Nam 2014-2015) Cho các số thực dương a, b, c. Chứng minh rằng √ √ √ √ √ √ 5a2 + 4bc + 5b2 + 4ca +
5c2 + 4ab ≥ p3 (a2 + b2 + c2) + 2 ab + bc + ca .
Bài 1.48. Cho các số thực dương a, b, c thỏa mãn a + b + c = 3. Chứng minh rằng a2b b2c c2a + + ≤ 1. 1 + a + b 1 + b + c 1 + c + a
Bài 1.49 (P122, Tạp chí Pi, tháng 12 năm 2017). Chứng minh rằng, với mọi số thực dương
a, b, c ta luôn có bất đẳng thức: s s s a2 + bc b2 + ca c2 + ab + + ≥ 3. a(b + c) b(c + a) c(a + b)
Hỏi đẳng thức xảy ra khi nào? 1 1 1
Bài 1.50. Cho 2018 số dương a1,a2, . . . ,a2018 thỏa: a1 + a2 + · · · + a2018 = + + · · · + . a1 a2 a2018
Chứng minh rằng: a1 + a2 + · · · + a2018 ≥ 2018. 18
2. BẤT ĐẲNG THỨC CAUCHY - SCHWARZ
§2. Bất đẳng thức Cauchy - Schwarz I.
Bất đẳng thức Cauchy-Schwarz dạng đa thức
Định lí 1. Cho 2n số thực a1,a2, · · · ,an,b1,b2, · · · ,bn. Khi đó, ta có
a2 + a2 + · · · + a2 b2 + b2 + · · · + b2 ≥ (a 1 2 n 1 2 n
1b1 + a2b2 + · · · + anbn)2 .
Đẳng thức xảy ra khi ai = kbi với mọi i = 1,2, · · · ,n.
Chứng minh. Nếu ai = 0 ∀i = 1,n thì bất đẳng thức hiển nhiên đúng. n
Nếu P a2 > 0, ta xét tam thức i i=1 n ! n ! n X X X f (x) = a2 x2 − 2 a x + b2 i i.bi i i=1 i=1 i=1 Ta có n X f (x) = (aix − bi)2 ≥ ∀x ∈ R i=1 Do đó n !2 n ! n ! X X X ∆0 = aibi − a2 b2 ≤ 0 i i i=1 i=1 i=1
Hay bất đẳng thức được chứng minh.
Đẳng thức xảy ra khi aix − bi = 0 ⇔ ai = k.bi. II.
Bất đẳng thức Cauchy-Schwarz dạng phân thức
Định lí 2. Cho các n số thực a1,a2, · · · , an và n số thực dương b1,b2, · · · ,bn. Khi đó, ta có a2 a2 a2 (a 1 + 2 + · · · + n ≥ 1 + a2 + · · · + an)2 . b1 b2 bn b1 + b2 + · · · + bn a1 a2 an
Đẳng thức xảy ra khi và chỉ khi = = · · · = . b1 b2 bn
Chứng minh. Áp dụng bất đẳng thức Cauchy-Schwarz dạng đa thức ta có n !2 n !2 n ! n ! X X p a X X a2 a i i i = bi. √ ≤ bi b bi i=1 i=1 i i=1 i=1 Hay a2 a2 a2 (a 1 + 2 + · · · + n ≥
1 + a2 + · · · + an)2 (đpcm). b1 b2 bn b1 + b2 + · · · + bn III. Các ví dụ minh họa
Ví dụ 2.1. Cho a, b, c > 0 thỏa mãn a + b + c = 1. Chứng minh rằng r r r 1 1 1 √ a2 + + b2 + + c2 + ≥ 82 b2 c2 a2 19
2. BẤT ĐẲNG THỨC CAUCHY - SCHWARZ
Áp dụng bất đẳng thức Cauchy – Schwarz ta có 1 1 a 3 2 a2 + + 9 ≥ + b2 9 3 b hay r 1 3 a 3 a2 + ≥ √ + . b2 82 3 b Tương tự, ta cũng có r r 1 3 b 3 1 3 c 3 b2 + ≥ √ + và c2 + ≥ √ + . c2 82 3 c a2 82 3 a
Công ba bất đẳng thức theo vế ta có r r r 1 1 1 3 a + b + c 1 1 1 a2 + + b2 + + c2 + ≥ √ + 3 + + . b2 c2 a2 82 3 a b c 1 1 1 9 Lại có + + ≥ = 9 nên ta suy ra được a b c a + b + c r r r 1 1 1 3 1 √ a2 + + b2 + + c2 + ≥ √ + 27 = 82. b2 c2 a2 82 3 1
Đẳng thức xảy ra khi a = b = c = . 3
Ví dụ 2.2. Cho các số thực dương a,b,c thỏa abc = 1. Chứng minh rằng 1 + a2 1 + b2 1 + c2 ≥ 4 3 p(a + b) (b + c) (c + a).
Áp dụng bất đẳng thức Bunhiacopsky cho hai bộ số (1; a) và (b; 1) ta có
1 + a2 1 + b2 = 1 + a2 b2 + 1 ≥ (a + b)2 . Tương tự 1 + b2 1 + c2 ≥ (b + c)2 , 1 + c2 1 + a2 ≥ (a + c)2 .
Nhận các bất đẳng thức trên theo vế ta được
1 + a2 1 + b2 1 + c2 ≥ (a + b) (b + c) (c + a) . Mặt khác √ √ √
(a + b) (b + c) (c + a) ≥ 2 ab.2 bc.2 ca = 8abc = 8. Suy ra q (a + b) (b + c) (c + a) = 3
p(a + b) (b + c) (c + a). 3 [(a + b) (b + c) (c + a)]2 √ ≥ 3
p(a + b) (b + c) (c + a). 3 82 = 4 3p(a + b) (b + c) (c + a) (đpcm). 20
2. BẤT ĐẲNG THỨC CAUCHY - SCHWARZ
Ví dụ 2.3. Cho a,b,c > 0 thỏa 1 1 1 + + ≥ 1. a2 + b2 + 1 b2 + c2 + 1 c2 + a2 + 1
Chứng minh rằng ab + bc + ca ≤ 3.
Áp dụng bất đẳng thức Cauchy-Schwarz cho hai bộ số (a; b; 1) và (1; 1; c) ta có
a2 + b2 + 1 1 + 1 + c2 ≥ (a + b + c)2 . Suy ra 1 2 + c2 ≤ . a2 + b2 + 1 (a + b + c)2 Tương tự: 1 2 + a2 1 2 + b2 ≤ , ≤ . b2 + c2 + 1 (a + b + c)2 c2 + a2 + 1 (a + b + c)2 Suy ra 2 + a2 2 + b2 2 + c2 + + ≥ 1, (a + b + c)2 (a + b + c)2 (a + b + c)2 Do đó ta có
6 + a2 + b2 + c2 ≥ (a + b + c)2 ⇒ ab + bc + ca ≤ 3 (đpcm). 1 1 1
Ví dụ 2.4. Cho các số thực dương a,b,c thỏa mãn a + b + c = + + . Chứng minh a b c rằng √ √ √ √ a2 + 1 + b2 + 1 + c2 + 1 ≤ 2 (a + b + c) .
Áp dụng bất đẳng thức Cauchy – Schwarz ta có √ √ √ √ r r r 1 √ 1 √ 1 a2 + 1 + b2 + 1 + c2 + 1 = a. a + + b. b + + c. c + a b c s 1 1 1 ≤ (a + b + c) a + + b + + c + a b c √ = 2 (a + b + c) .
Đẳng thức xảy ra khi a = b = c = 1.
Ví dụ 2.5. Cho các số thực a, b, c, x, y, z. Chứng minh rằng p 2 ax + by + cz +
(a2 + b2 + c2)(x2 + y2 + z2) ≥ (a + b + c)(x + y + z). 3 Ta có
2 (a + b + c)(x + y + z) − (ax + by + cz) 3 2y + 2z − x 2z + 2x − y 2x + 2y − z = a · + b · + c · 3 3 3 v u 2 2 2! u 2y + 2z − x 2z + 2x − y 2x + 2y − z ≤ t(a2 + b2 + c2) + + . 3 3 3 21
2. BẤT ĐẲNG THỨC CAUCHY - SCHWARZ Hơn nữa 2y + 2z − x 2 2z + 2x − y 2 2x + 2y − z 2 + + = x2 + y2 + z2. 3 3 3 Nên ta có đpcm.
Ví dụ 2.6. Cho các số thực a,b,c thỏa a2 + b2 + c2 = 9. Chứng minh rằng 2 (a + b + c) − abc ≤ 10.
Không mất tính tổng quát, ta giả sử |a| ≤ |b| ≤ |c|
3 a2 + b2 ≤ 2 a2 + b2 + c2 = 18 ⇒ a2 + b2 ≤ 6.
Áp dụng bất đẳng thức Cauchy-Schwarz cho hai bộ số q
2 (a + b + c) − abc = 2 (a + b) + c (2 − ab) ≤ 4 + (2 − ab)2 (a + b)2 + c2 p =
(8 − 4ab + a2b2) (a2 + b2 + c2 + 2ab) p = (8 − 4ab + a2b2) (9 + 2ab).
Do đó, ta chỉ cần chứng minh
p(8 − 4ab + a2b2) (9 + 2ab) ≤ 10
⇔ 2a3b3 + a2b2 − 20ab − 28 ≤ 0
⇔ (ab + 2)2 (2ab − 7) ≤ 0. (*)
Vì 2ab ≤ a2 + b2 ≤ 6 ⇒ 2ab − 7 < 0, do đó (*) đúng.
Ví dụ 2.7 (VQB Cẩn). Cho các số thực dương a,b,c thỏa mãn a + b + c = 6 và a2 + b2 + c2 = 14. Chứng minh rằng 4a + b 31 2 ≤ ≤ . c 2 Ta có
4a + b ≥ 2 ⇔ −4a − b + 2c ≤ 0 ⇔ 3a + 6b + 9c ≤ 7 (a + b + c) = 42 (1). c
Áp dụng bất đẳng thức Cauchy – Schwarz ta có
3a + 6b + 9c ≤ p(32 + 62 + 92) (a2 + b2 + c2) = 42.
Suy ra (1) đúng. Đẳng thức xảy ra khi a = 1,b = 2,c = 3. Tương tự 4a + b 31 ≤
⇔ 8a + 2b − 31c ≤ 0 ⇔ 57a + 51b + 18c ≤ 49 (a + b + c) = 294 (2). c 2
Áp dụng bất đẳng thức Cauchy – Schwarz ta có
57a + 51b + 18c ≤ p(572 + 512 + 182) (a2 + b2 + c2) = 294. 19 17 6
Hay (2) được chứng minh. Đẳng thức xảy ra khi a = ,b = ,c = . 7 7 7 22
2. BẤT ĐẲNG THỨC CAUCHY - SCHWARZ
Ví dụ 2.8. Cho các số thực dương a, b, c. Chứng minh rằng r r 2a 2b r 2c + + ≤ 3. a + b b + c c + a
Áp dụng bất đẳng thức Cauchy – Schwarz ta có 2 s √ ! r a √ b √ r c V T 2 = a + c + b + a + c + b (a + b)(a + c) (b + c)(b + a) (c + a)(c + b) a b c ≤ 2 (a + b + c) + + (a + b) (a + c) (b + a) (b + c) (c + a) (c + b) 4 (a + b + c) [ab + bc + ca] = . (a + b) (b + c) (c + a)
Do đó, ta chỉ cần chứng minh 4 (a + b + c) (ab + bc + ca) 9 (a + b + c) (ab + bc + ca) 9 ≤ ⇔ ≤ . (a + b) (b + c) (c + a) 2 (a + b) (b + c) (c + a) 8
Đây là một kết quả quen thuộc.
Ví dụ 2.9. Cho các số thực dương a,b,c thỏa 1 1 1 + + = 1. a2 + 2 b2 + 2 c2 + 2
Chứng minh rằng: ab + bc + ca ≤ 3. Từ giả thiết suy ra: a2 b2 c2 (a + b + c)2 1 = + + ≥ a2 + 2 b2 + 2 c2 + 2 a2 + b2 + c2 + 6 Do đó:
a2 + b2 + c2 + 6 ≥ (a + b + c)2 ⇔ ab + bc + ca ≤ 3 (đpcm).
Ví dụ 2.10. Cho các số thực a,b,c > 0 thỏa mãn a + b + c = 3. Chứng minh rằng a2 b2 c2 + + ≥ 1. a + 2b2 b + 2c2 c + 2a2
Gọi P là vế trái của bất đẳng thức cần chứng minh. Áp dụng bất đẳng thức Cauchy – Schwarz ta có a4 b4 c4 P = + + a3 + 2a2b2 b3 + 2b2c2 c3 + 2c2a2 2 (a2 + b2 + c2) ≥ .
a3 + b3 + c3 + 2 (a2b2 + b2c2 + c2a2) Với a + b + c = 3 ta có
a4 + b4 + c4 a2 + b2 + c2 ≥ a3 + b3 + c32
a3 + b3 + c3 (a + b + c) ≥ a2 + b2 + c22
3 a2 + b2 + c2 ≥ (a + b + c)2 . 23
2. BẤT ĐẲNG THỨC CAUCHY - SCHWARZ
Nhân ba bất đẳng thức trên theo vế ta được
3 a4 + b4 + c3 ≥ (a + b + c) a3 + b3 + c3
Hay a4 + b4 + c4 ≥ a3 + b3 + c3. Do đó
a2 + b2 + c22 = a4 + b4 + c4 + +2 a2b2 + b2c2 + c2a2
≥ a3 + b3 + c3 + 2 a2b2 + b2c2 + c2a2 .
Vậy P ≥ 1. Đẳng thức xảy ra khi a = b = c = 1.
Ví dụ 2.11. Cho các số thực dương a,b,c. Chứng minh rằng: a 2 b 2 c 2 3 + + ≥ . a + b b + c c + a 4 b c a Vì . .
= 1 nên tồn tại các số thực dương x,y,z sao cho a b c b yz c zx a xy = , = , = . a x2 b y2 c z2
Bất đẳng thức cần chứng minh trở thành x4 y4 z4 3 + + ≥ . (x2 + yz)2 (y2 + zx)2 (z2 + xy)2 4
Áp dụng bất đẳng thức Cauchy – Schwarz ta có 2 x4 y4 z4 (x2 + y2 + z2) + + ≥ . (x2 + yz)2 (y2 + zx)2 (z2 + xy)2
(x2 + yz)2 + (y2 + zx)2 + (z2 + xy)2 Ta chứng minh 2 (x2 + y2 + z2) 3 ≥
(x2 + yz)2 + (y2 + zx)2 + (z2 + xy)2 4
Biến đổi và rút gọn ta thu được bất đẳng thức
x4 + y4 + z4 + 5 x2y2 + y2z2 + z2x2 ≥ 6xyz (x + y + z) (∗). Ta có
x4 + y4 + z4 ≥ x2y2 + y2z2 + z2x2 ≥ xyz (x + y + z) .
Nên suy ra (∗) đúng. Vậy bài toán được chứng minh.
Ví dụ 2.12 (P61, Tạp chí Pi, tháng 6 năm 2017). Cho a, b, c là độ dài ba cạnh của
một tam giác. Chứng minh rằng a b c 2(ab + bc + ca) 7 + + + ≤ . b + c c + a a + b a2 + b2 + c2 2
Hỏi dấu bằng xảy ra khi và chỉ khi nào?
Ta biết rằng với a, b, c là ba số thực tùy ý, luôn có ab + bc + ca ≤ a2 + b2 + c2 24
2. BẤT ĐẲNG THỨC CAUCHY - SCHWARZ Do đó 2(ab + bc + ca) ab + bc + ca ≤ + 1. (1) a2 + b2 + c2 a2 + b2 + c2
Tiếp theo ta sẽ chứng minh a b c ab + bc + ca 5 + + + ≤ (2) b + c c + a a + b a2 + b2 + c2 2 Thật vậy, ta có a b c 1 ab + bc + ca (2) ⇔ 1 − + 1 − + 1 − ≥ + . b + c c + a a + b 2 a2 + b2 + c2 b + c − a c + a − b a + b − c (a + b + c)2 ⇔ + + ≥ b + c c + a a + b 2(a2 + b2 + c2) (b + c − a)2 (c + a − b)2 (a + b − c)2 (a + b + c)2 ⇔ + + ≥ . (3) (b + c − a)(b + c) (c + a − b)(c + a) (a + b − c)(a + b) 2(a2 + b2 + c2)
Do đó a, b, c là độ dài 3 cạnh của một tam giác nên b + c > a, a + c > b và a + b > c. Do đó, tất
cả các phân thức nằm ở vế trái của (3) đều có mẫu thức dương. Vì thế, ký hiệu VT là biểu thức
nằm ở vế trái của (3), theo một hệ quả của bất đẳng thức Cauchy - Schwarz, ta có (a + b + c)2 V T ≥ . 2(a2 + b2 + c2) Vì
(b + c − a) + (c + a − b) + (a + b − c) = a + b + c
(b + c − a)(b + c) + (c + a − b)(c + a) + (a + b − c)(a + b) = 2(a2 + b2 + c2).
nên (3) được chứng minh và vì thế (2) được chứng minh. Từ (1) và (2), hiển nhiên, suy ra bất
đẳng thức cần chứng minh theo yêu cầu bài toán. Từ điều kiện cần và đủ để dấu bằng xảy ra ở
các bất đẳng thức trên, dễ dàng suy ra dấu bằng ở bất đẳng thức đề bài xảy ra khi và chỉ khi a,
b, c là độ dài ba cạnh của tam giác đều.
Ví dụ 2.13. Cho a,b,c > 0 thỏa a + b + c = 2. Chứng minh rằng: a b c √ + √ + √ ≤ 1. 4a + 3bc 4b + 3ca 4c + 3ab
Áp dụng bất đẳng thức Cauchy – Schwarz ta có: a b c 2 a b c √ + √ + √ ≤ (a + b + c) + + 4a + 3bc 4b + 3ca 4c + 3ab 4a + 3bc 4b + 3ca 4c + 3ab a b c = 2 + + . 4a + 3bc 4b + 3ca 4c + 3ab Ta chứng minh: a b c 1 + + ≤ 4a + 3bc 4b + 3ca 4c + 3ab 2 bc ca ab 1 ⇔ + + ≥ (1) 4a + 3bc 4b + 3ca 4c + 3ab 3 Ta có (ab + bc + ca)2
V T (1) ≥ bc(4a + bc) + ca(4b + ca) + ab(4c + ab) 25
2. BẤT ĐẲNG THỨC CAUCHY - SCHWARZ Do
bc(4a + bc) + ca(4b + ca) + ab(4c + ab) = 3(ab + bc + ca)2. 1 2 Nên ta có: V T (1) ≥
(đpcm). Đẳng thức xảy ra khi và chỉ khi a = b = c = . 3 3
Ví dụ 2.14. Cho các số thực x,y,z > 0. Chứng minh rằng x + y y + z z + x + + ≤ 3. px2 + y2 + zx + zy py2 + z2 + xy + xz pz2 + x2 + yz + xy
Áp dụng bất đẳng thức Cauchy – Schwarz ta có " # (x + y)2 (y + z)2 (z + x)2 V T 2 ≤ 3 + + . x2 + y2 + zx + yz y2 + z2 + xy + xz z2 + x2 + zy + yx Mặt khác (x + y)2 (x + y)2 x2 y2 x y = ≤ + = + x2 + y2 + zx + yz x(x + z) + y(y + z) x (x + z) y (y + z) x + z y + z Tương tự (y + z)2 y z (z + x)2 z x ≤ + và ≤ + y2 + z2 + xy + xz y + x z + x z2 + x2 + zy + yx z + y x + y
Suy ra V T 2 ≤ 9 ⇔ V T ≤ 3, từ đây ta có đpcm.
Ví dụ 2.15. Cho a, b, c là các số thực không âm và không có hai số nào đồng thời bằng 0. Chứng minh rằng a2 b2 c2 1 + + ≤ . (2a + b)(2a + c) (2b + c)(2b + a) (2c + a)(2c + b) 3
Áp dụng bất đẳng thức Cauchy-Schwarz, ta có 9a2 (2a + a)2 = (2a + b)(2a + c) 2a(a + b + c) + (2a2 + bc) 4a2 a2 ≤ + 2a(a + b + c) 2a2 + bc 2a a2 = + . a + b + c 2a2 + bc Do đó a2 b2 c2 9V T ≤ 2 + + + . 2a2 + bc 2b2 + ca 2c2 + ab Nên ta chứng minh a2 b2 c2 + + ≤ 1 2a2 + bc 2b2 + ca 2c2 + ab bc ca ab ⇔ + + ≥ 1. (1) 2a2 + bc 2b2 + ca 2c2 + ab
Áp dụng bất đẳng thức Cauchy-Schwarz ta có (ab + bc + ca)2 (ab + bc + ca)2 V T (1) ≥ = = 1.
2abc(a + b + c) + a2b2 + b2c2 + c2a2 (ab + bc + ca)2 26
2. BẤT ĐẲNG THỨC CAUCHY - SCHWARZ IV. Bài tập
Bài 2.1 (Bất đẳng thức Mincopski). Cho các 2n số thực a1,a2, · · · ,an,b1,b2, · · · ,bn. Chứng minh rằng q q q a2 + a2 + · · · + a2 + b2 + b2 + · · · + b2 ≥ (a 1 2 n 1 2 n
1 + b1)2 + (a2 + b2)2 + · · · + (an + bn)2.
Bài 2.2. Cho các số thực dương a,b,c. Chứng minh rằng
a2 + 1 b2 + 1 c2 + 1 ≥ (a + b) (b + c) (c + a) .
Bài 2.3. Cho các số thực dương a,b,c thỏa mãn a + b + c = 3. Chứng minh rằng 1 1 1 2 a2 + b2 + c2 + 3 ≥ 9 + + . a2 + 2 b2 + 2 c2 + 2
Bài 2.4. Cho a,b,c > 0 và a + b + c = 1. Chứng minh rằng √ √ √
a a2 + 8bc + b b2 + 8ca + c c2 + 8ab ≤ 1.
Bài 2.5. Cho các số thực dương a, b, c thỏa a2 + b2 + c2 = 3. Chứng minh rằng: a3 b3 c3 + + ≥ 1. b + 2c c + 2a a + 2b
Bài 2.6. Cho a, b, c là các số thực dương thỏa: a2 + b2 + c2 ≥ 3. Chứng minh rằng: a3 b3 c3 √ + √ + √ ≥ 1. b2 + c2 + 7 c2 + a2 + 7 a2 + b2 + 7
Bài 2.7. Cho các số thực dương a,b,c có tổng bằng 3. Chứng minh rằng: 1 1 1 1 + + ≤ . 4a2 + b2 + c2 a2 + 4b2 + c2 a2 + b2 + 4c2 2
Bài 2.8. Cho các số thực không âm a, b, c thỏa mãn a + b + c = 3. Chứng minh rằng 1 1 1 3 + + ≥ . 2a2 + 3 2b2 + 3 2c2 + 3 5
Bài 2.9. Cho các số thực không âm a, b, c thỏa mãn ab + bc + ca = 3. Chứng minh rằng 1 1 1 3 + + ≥ . a2 + 1 b2 + 1 c2 + 1 2
Bài 2.10. Cho các số thực dương a, b, c thỏa mãn a2 + b2 + c2 = 3. Chứng minh rằng 1 1 1 3 + + ≤ . 3 − ab 3 − bc 3 − ca 2 27
2. BẤT ĐẲNG THỨC CAUCHY - SCHWARZ
Bài 2.11. Cho ba số thực dương x, y, z thỏa mãn x + y + z = 3. Chứng minh rằng: 4x + 5 4y + 5 4z + 5 162 + + ≥ . x3 + xy2 + 3xyz y3 + yz2 + 3xyz z3 + zx2 + 3xyz x2 + y2 + z2 + 27
Bài 2.12. Cho a, b, c > 0 thỏa mãn a + b + c = 1. Chứng minh rằng a2 b2 c2 1 + + ≥ . (b + 2c)2 (a + b) (c + 2a)2(b + c) (a + 2b)2(c + a) 2
Bài 2.13. Cho a,b,c ∈ (1; 2). Chứng minh rằng √ √ √ b a c b a c √ √ + √ √ + √ √ ≥ 1. 4b c − c a 4c a − a b 4a b − b c
Bài 2.14. Cho a,b,c là các số thực dương thỏa mãn a2b2 + b2c2 + c2a2 ≥ a2b2c2. Chứng minh rằng √ a2b2 b2c2 c2a2 3 + + ≥ c3 (a2 + b2) a3 (b2 + c2) b3 (c2 + a2) 2
Bài 2.15 (IMO Shortlist-2007). Cho các số thực không âm a1, a2, . . . , a100 thỏa mãn điều kiện a2 + a2 + · · · + a2 = 1. Chứng minh rằng 1 2 100 √2 S = a2a a a . 1 2 + a2 2 3 + · · · + a2 100 1 ≤ 3 1
Bài 2.16. (China TST 2005) Cho các số thực không âm a,b,c thỏa ab + bc + ca = . Chứng 3 minh rằng 1 1 1 + + ≤ 3. a2 − bc + 1 b2 − ca + 1 c2 − ab + 1
Bài 2.17. Cho các số thực dương a, b, c. Chứng minh rằng r r r ab bc ca b2 c2 a2 2 + + ≥ a 3 + + b 3 + + c 3 + ≥ 2(a + b + c). c a b c2 a2 b2
Bài 2.18. Cho x,y,z > −1. Chứng minh rằng 1 + x2 1 + y2 1 + z2 + + ≥ 2. 1 + y + z2 1 + z + x2 1 + x + y2
Bài 2.19 (P77, Tạp chí Pi, tháng 7 năm 2017). Cho a, b, c là các số thực dương. Chứng minh rằng a b c 3 + + ≥ . 3 p4(b3 + c3) c + a a + b 2
Hỏi đẳng thức xảy ra khi và chỉ khi nào?
Bài 2.20. Cho ba số thực không âm a, b, c thỏa mãn điều kiện (b + c)(c + 2a)(c + 4a) > 0. Chứng minh rằng a b 2c + + ≥ 1. b + c c + 4a c + 2a
Hỏi đẳng thức xảy ra khi và chỉ khi nào? 28
2. BẤT ĐẲNG THỨC CAUCHY - SCHWARZ
Bài 2.21. Cho a,b,c > 0 thỏa mãn a2 + b2 + c2 = 3. Chứng minh rằng 1 1 1 + + ≥ 3. 2 − a 2 − b 2 − c
Bài 2.22. Cho bốn số thực a,b,c,d thỏa mãn a2 + b2 + c2 + d2 = 1. Chứng minh rằng 1 1 1 1 16 + + + ≤ . 1 − ab 1 − bc 1 − cd 1 − da 3
Bài 2.23. Cho x,y,z > 0 thỏa mãn xyz = 1. Chứng minh rằng 1 1 1 + + ≥ 1. 1 + x + x2 1 + y + y2 1 + z + z2
Bài 2.24. Cho x,y,z > 0 thỏa mãn xyz = 8. Chứng minh rằng x2 y2 z2 + + ≥ 1. x2 + 2x + 4 y2 + 2y + 4 z2 + 2z + 4
Bài 2.25 (IMO 2008). Cho các số thực x,y,z 6= 1 và xyz = 1. Chứng minh rằng x 2 y 2 z 2 + + ≥ 1. x − 1 y − 1 z − 1
Bài 2.26. Cho a, b, clà các số thực dương. Chứng minh rằng s s bc r ac ab 3 + + ≥ . a(3b + a) b(3c + b) c(3a + c) 2
Bài 2.27. Cho các số thực a, b, c tất cả không cùng dấu. Chứng minh rằng
(a2 + ab + b2)(b2 + bc + c2)(c2 + ca + a2) ≥ 3(ab + bc + ca)3.
Bài 2.28. Cho a, b, c ≥ 0 và không có hai số nào đồng thời bằng 0. Chứng minh rằng a2 b2 c2 + + ≥ 1. 2b2 − bc + 2c2 2c2 − ca + 2a2 2a2 − ab + 2b2
Bài 2.29. Cho a, b, c là các số thực thỏa mãn điều kiện 3a2 + 2b2 + c2 = 6. Tìm giá trị lớn nhất
và giá trị nhỏ nhất của biểu thức P = 2(a + b + c) − abc.
Bài 2.30. (Iran MO 2016 day 3) Cho các số thực dương a, b, c thỏa mãn abc = 1. Chứng minh rằng a + b b + c c + a 2 + + ≥ . (a + b + 1)2 (b + c + 1)2 (c + a + 1)2 a + b + c 29
2. BẤT ĐẲNG THỨC CAUCHY - SCHWARZ
Bài 2.31. Cho các số thực dương x, y, z thỏa mãn xyz ≥ 1. Chứng minh rằng
(x4 + y)(y4 + z)(z4 + x) ≥ (x + y2)(y + z2)(z + x2).
Bài 2.32. (Serbia TST 2016) Cho các số thực dương a, b, c thỏa mãn a + b + c = 3. Chứng minh rằng a b c 3 √ + √ + √ ≤ . 3a + b 3b + c 3c + a 2
Bài 2.33. (Hải Dương TST 2016)
Cho a, b, c ∈ [−1,1] thỏa mãn: 1 + 2abc ≥ a2 + b2 + c2. Chứng minh rằng : 1 + 2a3b3c3 ≥ a6 + b6 + c6.
Bài 2.34. Cho các số thực dương a, b, c thỏa mãn a2 + b2 + c2 = 3. Chứng minh rằng 1 1 1 b + c c + a a + b 2 + + ≥ + + . a2 b2 c2 a b c
Bài 2.35. Cho n (n ≥ 1) x1, x2, . . . , xn thỏa mãn x1 + x2 + · · · + xn = 0. Chứng minh rằng (n − 2)x2 + 2x (n − 2)x2 + 2x (n − 2)x2 + 2x 1 1 + 2 2 + · · · + n n ≥ 0. (n − 1)x2 + 1 (n − 1)x2 + 1 (n − 1)x2 + 1 1 2 n
Bài 2.36. Cho các số thực dương a,b,c thỏa mãn a2 + b2 + c2 = 1. Chứng minh rằng 1 1 1 (a + b + c)2 + + ≥ . 2a2 + bc 2b2 + ac 2c2 + ab ab + bc + ac
Bài 2.37. Cho a, b, c là các số thực dương và n ∈ N, n ≥ 2. Tìm giá trị nhỏ nhất của biểu thức s s s a2 + bc b2 + ac c2 + ab P = n + n + n . a(b + c) b(a + c) c(a + b)
Bài 2.38. Cho các số dương a, b, c thỏa mãn abc = 1. Chứng minh rằng: a3 + 5 b3 + 5 c3 + 5 + + ≥ 9. a3(b + c) b3(c + a) c3(a + b)
Bài 2.39. Cho số nguyên dương n ≥ 3 và 2n số thực dương a1; a2; . . . ; an; b1; b2; . . . ; bn thỏa mãn: n n X X ak = 1; b2 = 1. k k=1 k=1 n X Chứng minh rằng:
ak(bk + ak+1) < 1 (với an+1 = a1). k=1 30
3. MỘT SỐ BẤT ĐẲNG THỨC KHÁC
§3. Một số bất đẳng thức khác I. Bất đẳng thức Schur 1. Bất đẳng thức Schur
Định lí 1. Cho các số thực không âm x,y,z và số thực dương r. Khi đó, ta có bất đẳng thức sau
xr(x − y)(x − z) + yr(y − x)(y − z) + zr(z − x)(z − y) ≥ 0.
Đẳng thức xảy ra khi a = b = c hoặc c = 0,a = b và các hoán vị.
Chứng minh. Vì bất đẳng thức cần chứng minh là đối xứng ba biến nên ta giả sử x ≥ y ≥ z,
khi đó zr(z − x)(z − y) ≥ 0 và
xr(x − y)(x − z) + yr(y − x)(y − z) ≥ (x − y) (xr(y − z) − yr(y − z))
= (x − y)(y − z)(xr − yr) ≥ 0.
Từ hai bất đẳng thức trên ta suy ra đpcm. 2.
Các trường hợp đặc biệt
• Xét r = 1 ta có các dạng sau
a) x3 + y3 + z3 + 3xyz ≥ xy(x + y) + yz(y + z) + zx(z + x)
b) 4(a3 + b3 + c3) + 15abc ≥ (a + b + c)3
c) xyz ≥ (x + y − z)(y + z − x)(z + x − y) 9xyz d) x2 + y2 + z2 + ≥ 2(xy + yz + zx) x + y + z
e) (x + y + z)3 + 9xyz ≥ 4(x + y + z)(xy + yz + zx)
• r = 2 ta có các dạng sau
a) x4 + y4 + z4 + xyz(x + y + z) ≥ xy(x2 + y2) + yz(y2 + z2) + zx(z2 + x2)
b) 6xyz(x + y + z) ≥ [2(xy + yz + zx) − (x2 + y2 + z2)] (x2 + y2 + z2 + xy + yz + zx). 3.
Bất đẳng thức Schur mở rộng
Định lí 2. Cho các số thực dương a, b, c, x, y, z sao cho các bộ (a, b, c) và (x, y, z) là các bộ
đơn điệu. Khi đó, ta có bất đẳng thức
a(x − y)(x − z) + b(y − z)(y − x) + c(z − x)(z − y) ≥ 0.
Chứng minh. Việc chứng minh bất đẳng thức này tương tự như chứng minh bất đẳng thức Schur ở trên. 4. Các ví dụ
Ví dụ 3.1 (Đồng Nai TST 2016). Cho các số thực dương a, b, c thỏa mãn abc = 1. Chứng minh rằng: √ r r a b r c 3 3 + + ≥ √ . b + c c + a a + b a3 + b3 + c3 + 3 31
3. MỘT SỐ BẤT ĐẲNG THỨC KHÁC Với x, y, z > 0 ta có: 1 1 1 r 1 (x + y + z)2 + + ≥ 9 3 px2y2z2 · 3 3 = 27. x2 y2 z2 x2y2z2 Suy ra : √ 3 3 x + y + z ≥ . (∗) s 1 2 1 2 1 2 + + x y z r a r b r c Áp dụng (*) với x = , y = , z = ta có b + c c + a a + b √ r r a b r c 3 3 + + ≥ b + c c + a a + b r b + c c + a a + b + + a b c √ 3 3 = .
pab (a + b) + bc (b + c) + ca (c + a)
Mặt khác, theo bất đẳng thức Schur ta có
a3 + b3 + c3 + 3abc ≥ ab (a + b) + bc (b + c) + ca (c + a) . √ r a r b r c 3 3 Nên ta có : + + ≥ √ . b + c c + a a + b a3 + b3 + c3 + 3
Đẳng thức có a = b = c = 1.
Ví dụ 3.2 (Nghệ an TST 2014, ngày 2). Cho các số thực x, y, z > 0 thỏa xyz = 1. Chứng minh rằng r x + y r y + z r z + x 5(x + y + z) + 9 3 + 3 + 3 ≤ . 2z 2x 2y 8
Đặt x = a3, y = b3, z = c3 bất đẳng thức cần chứng minh trở thành r r r 3 a3 + b3 3 b3 + c3 3 c3 + a3 5(a3 + b3 + c3) + 9 + + ≤ . 2c3 2a3 2b3 8
Theo bất đẳng thức Schur ta có
a3 + b3 + c3 + 3 = a3 + b3 + c3 + 3abc ≥ ab(a + b) + bc(b + c) + ca(c + a). Do đó
5(a3 + b3 + c3) + 9 = 2(a3 + b3 + c3) + 3(a3 + b3 + c3 + 3)
≥ 2(a3 + bb + c3) + 3ab(a + b) + 3bc(b + c) + 3ca(c + a)
= (a + b)3 + (b + c)3 + (c + a)3. (1) Ta chứng minh r (a + b)3 a3 + b3 ≥ 3 . (2) 8 2c3 32
3. MỘT SỐ BẤT ĐẲNG THỨC KHÁC
Thật vậy (1) tương đương với r (a + b)3 1 a3 + b3 a3 + b3 ≥ 3 ⇔ (a + b)9 ≥ 83a3b3 . (3) 8abc c 2 2 Ta có
4 · 82 · a3b3(a3 + b3) = ab · ab · ab(a2 − ab + b2)(a + b)
ab + ab + ab + a2 − ab + b2 4 ≤ 4 · 82 (a + b) = (a + b)9. 4
Do đó (2) đúng. Tương tự ta có r r (b + c)3 b3 + c3 (c + a)3 c3 + a3 ≥ 3 , ≥ 3 . (4) 8 2a3 8 b3
Công các bất đẳng thức (2), (4) và từ (1) ta có đpcm.
Ví dụ 3.3. (VMO 2014) Cho a, b, c ≥ 0. Chứng minh rằng
3(a2 + b2 + c2) ≥ P ≥ (a + b + c)2, √ √ √ với P = (a + b + c) ab + bc +
ca + (a − b)2 + (b − c)2 + (c − a)2. Ta có √ √ √
3(a2 + b2 + c2) ≥ P ⇔ a + b + c ≥ ab + bc + ca.
Bất đẳng thức này là kết quả quen thuộc. √ √ √ Đặt x = a, y = b, z =
c. Khi đó, bất đẳng thức X X X X P ≥ (a + b + c)2 ⇔ x4 + xyz x + xy(x2 + y2) ≥ 4 x2y2 (1)
Sử dụng bất đẳng thức Schur (với trường hợp r = 2) ta có X X X x4 + xyz x ≥ xy(x2 + y2) do đó X X X V T (1) ≥ 2 xy(x2 + y2) ≥ 2. xy.2xy = 4 x2y2. Hay (1) được chứng minh.
Ví dụ 3.4. Cho a,b,c > 0. Chứng minh rằng a2 + bc b2 + ca c2 + ab 1 1 1 + + ≥ + + . a2(b + c) b2(c + a) c2(a + b) a b c Ta có a2 + bc 1 a2 + bc − a(b + c) (a − b)(a − c) − = = . a2(b + c) a a2(b + c) a2(b + c)
Do đó, bất đẳng thức cần chứng minh tương đương với
x(a − b)(a − c) + y(b − c)(b − a) + z(c − a)(c − b) ≥ 0 (1). 1 1 1 Với x = , y = , z = . a2(b + c) b2(c + a) c2(a + b) 1 1 ab(b − a) + c(b2 − a2)
Giả sử a > b > c, ta có − = > 0 hay x < y. a2(b + c) b2(c + a) a2b2(b + c)(c + a)
Do đó, bộ (x,y,z) là bộ đơn điệu giảm. Do đó, theo bất đẳng thức Schur suy rộng, ta có (1) đúng. 33
3. MỘT SỐ BẤT ĐẲNG THỨC KHÁC II. Bất đẳng thức Holder 1. Bất đẳng thức Holder
Định lí 3. Cho mn số thực dương ai với i = 1,m và j = 1,n. Khi đó ta có bất đẳng thức sau j m n ! n m !m Y X X Y am ≥ a . i i j j i=1 j=1 j=1 i=1 2. Trường hợp đặc biệt
• m = 2 ta có bất đẳng thức Cauchy-Schwarz • m = 3 ta có
(a3 + a3 + · · · + a3 )(b3 + b3 + · · · + b3 )(c3 + c3 + · · · + c3 ) ≥ (a 1 2 n 1 2 n 1 2 n 1b1c1 + · · · + anbncn)3. 3. Ví dụ minh họa
Ví dụ 3.5. Cho các số thực dương a, b, c. Chứng minh rằng
(a3 + 2)(b3 + 2)(c3 + 2) ≥ (a + b + c)3.
Áp dụng bất đẳng thức Holder ta có
(a3 + 2)(b3 + 2)(c3 + 2) = (a3 + 1 + 1)(1 + b3 + 1)(1 + 1 + c3) ≥ (a + b + c)3.
Ví dụ 3.6. Cho các số thực dương a, b, c. Chứng minh rằng a + b b + c c + a √ √ + √ + √ ≥ 2 a + b + c. a + 2c b + 2a c + 2b
Áp dụng bất đẳng thức Holder ta có a + b b + c c + a 2 √ X + √ + √
(a + b)(a + 2c) ≥ 8(a + b + c)3. a + 2c b + 2a c + 2b Mặt khác
X(a + b)(a + 2c) = (a + b + c)2 + 3(ab + bc + ca) ≤ 2(a + b + c)2. Do đó a + b b + c c + a √ √ + √ + √ ≥ 2 a + b + c. a + 2c b + 2a c + 2b
Ví dụ 3.7. Cho các số thực dương a, b, c. Chứng minh rằng a b c √ + √ + √ ≥ 1. a2 + 8bc b2 + 8ca c2 + 8ab 34
3. MỘT SỐ BẤT ĐẲNG THỨC KHÁC
Gọi P là vế trái của bất đẳng thức. áp dụng bất đẳng thức Holder ta có X P 2 a(a2 + 8bc) ≥ (a + b + c)3. Mà
X a(a2 + 8bc) = a3 + b3 + c3 + 24abc ≤ (a + b + c)3, nên ta có P ≥ 1. III. Bất đẳng thức Chebyshev 1. Bất đẳng thức Chebyshev Định lí 4.
a) Với hai dãy n số thực a1 ≥ a2 ≥ · · · ≥ an và b1 ≥ b2 ≥ · · · ≥ bn cùng tăng hoặc cùng giảm, tức là ( ( a1 ≥ a2 ≥ · · · ≥ an a1 ≤ a2 ≤ · · · ≤ an hoặc b1 ≥ b2 ≥ · · · ≥ bn b1 ≤ b2 ≤ · · · ≤ bn ta luôn có a1b1 + a2b2 + · · · + anbn a b
≥ 1 + a2 + · · · + an · · · 1 + b2 + · · · + bn . n n n
b) Với hai dãy n số thực a1 ≥ a2 ≥ · · · ≥ an và b1 ≥ b2 ≥ · · · ≥ bn có một dãy tăng và một dãy giảm, tức là ( ( a1 ≥ a2 ≥ · · · ≥ an a1 ≤ a2 ≤ · · · ≤ an hoặc b1 ≤ b2 ≤ · · · ≤ bn b1 ≥ b2 ≥ · · · ≥ bn ta luôn có a1b1 + a2b2 + · · · + anbn a b
≤ 1 + a2 + · · · + an · · · 1 + b2 + · · · + bn . n n n
Chứng minh. Ta chứng minh cho trường hợp
(a1 ≥ a2 ≥ · · · ≥ an . b1 ≥ b2 ≥ · · · ≥ bn a1 + a2 + · · · + an Đặt a =
, khi đó tồn tại chỉ số k sao cho ak ≤ a ≤ ak+1, với k đó ta chọn số b n
sao cho bk ≤ b ≤ bk+1. Khi đó ta có
(a − ai)(b − bi) ≥ 0 ⇔ ab − abi − ai + aibi.
Cho i chạy từ 1 đến n và cộng n bất đẳng thức đó ta được n n n X X X nab − a bi − b ai + aibi ≥ 0. i=1 i=1 i=1 n Mà b P ai = nab nên ta có i=1 n n n ! n ! X X 1 X X aibi ≥ a bi = ai bi . n i=1 i=1 i=1 i=1 Ta có đpcm. 35
3. MỘT SỐ BẤT ĐẲNG THỨC KHÁC 2. Ví dụ minh họa
Ví dụ 3.8. Cho các số thực dương a, b, c thỏa a2 + b2 + c2 ≥ 1. Chứng minh rằng a3 b3 c3 1 + + ≥ . b + c c + a a + b 2 a b c
Gải sử a ≥ b ≥ c, khi đó a2 ≥ b2 ≥ c2 và ≥ ≥
, nên áp dụng bất đẳng thức b + c c + a a + b Trebyshev ta có a b c V T = a2 + b2 + c2 b + c c + a a + b 1 a b c 1 ≥ (a2 + b2 + c2) + + ≥ . 3 b + c c + a a + b 2
Ví dụ 3.9. Cho các số thực không âm x, y, z thỏa mãn x + y + z = 1. Chứng minh rằng 1 1 1 27 + + ≤ . 1 + x2 1 + y2 z2 + 1 10
Bất đẳng thức cần chứng minh tương đương với 9 1 9 1 9 1 − + − + − ≥ 0 10 1 + x2 10 1 + y2 10 1 + z2 3x + 1 3y + 1 3z + 1 ⇔ (3x − 1) + (3y − 1) + (3z − 1) ≥ 0. (1) 1 + x2 1 + y2 1 + z2
Với 1 ≥ a ≥ b ≥ 0 ta có 3a + 1 3b + 1
(3a + 1)(b2 + 1) − (3b + 1)(a2 + 1) = (a − b)(3ab − a − b + 3) ≥ 0 ⇒ ≥ . 1 + a2 1 + b2 3x + 1 3y + 1 3z + 1
Giả sử x ≥ y ≥ z, ta có 3x − 1 ≥ 3y − 1 ≥ 3z − 1 và ≥ ≥ . Do đó áp dụng 1 + x2 1 + z2 1 + z2
bất đẳng thức Chebyshev ta có 3x + 1 3y + 1 3z + 1
V T (1) ≥ (3x − 1 + 3y − 1 + 3z − 1) + + = 0. 1 + x2 1 + z2 1 + z2
Vậy bài toán được chứng minh. IV. Bài tập
Bài 3.1. Cho các số thực dương a, b, c. Chứng minh rằng r 2a 2b 2c 3abc + + + 3 ≥ 4. b + c c + a a + b a3 + b3 + c3
Bài 3.2. Cho các số thực dương a, b, c thỏa mãn a + b + c = 1. Chứng minh rằng ab bc ca 2 + + + 1 ≥ 6(ab + bc + ca). a + b b + c c + a 36
3. MỘT SỐ BẤT ĐẲNG THỨC KHÁC
Bài 3.3. Cho các số thực dương a, b, c. Chứng minh rằng s √ X a2 + bc 9 3 abc 3 ≥ . b2 + c2 a + b + c cyc
Bài 3.4. (APMO 2004) Cho các số thực dương a,b,c. Chứng minh rằng
(a2 + 2)(b2 + 2)(c2 + 2) ≥ 9(ab + bc + ca).
Bài 3.5. Cho các số thực không âm a, b, c. Chứng minh rằng s s s (a + b)3 (b + c)3 (c + a)3 √ + + ≥ 2 2. ab(4a + 4b + c) bc(4b + 4c + a) ca(4c + 4a + b)
Bài 3.6. (Hello IMO 2007- Trần Nam Dũng) Chứng minh rằng với mọi a, b, c ≥ 0,ta có:
2(a2 + b2 + c2) + abc + 8 ≥ 5(a + b + c).
Bài 3.7. Cho các số thực dương a,b,c thỏa mãn abc = 1. Chứng minh rằng 1 1 1 + + + 3 ≥ 2(a + b + c). a2 b2 c2
Bài 3.8. Cho các số thực dương a, b, c thỏa mãn abc = 1. Chứng minh rằng
a2 + b2 + c2 + 3 ≥ 2(ab + bc + ca).
Bài 3.9. Cho các số thực dương a, b, c thỏa a + b + c = 1. Chứng minh rằng 4a3 + 9b3 + 36c3 ≥ 1.
Bài 3.10. Cho các số thực dương a, b, c có a + b + c = 1. Chứng minh rằng a b c √ + √ + √ ≥ 1. 3 a + 2b 3 b + 2c 3 c + 2a a b c
Bài 3.11. Cho các số thực dương x, y, z và a, b, c thỏa mãn + + = 1. Chứng minh rằng x y z √ √ √ 3 x2 + y2 + z2 ≥ 3 a2 + 3 b2 + 3 c2 .
Bài 3.12. Cho a, b, c, x, y, z là các số thực dương thỏa ax + by + cz = 1. Chứng minh rằng √ √ √ 1−n xn + yn + zn ≥ n−1 an + n−1 bn + n−1 cn .
Bài 3.13. Cho 3 số thực dương a,b,c. Chứng minh bất đẳng thức: 1 1 1 √ ab + bc + ca √ (a + b + c) + + + 4 2 ≥ 9 + 4 2. a b c a2 + b2 + c2 37 4. PHƯƠNG PHÁP QUY NẠP §4. Phương pháp quy nạp I. Lý thuyết
Quy nạp toán học là một phương pháp mạnh để chứng minh các phát biểu phụ thuộc vào một số tự nhiên. Cho (P (n))
là một dãy các mệnh đề. Phương pháp quy nạp toán học được sử dụng để chứng n≥0
minh P (n) đúng với mọi n ≥ n0 với n0 là một số tự nhiên.
Phương pháp quy nạp toán học (dạng yếu): Giả sử • P (n0) đúng.
• Với mọi k ≥ n0 và P (k) đúng thì P (k + 1) đúng.
Khi đó P (n) đúng với mọi n ≥ n0.
Phương pháp quy nạp toán học (bước nhảy s): Cho s là số nguyên dương. Giả sử
• P (n0) , P (n0 + 1) ,...,P (n0 + s − 1) đúng.
• Với mọi k ≥ n0, P (k) đúng kéo theo P (k + s) đúng .
Khi đó P (n) đúng với mọi n ≥ n0.
Phương pháp quy nạp toán học (Dạng mạnh): Giả sử • P (n0) đúng
• Với mọi k ≥ n0, P (m) đúng với mọi m mà n0 ≤ m ≤ k kéo theo P (k + 1) đúng.
Khi đó P (n) đúng với mọi n ≥ n0. II. Ví dụ minh họa
Ví dụ 4.1. Chứng minh rằng √ 1 1 1 √
n ≤ 1 + √ + √ + · · · + √ ≤ 2 n (1.1) 2 3 n
với mọi số nguyên dương n.
Ta thấy (1.1) đúng với n = 1.
Giả sử (1.1) đúng với n = k ≥ 1, tức là √ 1 1 1 √
k ≤ 1 + √ + √ + · · · + √ ≤ 2 k, (1.2) 2 3 k
ta chứng minh (1.1) đúng với n = k + 1, tức là √ 1 1 1 √
k + 1 ≤ 1 + √ + √ + · · · + √ ≤ 2 k + 1. (1.3) 2 3 k + 1
Thật vậy, dựa vào (1.2) ta có √ 1 1 1 1 √ 1 k + √
≤ 1 + √ + √ + · · · + √ ≤ 2 k + √ . k + 1 2 3 k + 1 k + 1 Mặt khác √ 1 √ 1 1 k + √ − k + 1 = √ − √ √ > 0, k + 1 k + 1 k + k + 1 38 4. PHƯƠNG PHÁP QUY NẠP và √ √ 1 2 1 2 k + 1 − 2 k − √ = √ √ − √ > 0. k + 1 k + k + 1 k + 1 Từ đó ta có đpcm.
Ví dụ 4.2 (VMO 2011). Chứng minh rằng với ∀n ≥ 1,∀x > 0 ta có bất đẳng thức: xn(xn+1 + 1) x + 1 2n+1 ≤ . (1.4) xn + 1 2
Đẳng thức xảy ra khi nào?
• Với n = 1 ta cần chứng minh: x(x2 + 1) x + 1 3 ≤ ⇔ 8x(x2 + 1) ≤ (x + 1)4 x + 1 2 hay là:
x4 − 4x3 + 6x2 − 4x + 1 ≥ 0 ⇔ (x − 1)4 ≥ 0 (đúng).
Suy ra (1.4) đúng với n = 1. Đẳng thức xảy ra khi x = 1.
• Giả sử (1.4) đúng với n = k ≥ 1, tức là: xk(xk+1 + 1) x + 1 2k+1 ≤ . (1.5) xk + 1 2 Ta cần chứng minh: xk+1(xk+2 + 1) x + 1 2k+3 ≤ . (1.6) xk+1 + 1 2 Thật vậy, ta có: x + 1 2k+3 x + 1 2 x + 1 2k+1 x + 1 2 xk(xk+1 + 1) = ≥ . 2 2 2 2 xk + 1
Nên để chứng minh (1.6) ta chỉ cần chứng minh x + 1 2 xk(xk+1 + 1) xk+1(xk+2 + 1) ≥ , 2 xk + 1 xk+1 + 1 hay
x + 1 2 (xk+1 + 1)2 ≥ x(xk+2 + 1)(xk + 1). (1.7) 2
Khai triển (1.7), biến đổi và rút gọn ta thu được:
x2k+2(x − 1)2 − 2xk+1(x − 1)2 + (x − 1)2 ≥ 0 ⇔ (x − 1)2(xk+1 − 1)2 ≥ 0,
bất đẳng thức cuối hiển nhiên đúng. Đẳng thức có khi x = 1.
Vậy bài toán được chứng minh. 39 4. PHƯƠNG PHÁP QUY NẠP
Ví dụ 4.3. Cho n ≥ 2 số thực không âm a1 ≥ a2 ≥ · · · ≥ an có tổng bằng 1. Chứng minh rằng: n n !2 X n − 1 X (i − 1)a2 ≤ a . (1.8) i i 2n i=1 i=1 1 • Với n = 2 ta có: a2 ≤ (a 2
1 + a2)2 đúng do a2 ≤ a1, nên (1.8) đúng với n = 2. 4
• Giả sử (1.8) đúng với n = k, tức là: k k !2 X k − 1 X (i − 1)a2 ≤ a . (1.9) i i 2k i=1 i=1
Ta chứng minh (1.8) đúng với n = k + 1, tức là: k+1 k+1 !2 X k X (i − 1)a2 ≤ a i i 2(k + 1) i=1 i=1 k k !2 X k X ⇔ (i − 1)a2 + ka2 ≤ a . (1.10) i k+1 i + ak+1 2(k + 1) i=1 i=1 1 k Đặt x =
P xi,x ≥ xk+1. Sử dụng (1.9), ta chỉ cần chứng minh: k i=1 k !2 k !2 k − 1 X k X ai + ka2 ≤ ai + ak+1 2k k+1 2(k + 1) i=1 i=1 k(k − 1) k ⇔ x2 + kx2 ≤ (kx + xk+1)2 2 k+1 2(k + 1)
⇔ (k2 − 1)x2 + 2(k + 1)xk+1 ≤ (kx + xk+1)2
⇔ x2 + 2kx · xk+1 − (2k + 1)x2 ≥ 0 k+1
⇔ (x − xk+1) [x + (2k + 1)xk+1] ≥ 0 (đúng). Vậy (1.8) luôn đúng.
Ví dụ 4.4. Cho các số nguyên dương phân biệt a1, a2, · · · , an. Chứng minh rằng a3 + a3 + · · · + a3 ≥ (a 1 2 n 1 + a2 + · · · + an)2 . (1.11) đúng với mọi n ≥ 1.
Không mất tính tổng quát, ta giả sử a1 < a2 < · · · < an.
Với n = 1 ta có a3 ≥ a2 nên (1.11) đúng với n = 1. 1 1
Giả sử (1.11) đúng với n = k và a1 < a2 < · · · < ak < ak+1 là các số nguyên dương. Khi đó ak+1 ≥ ak + 1, nên (ak+1 − 1) ak+1 a
≥ k (ak + 1) = 1 + 2 + · · · + ak. 2 2
Vì 1 + 2 + · · · + ak là tổng của ak số nguyên dương đầu tiên, nên
1 + 2 + · · · + ak ≥ a1 + a2 + · · · + ak. 40 4. PHƯƠNG PHÁP QUY NẠP Suy ra
(ak+1 − 1) ak+1 ≥ a1 + a2 + · · · + ak. 2 Hay a3 ≥ 2a . k+1
k+1 (a1 + a2 + · · · + ak ) + a2 k+1 Do đó a3 + a3 + · · · + a3 + a3 ≥ (a 1 2 k k+1
1 + a2 + · · · + ak )2 + 2ak+1 (a1 + a2 + · · · + ak ) + a2 k+1
= (a1 + a2 + · · · + ak+1)2 .
Suy ra (1.11) được chứng minh.
Ví dụ 4.5. Cho 2n số thực dương a1, a2, · · · , an và b1, b2, · · · , bn. Chứng minh rằng n n X (b1 + b2 + · · · + bk )bk X b2 < 2 i . (1.12) a1 + a2 + · · · + ak ai k=1 i=1
Ta thấy bất đẳng thức (1.12) đúng với n = 1.
Giả sử bất đẳng thức (1.12) đúng với n ≥, tức là n n X (b1 + b2 + · · · + bk )bk X b2 < 2 i , (1.13) a1 + a2 + · · · + ak ai k=1 i=1
ta chứng minh (1.12) đúng với n + 1, tức là n+1 n+1 X (b1 + b2 + · · · + bk )bk X b2 < 2 i . (1.14) a1 + a2 + · · · + ak ai k=1 i=1
Áp dụng cho trường hợp 2n số
(a1 + a2, a3, a4 . . . , an+1, b1 + b2, b3, b4, . . . , bn+1) ta có n+1 n+1 X (b1 + b2 + · · · + bk )bk X b2 < 2 i . a1 + a2 + · · · + ak ai k=1 i=1 n n (b1 + b2)2 X (b 3 (b X b2 + 1 + · · · + bk ) bk ≤ 1 + b2)2 + 2 k . a1 + a2 a1 + · · · + ak 2 a1 + a2 ak k=3 k=3
Bất đẳng thức tương đương với b2 (b 1 (b 3 b2 b2 1 + 1 + b2) b2 + 1 + b2)2 ≤ 1 + 2 2 a1 a1 + a2 2 a1 + a2 2 a1 a2 b2 + 4b b2 b2 ⇔ 1 1b2 + 3b2 2 ≤ 1 + 4 2 a1 + a2 a1 a2 a a ⇔ b2 + 4b ≤ b2 + 2 b2 + 4 1 b2 + 4b2 1 1b2 + 3b2 2 1 a 1 2 2 1 a2 a a ⇔ 4b 2 1 1b2 ≤ b2 + 4 b2 + b2 a 1 2 2 1 a2
Bất đẳng thức cuối đúng, bằng cách áp dụng bất đẳng thức AM-GM a r 2 a a a b2 + 4 1 b2 ≥ 2 2 b2 · 4 1 b2 = 4b1b2. a 1 2 1 2 1 a2 a1 a2 41
5. PHƯƠNG PHÁP PHÂN TÍCH BÌNH PHƯƠNG SOS
§5. Phương pháp phân tích bình phương SOS I. Lý thuyết 1.
Một số tiêu chuẩn đánh giá Xét biểu thức:
S = f (a, b, c) = Sa(b − c)2 + Sb(c − a)2 + Sc(a − b)2.
trong đó Sa, Sb, Sc là các biểu thức chứa a, b, c là các số thực không âm. Tính chất 1.
Nếu Sa, Sb, Sc ≥ 0 thì S ≥ 0. Tính chất 2.
Nếu a ≥ b ≥ c và Sb, Sb + Sc ≥ 0, Sb + Sa ≥ 0 thì S ≥ 0. Thật vậy: Vì a ≥ b ≥ c nên
(c − a)2 = [(a − b) + (b − c)]2 ≥ (a − b)2 + (b − c)2 . Suy ra S ≥ S a (b − c)2 + Sb
(a − b)2 + (b − c)2 + Sc (a − b)2
= (Sa + Sb) (b − c)2 + (Sb + Sc) (a − b)2 ≥ 0. Tính chất 3.
Nếu a ≥ b ≥ c và Sb ≤ 0, Sa + 2Sb ≥ 0, Sc + 2Sb ≥ 0 thì S ≥ 0. Thật vậy, ta có
(a − c)2 = [(a − b) + (b − c)]2 ≤ 2 (a − b)2 + 2 (b − c)2 . Suy ra
S ≥ (Sa + 2Sb) (b − c)2 + (Sc + 2Sb) (a − b)2 ≥ 0. Tính chất 4.
Nếu a ≥ b ≥ c và Sb ≥ 0, Sc ≥ 0, a2Sb + b2Sa ≥ 0 thì S ≥ 0. a − c b Ta có a ≥ b ≥ c ⇒ ≥ . Suy ra b − c a " c − a 2# b2
Sa (b − c)2 + Sb (c − a)2 = (b − c)2 Sa + Sb ≥ (b − c)2 Sa + Sb ≥ 0. b − c a2 2.
Một số biểu diễn cơ sở a b 1 • + − 2 = (a − b)2. b a ab 1
• a2 + b2 + c2 − ab − bc − ca =
(a − b)2 + (b − c)2 + (c − a)2. 2 (a − b)2 • p2 (a2 + b2) − (a + b) = . a + b + p2 (a2 + b2) 1 • a3 + b3 + c3 − 3abc =
(a + b + c) (a − b)2 + (b − c)2 + (c − a)2. 2
• a3 + b3 − ab (a + b) = (a + b) (a − b)2. 42
5. PHƯƠNG PHÁP PHÂN TÍCH BÌNH PHƯƠNG SOS 1
• a3 + b3 + c3 − a2b − b2c − c2a =
(2a + b) (a − b)2 + (2b + c) (b − c)2 + (2c + a) (c − a)2. 3
• (a + b) (b + c) (c + a) − 8abc = a (b − c)2 + b (c − a)2 + c (a − b)2. 1
• a2b + b2c + c2a − ab2 − bc2 − ca2 = − (a − b)3 + (b − c)3 + (c − a)3. 3 a b c 3 (a − b)2 (b − c)2 (c − a)2 • + + − = + + . b + c c + a a + b 2 2 (a + c) (b + c) 2 (b + a) (c + a) 2 (c + b) (a + c) II. Các ví dụ
Ví dụ 5.1 (IMO 2005). Cho x, y, z > 0 thỏa mãn xyz ≥ 1. Chứng minh rằng x5 − x2 y5 − y2 z5 − z2 + + ≥ 0. (1) x5 + y2 + z2 y5 + z2 + x2 z5 + x2 + y2 Ta có x5 − x2 x5 − x2xyz x4 − x2yz ≥ = x5 + y2 + z2 x5 + (y2 + z2) xyz x4 + yz (y2 + z2) 2x4 − x2 (y2 + z2) 2a2 − a (b + c) ≥ = , 2x4 + (y2 + z2)2 2a2 + (b + c)2 với a = x2, b = y2, c = z2. Suy ra X 2a2 − a (b + c) X a (a − b) + a (a − c) V T (1) ≥ = 2a2 + (b + c)2 2a2 + (b + c)2 a, b, c a, b, c X a b = (a − b) − 2a2 + (b + c)2 2b2 + (c + a)2 X =
(a − b)2 c (2a + 2b + c) + a2 − ab + b2 ≥ 0 (đúng).
2a2 + (b + c)2 (2b2 + (c + a)2) a, b, c
Ví dụ 5.2 (VMO 2015). Cho a, b, c ≥ 0. Chứng minh rằng √ √ √ (a + b + c) ab + bc +
ca + (a − b)2 + (b − c)2 + (c − a)2 ≥ (a + b + c)2 .
Bất đẳng thức cần chứng minh tương đương với √ √ √
(a − b)2 + (b − c)2 + (c − a)2 ≥ (a + b + c) a + b + c − ab − bc − ca . √ √ √ Đặt x = a,y = b,z = c ta cần chứng minh X
x2 − y22 ≥ x2 + y2 + z2 x2 + y2 + z2 − xy − yz − zx . (2) Mà 1 1 1
x2 + y2 + z2 − xy − yz − zx = (x − y)2 + (y − z)2 + (z − x)2 . 2 2 2 43
5. PHƯƠNG PHÁP PHÂN TÍCH BÌNH PHƯƠNG SOS Nên (2) trở thành
Sx (y − z)2 + Sy (z − x)2 + Sz (x − y)2 ≥ 0, (3)
với Sx = y2 + z2 + 4yz − x2,Sy = z2 + x2 + 4zx − y2 , Sz = x2 + y2 + 4xy − z2.
Gải sử x ≥ y ≥ z , khi đó Sy ≥ 0,Sz ≥ 0 và Sx + Sy = 2z2 + 4yz + 4zx ≥ 0 .
Lại có (x − z)2 ≥ (y − z)2 nên
V T (3) ≥ (Sx + Sy) (y − z)2 ≥ 0 nên (3) đúng.
Ví dụ 5.3. Cho a, b, c > 0. Chứng minh rằng: a2 + b2 + c2 8abc + ≥ 2. ab + bc + ca (a + b)(b + c)(c + a) Ta có 1
a2 + b2 + c2 − (ab + bc + ca) =
(a − b)2 + (b − c)2 + (c − a)2 2
(a + b) (b + c) (c + a) − 8abc = a (b − c)2 + b (c − a)2 + c (a − b)2 .
Bất đẳng thức cần chứng minh tương đương với
(a − b)2 + (b − c)2 + (c − a)2
2a (b − c)2 + 2b (c − a)2 + 2c (a − b)2 ≥ ab + bc + ca (a + b) (b + c) (c + a) X 1 2c ⇔ (a − b)2 − ≥ 0 ab + bc + ca (a + b) (b + c) (c + a) X ⇔ (a − b)2 Sc ≥ 0. (4) Với 1 2a Sa = − ab + bc + ca (a + b) (b + c) (c + a) 1 2b Sb = − ab + bc + ca (a + b) (b + c) (c + a) 1 2c Sc = − . ab + bc + ca (a + b) (b + c) (c + a)
Giả sử a ≥ b ≥ c ta có Sb, Sc ≥ 0 và 2 2 Sa + Sb = − ≥ 0. ab + bc + ca (a + c) (b + c) Suy ra (4) đúng.
Ví dụ 5.4. Cho a, b, c > 0. Chứng minh rằng: a2 b2 c2 3(a3 + b3 + c3) + + ≥ . b c a a2 + b2 + c2
Ta giả sử b nằm giữa a và c. Ta có a2 b2 c2 (a − b)2 (b − c)2 (c − a)2 + + − (a + b + c) = + + b c a b c a 3(a3 + b3 + c3)
(a + b) (a − b)2 + (b + c) (b − c)2 + (c + a) (c − a)2 − (a + b + c) = . a2 + b2 + c2 a2 + b2 + c2 44
5. PHƯƠNG PHÁP PHÂN TÍCH BÌNH PHƯƠNG SOS
Nên bất đẳng thức cần chứng minh tương đương với
Sa (b − c)2 + Sb (c − a)2 + Sc (a − b)2 ≥ 0. (5) a2 + b2 − bc b2 + c2 − ac a2 + c2 − ab Với Sa = ; Sb = ; Sc = . c a b
• Nếu a ≥ b ≥ c ⇒ Sa ≥ 0,Sc ≥ 0 và a2 + b2 − bc b2 + c2 − ca Sa + 2Sb = + 2 > 0. c a Ta có c2 + a2 − ab b2 + c2 − ca Sc + 2Sb = + 2 b a c2 b2 + c2 − ca ≥ + a − b + 2 b a c2 b2 + c2 − ca ≥ + a − b + 2 , a a hay a2 + 2b2 + 3c2 − ab − 2ca Sc + 2Sb ≥ . a Mặt khác  1   a2 + 3c2 ≥ 2ca 3 2
⇒ a2 + 2b2 + 3c2 ≥ 2ca + ab.  a2 + 2b2 > ab  3
Do đó, ta có Sc + 2Sb > 0.
• Nếu a ≤ b ≤ c ⇒ Sb ≥ 0 thì ta có b2 + c2 − ca a2 + b2 − bc Sb + Sa = + > 0 a c b2 + c2 − ca c2 + a2 − ab Sb + Sc = + > 0. a b
Ví dụ 5.5 (Iran 1996). Cho a, b, c là số dương. Chứng minh rằng : 1 1 1 9 (ab + bc + ca) + + ≥ . (a + b)2 (b + c)2 (c + a)2 4 x + z − y x + y − z z + y − x
Đặt x = a + b, y = b + c, z = c + a ⇒ a = ,b = ,c = . 2 2 2
Bất đẳng thức cần chứng minh trở thành 1 1 1
2xy + 2yz + 2zx − x2 − y2 − z2 + + ≥ 9 x2 y2 z2 1 1 1 1 1 1
⇔ 2 xy + yz + zx − x2 − y2 − z2 + + + x2 + y2 + z2 + + − 9 ≥ 0 x2 y2 z2 x2 y2 z2 1 1 1 x2 y2 y2 z2
⇔ − (x − y)2 + (y − z)2 + (z − x)2 + + + + − 2 + + − 2 + x2 y2 z2 y2 x2 z2 y2 z2 x2 + + − 2 ≥ 0 x2 z2 2 1 2 1 2 1
⇔ Sz (x − y)2 + Sx (y − z)2 + Sy (z − x)2 ≥ 0; Sx = − , Sy = − , Sz = − . yz x2 zx y2 xy z2 45
5. PHƯƠNG PHÁP PHÂN TÍCH BÌNH PHƯƠNG SOS
Giả sử x ≥ y ≥ z suy ra Sx ≥ 0,Sx ≥ Sy ≥ Sz. Mặt khác
y + z ≥ x ⇒ zx ≤ z (y + z) ≤ 2y2 ⇒ Sy ≥ 0 y2 (x − z)2 ≥
(x − y)2 ⇔ (y − z) (y + z − x) ≥ 0 (đúng). z2 Suy ra
Sz (x − y)2 + Sx (y − z)2 + Sy (z − x)2 ≥ Sy (z − x)2 + Sz (x − y)2 y2 ≥ (x − y)2 Sy + Sz (x − y)2 z2 Mà 2 1 2 2 1 2 2y2 2z2 y2Sy + z2Sz = y2 − + z2 − = + − 2 ≥ 0 zx y xy z zx xy ⇔ y3 + z3 ≥ xyz.
Do y + z ≥ x,y2 + z2 − yz ≥ yz suy ra y2Sy + z2Sz ≥ 0. III. Bài tập
Bài 5.1 (Bất đẳng thức Schur). Cho a, b, c là các không âm . Chứng minh rằng :
a3 + b3 + c3 + 3abc ≥ ab (a + b) + bc (b + c) + ca (c + a) .
Bài 5.2. Cho a, b, c ≥ 0 và không có hai số nào đồng thời bằng 0. Chứng minh rằng ab − bc + ca bc − ca + ab ca − ab + bc 3 + + ≥ . b2 + c2 c2 + a2 a2 + b2 2
Bài 5.3. Cho a, b, c là các không âm . Chứng minh rằng : p p p a3 + b3 + c3 + 3abc ≥ ab 2 (a2 + b2) + bc 2 (b2 + c2) + ca 2 (c2 + a2).
Bài 5.4. Cho a, b, c là số không âm.Chứng minh rằng : 2a2 + bc 2b2 + ca 2c2 + ab 9 + + ≥ . b2 + c2 c2 + a2 a2 + b2 2
Bài 5.5. Cho a, b, c là các số dương. Chứng minh rằng
a3 + b3 + c3 + 6 ab2 + bc2 + ca2 ≥ 3 a2b + b2c + c2a + 12abc.
Bài 5.6. (VN TST 2006) Chứng minh rằng với mọi số thực x,y,z ∈ [1; 2] , ta luôn có bất đẳng thức sau : 1 1 1 x y z (x + y + z) + + ≥ 6 + + . x y z y + z z + x x + y
Hỏi đẳng thức xảy ra khi và chỉ khi nào ?
Bài 5.7. Bài 5.8. Cho các số thực a,b,c ∈ [0; 1] thỏa mãn a + b + c = 2. 2 4 Chứng minh rằng
(a2 + b2 + c2) ≤ a3 + b3 + c3 ≤ (a2 + b2 + c2) − 3abc. 3 3 46
5. PHƯƠNG PHÁP PHÂN TÍCH BÌNH PHƯƠNG SOS
Bài 5.9. Cho a, b, c > 0. Chứng minh rằng: 1 a2 + b2 + c2 a b c + ≥ + + . 2 ab + bc + ca b + c c + a a + b
Bài 5.10. Cho a, b, c > 0. Chứng minh rằng: p a2 + b2 b2 + c2 c2 + a2 3(a2 + b2 + c2) ≤ + + . a + b b + c c + a
Bài 5.11. Cho a, b, c > 0. Chứng minh rằng: 3(a2 + b2 + c2) a2 + b2 b2 + c2 c2 + a2 ≥ + + . a + b + c a + b b + c c + a
Bài 5.12. Cho a, b, c > 0. Chứng minh rằng: a2 b2 c2 3 p + + + a + b + c ≥ 3(a2 + b2 + c2). b + c c + a a + b 2
Bài 5.13. Cho a, b, c > 0. Chứng minh rằng: a b c 1 ab + bc + ca + + ≥ 4 − . b + c c + a a + b 2 a2 + b2 + c2
Bài 5.14. Cho a, b, c > 0. Chứng minh rằng: 2(a3 + b3 + c3) 9(a + b + c)2 + ≥ 33. abc a2 + b2 + c2 47 6. PHƯƠNG PHÁP DỒN BIẾN
§6. Phương pháp dồn biến I. Lý thuyết
Mục đích của phương pháp này là làm giảm biến trong bất đẳng thức cần chứng minh. II. Ví dụ minh họa
Ví dụ 6.1. Cho a, b, c ≥ −3 và a + b + c = 3. Chứng minh rằng 1 1 1 1 1 1 + + ≥ + + . a2 b2 c2 a b c 1 1
Giả sử a = min{a, b, c} và đặt f (a, b, c) = P − . cyc a2 a Ta có −3 ≤ a ≤ 1 và b + c b + c 1 1 8 1 1 4 f (a, b, c) − f a, , = + − − + − 2 2 b2 c2 (b + c)2 b c b + c
(b − c)2(b2 + 4bc + c2 − bc(b + c)) = b2c2(b + c)2
(b − c)2((3 − a)2 + bc(a − 1)) = b2c2(b + c)2 (3 − a)2(a − 1) (b − c)2 (3 − a)2 + 4 ≥ b2c2(b + c)2 (b − c)2(3 − a)2(a + 3) = ≥ 0. 4b2c2(b + c)2 a + b 3 − a
Tiếp theo ta chứng minh f (a,t,t) ≥ 0 với t = = . Thật vậy 2 2 1 2 1 2 f (a,t,t) = + − − a2 t2 a t t2 + 2a2 − at2 − 2ta2 = a2t2 (a + 3)(a − 1)2 = ≥ 0 4a2t2
Vậy bài toán được chứng minh.
Ví dụ 6.2. Cho các số thực dương a, b, c có tích bằng 1. Chứng minh rằng 1 1 1 6 + + + ≥ 5. a b c a + b + c 48 6. PHƯƠNG PHÁP DỒN BIẾN
Gải sử a = max{a, b, c}, suy ra bc ≤ 1 ≤ a. 1 1 1 6 √ Đặt f (a, b, c) = + + + và x = bc. Ta có a b c a + b + c 1 1 2 6 6 f (a, b, c) − f (a,x,x) = + − + − b c x a + b + c a + 2x √ √ 2 √ √ 2 b − c 6 b − c = − bc (a + b + c)(a + 2x) √ √ 2 1 6 = b − c − . bc (a + b + c)(a + 2x) Ta có 1 1
(a + b + c)(a + 2x) − 6bc ≥ + 2x + 2x
− 6x2 ≥ 3 · 3 − 6x2 > 0. x2 x2 Suy ra 1 f (a, b, c) ≥ f (a,x,x) = f ,x,x . x2 1 Ta chứng minh f ,x,x ≥ 5. (1) x2
Tuy nhiên (1) tương đương với
(x − 1)2(2x4 + 4x3 − 4x2 − x + 2) ≥ 0. (2)
Bất đẳng thức (2) đúng do 0 < x ≤ 1.
Ví dụ 6.3. Cho các số thực dương a, b, c thỏa mãn a2 + b2 + c2 = 3. Chứng minh rằng
a3(b + c) + b3(c + a) + c3(a + b) ≤ 6. r b2 + c2
Giả sử a = min{a, b, c}. Đặt f (a, b, c) = a3(b + c) + b3(c + a) + c3(a + b) và t = ≥ a. 2 Xét
P = f (a, b, c) − f (a,t,t) = a3(b + c − 2t) + a(b3 + c3 − 2t3) + t2(2bc − 2t2) Ta có (b + c)2 1 b2 + c2 ≥ ⇒ t ≥
(b + c) ⇒ b + c − 2t ≤ 0, 2 2
b3 + c3 − 2t3 = (b + c)(b2 + c2 − bc) − 2t3 ≤ 2t(b2 + c2 − bc) − 2t3 = t(b − c)2 2bc − 2t2 = −(b − c)2. Suy ra
P ≤ 0 + ta(b − c)2 − t2(b − c)2 = t(b − c)2(a − t) ≤ 0 ⇒ f (a, b, c) ≤ f (a,t,t).
Mặt khác, ta có a2 + 2t2 = 3 nên
f (a,t,t) ≤ 6 ⇔ a3t + t3a ≤ 3 − t4 ⇔ at(a2 + t2) ≤ 3 − t4
⇔ a2t2(3 − t2)2 ≤ (3 − t4)2 ⇔ (t2 − 1)2(t4 − 3t2 + 3) ≥ 0 (bđt này luôn đúng). 49 6. PHƯƠNG PHÁP DỒN BIẾN
Ví dụ 6.4. Cho các số thực không âm a, b, c. Chứng minh rằng
(a2 + b2 + c2)2 ≥ 4(a + b + c)(a − b)(b − c)(c − a).
Kí hiệu (1) là bất đẳng thức cần chứng minh Không mất tính tổng quát, ta giả sử a = min {a, b, c}.
+) b > c thì (1) luôn đúng. +) Xét c > b. Khi đó
V T (1) > b2 + c22 và c − a 6 c Ta chứng minh :
(a + b + c) (b − a) 6 b (b + c) (2) Thật vậy
(2) ⇔ ab − a2 + b2 − ab + bc − ca 6 b2 + bc ⇔ a (c + a) > 0 (luôn đúng) Từ đó, suy ra
V P (1) 6 bc (b + c) (c − b) = bc c2 − b2 .
Để chứng minh (1), ta chứng minh: b2 + c22 > 4bc b2 − c2 (3). c
Nếu b = 0 ⇒ (3) đúng. Xét b 6= 0 ta đặt t =
> 1. Khi đó (3) trở thành b
t2 + 12 > 4t t2 − 1 ⇔ t4 − 4t3 + 2t2 + 4t + 1 = 0 ⇔ t2 − 2t − 12 > 0 (luôn đúng). a = 0 
Suy ra (1) được chứng minh. Đẳng thức xảy khi √ và các hoán vị. c = 1 + 2 b 
Ví dụ 6.5 (Hojoo Lee). Cho các số thực không âm a, b, c thỏa mãn ab + bc + ca = 1. Chứng minh rằng 1 1 1 5 + + ≥ . a + b b + c c + a 2 1 1 1
Giả sử c = max{a, b, c}. Đặt f (a, b, c) = + + . a + b b + c c + a
Nếu c = 0, ta có ab = 1 nên a + b ≥ 2 và 1 1 1 1 f (a, b, c) = + + = + a + b a + b a b a + b 1 a + b 3(a + b) 3 5 = + + ≥ 1 + = . a + b 4 4 2 2 50 6. PHƯƠNG PHÁP DỒN BIẾN Xét 1 P = f (a, b, c) − f a + b, ,0 a + b     1 1 1 1 1 =  + +  −  + a + b +   a + b 1 − ab 1 − ab   1 a + b  b + c + a + b + a + b a + b a + b 1 1 1 = (a + b) + − 1 − 1 + a2 1 + b2 1 + (a + b)2 a (2 − 2ab − ab(a + b)2) = (a + b) . (1 + a2)(1 + b2)(1 + (a + b)2)
Ta có 2 − 2ab = 2(1 − ab) = 2(bc + ac) = 2c(a + b) ≥ ab(a + b)2. Suy ra P ≥ 0, từ đó ta có 1 5 f (a, b, c) ≥ f (t, ,0) ≥ . t 2 III. Bài tập
Bài 6.1. Xét các số thực dương a, b, c thỏa mãn a + b + c = 3. 12
a) Tìm giá trị nhỏ nhất của biểu thức P = abc + . ab + bc + ca
b) Chứng minh số nguyên k nhỏ nhất sao cho k k abc + ≥ 1 + ab + bc + ca 3
với mọi a, b, c thỏa mãn điều kiện trên là k = 10.
Bài 6.2 (Nguyễn Văn Quý). Cho các số thực dương a,b,c thỏa mãn điều kiện a + b + c = 3. Chứng minh rằng √ √ √ 3a2 + 4bc + 9 + 3b2 + 4ca + 9 + 3c2 + 4ab + 9 ≥ 12.
Bài 6.3 (Liu Quan Bao). Cho a, b, c, d là các số thực dương thỏa mãn điều kiện a + b + c = 3. Chứng minh rằng √ √ √ √ 9 − 6ab + a2 + b2 + 9 − 6bc + b2 + c2 + 9 − 6ca + c2 + a2 ≥ 3 5.
Bài 6.4. (Yi Chang) Cho các số thực dương a,b,c thỏa mãn điều kiện a + b + c = 3. Chứng minh r 7 r 7 r 7 rằng 1 + 3bc + (b − c)2 + 1 + 3ca + (c − a)2 + 1 + 3ab + (a − b)2 ≤ 6. 12 12 12
Bài 6.5. (Võ Quốc Bá Cẩn) Cho các số thực dương a,b,c thỏa mãn điều kiện a + b + c = 3.
Chứng minh rằng p2(a2 + b2) + 21c + p2(b2 + c2) + 21a + p2(c2 + a2) + 21b ≥ 15.
Bài 6.6. (Phạm Thanh Tùng) Cho các số thực dương a,b,c có tổng bằng 3. Chứng minh rằng √ √ √ 3a2 − a + 1 + 3b2 − b + 1 +
3c2 − c + 1 ≤ p6(a2 + b2 + c2) + 9 51 6. PHƯƠNG PHÁP DỒN BIẾN
Bài 6.7. (Phạm Kim Hùng) Cho các số thực a,b,c thỏa mãn điều kiện a + b + c = 2. Chứng minh rằng √ √ √ a + b − 2ab + b + c − 2bc + c + a − 2ca ≥ 2.
Để kết thúc bài viết, chúng tôi xin giới thiệu một bài toán rất chặt và khó của Liu Quan Ban.
Ngoài lời giải bằng dồn biến mà chúng tôi biết và giới thiệu ở đây chúng tôi chưa thấy có lời giải nào khác.
Bài 6.8. (Liu Quan Bao) Cho các số thực a, b,c thỏa mãn điều kiện a + b + c = 3. Chứng minh rằng √ √ √ 1 p 2a2 − a + 1 + 2b2 − b + 1 + 2c2 − c + 1 ≥ 21(a2 + b2 + c2) + 99. 3
Bài 6.9. Cho các số thực dương a, b, c có tích bằng 1. Chứng minh rằng 1 1 1 13 25 + + + ≥ . a b c a + b + c 4
Bài 6.10. Cho các số thực dương a, b, c thỏa mãn a2 + b2 + c2 = 9. Chứng minh rằng 2(a + b − c) + abc ≤ 10.
Bài 6.11. Cho các số thực dương a, b, c thỏa a + b + c = 3. Chứng minh rằng
2(a4 + b4 + c4) + 36 ≥ 7 a3 + b3 + c3 + 3abc .
Bài 6.12. Cho các số thực a, b, c > 0. Chứng minh rằng √
2(a2 + b2 + c2) + 3 3 a2b2c2 ≥ (a + b + c)2.
Bài 6.13. Cho các số thực a, b, c ≥ −3 thỏa mãn a + b + c = 3. Chứng minh rằng 1 1 1 1 1 1 + + ≥ + + . a2 b2 c2 a b c
Bài 6.14. Cho các số thực a, b, c thỏa mãn a2 + b2 + c2 = 9. Chứng minh rằng 2(a + b + c) − abc ≤ 10.
Bài 6.15 (Iran 1996). Cho a, b, c là số dương. Chứng minh rằng : 1 1 1 9 (ab + bc + ca) + + ≥ . (a + b)2 (b + c)2 (c + a)2 4 52 Chương 2
Các phương pháp chứng minh bất đẳng thức hiện đại 53 1. PHƯƠNG PHÁP P, Q, R §1. Phương pháp p, q, r I. Lý thuyết 1. Bất đẳng thức Schur
Cho các số thực không âm x,y,z và số thực dương r. Khi đó, ta có bất đẳng thức sau
xr(x − y)(x − z) + yr(y − x)(y − z) + zr(z − x)(z − y) ≥ 0.
Đẳng thức xảy ra khi x = y = z hoặc z = 0,x = y và các hoán vị. Hệ quả 1.
a) Xét r = 1 ta có các dạng sau
• x3 + y3 + z3 + 3xyz ≥ xy(x + y) + yz(y + z) + zx(z + x)
• 4(x3 + y3 + z3) + 15xyz ≥ (x + y + z)3
• xyz ≥ (x + y − z)(y + z − x)(z + x − y) 9xyz • x2 + y2 + z2 + ≥ 2(xy + yz + zx) x + y + z
• (x + y + z)3 + 9xyz ≥ 4(x + y + z)(xy + yz + zx)
b) r = 2 ta có các dạng sau
• x4 + y4 + z4 + xyz(x + y + z) ≥ xy(x2 + y2) + yz(y2 + z2) + zx(z2 + x2)
• 6xyz(x + y + z) ≥ [2(xy + yz + zx) − (x2 + y2 + z2)] (x2 + y2 + z2 + xy + yz + zx). 2.
Một số biểu diễn đa thức đối xứng ba biến qua p, q, r
Cho các số thực a, b, c. Đặt p = a + b + c, p = ab + bc + ca và r = abc. Khi đó ta có các biểu diễn sau
• ab(a + b) + bc(b + c) + ca(c + a) = pq − 3r.
• (a + b)(b + c)(c + a) = pq − r.
• ab(a2 + b2) + bc(b2 + c2) + ca(c2 + a2) = p2q − 2q2 − pr.
• (a + b)(a + c) + (b + c)(b + a) + (c + a)(c + b) = p2 + q. • a2 + b2 + c2 = p2 − 2q.
• a3 + b3 + c3 = p3 − 3pq + 3r.
• a4 + b4 + c4 = p4 − 4p2q + 2q2 + 4pr.
• a2b2 + b2c2 + c2a2 = q2 − 2pr.
• a3b3 + b3c3 + c3a3 = q3 − 3pqr + 3r2.
• a4b4 + b4c4 + c4a4 = q4 − 4pq2r + 2p2r2 + 4qr2. 54 1. PHƯƠNG PHÁP P, Q, R 3.
Một số đánh giá giữa p, q, r
Dựa vào các bất đẳng thức cơ bản ba biến và bất đẳng thức Schur ta có các đánh giá sau
• (a + b + c)2 ≥ 3(ab + bc + ca) ⇒ p2 ≥ 3q.
• (a + b + c)3 ≥ 27abc ⇒ p3 ≥ 27r.
• (ab + bc + ca)2 ≥ 3abc(a + b + c) ⇒ q2 ≥ 3pr.
• (a + b + c)(ab + bc + ca) ≥ 9abc ⇒ pq ≥ 9r.
• p3 + 9r ≥ 4pq (BĐT Schur với r = 1).
• p4 + 4q2 + 6pr ≥ 5p2q (BĐT Schur với r = 2). p(4q − p2) • r ≥ max 0, (BĐT Schur với r = 1). 9 (4q − p2)(p2 − q) • r ≥ max 0, (BĐT Schur với r = 2). 6p II. Một số ví dụ
Ví dụ 1.1. Cho các số thực dương a, b, c thỏa mãn a2 + b2 + c2 = 3. Chứng minh rằng 3 5(a + b + c) + ≥ 18. abc
Ta có p2 − 2q = 3 và bất đẳng thức cần chứng minh trở thành 3 5p + ≥ 18. (1) r Ta có 3 9p 36p q2 ≥ 3rq ⇒ ≥ = . r q2 (p2 − 3)2 Do đó 36p V T (1) ≥ 5p + . (p2 − 3)2 Nên ta đi chứng minh 36p 5p +
⇔ 5p5 − 18p4 − 30p3 + 108p2 + 81p − 162 ≥ 0 (p2 − 3)2
⇔ (p − 3)2(5p3 + 12p2 − 3p − 18) ≥ 0 (2) Ta có √
3 < p2 ≤ 3(a2 + b2 + c2) = 9 ⇒
3 < p ≤ 3 ⇒ 5p3 + 12p2 − 3p − 18 > 0, nên (2) luôn đúng. 55 1. PHƯƠNG PHÁP P, Q, R
Ví dụ 1.2. Cho các số thực dương a, b, c thỏa mãn ab + bc + ca = 1. Chứng minh rằng 1 1 1 5 + + ≥ . a + b b + c c + a 2
Bất đẳng thức cần chứng minh tương đương với
(a + b)(b + c) + (b + c)(c + a) + (c + a)(a + b) 5 ≥ (a + b)(b + c)(c + a) 2 p2 + q 5 p2 + 1 5 ⇔ ≥ ⇔ ≥ ⇔ 2p2 − 5p + 5r + 2 ≥ 0. (1) pq − r 2 p − r 2 Nếu p ≥ 2 thì
2p2 − 5p + 2 + 5r = (p − 2)(2p − 1) + 5r ≥ 0, suy ra (1) đúng. √ 4pq − p3 4p − p3 Xét 3 ≤ p < 2 ta có r ≥ =
. Nên để chứng minh (1) ta chứng minh 9 9 4p − p3 2p2 − 5p + 2 + 5
≥ 0 ⇔ (p − 2)(5p2 − 8p + 9) ≤ (luôn đúng). 9
Ví dụ 1.3. Chứng minh rằng nếu x, y, z > 0 thì 1 1 1 9 (xy + yz + zx)( + + ) ≥ . (x + y)2 (y + z)2 (z + x)2 4 Ta có
(x + y)2(y + z)2 + (y + z)2(z + x)2 + (z + x)2(x + y)2
= ((x + y)(y + z) + (y + z)(z + x) + (z + x)(x + y))2 − 4(x + y)(y + z)(z + x)(x + y + z) = (p2 + q)2 − 4p(pq − r).
Do đó bất đẳng thức cần chứng minh tương đương với (p2 + q)2 − 4p(pq − r) 9 q( ) ≥ (pq − r)2 4
⇔ 4p4q − 17p2q2 + 4q3 + 34pqr − 9r2 ≥ 0
⇔ 3pq(p3 − 4pq + 9r) + q(p4 − 5p2q + 4q2 + 6pr) + r(pq − 9r) ≥ 0.
Bất đẳng thức cuối đúng nên ta có đpcm. III. Bài tập
Bài 1.1. Cho các số dương a, b, c thỏa abc = 1. Chứng minh rằng : 3 6 1 + ≥ . a + b + c ab + bc + ca 56 1. PHƯƠNG PHÁP P, Q, R
Bài 1.2. Cho các số thực dương a, b, c thỏa mãn ab + bc + ca + 6abc = 9. Chứng minh rằng a + b + c + 3abc ≥ 6.
Bài 1.3. Cho a, b, c là các số thực không âm thỏa mãn ab + bc + ca = 3.Chứng minh rằng: a3 + b3 + c3 + 7abc ≥ 10.
Bài 1.4. Cho a, b, c > 0 thỏa a + b + c = 3.Chứng minh rằng: 12 1 1 1 3 + ≥ 5 + + . abc a b c
Bài 1.5. Cho a, b, c là các số thực dương thỏa mãn a2 + b2 + c2 = 3.Chứng minh rằng: 1 1 1 + + ≥ 3. 2 − a 2 − b 2 − c
Bài 1.6. Cho a, b, c là các số thực không âm thỏa mãn a + b + c = 3.Chứng minh rằng: 1 1 1 3 + + ≤ . 9 − ab 9 − bc 9 − ca 8
Bài 1.7. Cho các số thực không âm x, y, z thỏa mãn xy + yz + zx + xyz = 4. Chứng minh rằng x2 + y2 + z2 + 5xyz ≥ 8. 57
2. PHƯƠNG PHÁP SỬ DỤNG TIẾP TUYẾN VÀ CÁT TUYẾN
§2. Phương pháp sử dụng tiếp tuyến và cát tuyến I. Lý thuyết 1.
Hàm lồi - Dấu hiệu hàm lồi
Định nghĩa 1. Cho hàm số y = f (x) liên tục [a; b] và có đồ thị là (C). Khi đó ta có hai điểm
A(a; f (a)), B(b; f (b)) nằm trên đồ thị (C).
i) Đồ thị (C) gọi là lồi trên (a; b) nếu tiếp tuyến tại mọi điểm nằm trên cung AB luôn nằm phía trên đồ thị (C).
ii) Đồ thị (C) gọi là lõm trên (a; b) nếu tiếp tuyến tại mọi điểm nằm trên cung AB luôn nằm phía dưới đồ thị (C). y y M B A x O A x O B Đồ thị hàm số lõm M Đồ thị hàm số lồi
Định lí 1 (Dấu hiệu hàm lồi, lõm). Cho hàm số y = f (x) có đạo hàm cấp hai liên tục trên (a; b). Khi đó
• Nếu f 00(x) > 0 ∀x ∈ (a; b) thì đồ thị hàm số lõm trên (a; b).
• Nếu f 00(x) < 0 ∀x ∈ (a; b) thì đồ thị hàm số lồi trên (a; b). 2.
Bất đẳng thức tiếp tuyến - Bất đẳng thức cát tuyến
Định lí 2 (Bất đẳng thức tiếp tuyến). Cho hàm số y = f (x) liên tục và có đạo hàm đến
cấp hai trên [a;b]. Khi đó ta có
• Nếu f 00(x) ≥ 0 ∀x ∈ [a; b] thì f (x) ≥ f 0(x0)(x − x0) + f (x0) ∀x0 ∈ [a; b]
• Nếu f 00(x) ≤ 0 ∀x ∈ [a; b] thì f (x) ≤ f 0(x0)(x − x0) + f (x0) ∀x0 ∈ [a; b]
Đẳng thức trong hai bất đẳng thức trên xảy ra khi x = x0.
Chứng minh: Ta chứng minh trường hợp thứ nhất, trường hợp thứ hai chứng minh tương tự.
Xét hàm số g(x) = f (x) − f 0(x0)(x − x0) − f (x0), x ∈ [a; b].
Ta có : g0(x) = f 0(x) − f 0(x0) ⇒ g00(x) = f 00(x) ≥ 0 ∀x ∈ [a; b].
Suy ra g0(x) = 0 ⇔ x = x0 và g0(x) đổi dấu từ − sang + khi x qua x0 nên ta có :
g(x) ≥ g(x0) = 0 ∀x ∈ [a; b].
Định lí 3 (Bất đẳng thức cát tuyến). Cho hàm số y = f (x) liên tục và có đạo hàm đến cấp hai trên [a; b]. Khi đó 58
2. PHƯƠNG PHÁP SỬ DỤNG TIẾP TUYẾN VÀ CÁT TUYẾN f (a) − f (b)
• Nếu f 00(x) ≥ 0 ∀x ∈ [a; b] thì f (x) ≥
(x − a) + f (a) ∀x0 ∈ [a; b] a − b f (a) − f (b)
• Nếu f 00(x) ≤ 0 ∀x ∈ [a; b] thì f (x) ≤
(x − a) + f (a) ∀x0 ∈ [a; b]. a − b
Đẳng thức trong các bất đẳng thức trên có khi và chỉ khi x = a hoặc x = b. II. Các ví dụ minh họa
Ví dụ 2.1. Cho các số thực dương a,b,c thỏa a + b + c = 1. Chứng minh rằng a b c 3 √ + √ + √ ≤ √ . a2 + 1 b2 + 1 c2 + 1 10 x Xét hàm số f (x) = √ với x ∈ (0; 1). Ta có: x2 + 1 1 3x f 0(x) = ⇒ f 00(x) = − < 0 ∀x ∈ (0; 1). q q (x2 + 1)3 (x2 + 1)5 Nên ta có: 1 1 1 f (a) ≤ f 0( )(a − ) + f ( ) 3 3 3 1 1 1 f (b) ≤ f 0( )(b − ) + f ( ) 3 3 3 1 1 1 f (c) ≤ f 0( )(c − ) + f ( ). 3 3 3 Suy ra : 1 1 3 f (a) + f (b) + f (c) ≤ f 0
(a + b + c − 1) + 3f ( ) = √ . 3 3 10 1
Đẳng thức xảy ra khi a = b = c = . 3
Ví dụ 2.2. Cho các số thực dương a, b, c thỏa : a2 + b2 + c2 = 3. Chứng minh rằng 1 1 1 √ + √ + √ ≥ 1. 1 + 8a 1 + 8b 1 + 8b 1 √ Xét hàm số : f (x) = √ , 0 < a ≤ 3. Ta có : 1 + 8a 4 48 1 √ f 0(x) = − ⇒ f 00(x) = > 0 ∀x ∈ (− ; 3]. q q 8 (1 + 8x)3 (1 + 8x)5 Nên ta có :
f (a) ≥ f 0(1)(a − 1) + f (1)
f (b) ≥ f 0(1)(b − 1) + f (1)
f (c) ≥ f 0(1)(c − 1) + f (1) 59
2. PHƯƠNG PHÁP SỬ DỤNG TIẾP TUYẾN VÀ CÁT TUYẾN Suy ra
f (a) + f (b) + f (c) ≥ f 0(1)(a + b + c − 3) + 3f (1). (∗) Mặt khác
(a + b + c)2 ≤ 3(a2 + b2 + c2) = 9 ⇒ −3 ≤ a + b + c ≤ 3 ⇒ a + b + c − 3 ≤ 0 4 và f 0(1) = −
< 0 nên từ (*) ta suy ra : f (a) + f (b) + f (c) ≥ 3f (1) = 1. 27
Nhận xét 1. Dấu hiệu giúp chúng ta nhận ra phương pháp trên là bất đẳng thức cần chứng minh có dạng
f (a1) + f (a2) + · · · + f (an) ≥ k hoặc
f (a1) + f (a2) + · · · + f (an) ≤ k,
trong đó ai (i = 1,..,n) là các số thực cho trước. Trong một số trường hợp BĐT chưa có dạng
trên, ta phải thực hiện một số phép biến đổi mới đưa về dạng trên.Chúng ta cần chú ý một số dấu hiệu sau.
• Nếu bất đẳng thức có dạng f (a1) · f (a2) · · · f (an) ≥ k thì ta lấy ln hai vế
• Nếu bất đẳng thức cần chứng minh đồng bậc thì ta có thể chuẩn hóa. Tùy thuộc vào từng
bài toán mà ta lựa chọn cách chuẩn hóa phù hợp.
Ví dụ 2.3. Cho các số thực dương a,b,c thỏa : a + b + c = 3. Tìm GTLN của biểu thức : √ √ √ b c a P = a + 1 + a2 b + 1 + b2 c + 1 + c2 . Ta có : √ √ √ ln P = b ln(a + 1 + a2) + c ln b + 1 + b2 + a ln c + 1 + c2 . √
Xét hàm số : f (x) = ln x +
1 + x2 , 0 < x < 1. Ta có : 1 −x f 0(x) = √ ⇒ f 00(x) = < 0, ∀x ∈ (0; 1). x2 + 1 q (1 + x2)3 Suy ra :
f (a) ≤ f 0(1) (a − 1) + f (1) = f 0(1)a + f (1) − f 0(1). Do đó
bf (a) ≤ f 0(1)ab + [f (1) − f 0(1)] b. Tương tự
cf (b) ≤ f 0(1)cb + [f (1) − f 0(1)] c và af (c) ≤ f 0(1)ac + [f (1) − f 0(1)] a.
Công các bất đẳng thức theo vế ta được √
ln P ≤ f 0(1) (ab + bc + ca − (a + b + c)) + f (1)(a + b + c) ≤ 3 ln(1 + 2).
(Do ab + bc + ca ≤ 3 = a + b + c) √ √ Suy ra ln P ≤ 3 ln(1 + 2) ⇒ P ≤ (1 +
2)3. Đẳng thức xảy ra ⇔ a = b = c = 1. √ Vậy GTLN của P = (1 + 2)3. 60
2. PHƯƠNG PHÁP SỬ DỤNG TIẾP TUYẾN VÀ CÁT TUYẾN
Ví dụ 2.4. Cho x,y > 0 thỏa x+y+z = 1. Tìm GTNN của biểu thức P = x−y +y−z +z−x.
Áp dụng bất đẳng thức AM-GM, ta có : 3 P ≥ √ . 3 xy.yz.zx
Đặt A = xy.yz.zx ⇒ ln A = y ln x + z ln y + x ln z. 1
Vì hàm số f (t) = ln t có f 00(t) = − < 0. Suy ra t2 1 1 1 ln x ≤ f 0 x − + f ( ) = 3x − 1 − ln 3. 3 3 3 Do đó
ln A ≤ y(3x − 1 − ln 3) + z(3y − 1 − ln 3) + x(3z − 1 − ln 3)
= 3(xy + yz + zx) − 1 − 3 ln 3
≤ (x + y + z)2 − 1 − 3 ln 3 = −3 ln 3. 1 √ 1 Suy ra A ≤
⇒ P ≥ 3 3 3. Đẳng thức xảy ra ⇔ x = y = z = . 3 √ 3 Vậy GTNN của P = 3 3 3. 1 Ví dụ 2.5. Cho a,b,c ≥
thỏa a + b + c = 2. Tìm GTNN của biểu thức 2 P = aa + bb + cc. 1 Xét hàm số f (t) = tt,
≤ t ≤ 1. Ta có : ln f (t) = t ln t lấy đạo hàm hai vế ta được 2
f 0(t) = (1 + ln t)f (t) ⇒ ln f 0(t) = ln f (t) + ln (ln t + 1) f 00(t) f 0(t) 1 1 ⇒ = + = 1 + ln t + f 0(t) f (t) t(ln t + 1) t(ln t + 1) 1 1
⇒ f 00(t) = (1 + ln t)f (t) 1 + ln t + > 0 ∀t ∈ [ ; 1]. t(1 + ln t) 2 1 Vì a,b,c ∈
; 1 nên áp dụng bất đẳng thức tiếp tuyến, ta có : 2 2 2 2 f (a) ≥ f 0( )(a − ) + f ( ) 3 3 3 2 2 2 f (b) ≥ f 0( )(b − ) + f ( ) 3 3 3 2 2 2 f (c) ≥ f 0( )(c − ) + f ( ). 3 3 3
Cộng ba bất đẳng thức trên ta có : r 2 2 4
f (a) + f (b) + f (c) ≥ f 0( ) (a + b + c − 2) + 3f ( ) = 3 3 . 3 3 9 r 4 2 Vậy GTNN của P = 3 3
đạt được ⇔ a = b = c = . 9 3 61
2. PHƯƠNG PHÁP SỬ DỤNG TIẾP TUYẾN VÀ CÁT TUYẾN 2π
Ví dụ 2.6. Cho tam giác ABC có một góc không nhỏ hơn . Chứng minh rằng : 3 A B C √ tan + tan + tan ≥ 4 − 3. 2 2 2 2π π
Không mất tính tổng quát, ta giả sử A ≥ > B ≥ C ⇒ C ≤ . 3 6 π
Hàm số f (x) = tan x, x ∈ 0; có 3 π f 00(x) > 0 ∀x ∈ 0; . 3
Áp dụng BĐT tiếp tuyến, ta có A π A π π f ( ) ≥ f 0( )( − ) + f ( ) 2 3 2 3 3 B π B π π f ( ) ≥ f 0( )( − ) + f ( ) 2 12 2 12 12 C π C π π f ( ) ≥ f 0( )( − ) + f ( ). 2 12 2 12 12 Suy ra A B C h π π i A 2π π A + B + C π f + f + f ≥ f 0( ) − f 0( ) − + f 0( ) − 2 2 2 3 12 2 3 12 2 2 π π + f + 2f . 3 12 π π A π A + B + C π Do f 0 − f 0 > 0; − ≥ 0 và = nên ta có : 3 12 2 3 2 2 A B C √ π π f + f + f ≥ f + 2f = 4 − 3. 2 2 2 3 12 2π π
Đẳng thức xảy ra ⇔ A = ; B = C = và các hoán vị. 3 6 3
Ví dụ 2.7. Cho các số thực không âm a,b,c thỏa max {a,b,c} ≥ và a + b + c = 1. Tìm 4 GTNN của biểu thức : √ √ √
P = 3 1 + 3a2 + 3 1 + 3b2 + 3 1 + 3c2. 3 1
Không mất tính tổng quát, ta giả sử a = max {a,b,c} ⇒ a ≥ ,c ≤ . √ 4 8
Xét hàm số f (x) = 3 1 + 3x2, x ∈ (0; 1) có 2x 2 − 2x2 f 0(x) = ⇒ f 00(x) = > 0 ∀x ∈ (0; 1). q q 3 (1 + 3x2)2 3 (1 + 3x2)5 62
2. PHƯƠNG PHÁP SỬ DỤNG TIẾP TUYẾN VÀ CÁT TUYẾN
Áp dụng bất đẳng thức tiếp tuyến, ta có : 3 3 3 f (a) ≥ f 0 a − + f 4 4 4 1 1 1 f (b) ≥ f 0 b − + f 8 8 8 1 1 1 f (c) ≥ f 0 c − + f 8 8 8
Cộng các bất đẳng thức trên ta có 3 1 3 3 1 f (a) + f (b) + f (c) ≥ f 0 − f 0 a − + f + 2f 4 8 4 4 8 √ √ 3 1 3 172 + 2 3 67 ≥ f + 2f = . 4 8 4 3 1
Đẳng thức xảy ra ⇔ a = ; b = c = và các hoán vị. √ √ 4 8 3 172 + 2 3 67 Vậy min P = . 4
Nhận xét 2. Trong một số trường hợp đồ thị hàm số y = f (x) có khoảng lồi, lõm trên [a; b]
nhưng ta vẫn có được đánh giá :
f (x) ≥ f 0(x0)(x − x0) + f (x0) ,x0 ∈ (a; b).
Ví dụ 2.8. Cho a,b,c ∈ R và a + b + c = 6. Chứng minh rằng :
a4 + b4 + c4 ≥ 2(a3 + b3 + c3).
Bất đẳng thức đã cho tương đương với
a4 − 2a3 + b4 − 2b3 + c4 − 2c3 ≥ 0 ⇔ f (a) + f (b) + f (c) ≥ 0.
Trong đó f (x) = x4 − 2x3 . Ta thấy f 00(x) = 12x2 − 12x nên đồ thị hàm số f có khoảng lồi
và khoảng lõm do đó ta không thể áp dụng BĐT tiếp tuyến được. Tuy nhiên ta vẫn có thể
đánh giá được f (x) qua tiếp tuyến của nó tại điểm có hoành độ x = 2 (vì đẳng thức xảy ra khi a = b = c = 2).
Ta có tiếp tuyến của đồ thị hàm số tại y = f (x) điểm có hoành độ x = 2 là: y = 8x − 16.
f (x) − (8x − 16) = x4 − 2x3 − 8x + 16 = (x − 2)2 x2 − 2x + 4 ≥ 0 ∀x ∈ R. Suy ra
f (a) + f (b) + f (c) ≥ 8 (a + b + c) − 48 = 0.
Vậy bài toán được chứng minh. 3
Ví dụ 2.9 (Ba Lan 1996). Cho a,b,c ≥ −
và a + b + c = 1. Chứng minh rằng: 4 a b c 9 + + ≤ . a2 + 1 b2 + 1 c2 + 1 10 63
2. PHƯƠNG PHÁP SỬ DỤNG TIẾP TUYẾN VÀ CÁT TUYẾN 1
Ta thấy đẳng thức xảy ra khi a = b = c =
và bất đẳng thức đã cho có dạng: f (a)+f (b)+f (c) ≤ 3 9 x 3 5 trong đó f (x) = với x ∈ [− ; ] . 10 x2 + 1 4 2 1 36x + 3
Tiếp tuyến của đồ thị hàm số y = f (x) tại điểm có hoành độ x = là : y = . 3 50 Ta có: 36x + 3 36x + 3 x (3x − 1)2(4x + 3) 3 5 − f (x) = − = ≥ 0 ∀x ∈ [− ; ]. 50 50 x2 + 1 50(x2 + 1) 4 2 Vậy a b c 36(a + b + c) + 9 9 + + ≤ = . a2 + 1 b2 + 1 c2 + 1 50 10
Bài toán được chứng minh.
Ví dụ 2.10. Cho a,b,c là độ dài ba cạnh tam giác. Chứng minh rằng : 1 1 1 9 1 1 1 + + + ≥ 4 + + . a b c a + b + c a + b b + c c + a
Không làm mất tính tổng quát ta giả sử , khi đó Bđt đã cho trở thành 5a − 1 5a − 1 5c − 1 + + ≤ 9. a − a2 b − b2 c − c2 1
Vì a,b,c là độ dài ba cạnh tam giác và a + b + c = 1 suy ra a,b,c ∈ (0; ). Ta có 2 5a − 1 (3a − 1)2(2a − 1) 1 − (18a − 3) = ≤ 0 ∀a ∈ (0; ). a − a2 a − a2 2 5a − 1 1 Suy ra ≤ 18a − 3 ∀a ∈ (0; ). a − a2 2
Ta cũng có hai bất đẳng thức tương tự. Cộng các bất đẳng thức này lại với nhau ta có 5a − 1 5a − 1 5c − 1 + + ≤ 18(a + b + c) − 9 = 9. a − a2 b − b2 c − c2 1
Bài toán được chứng minh. Đẳng thức xảy ra khia = b = c = . 3
Ví dụ 2.11 (Trung Quốc 2005). Cho a,b,c > 0 và a + b + c = 1. Chứng minh rằng:
10 a3 + b3 + c3) − 9(a5 + b5 + c5 ≥ 1. Giả sử a ≥ b ≥ c.
Xét hàm số f (x) = 10x3 − 9x4, x ∈ (0; 1) có
f 0(x) = 30x2 − 45x4 ⇒ f 00(x) = 60x − 180x3. r 1
Suy ra f 00(x) = 0 ⇔ x = x0 = đồng thời 3
f 00(x) > 0 ∀x ∈ (0; x0) và f 00(x) < 0 ∀x ∈ (x0; 1). 64
2. PHƯƠNG PHÁP SỬ DỤNG TIẾP TUYẾN VÀ CÁT TUYẾN
• Nếu a < x0. Áp dụng bất đẳng thức tiếp tuyến ,ta có: 1 1 1 f (a) ≥ f 0 a − + f 3 3 3 1 1 1 f (b) ≥ f 0 b − + f 3 3 3 1 1 1 f (c) ≥ f 0 c − + f . 3 3 3 Suy ra 1 1 f (a) + f (b) + f (c) ≥ f 0 (a + b + c − 1) + 3f = 1. 3 3
• Nếu a > x0. Áp dụng bất đẳng thức tiếp tuyến và cát tuyến ta có: f (1) − f (x f (a) ≥
0) (a − 1) + f (1) > f(1) = 1 1 − x0
f (b) ≥ f 0 (0) (b − 0) + f (0) = 0
f (c) ≥ f 0 (0) (c − 0) + f (0) = 0.
Do đó f (a) + f (b) + f (c) > 1.
Vậy bài toán được chứng minh.
Ví dụ 2.12. Cho ∆ABC nhọn. Tìm GTLN của biểu thức F = sin A · sin2 B · sin3 C. Ta có
ln F = ln sin A + 2 ln sin B + 3 ln sin C. π
Xét hàm số f (x) = ln sin x, x ∈ (0; ), ta có 2 1 π
f 0(x) = cot x ⇒ f 00(x) = − ∀x ∈ 0; . sin2x 2
Áp dụng bất đẳng thức tiếp tuyến với ∆M N P nhọn, ta có :
f (A) ≤ f 0(M ) (A − M ) + f (M ) = (A − M ) cot M + ln sin M
f (B) ≤ f 0(N ) (B − N ) + f (N ) = (B − N ) cot N + ln sin N
f (C) ≤ f 0(P ) (C − P ) + f (P ) = (C − P ) cot P + ln sin P. Suy ra
tan M · f (A) + tan N · f (B) + tan P · f (C) ≥ tan M · ln sin M + tan N · ln sin N + tan P · ln sin P.
Chọn ba góc M, N, P sao cho : tan M tan N tan P = =
= k ⇒ tan M = k; tan N = 2k; tan P = 3k. 1 2 3
Mặt khác : tan M + tan N + tan P = tan M. tan N. tan P , suy ra tan M 1 2 3
6k = 6k3 ⇒ k = 1 ⇒ sin M = √
= √ ; sin N = √ ; sin P = √ . 1 + tan2M 2 5 10 65
2. PHƯƠNG PHÁP SỬ DỤNG TIẾP TUYẾN VÀ CÁT TUYẾN Do đó, ta có 1 2 3 27
f (A) + f (B) + f (C) ≤ ln √ + 2 ln √ + 3 ln √ = ln √ , 2 5 10 25 5 27 hay F ≤
√ . Đẳng thức xảy ra ⇔ A = M ; B = N ; C = P . 25 5 27 Vậy GTLN của F = √ . 25 5 III. Bài tập
Bài 2.1 (Albania 2002). Cho a,b,c > 0. Chứng minh rằng : √ 1 + 3 1 1 1 √ √ (a2 + b2 + c2)( + + ) ≥ a + b + c + a2 + b2 + c2. 3 3 a b c π
Bài 2.2. Cho n số thực x1,x2, . . . ,xn thuộc khoảng (0; ) thỏa : 2
tan x1 + tan x2 + · · · + tan xn ≤ n. Chứng minh rằng 1
sin x1 · sin x2 · · · sin xn ≤ √ . 2n
Bài 2.3. Cho các số thực a,b,c > 0 thoả mãna + b + c = 1. Chứng minh : a b c 9 + + ≥ . 1 + bc 1 + ac 1 + ab 10
Bài 2.4. Cho a,b,c > 0. Chứng minh rằng : (b + c − a)2 (c + a − b)2 (a + b − c)2 3 + + ≥ . (b + c)2 + a2 (c + a)2 + b2 (a + b)2 + c2 5
Bài 2.5. Cho a, b, c > −1 và a + b + c = 1. Tìm giá trị nhỏ nhất của
S = a3 + b3 + c3 + 5(a2 + b2 + c2).
Bài 2.6. Cho a + b + c = −6 với a, b, c < −1. Tìm giá trị lớn nhất của a b c S = + + . a2 + a + 1 b2 + b + 1 c2 + c + 1 2 Bài 2.7. Cho a, b, c > −
và 4(ab + bc + ca) + a + b + c ≥ 15. Tính giá trị nhỏ nhất của biểu 3 thức
S = a3 + b3 + c3 + 2(a2b + b2c + c2a). 66
2. PHƯƠNG PHÁP SỬ DỤNG TIẾP TUYẾN VÀ CÁT TUYẾN 4 Bài 2.8. Cho a, b, c ∈ 0,
và −2(ab + bc + ca) + 3(a + b + c) = 3. Tìm giá trị nhỏ nhất 3
S = 2(a3 + b3 + c3) − 3(a2b + b2c + c2a).
Bài 2.9. Cho các số dương a, b, c thỏa mãn ab + bc + ca = 3. Tìm giá trị nhỏ nhất của
S = 5(a3 + b3 + c3) + 2(a2b + b2c + c2a). 1 √ Bài 2.10. Cho a, b, c ∈
, 2 và ab + bc + ca + 9 = 4(a + b + c). Tìm giá trị nhỏ nhất của 2 1 1 1 b a c T = + + − + + . a2 b2 c2 a c b
Bài 2.11. Cho các số dương a, b, c thỏa mãn 3(ab + bc + ca) − (a + b + c) ≥ 6. Tính giá trị nhỏ nhất của
A = a5 + b5 + c5 + 3(a4b + b4c + c4a).
Bài 2.12. Cho a, b, c ∈ [0,4] thỏa mãn ab + bc + ca ≥ 3. Tìm giá trị nhỏ nhất của √ √ √
B = (a2 + b2 + c2) + b 3a + 1 + c 3b + 1 + a 3c + 1.
Bài 2.13. Cho tam giác ABC nhọn. Tìm GTNN của biểu thức : F = tan A + 2 tan B + 3 tan C.
Bài 2.14. Cho x, y, z > 0 thỏa x + y + z = 1. Tìm GTNN của : √ p P = x3 + 1 + y2 + 4 1 + z4. 67 Chương 3 Một số chuyên đề
§1. Ứng dụng điều kiện có nghiệm của phương trình bậc
ba trong chứng minh bất đẳng thức I. Lý thuyết 1. Mở đầu
Định lí Vi-ét đảo đối với phương trình bậc hai được phát biểu như sau:
Định lí 1. Nếu hai số a,b có tổng là S và tích là P thì hai số đó là hai nghiệm của phương trình x2 − Sx + P = 0. (3.1)
Phương trình (3.1) có nghiệm khi và chỉ khi ∆ = S2 − 4P ≥ 0 hay S2 ≥ 4P .
Mà S = a + b,P = ab nên ta có điều kiện để tồn tại hai số a,b (tức là phương trình (3.1) có nghiệm) là : (a + b)2 ≥ 4ab.
Đây chính là bất đẳng thức quen thuộc. Nếu a, b ≥ 0 thì ta thu được bất đẳng thức AM-GM.
Tương tự đối với định lí Vi-ét đảo của phương trình bậc ba như sau:
Đặt m = a + b + c,n = ab + bc + ca,p = abc . Khi đó, a, b, c là nghiệm của phương trình x3 − mx2 + nx − p = 0. (3.2)
Ta đi tìm điều kiện để phương trình (3.2) có ba nghiệm (có thể trùng nhau). 2. Một số kết quả m m2 9mn − 2m3 − 27p Đặt: x = y + ; α = − n; β =
. Từ (3.2) ta thu được phương trình 3 3 27 y3 − αy + β = 0. (3.3)
Số nghiệm của (3.3) chính là số giao điểm của đồ thị (C) : f (y) = y3 − αy + β với trục hoành. Ta có: f 0(y) = 3y2 − α.
• Nếu α < 0 thì f 0(y) > 0, ∀y nên phương trình (3.3) có đúng 1 nghiệm.
• Nếu α = 0 thì phương trình (3.3) có nghiệm bội ba. 68
1. ỨNG DỤNG ĐIỀU KIỆN CÓ NGHIỆM CỦA PHƯƠNG TRÌNH BẬC BA TRONG
CHỨNG MINH BẤT ĐẲNG THỨC r α r α
• Nếu α > 0 thì f 0(y) = 0 có hai nghiệm y1 = − ; y2 = , khi đó 3 3 2α r α 2α r α f (y1) = + β, f (y2) = − + β. 3 3 3 3 Suy ra 4α3 27β2 − 4α3 f (y1) .f (y2) = β2 − = . 27 27
Do đó, phương trình (3.3) có ba nghiệm khi và chỉ khi:
f (y1) .f (y2) ≤ 0 ⇔ 4α3 − 27β2 ≥ 0. Hay là: q 9mn − 27p − 2m3 ≤ 2 (m2 − 3n)3.
Kết quả 1. Cho các số thực a, b, c. Đặt a + b + c = m, ab + bc + ca = n, abc = p. Khi đó, ta có đánh giá sau: q 9mn − 27p − 2m3 ≤ 2 (m2 − 3n)3. (3.4)
Với a, b, c ≥ 0, đặt a + b + c = 3u, ab + bc + ca = 3v2 và abc = w3. Vì q
(a + b + c)2 ≥ 3 (ab + bc + ca) ≥ 9 3 (abc)2 nên ta có u ≥ v ≥ w. Khi đó (3.4) trở thành q
9 · 3u · 3v2 − 27w3 − 2 · 27u3 ≤ 2 (9u2 − 9v2)3. Hay q 3uv2 − w3 − 2u3 ≤ 2 (u2 − v2)3. (3.5)
Chia hai vế của (3.5) cho u3 ta có s v 2 w 3 v 23 3 − − 2 ≤ 2 1 − . u u u Hay là s s v 23 v 2 w 3 v 23 −2 1 − ≤ 3 − − 2 ≤ 2 1 − . u u u u Suy ra s s v 2 v 23 w 3 v 2 v 23 3 − 2 1 − − 2 ≤ ≤ 3 + 2 1 − − 2. u u u u u
Kết quả 2: Cho các số thực dương a,b,c. Đặt a + b + c = 3u, ab + bc + ca = 3v2 và abc = w3 với
u,v,w là các số thực dương. Khi đó u ≥ v ≥ w và s s v 2 v 23 w 3 v 2 v 23 3 − 2 1 − − 2 ≤ ≤ 3 + 2 1 − − 2. (3.6) u u u u u 69
1. ỨNG DỤNG ĐIỀU KIỆN CÓ NGHIỆM CỦA PHƯƠNG TRÌNH BẬC BA TRONG
CHỨNG MINH BẤT ĐẲNG THỨC II. Ví dụ minh họa
Ví dụ 1.1. Cho các số thực a, b, c thỏa mãn a + b + c = 0. Chứng minh rằng
a2 + b2 + c23 ≥ 24 a3 + b3 + c3 − 1 .
Ta có m = 0 nên (3.4) trở thành q 27 3 2 |27p| ≤ 2 (−3n)3 ⇔ −n3 ≥ p2 = 3 p − 1 + 9p − 3 ≥ 9p − 3. 4 2 Hay là
− (ab + bc + ca)3 ≥ 9abc − 3.
Mặt khác a + b + c = 0 nên ta có a3 + b3 + c3 = 3abc và a2 + b2 + c2 − (ab + bc + ca) = . 2 Do vậy, ta có
a2 + b2 + c23 ≥ 24 a3 + b3 + c3 − 1 .
Vậy bài toán được chứng minh.
Ví dụ 1.2. Cho các số thực a, b, c có tổng bằng −1. Tìm giá trị nhỏ nhất của biểu thức P = 2abc + (ab + bc + ca)2 .
Ta có m = −1 nên áp dụng (3.4) ta có q 1 2 2 q |9n + 27p − 2| ≤ 2 (1 − 3n)3 ⇒ p ≥ − n + − (1 − 3n)3. 3 27 27 Do đó q q 27P ≥ −18n + 4 − 4
(1 − 3n)3 + 27n2 = 3 (3n − 1)2 − 4 (1 − 3n)3 + 1. √ Đặt t = 1 − 3n,t ≥ 0 ta có
27P ≥ 3t4 − 4t3 + 1 = (t − 1)2 3t2 − t + 1 ≥ 0.
Do đó P ≥ 0. Đẳng thức xảy ra khi a = −1, b = c = 0 và các hoán vị. Vậy min P = 0.
Ví dụ 1.3. Cho các số thực a,b,c thỏa mãn a2 + b2 + c2 = 3.Chứng minh rằng
3 (abc − 2) ≤ (a + b + c) (ab + bc + ca) ≤ 3 (abc + 2) . m2 − 3
Ta có a2 + b2 + c2 = 3 nên (a + b + c)2 = 2 (ab + bc + ca) + 3 hay n = . 2 Từ (3.4) ta suy ra  s   s  9 − m2 3 9 − m2 3 2 m3 − ≤ 9mn − 27p ≤ 2 m3 + .     2 2 70
1. ỨNG DỤNG ĐIỀU KIỆN CÓ NGHIỆM CỦA PHƯƠNG TRÌNH BẬC BA TRONG
CHỨNG MINH BẤT ĐẲNG THỨC Ta chứng minh s 9 − m2 3 9 − m2 3 m3 + ≤ 27 ⇔ ≤ 27 − m32 . 2 2
Khai triển và biến đổi ta được
(m − 3)2 m4 + 6m3 + 24m2 + 42m + 63 ≥ 0.
Bất đẳng thức này hiển nhiên đúng do
m4 + 6m3 + 24m2 + 42m + 63 = m2 + 3m + 72 + m2 + 14 > 0.
Chứng minh tương tự, ta có s 9 − m2 3 m3 − ≥ −27. 2
Do vậy ta có được bất đẳng thức
−54 ≤ 9mn − 27p ≤ 54 ⇔ 3p − 6 ≤ mn ≤ 3p + 6. Hay là
3 (abc − 2) ≤ (a + b + c) (ab + bc + ca) ≤ 3 (abc + 2) .
Bài toán được chứng minh.
Ví dụ 1.4. Cho các số thực a, b, c thỏa mãn abc = 1. Tìm giá trị lớn nhất của biểu thức (a + b + c)3 + (ab + bc + ca)3 P = .
(a + b + c)2(ab + bc + ca)2 + 27
Ta có p = 1 nên từ (3.4) ta được q 9mn − 27 − 2m3 ≤ 2 (m2 − 3n)3.
Bình phương hai vế và rút gọn ta thu được
(mn)2 + 18mn ≥ 4 m3 + n3 + 27.
Mặt khác 18mn ≤ (mn)2 + 81 nên ta có
2 (mn)2 + 81 ≥ 4 m3 + n3 + 27 ⇔ m2n2 + 27 ≥ 2 m3 + n3 . Do đó m3 + n3 1 P = ≤ . m2n2 + 27 2 (abc = 1 Đẳng thức xảy ra khi , chẳng hạn a = b = c = 1. (ab + bc + ca) (a + b + c) = 9 1 Vậy max P = . 2
Ví dụ 1.5. Cho các số thực a,b,c thỏa mãn ab + bc + ca = 3. Tìm giá trị lớn nhất của biểu thức abc(a + b + c)3 + 27 P = . (a + b + c + 3abc)2 71
1. ỨNG DỤNG ĐIỀU KIỆN CÓ NGHIỆM CỦA PHƯƠNG TRÌNH BẬC BA TRONG
CHỨNG MINH BẤT ĐẲNG THỨC
Ta có n = 3 nên từ (3.4), suy ra q 27m − 27p − 2m3 ≤ 2 (m2 − 9)3.
Bình phương hai vế và rút gọn ta thu được
27p2 + 4m3p + 108 ≤ 54mp + 9m2, hay
108p2 + 4m3p + 108 ≤ 9 (m + 3p)2 . Suy ra m3p + 27 9
4m3p + 108 ≤ 9 (m + 3p)2 ⇒ P = ≤ . (m + 3p) 4 abc = 0   √ √ Đẳng thức xảy ra khi
a + b + c = ±2 3 , chẳng hạn ta chọn a = 0,b = c = 3.  ab + bc + ca = 3 9 Vậy max P = . 4
Ví dụ 1.6. Cho các số thực a,b,c thoả
2 a2 + b2 + c2 = 5 (ab + bc + ca) . Chứng minh rằng: √
(a + b + c)2 + 27. 3 abc + 1 ≥ 0. Ta có 2 2 m2 − 2n = 5n ⇒ n = m2. 9 Khi đó (3.4) trở thành s r 2 3 4 p2 | 3 27p| ≤ 2 m2 − m2 ⇒ 272p2 ≤ m6 ⇒ m2 ≥ 27 . 3 27 4 p 2 Mặt khác + 1 ≥ 0 nên 2 p2 ≥ − (p + 1) , 4 suy ra m2 ≥ −27 3 pp + 1 hay √
(a + b + c)2 + 27 3 abc + 1 ≥ 0.
Bài toán được chứng minh.
Ví dụ 1.7. Cho các số thực a,b,c thoả a2 + b2 + c2 = ab + bc + ca + 1. Chứng minh rằng:
(a + b + c)2 ≤ 4 + 3 (ab + bc + ca)2 + 18abc. 72
1. ỨNG DỤNG ĐIỀU KIỆN CÓ NGHIỆM CỦA PHƯƠNG TRÌNH BẬC BA TRONG
CHỨNG MINH BẤT ĐẲNG THỨC
Ta có (a + b + c)2 = 3 (ab + bc + ca) + 1 nên m2 = 3n + 1. Khi đó (2) trở thành: m2 − 1 9m
− 27p − 2m3 ≤ 2 ⇒ 27p ≥ m3 − 3m − 2. 3
Đặt T = (a + b + c)2 − 3 (ab + bc + ca)2 − 18abc, ta cần chứng minh T ≤ 4. 3T = 3m2 − 9n2 − 54p
≤ 3m2 − m2 − 12 − 2 m3 − 3m − 2 = −m4 − 2m3 + 5m2 + 6m + 3
= − m2 + m − 32 + 12 ≤ 12.
Suy ra T ≤ 4. Bài toán được chứng minh.
Ví dụ 1.8 (Iran MO 2014, vòng 2). Cho các số thực không âm x,y,z thỏa mãn điều kiện:
x2 + y2 + z2 = 2(xy + yz + zx). x + y + z √ Chứng minh rằng: ≥ 3 2xyz. 3
Nếu x = y = z = 0 thì bất đẳng thức cần chứng minh hiển nhiên đúng. Ta xét x + y + z > 0.
Bất đẳng thức cần chứng minh tương đương với √ w 3 1 u ≥ 3 2.w ⇔ ≤ . u 2
Áp dụng (3.6) ta chỉ cần chứng minh s w 3 v 2 v 23 ≤ 3 + 2 1 − − 2. u u u
Mà x2 + y2 + z2 = 2 (xy + yz + zx) nên v 2 3 9u2 = 4.3v2 ⇒ = . u 4 Do đó s w 3 v 2 v 23 1 ≤ 3 + 2 1 − − 2 = . u u u 2
Bài toán được chứng minh.
Ví dụ 1.9. Cho các số thực không âm a,b,c. Chứng minh rằng a4 + b4 + c4 3abc 2 + ≥ a2 + b2 + c2 . ab + bc + ca a + b + c 3 Ta có
a4 + b4 + c4 = 81u4 − 108u2v2 + 18v4 + 12uw3, và a2 + b2 + c2 = 9u2 − 6v2. 73
1. ỨNG DỤNG ĐIỀU KIỆN CÓ NGHIỆM CỦA PHƯƠNG TRÌNH BẬC BA TRONG
CHỨNG MINH BẤT ĐẲNG THỨC
Nên bất đẳng thức cần chứng minh trở thành
81u4 − 108u2v2 + 18v4 + 12uw3 3w3 2 + ≥ 9u2 − 6v2 3v2 3u 3 27 − 36x + 6x2 + 4y ⇔ + y + 4x ≥ 6. (3.7) x v 2 w 3 Trong đó x = và y = . u u Theo (3.6), ta có q y ≥ 3x − 2 (1 − x)3 − 2 vàx ≤ 1 nên 27 4 27 V T (3.7) = − 36 + 10x + y 1 + ≥ − 36 + 10x + 5y x x x 27 q ≥ − 36 + 10x + 5 3x − 2 (1 − x)3 − 2 x 25x2 − 52x + 27 q = − 10 (1 − x)2 + 6 x q
≥ (1 − x) (27 − 25x) − 10 (1 − x)3 + 6 q
≥ (1 − x) (1 + 25(1 − x)) − 10 (1 − x)3 + 6 √ q
≥ (1 − x) 10 1 − x − 10 (1 − x)3 + 6 = 6.
Bài toán được chứng minh. Đẳng thức xảy ra khi a = b = c. III. Bài tập
Bài 1.1. Cho các số thực a,b,c không đồng thời bằng 0 thỏa a + b + c = 0. Tìm giá trị lớn nhất của biểu thức: 13a2b2c2 − 2abc − 2 P = . (a2 + b2 + c2)3
Bài 1.2. Cho các số thực a,b,c có tổng bằng 0. Tìm giá trị nhỏ nhất của biểu thức
P = a2 + b2 + c25 − 32 (ab + bc + ca) a2b2c2 − 8 |abc| .
Bài 1.3. Cho các số thực a,b,c thoả a2 + b2 + c2 = 2(ab + bc + ca). Tìm giá trị nhỏ nhất của biểu thức: 1 P = abc (a + b + c)3 + . (abc)4
Bài 1.4. Cho các số thực a,b,c thoả a2 + b2 + c2 = ab + bc + ca + 4. Tìm giá trị nhỏ nhất của biểu thức:
P = 18 (ab + bc + ca)2 − (ab + bc + ca) (a + b + c − 48) + 9abc.
Bài 1.5. Cho các số thực dương a,b,c thoả (a + b + c)3 = 32abc. Tìm giá trị lớn nhất, giá trị
nhỏ nhất của biểu thức: a4 + b4 + c4 P = . (a + b + c)4 74
2. BÀI TOÁN TÌM HẰNG SỐ TỐT NHẤT TRONG BẤT ĐẲNG THỨC
§2. Bài toán tìm hằng số tốt nhất trong bất đẳng thức I. Lý thuyết
Trong chuyên đề nyaf ta đi giải quyết bài toán:
Tìm hằng số k lớn nhất (nhỏ nhất) để một BĐT luôn đúng với một giả thiết nào đó của các biến.
Để giải dạng toán này, ta thường giải quyết theo hai hướng sau: Hướng 1:
• Bước 1: Chọn giá trị đặc biệt của các biến hoặc đánh giá trực tiếp các biến để chỉ ra điều kiện cần của k.
• Bước 2: Chứng minh bất đẳng thức đã cho đúng với giá trị của k ( lớn nhất, nhỏ nhất) vừa tìm được.
Hướng 2: Giả sử ta cần tìm k nhỏ nhất để bất đẳng thức f (a1,a2, . . . ,an) ≤ k
luôn đúng với mọi a1, a2, . . . , an ∈ D. Ta đi tìm giá trị lớn nhất M của f (a1,a2, . . . ,an) với
a1, a2, . . . , an ∈ D. Khi đó kmin = M. II. Ví dụ minh họa
Ví dụ 2.1. Tìm hằng số k lớn nhất sao cho bất đẳng thức sau luôn đúng q q q a + k|b − c|α + b + k|c − a|α + c + k|a − b|α ≤ 2,
với mọi α ≥ 1 và a,b,c là các số thực không âm thỏa mãn a + b + c = 1.
Cho a = b = 0,c = 1,α = 1. Ta có √ √ 1 1 + k + k ≤ 1 ⇔ 0 ≤ k ≤ . 4
Ta chứng minh bất đẳng thức sau đúng q q q 4a + |b − c|α + 4b + |c − a|α + 4c + |a − b|α ≤ 4. (3.8)
đúng với mọi α ≥ 1 và a,b,c ≥ 0 thỏa a + b + c = 1.
Không mất tính tổng quát ta giả sử a ≥ b ≥ c. Ta có
0 ≤ |a − b| , |b − c| , |c − a| ≤ 1, α ≥ 1 nên ta có p p
V T (3.8) ≤ p4a + |b − c| + 4b + |c − a| + 4c + |a − b| 1 √ √ p = 4(4a + b − c) + 4b + a − c + 4c + a − b 2 1 4 + 4a + b − c 1 + 4b + a − c 1 + 4c + a − b ≤ . + + 2 2 2 2 8a + 7b + 5c + 8 8 (a + b + c) + 8 = ≤ = 4. 4 4 1 Vậy kmax = . 4 75
2. BÀI TOÁN TÌM HẰNG SỐ TỐT NHẤT TRONG BẤT ĐẲNG THỨC
Ví dụ 2.2. Tìm số k nhỏ nhất sao cho bất đẳng thức k + 3 a3 + b3 + c3 + kabc ≤
a2 (b + c) + b2 (c + a) + c2 (a + b) 6
đúng với mọi a,b,c là độ dài ba cạnh của tam giác. 2 3 3 + 1 1 n Cho c = 1, a = b = + ta có: k ≥
→ 9. Ta chứng minh bất đẳng thức 2 n 2 1 + n
a3 + b3 + c3 + 9abc ≤ 2 a2 (b + c) + b2 (c + a) + c2 (a + b) .
Giả sử a = max {a,b,c}, ta có
a3 + b3 + c3 + 9abc − 2 a2 (b + c) + b2 (c + a) + c2 (a + b)
= (a − b − c) (a − b) (a − c) + (b − c)2 (b + c − 3a) ≤ 0. Vậy kmax = 9.
Ví dụ 2.3. Cho a, b, c > 0. Tìm hằng số k lớn nhất sao cho bất đẳng thức sau đúng a b c a2 + b2 + c2 + + − 3 ≥ k − 1 . b c a ab + bc + ca Cho a = 1, b = c3 6= 1, ta có
(c3 + c2 + 1) (c3 + 3c2 + 2c + 1) k ≤ → 1. c2 (c4 + 2c3 + 2c2 + c + 1)
Ta chứng minh bất đẳng thức a b c a2 + b2 + c2 + + − 3 ≥ − 1 , b c a ab + bc + ca hay a b c a2 + b2 + c2 + + ≥ + 2 b c a ab + bc + ca a b c ⇔ (ab + bc + ca) + + ≥ (a + b + c)2 . b c a
Bất đẳng thức cuối dễ dàng chứng minh được bằng cách áp dụng bất đẳng thức Cauchy – Schwarz.
Ví dụ 2.4. Tìm số thực dương k lớn nhất để bất đẳng thức sau đúng với mọi số thực
dương x,y,z thỏa mãn điều kiện xyz = 1 : x y z k 3 k + + + ≥ + √ . xy + 1 yz + 1 zx + 1 3 pxy2 + yz2 + zx2 2 3 3 76
2. BÀI TOÁN TÌM HẰNG SỐ TỐT NHẤT TRONG BẤT ĐẲNG THỨC
Do bất đẳng thức đã cho đúng với mọi x,y,z > 0 thỏa mãn xyz = 1 nên nó sẽ đúng khi 1 x = n; y = 1; z =
với mọi n > 0. Khi đó ta có n 2n 1 k 3 k + + ≥ + √ ; ∀n > 0. n + 1 2n r 1 2 3 3 3 2n + n2 Cho n → +∞ ta được √ 3 k 3 3 2 ≥ + √ ⇔ k ≤ . 2 3 3 2 √ 3 3 Ta sẽ chứng minh k =
thỏa mãn yêu cầu bài toán, tức là 2 2x 2y 2z r 3 + + + 3 ≥ 4. xy + 1 yz + 1 zx + 1 xy2 + yz2 + zx2 b c a
Do x, y, z > 0 thỏa mãn xyz = 1 nên tồn tại a,b,c > 0 thỏa mãn x = ; y = ; z = . Khi đó a b c
bất đẳng thức cần chứng minh trở thành r 2a 2b 2c 3abc + + + 3 ≥ 4. b + c c + a a + b a3 + b3 + c3 Ta có r 2a 2b 2c 3abc + + + 3 b + c c + a a + b a3 + b3 + c3 (a + b + c)2 9abc ≥ + ab + bc + ca q 3 3 (3abc)2(a3 + b3 + c3) (a + b + c)2 9abc ≥ + . ab + bc + ca a3 + b3 + c3 + 6abc Ta chỉ cần chứng minh (a + b + c)2 9abc − 3 ≥ 1 − ab + bc + ca a3 + b3 + c3 + 6abc 1 a + b + c ⇔ ≥ . ab + bc + ca a3 + b3 + c3 + 6abc √ 3 3
Bất đẳng thức này hiển nhiên đúng theo Schur. Vậy bất đẳng thức đã cho đúng khi k = . 2 1
Đẳng thức xảy ra khi x = y = z = 1 hoặc x → +∞; y = 1; z = . √ x 3 3
Tóm lại giá trị k tốt nhất cần tìm là k = . 2 √
Ví dụ 2.5 (VN TST 2012). Chứng minh rằng C = 10 24 là hằng số lớn nhất sao cho
nếu có 17 số thực dương a1,a2, . . . ,a17 thỏa các điều kiện (a2 + a2 + · · · + a2 = 24 1 2 17 . a3 + · · · + a3 + a 1 17 1 + · · · + a17 < C
thì với mọi 1 ≤ i ≤ j ≤ k ≤ 17 ta có ai, aj, ak là độ dài ba cạnh của một tam giác.
Trước hết ta có bổ đề sau 77
2. BÀI TOÁN TÌM HẰNG SỐ TỐT NHẤT TRONG BẤT ĐẲNG THỨC
Bổ đề 1. Cho số nguyên n ≥ 3. Giả sử n số dương a1,a2, . . . ,anthỏa mãn bất đẳng thức
(n − 1) a4 + a4 + ... + a4 < a2 + a2 + · · · + a2 2 . 1 2 n 1 2 n
Hãy chứng minh ba số bất kì ai, aj, ak (1 ≤ i < j < k ≤ n) là độ dài các cạnh của một tam giác. Chứng minh. • Với n = 3 thì ta có
2 a4 + a4 + a4 < a2 + a2 + a22 1 2 3 1 2 3
⇔ (a1 + a2 + a3)(a2 + a3 − a1)(a3 + a1 − a2)(a1 + a2 − a3) > 0,
suy ra a1,a2,a3 là độ dài 3 cạnh của một tam giác.
• Với n > 3, không mất tính tổng quát ta chứng minh a1, a2, a3 là độ dài ba cạnh của một tam giác. Ta có
(n − 1) a4 + a4 + · · · + a4 < a2 + a2 + · · · + a2 2 1 2 n 1 2 n √ a2 + a2 + a2 2 = 2 1 2 3 √ + a2 + · · · + a2 4 n 2   " 2 # (a2 + a2 + a2) ≤ 1 2 3 2 + 1 + · · · + 1 + a4 + · · · + a4 4 n | {z } 2 n−3 " 2 # (a2 + a2 + a2) = (n − 1) 1 2 3 + a4 + · · · + a4 . 2 4 n Suy ra
2 a4 + a4 + a4 < a2 + a2 + a22 , 1 2 3 1 2 3
do đó a1,a2,a3 là độ dài ba cạnh của một tam giác.
Bổ đề được chứng minh. Trở lại bài toán. ai Đặt xi = √
,i = 1,17, khi đó các số dương x1,x2 . . . , x17 thỏa 24 (x2 + x2 + · · · + x2 = 1 1 2 17 24 x3 + x3 + · · · + x3 + x 1 2 17 1 + · · · + x17 < 10
Để chứng minh ai, aj, ak là độ dài ba cạnh của một tam giác, ta chỉ cần chứng minh xi, xj, xk
là độ dài ba cạnh của một tam giác.
• Ta chứng minh bài toán đúng với C = 10.
Ta đi tìm số thực dương a thỏa : a 16x4 + (a − 1)x2 < 24x3 + x , ∀x ∈ (0; 1) . (1) 10
Vì nếu có bất đẳng thức (1) thì ta suy ra a 16x4 + (a − 1)x2 < 24x3 + x , ∀i = 1,17. i i i 10 i 78
2. BÀI TOÁN TÌM HẰNG SỐ TỐT NHẤT TRONG BẤT ĐẲNG THỨC Do đó 17 17 17 X X a X 16 x4 + (a − 1) x2 < 24x3 + x . i i i 10 i i=1 i=1 i=1 Hay 17 17 17 !2 X X X 16 x4 + (a − 1) < a ⇒ 16 x4 < 1 = x2 , i i i i=1 i=1 i=1
khi đó theo bổ đề ta có đpcm.
Ta tìm a để (1) đúng? Ta viết lại (1) như sau a 16x3 + (a − 1)x − 24x2 + 1 < 0. (2) 10
Vì x ∈ (0; 1) nên ta chọn a sao cho vế trái của (2) có thừa số x − 1 hay 5a 16 + a − 1 − = 0 ⇒ a = 10. 2 Khi đó (2) trở thành:
16x3 − 24x2 + 9x − 1 < 0 ⇔ (x − 1) 16x2 − 8x + 1 < 0. (3)
Rõ ràng (3) đúng với x ∈ (0; 1). Từ đó, ta có đpcm.
• Ta chứng minh C = 10 là số lớn nhất. Giả sử tồn tại số C0 > 10 sao cho với 17 số thực
dương x1, x2, . . . , x17 thỏa mãn (x2 + x2 + · · · + x2 = 1 1 2 17 24 x3 + x3 + · · · + x3 + x 1 2 17 1 + · · · + x17 < C 0
thì xi, xj, xk là độ dài ba cạnh của một tam giác với mọi 1 ≤ i < j < k ≤ 17. Ta xét 17 số thực dương r r 1 1 1 a − a2 1 x1 = , x2 =
− a, x3 = a, x4 = · · · = x17 = + , 0 < a < . 4 16 16 14 16 Ta có x2 + x2 + · · · + x2 = 1, 1 2 17 và s r 1 1 2 1 x1 − x3 = − a = − a <
− a = x2 ⇒ x1 > x2 + x3, 4 4 16
hay x1, x2, x3 không là độ dài ba cạnh của một tam giác. Ta có 17 17 X X S(a) = 24 x3 + x i i i=1 i=1  s s  1 1 3 1 a − a2 3 = 24 −  + a + a3 + 14 +  43 16 16 14 r r 1 1 1 a − a2 + + − a + a + 14 + 4 16 16 14  s s  r r 1 1 3 1 3 1 1 1 → 24 + + 14 + + + 14 = 10   khi a → 0+. 43 16 16 4 16 16 79
2. BÀI TOÁN TÌM HẰNG SỐ TỐT NHẤT TRONG BẤT ĐẲNG THỨC 1
Do tính liên tục của S, nên tồn tại a0 ∈ 0;
sao cho S(a0) < C0 nhưng bộ 16 r r r ! 1 1 1 a 1 a , − a 0 − a2 0 0 − a2 0 0, a0, + , . . . , + 4 16 16 14 16 14
thỏa mãn các điều kiện của bài toán, nhưng bộ (x1,x2,x3) không tạo thành 3 cạnh của một tam giác.
Vậy bài toán được chứng minh.
Ví dụ 2.6. Tìm số thực k lớn nhất sao cho bất đẳng thức sau luôn đúng với mọi a,b là
các số thực dương phân biệt thỏa mãn ab = 1 a2 + b2 − 2 ≥ k. p2(a + b) − 2
Bài toán chuyển về tìm GTNN của biểu thức a2 + b2 − 2 P = , p2(a + b) − 2 với a,b > 0 và ab = 1.
Đặt t = p2 (a + b) > 2, ta có t4 − 16 (t + 2) (t2 + 4) P = = > 8 4 (t − 2) 4 khi t > 2. 1
Giả sử tồn tại k > 8 thỏa bài toán. Cho t = 2 + , suy ra n " # 1 1 1 2 k ≤ 4 + 2 + + 4 → 8 (vô lí). 4 n n Vậy kmax = 8.
Ví dụ 2.7. Tìm hằng số dương k lớn nhất sao cho bất đẳng thức
a2 − b2 b2 − c2 c2 − a2 ≤ k (a + b + c)6
đúng với mọi a,b,c ≥ 0.
Giả sử a ≥ b ≥ c, ta có
a2 − b2 b2 − c2 c2 − a2 = a2 − b2 b2 − c2 a2 − c2 q ≤ a2 − b2 a2b2 = (a + b) (a − b)2.ab.ab.ab.ab v u !5 u (a − b)2 + 4ab ≤ (a + b) t 5 (a + b)6 1 = √ ≤ √ (a + b + c)6 . 25 5 25 5 1 Từ đó ta có kmax = √ . 25 5 80
2. BÀI TOÁN TÌM HẰNG SỐ TỐT NHẤT TRONG BẤT ĐẲNG THỨC
Ví dụ 2.8 (IMO 2006). Tìm hằng số M nhỏ nhất sao cho với mọi số thực a,b,c ta đều có ab
a2 − b2 + bc b2 − c2 + ca c2 − a2 ≤ M a2 + b2 + c22 .
Bằng biến đổi đơn giản, ta có ab
a2 − b2 + bc b2 − c2 + ca c2 − a2 = |(b − c) (a − b) (a − c) (a + b + c)|
Bài toán trở thành: Tìm M nhỏ nhất để
|(b − c) (a − b) (a − c) (a + b + c)| ≤ M a2 + b2 + c2 . (1)
Giả sử a = max {a,b,c} , ta có
9 a2 + b2 + c22 = (a + b + c)2 + 2 (b − c)2 + 2 (a − b) (a − c)2 √ h i2
≥ 2 2 |(a + b + c) (b − c)| + 2 |(a − b) (a − c)| √
≥ 16 2 |(b − c) (a − b) (a − c) (a + b + c)| . Do đó, ta có √ 9 2
|(b − c) (a − b) (a − c) (a + b + c)| ≤ a2 + b2 + c22 . 32 √ √ 3 2 3 2
Có thể chọn b = 1 thì a = 1 + ; c = 1 − để đẳng thức xảy ra. √ 2 2 9 2 Vậy min M = . 32
Ví dụ 2.9 (Tổng quát IMO 2004). Với mỗi số nguyên dương n ≥ 3, tìm hằng số dương
k = k(n) lớn nhất sao cho nếu n số thực dương t1, t2, . . . , tn thỏa mãn 1 1 1 (t1 + t2 + · · · + tn) + + · · · + < k t1 t2 tn
thì ti, tj, tk là độ dài ba cạnh của một tam giác với mọi i, j, k thỏa mãn 1 ≤ i < j < k ≤ n.
Giả sử tồn tại ba số chẳng hạn t1, t2, t3 thỏa mãn t1 + t2 ≤ t3. Khi đó 1 1 1 t t t 1 1 (t 1 2 1 + t2 1 + t2 + t3) + + = 3 + + + + t3 + t1 t2 t3 t2 t1 t3 t1 t2 t 4t ≥ 3 + 2 + 1 + t2 + 3 t3 t1 + t2 s t t 4 ≥ 5 + 5 5 1 + t2 3 ≥ 10. t3 t1 + t2 81
2. BÀI TOÁN TÌM HẰNG SỐ TỐT NHẤT TRONG BẤT ĐẲNG THỨC Xét n ≥ 4, ta có n ! n ! X X 1 1 1 1 ti = (t1 + t2 + t3) + + + ti t1 t2 t3 i=1 i=1 n ! ! n n X 1 1 1 1 X X 1 + (t1 + t2 + t3) + + + ti + ti ti t1 t2 t3 ti i=4 i=4 i=4 s 1 1 1 ≥ 10 + 2(n − 3) (t1 + t2 + t3) + + + (n − 3)2 t1 t2 t3 √ √ 2
≥ 10 + 2 10(n − 3) + (n − 3)2 = n + 10 − 3 . √ 2 10t1
Đẳng thức xảy ra khi t1 = t2, t3 = 2t1, ti = , ∀i = 4, . . . ,n. 5 √ Từ đó suy ra kmax = n + 10 − 32. Nhận xét 1.
a) Nội dung bài toán IMO 2004 như sau:
Cho số nguyên n ≥ 3. Giả sử t1,t2, . . . ,tn là các số thực dương sao cho 1 1 1 (t1 + t2 + · · · + tn) + + · · · + < n2 + 1. t1 t2 tn
Chứng minh rằng ti, tj, tk là độ dài ba cạnh của một tam giác với mọi 1 ≤ i < j < k ≤ n.
b) Ta có thể tổng quát bài toán trên theo hướng khác như sau:
Cho các số nguyên dương n, k với n ≥ k ≥ 3 và giả sử n số thực dương t1, t2, . . . , tn thỏa mãn 1 1 1 √ 2 (t1 + t2 + · · · + tn) + + · · · + < n + 2k2 − 4k + 4 − k . t1 t2 tn
Chứng minh rằng mỗi số trong k số ti , t , . . . ,t
đều nhỏ hơn tổng của k − 1 số còn lại 1 i2 ik
với 1 ≤ i1 < · · · < ik III. Bài tập
Bài 2.1. Tìm giá trị lớn nhất của k để bất đẳng thức sau đúng với mọi giá trị a, b, c:
a4 + b4 + c4 + abc (a + b + c) ≥ k (ab + bc + ca)2 .
Bài 2.2. Tìm số thực k lớn nhất sao cho với mọi số thực a, b, c không âm và thỏa mãn điều
kiện a + b + c = 1, ta luôn có a b c 1 + + ≥ . 1 + 9bc + k(b − c)2 1 + 9ca + k(c − a)2 1 + 9ab + k(a − b)2 2
Bài 2.3 (Turkey National Olympiad Second Round 2013). Tìm số thực M lớn nhất sao
cho với mọi a, b, c dương, ta luôn có
a3 + b3 + c3 − 3abc ≥ M ab2 + bc2 + ca2 − 3abc . 82
2. BÀI TOÁN TÌM HẰNG SỐ TỐT NHẤT TRONG BẤT ĐẲNG THỨC
Bài 2.4. Cho các số dương x, y, z. Tìm số k lớn nhất sao cho: x y z x + y + z + + + 3k ≥ (k + 1) · √ . y z x 3 xyz
Bài 2.5. Cho các số thực không âm a, b, c thỏa mãn ab + bc + ca = a + b + c. Tìm số thực k lớn
nhất sao cho ta luôn có bất đẳng thức: 1 1 1 (a + b + c) + + ≥ k · (a + b + c + 1) . a + b b + c c + a
Bài 2.6 (Phạm Kim Hùng). Tìm số thực dương k lớn nhất sao cho bất đẳng thức sau đúng
với mọi số dương a, b, c: 1 1 1 (ab + bc + ca) (a + b + c) + + + k · ≥ 9 + k. a b c a2 + b2 + c2
Bài 2.7 (Kiểm tra Trường Hè Lê Qúy Đôn năm 2014). Tìm hằng số k nhỏ nhất sao cho
với mọi số thực dương a, b, c thỏa mãn a + b + c = 3 thì ta luôn có bất đẳng thức:
k a4 + b4 + c4 − 3 ≥ a3 + b3 + c3 + 3abc − 6.
Bài 2.8. Với các số không âm a, b, c thỏa mãn a + b + c = ab + bc + ca. Tìm số thực k lớn nhất sao cho
a + b + c + k · abc ≥ 3 + k.
Bài 2.9. Cho a, b, c là các số thực dương. Tìm hằng số k lớn nhất sao cho bất đẳng thức sau đúng: a3 + b3 + c3 ab + bc + ca 3 k + k · ≥ + . (a + b) (b + c) (c + a) (a + b + c)2 8 3
Bài 2.10 (Vietnamese Team Selection Test 2009). Tìm tất cả các hằng số k sao cho bất
đẳng thức sau đúng với mọi số dương a, b, c : a b c 1 3 + k + k + k ≥ k + . b + c c + a a + b 2
Bài 2.11 (VN TST – 2013). Tìm số nguyên dương k lớn nhất để bất đẳng thức sau đúng với
mọi số thực dương a, b, c thỏa mãn điều kiện abc = 1: 1 1 1 k k + + + ≥ 3 + . a b c a + b + c + 1 4
Bài 2.12. Xét các số dương a, b, c thỏa mãn abc = 1. Tìm số k lớn nhất sao cho 1 1 1 a2 + b2 + c2 + 3k ≥ (k + 1) + + . a b c 83
2. BÀI TOÁN TÌM HẰNG SỐ TỐT NHẤT TRONG BẤT ĐẲNG THỨC
Bài 2.13. Xét các số dương a, b, c thỏa mãn a + b + c = 3. Tìm k lớn nhất sao cho 1 1 1 + +
− 3 ≥ k a2 + b2 + c2 − 3 . a b c
Bài 2.14 (Tạp chí Pi - P47 – 7/2017). Tìm số thực k bé nhất sao cho với mọi bộ ba số thực
a, b, c mà abc ≥ 0, ta luôn có
abc + k · (a − b)2 + (b − c)2 + (c − a)2 + 2 ≥ a + b + c.
Bài 2.15 (Tạp chí Pi - P68 – 9/2017). Tìm số thực k nhỏ nhất sao cho với mọi số thực
dương x, y, z mà min {xy,yz,zx} ≥ 1, ta luôn có x + y + z 2 3
p(x2 + 1) (y2 + 1) (z2 + 1) ≤ + k. 3
Bài 2.16 (Tạp chí Pi - P61 – 9/2017). Tìm số thực k nhỏ nhất sao cho với mọi số thực a,
b, c là độ dài ba cạnh của một tam giác, ta luôn có a b c ab + bc + ca 3 + + + k · ≤ + k. b + c c + a a + b a2 + b2 + c2 2
Bài 2.17 (P5 – 2/2017). Tìm số dương k nhỏ nhất sao cho với mọi số thực dương x, y, z thỏa √ √ √ mãn điều kiện xy + yz + zx = 1, ta luôn có 1 1 1 9k + 3 + + ≤ . x + y + k y + z + k z + x + k 2 1
Bài 2.18 (BMO 2012). Tìm số thực k lớn nhất để đối với mọi a,b,c khác 0 thỏa mãn + a 1 1 +
≤ 3, ta đều có bất đẳng thức b c
a2 + 4 b2 + c2 b2 + 4 a2 + c2 c2 + 4 a2 + b2 ≥ k.
Bài 2.19. Cho a1,a2, . . . ,a5 là các số thực bất kì có tổng bằng 0. Tìm hằng số c = c(n) lớn nhất
sao cho bất đẳng thức sau đúng: n X X c |ai| ≤ |ai − aj| . i=1
1≤iBài 2.20. Tìm hằng số M lớn nhất sao cho với mỗi n nguyên dương tồn tại các số a1,a2,...,an > 0
và b1,b2,...,bn > 0 đồng thời thỏa mãn các điều kiện; i) Pn b k=1
k = 1; 2bk ≥ bk−1 + bk+1, ∀k = 2,...,n; ii) a2 ≤ 1 + Pk a k i=1 ibi, k = 1,n; iii) an = M. 84
2. BÀI TOÁN TÌM HẰNG SỐ TỐT NHẤT TRONG BẤT ĐẲNG THỨC
ĐÁP SỐ VÀ HƯỚNG DẪN GIẢI 85 Chương 1
Các bất đẳng thức cổ điển
§1. Bất đẳng thức AM-GM Câu 1.1.
a) Bất đẳng thức cần chứng minh tương đương với s s 1.1.1 abc 3 + 3 ≤ 1. (1 + a) (1 + b) (1 + c) (1 + a) (1 + b) (1 + c) Đặt : s s 1.1.1 abc T = 3 + 3 (1 + a) (1 + b) (1 + c) (1 + a) (1 + b) (1 + c) 1 1 1 1 1 a b c T ≤ + + + + + 3 1 + a 1 + b 1 + c 3 1 + a 1 + b 1 + c 1 a + 1 b + 1 c + 1 1 T ≤ + + = .3 = 1 3 1 + a 1 + b 1 + c 3
Dấu đẳng thức xảy ra khi a = b = c ≥ 0. b) Ta có a b c 1 + 1 + 1 + b c a a b c a c b = 2 + + + + + + b c a c b a a + b b + c c + a = + 1 + + 1 + + 1 − 1 c a b 1 1 1 = (a + b + c) + + − 1 a b c 3(a + b + c) 2(a + b + c) ≥ √ − 1 ≥ √ + 2. 3 abc 3 abc
Câu 1.2. Bất đẳng thức cần chứng minh tương đương với 1 r a + 1a2 · · · an n ≤ 1. (1) n p(1 + a (1 + a 1)(1 + a2) · · · (1 + an) 1)(1 + a2) · · · (1 + an)
Áp dụng bất đẳng thức AM-GM ta có 1 1 1 X 1 1 X a V T (1) ≤ + i = 1. n 1 + ai n 1 + ai i=1 i=1 86 1. BẤT ĐẲNG THỨC AM-GM
Bài toán được chứng minh. a + b + c 3 1 Câu 1.3. Ta có abc ≤ = . 3 27 Khi đó 1 1 1 1 3 1 + 1 + 1 + ≥ 1 + √ ≥ 64. a b c 3 abc
Suy ra (1 + a) (1 + b) (1 + c) ≥ 64abc.
Câu 1.4. Bất đẳng thức cần chứng minh tương đương với s r a1a2 · · · an b1b2 · · · bn n + n ≤ 1. (1)
(a1 + b1)(a2 + b2) · · · (an + bn)
(a1 + b1)(a2 + b2) · · · (an + bn)
Áp dụng bất đẳng thức AM-GM ta có r a 1a2 · · · an 1 a1 an n ≤ + · · · +
(a1 + b1)(a2 + b2) · · · (an + bn) n a1 + b1 an + bn s b 1 b b n 1b2 · · · bn ≤ 1 + · · · + n
(a1 + b1)(a2 + b2) · · · (an + bn) n a1 + b1 an + bn
Cộng hai bất đẳng thức trên theo vế ta có đpcm. n Câu 1.5. Vì α P
i là các số hữu tỉ dương và
αi = 1 nên tồn tại các số nguyên dương N,k1,k2, · · · , kn i=1 ki sao cho αi =
. Áp dụng bất đẳng thức AM-GM cho N số, ta có N
a1 + a1 + · · · + a1 + · · · + an + an + · · · + an k1 kn n | {z } | {z } X k k α 1 số n số n n i · ai = ≥ a · · · an = aα1 · · · aαn. N 1 1 n i=1
Bất đẳng thức được chứng minh.
Câu 1.6. Chuẩn hóa a1 + a2 + · · · + an = n, ta cần chứng minh ak + ak + · · · ak ≥ n. (1) 1 2 n
Áp dụng bất đẳng thức AM − GM cho k số gồm k − 1 số 1 và ak ta có i n n n X X X ak + k − 1 ≥ ka ak + n(k − 1) ≥ k a ak ≥ n. i i ⇒ i i = kn ⇒ i i=1 i=1 i=1
Vậy (1) đúng, hay bài toán được chứng minh. 1 1 4
Câu 1.7. Áp dụng bất đẳng thức + ≥ ta có x y x + y 1 1 4 2 + ≥ = a + 3b b + 2c + a (a + 3b) + (b + 2c + a) a + 2b + c 1 1 2 + ≥ b + 3c 2a + b + c a + b + 2c 1 1 2 + ≥ . c + 3a a + 2b + c 2a + b + c
Cộng các bất đẳng thức trên theo vế ta có đpcm. 87 1. BẤT ĐẲNG THỨC AM-GM
Câu 1.8. Áp dụng bất đẳng thức Cô si cho hai số ta có √ √ √ √ √ √ 4 1 1 2 4 a + 4 b ≥ 2 8 ab ⇒ 4 a + 4 b ≥ 16 ab và + ≥ √ . a b ab Suy ra √ √ √ 4 1 1 2 4 a + 4 b + ≥ 16 ab. √ = 32. a b ab Dẫn tới 1 1 1 1 ≤ + . √ √ 4 32 a b 4 a + 4 b Tương tự: 1 1 1 1 1 1 1 1 ≤ + , √ √ ≤ + . √ √ 4 4 4 32 b c 32 c a b + 4 c ( 4 c + 4 a)
Cộng các bất đẳng thức trên ta có 1 1 1 1 1 1 1 + + √ √ ≤ + + . √ √ 4 √ √ 4 4 16 a b c 4 a + 4 b 4 b + 4 c ( 4 c + 4 a)
Mặt khác, theo giả thiết ta có ab + bc + ca ≤ 3abc nên suy ra 1 1 1 + + ≤ 3. a b c Suy ra 1 1 1 3 + + √ √ ≤ (đpcm). √ √ 4 √ √ 4 4 16 4 a + 4 b 4 b + 4 c ( 4 c + 4 a)
Câu 1.9. Bất đẳng thức cần chứng minh tương đương với 2a 2b 2c 2a 2b 2c + + ≥ 4 − − − b + 2c c + 2a a + 2b b + 2a c + 2b a + 2c 1 1 1 1 1 1 ⇔ a + + b + + c + ≥ 2. b + 2c b + 2a c + 2a c + 2b a + 2b a + 2c 1 1 4
Áp dụng bất đẳng thức + ≥ ta có x y x + y 1 1 4 2 + ≥ = . b + 2c b + 2a 2a + 2b + 2c a + b + c Suy ra 1 1 2a a + ≥ . b + 2c b + 2a a + b + c Tương tự: 1 1 2b 1 1 2c b + ≥ , c + ≥ . c + 2a c + 2b a + b + c a + 2b a + 2c a + b + c
Cộng các bất đẳng thức trên ta có 1 1 1 1 1 1 a + + b + + c + ≥ 2 (đpcm). b + 2c b + 2a c + 2a c + 2b a + 2b a + 2c 88 1. BẤT ĐẲNG THỨC AM-GM
Câu 1.10. Áp dụng AM – GM, ta có 2 2 (1 + x + 1 − x + x2) (2 + x2)
1 + x3 = (1 + x) 1 − x + x2 ≤ = . 4 4 Tương tự 1 2 1 2 ≥ ; √ ≥ . p1 + y3 2 + y2 1 + z3 2 + z2 Vậy 1 1 1 2 2 2 P = √ + + √ ≥ + + . 1 + x3 p1 + y3 1 + z3 2 + x2 2 + y2 2 + z2
Áp dụng Cauchy – Swarzt, ta được: 18 P ≥ ≥ 1. x2 + y2 + z2 + 6
Dấu ‘=’ xảy ra khi x = y = z = 2. Vậy GTNN của biểu thức là P = 1. Câu 1.11. Ta có: a a (1 + b2) − ab2 ab2 ab2 ab = = a − ≥ a − = a − . 1 + b2 1 + b2 1 + b2 2b 2 Do đó: a b c 1 + + ≥ a + b + c − (ab + bc + ca) . 1 + b2 1 + c2 1 + a2 2 Mà: 1 ab + bc + ca ≤ (a + b + c)2 = 3. 3 Nên suy ra: a b c 3 3 + + ≥ 3 − = . 1 + b2 1 + c2 1 + a2 2 2 Câu 1.12. Ta có: a2 a (a + 2b2) − 2ab2 2ab2 2ab2 1 √ = = a − ≥ a − √ = a − √ b. a. a + 2b2 a + 2b2 a + b2 + b2 2 2ab 2 Suy ra: a2 b2 c2 1 √ √ √ + + ≥ a + b + c − √ a.b + b.c + c.a . a + 2b2 b + 2c2 c + 2a2 2 Mặt khác: 1 3 ab + bc + ca ≤ (a + b + c)2 = . 3 4 Và √ √ √ √ √ √ 3 ab. b + bc. c +
ca. a ≤ p(ab + bc + ca) (a + b + c) ≤ √ . 2 2 Vậy: a2 b2 c2 3 3 3 + + ≥ − = . a + 2b2 b + 2c2 c + 2a2 2 4 4 89 1. BẤT ĐẲNG THỨC AM-GM
Câu 1.13. ax2 + ay2 ≥ 2axy. Đẳng thức xảy ra khi x = y. √
by2 + cz2 ≥ 2 bcyz.Đẳng thức xảy ra khi by2 = cz2. √
cz2 + bx2 ≥ 2 cbzx. Đẳng thức xảy ra khi cz2 = bx2 .
Bây giờ ta chọn a, b, c sao cho :  a + b = 3 a = 1      2c = 1 ⇔ b = 2 √   1 a = bc  c = 2
Suy ra: x2 + y2 ≥ 2xy. Đẳng thức xảy ra khi x = y. . 1 1 2y2 +
z2 ≥ 2yz.Đẳng thức xảy ra khi 2y2 = z2.. 2 2 1 1
z2 + 2x2 ≥ 2zx. Đẳng thức xảy ra khi z2 = 2x2.. 2 2
Cộng vế theo vế ta được :
3x2 + 3y2 + z2 ≥ 2 (xy + yz + zx) ⇒ 3x2 + 3y2 + z2 ≥ 10 (đpcm). x = y     1  ( 2y2 = z2  x = y = 1 Đẳng thức xảy ra khi : 2 ⇔ . 1 z = 2   z2 = 2x2   2   xy + yz + zx = 5
Câu 1.14. Áp dụng bất đẳng thức Cô si cho 3 số thực dương ta có s a3 a + 2b b + 2c a3 a + 2b b + 2c 1 + + ≥ 3 3 . . = a. (a + 2b) (b + 2c) 27 27 (a + 2b) (b + 2c) 27 27 3 Tương tự: b3 b + 2c c + 2a 1 + + ≥ b, (b + 2c) (c + 2a) 27 27 3 và c3 c + 2a a + 2b 1 + + ≥ c. (c + 2a) (a + 2b) 27 27 3
Cộng ba bất đẳng thức trên ta có a3 b3 c3 2(a + b + c) a + b + c + + + ≥ (a + 2b) (b + 2c) (b + 2c) (c + 2a) (c + 2a) (a + 2b) 9 3 Suy ra a3 b3 c3 a + b + c + + ≥ . (a + 2b) (b + 2c) (b + 2c) (c + 2a) (c + 2a) (a + 2b) 9
Đẳng thức xảy ra khi a = b = c.
Câu 1.15. Ta thấy đẳng thức xảy ra khi a = b = c = 1. a4 1 Khi đó =
,b = 1,c + 2 = 3 nên áp dụng bất đẳng thức Cô si cho 4 số ta được b2 (c + 2) 3 s a4 b b c + 2 a4 b b c + 2 4 + + + ≥ 4 4 + . . = a b2 (c + 2) 3 3 9 b2 (c + 2) 3 3 9 3 90 1. BẤT ĐẲNG THỨC AM-GM Tương tự: b4 2b a + 2 4 c4 2a b + 2 4 + + ≥ b, + + ≥ c. c2 (a + 2) 3 9 3 a2 (b + 2) 3 9 3
Cộng các bất đẳng thức trên ta có a4 b4 c4 7(a + b + c) + 6 4 (a + b + c) + + + ≥ . b2 (c + 2) c2 (a + 2) a2 (b + 2) 9 3 Hay a4 b4 c4 5(a + b + c) − 6 + + ≥ . b2 (c + 2) c2 (a + 2) a2 (b + 2) 9 √
Mà a + b + c ≥ 3 3 abc = 3 nên ta có a4 b4 c4 + + ≥ 1 (đpcm). b2 (c + 2) c2 (a + 2) a2 (b + 2) √ √
Câu 1.16. Áp dụng bất đẳng thức x + y ≤ p2 (x + y), ta có : √ √ √ √ r r √ ! ! √ ! a + b b + c r c + a 1 a b 1 b c 1 c b + + ≥ √ √ + √ + √ √ + √ + √ √ + √ c a b 2 c c 2 a a 2 a a √ √ √ a 1 1 b 1 1 c 1 1 = √ √ + √ + √ √ + √ + √ √ + √ . 2 c b 2 a c 2 a b 1 1 4
Áp dụng bất đẳng thức + ≥ , ta có : x y x + y √ √ √ √ √ √ a 1 1 b 1 1 c 1 1 2 2a 2 2b 2 2c √ √ + √ + √ √ + √ + √ √ + √ ≥ √ √ + √ √ + √ √ . 2 c b 2 a c 2 a b b + c a + c a + b √ √
Áp dụng bất đẳng thức x + y ≤ p2 (x + y), ta có : √ √ √ √ √ √ 2 2a 2 2b 2 2c 2 2a 2 2b 2 2c √ √ + √ √ + √ √ ≥ + + b + c a + c a + b p2 (b + c) p2 (a + c) p2 (a + b) ! r r c b r a = 2 + + . a + b a + c b + c
Câu 1.17. Áp dụng bất đẳng thức AM-GM ta có s a4 b + 2 a4 b + 2 2 + ≥ 2 . = a2. b + 2 9 b + 2 9 3 Tương tự: b4 c + 2 2 c4 a + 2 2 + ≥ b2, + ≥ c2. c + 2 9 3 a + 2 9 3
Cộng ba bất đẳng thức trên ta có a4 b4 c4 a + b + c + 6 2 + + + ≥ a2 + b2 + c2 = 2. b + 2 c + 2 a + 2 9 3 Suy ra a4 b4 c4 12 − (a + b + c) + + ≥ . b + 2 c + 2 a + 2 9 91 1. BẤT ĐẲNG THỨC AM-GM Mặt khác:
a + b + c ≤ p3 (a2 + b2 + c2) = 3 nên suy ra a4 b4 c4 12 − 3 + + ≥ = 1 (đpcm). b + 2 c + 2 a + 2 9
Đẳng thức xảy ra khi a = b = c = 1.
Câu 1.18. Bất đẳng thức cần chứng minh tương đương với 4 4 4 + 1 + 1 + 1 ≥ 3 (a + b + c)2 . 3 − a2 3 − b2 3 − c2
Từ đề bài, ta suy ra a2,b2,c2 < 3 .
Áp dụng bất đẳng thức Cô si ta có: 4 r 4 + 3 − a2 ≥ 2 (3 − a2) = 4, 3 − a2 3 − a2 suy ra 4 + 1 ≥ a2 + 2. 3 − a2 Tương tự: 4 4 + 1 ≥ b2 + 2, + 1 ≥ c2 + 2. 3 − b2 3 − c2 Do đó 4 4 4 + 1 + 1 + 1 ≥ a2 + 2 b2 + 2 c2 + 2 . 3 − a2 3 − b2 3 − c2 Mặt khác: " # a + b √ 2 (a + b)2 3 (a + b + c)2 = 3 √ . 2 + 1.c ≤ 3 + 1 c2 + 2 . 2 2 Ta chứng minh " # (a + b)2 3 + 1 ≤ a2 + 2 b2 + 2 . (∗) 2
Khai triển và rút gọn, bất đẳng thức (*) trở thành 2a2b2 + a2 + b2 + 2 ≥ 6ab.
Bất đẳng thức này hiển nhiên đúng vì:
2 a2b2 + 1 ≥ 4ab, a2 + b2 ≥ 2ab.
Vậy bài toán được chứng minh.
Câu 1.19. Ta có: b + c ≤ p2(b2 + c2) Suy ra a2 a2 + b2 + c2 1 √ ≥ − √ b2 + c2. b + c p2(b2 + c2) 2 Tương tự: b2 a2 + b2 + c2 1 √ c2 a2 + b2 + c2 1 √ ≥ − √ c2 + a2; ≥ − √ a2 + b2. c + a p2(c2 + a2) 2 a + b p2(a2 + b2) 2 92 1. BẤT ĐẲNG THỨC AM-GM Suy ra a2 + b2 + c2 1 1 1 1 √ √ √ V T ≥ √ √ + √ + √ − √ a2 + b2 + b2 + c2 + c2 + a2 . 2 a2 + b2 b2 + c2 c2 + a2 2 Ta có: 1 1 1 9 √ + √ + √ ≥ √ √ √ a2 + b2 b2 + c2 c2 + a2 a2 + b2 + b2 + c2 + c2 + a2 và 1 √ √ √ 2 a2 + b2 + c2 ≥ a2 + b2 + b2 + c2 + c2 + a2 . 6 Suy ra 1 √ √ √ V T ≥ √ a2 + b2 + b2 + c2 + c2 + a2 . 2 2 Đặt √ √ √ √ √ √ √ √ √ t = a2 + b2 + b2 + c2 + c2 + a2 ≥ 2 ab + bc + ca ≥ 3 2 3 abc. 1 Suy ra
√ t3 ≥ abc nên từ giả thiết ta suy ra 54 2 √ √ √ 7 − abc 7 1 t = a2 + b2 + b2 + c2 + c2 + a2 = √ ≥ √ − t3 2 2 108 √ ⇔ t3 + 108t − 378 2 ≥ 0 √ √ √ ⇔ t − 3 2
t2 + 3 2t + 126 ≥ 0 ⇔ t ≥ 3 2. √ 3 2 3 Suy ra V T ≥ √ = . 2 2 2 a = b = c (  a = b = c
Đẳng thức xảy ra khi và chỉ khi √ 7 − a3 ⇔ ⇔ a = b = c = 1. 3 2a = √ a3 + 6a − 7 = 0  2
Câu 1.20. Nhận thấy đẳng thức xảy ra khi a = b = c = 1 và 3 = 1 + 2 nên ta có đánh giá
a2 + 2b2 + 3 = a2 + b2 + b2 + 1 + 2 ≥ 2ab + 2b + 2. Do đó: 1 1 1 ≤ . . a2 + 2b2 + 3 2 ab + b + 1 Suy ra: 1 1 1 1 1 1 1 + + ≤ + + a2 + 2b2 + 3 b2 + 2c2 + 3 c2 + 2a2 + 3 2 ab + b + 1 bc + c + 1 ca + a + 1 Vậy ta cần chứng minh: 1 1 1 + + ≤ 1. ab + b + 1 bc + c + 1 ca + a + 1 1
Bất đẳng thức này hiển nhiên đúng vì đó là đẳng thức. Để chứng minh ta thay c = vào vế ab
trái và biến đổi ta có đpcm. Câu 1.21. Ta có s 3 X 1 1 1 1 1 3 1 3 √ X 1 X X ≤ 4 √ = √ · ≤ √ + = √ . a3 + b p2 a3b 2 a b 4 2 a b 2 93 1. BẤT ĐẲNG THỨC AM-GM Câu 1.22. Ta có
2ab + a2 + b2 + c2 + ab + bc + ca (a + b)2 + (c + a) (c + b) ab + 1 ≥ = . 2 2 Suy ra 1 (a + c)(b + c) (a + b)(a + c) (b + a)(b + c) V T ≥ 3 + + + ≥ 3. 2 (a + b)2 (b + c)2 (c + a)2
Câu 1.23. Áp dụng bất đẳng thức AM-GM ta có s b + c b + c 3 (b + c)2 a + b + c = a + + ≥ 3 a , 2 2 4 suy ra s 2a 2 3a 3 ≥ . b + c a + b + c
Chứng minh tương tự, ta cũng có s s 2b 3 3b 2c 2 3c 3 ≥ và 3 ≥ . c + a a + b + c a + b a + b + c
Cộng ba bất đẳng thức trên theo vế ta có đpcm.
Câu 1.24. Bài toán này có thể chứng minh bằng cách sử dụng đánh giá sau: r 3 a3 + b3 a2 + b2 ≤ . 2 a + b a2 + b2 2ab Chú ý rằng: = a + b − a + b a + b
Như vậy ta phải chứng minh: ab bc ca 2 + + + a + b + c ≥ 6. a + b b + c c + a
Áp dụng bất đẳng thức AM-GM với abc = 1,ta có ngay: 2ab a + b 2bc b + c 2ca c + a + + + + + ≥ 6. a + b 2 b + c 2 c + a 2
Vậy ta có điều phải chứng minh.Đẳng thức xảy ra khi a = b = c = 1. Câu 1.25. Ta có 13a2b2c2 − 2abc − 2 27a2b2c2 − (abc + 2)2 27a2b2c2 = ≤ . (a2 + b2 + c2)3 2(a2 + b2 + c2)3 2(a2 + b2 + c2)3 Ta chứng minh: 27a2b2c2 1 a2b2c2 1 ≤ ⇔ ≤ . 2(a2 + b2 + c2)3 4 (a2 + b2 + c2)3 54
Vì a + b + c = 0 nên trong ba số a,b,c có hai số cùng dấu, ta giả sử hai số đó là a,b. Khi đó ab ≥ 0 nên " #3 (a + b)2 (a + b)2
a2 + b2 + c23 = a2 + b2 + (a + b)23 = a2 + b2 + + 2 2 27 27 ≥ a2 + b2 (a + b)2 (a + b)2 ≥ .2ab.4ab.c2 = 54a2b2c2. 4 4 Suy ra a2b2c2 1 ≤ (đpcm). (a2 + b2 + c2)3 54 94 1. BẤT ĐẲNG THỨC AM-GM
Câu 1.26. Không mất tính tổng quát, ta giả sử c = min {a,b,c}. √
Nếu a ≥ b suy ra 6 3 (a − b) (b − c) (c − a) ≤ 0 nên bất đẳng thức cần chứng minh luôn đúng. Nếu a ≤ b, ta có: √ √
6 3 (a − b) (b − c) (c − a) = 6 3 (b − a) (b − c) (a − c) √ √ q ≤ 6 3 (b − a) ba = 3 3. (a − b)22ab.2ab v √ u !3 u (a − b)2 + 2ab + 2ab ≤ 3 3t 3 q q = (a + b)3 ≤ (a + b + c)3 (đpcm).  (c = 0 c = 0 
Đẳng thức xảy ra khi và chỉ khi ⇔ √ . (a − b)2 = 2ab a = 2 ± 2 b 
Câu 1.27. Trước hết ta có bất đẳng thức √ 1 1 2 2 + ≥ . x y px2 + y2
Không mất tính tổng quát, ta giả sử a > b > c. Khi đó 2 2 2 5 P = + + + √ a − b b − c a − c ab + bc + ca 8 2 5 ≥ + + √ a − b + b − c a − c ab + bc + ca 10 10 = + √ a − c 2 ab + bc + ca √ 20 2 ≥ q (a − c)2 + 4(ab + bc + ca) √ √ 20 2 20 2 = = . q p (a + c)2 + 4b (a + c) (1 − b) (1 + 3b) Mặt khác 1 1 3 − 3b + 1 + 3b 2 4 (1 − b) (1 + 3b) = (3 − 3b) (1 + 3b) ≤ = . 3 3 2 3  1 √ b =  √
Do đó P ≥ 10 6. Đẳng thức xảy ra khi 3 . Vậy min P = 10 6. 2  a = ,c = 0 3 Câu 1.28. Ta có 2 X 1 X 1 1 X X a X 1 a + ≥ a + b + = ab + + + 3. b b c c ab 1 a Áp dụng P = P a và P ab + P ≥ 2 P a. ta có đpcm. ab c Cách 2: Ta có r r r 1 b 1 b 3 a2 3 a2 · abc a2 + + ≥ 3 3 a2 · · = 3 = 3 = 3a. b2 c b2 c bc bc 95 1. BẤT ĐẲNG THỨC AM-GM Tương tự 1 c b2 + + ≥ 3b, c2 a và 1 a c2 + + ≥ 3c. a2 b Kết hợp với r a b c a b c + + ≥ 3 3 · · = 3. b c a b c a ta có đpcm. 2 5 3 Câu 1.29. Ta có b + 2a + ≥ + a nên b + 1 2 2 2 2 5 3 5 3 a + 2b + b + 2a + ≥ + a + b a + 1 b + 1 2 2 2 2 và 5 3 5 3 25 15 9 + a + b = + (a + b) + ab ≥ 16 2 2 2 2 4 4 4 nên ta có đpcm. Câu 1.30. Ta có p
2a + b + c = a + b + a + c ≥ 2 (a + b) (a + c). Suy ra 1 1 1 ≤ . . (2a + b + c)2 4 (a + b) (a + c) Do đó 1 a + b + c V T ≤ . . 2 (a + b) (b + c) (c + a) Mặt khác ta có:
9 (a + b) (b + c) (c + a) ≥ 8(a + b + c)(ab + bc + ca), và
(ab + bc + ca)2 ≥ 3abc (a + b + c) .
Từ giả thiết, ta suy ra ab + bc + ca = abc (a + b + c) nên ab + bc + ca ≥ 3. Suy ra a + b + c (a + b + c)(ab + bc + ca) 1 9 1 3 = ≤ . = . (a + b)(b + c)(c + a) (a + b)(b + c)(c + a) ab + bc + ca 8 3 8 Từ đó, suy ra đpcm.
Câu 1.31. Áp dụng bất đẳng thức Bunhia ta có: √ √ 2 a2 + b22 = a a · a + b b · b
≤ a3 + b3 (a + b) ≤ a3 + b3 p2 (a2 + b2) q √ q ⇒ (a2 + b2)3 ≤
2 a3 + b3 ⇒ a2 + b2 ≤ 3 2 (a3 + b3)2 c3 1 c3 1 a3 + b3 + c3 1 √ ⇒ ≥ √ = √ − √ 3 a3 + b3. (1) a2 + b2 3 2 q 3 q 3 3 (a3 + b3)2 2 3 (a3 + b3)2 2 96 1. BẤT ĐẲNG THỨC AM-GM
Ta cũng có hai bất đẳng thức tương tự b3 1 a3 + b3 + c3 1 √ ≥ √ − √ 3 c3 + a3, (2) c2 + a2 3 2 3 p(c3 + a3)2 3 2 a3 1 a3 + b3 + c3 1 √ ≥ √ − √ 3 b3 + c3. (3) b2 + c2 3 2 3 p(b3 + c3)2 3 2
Cộng ba bất đẳng thức (1), (2) và (3), ta được   1 1 1 1 P ≥ √ a3 + b3 + c3  + +  3 2 q q q 3 (a3 + b3)2 3 (a3 + b3)2 3 (a3 + b3)2 1 √ √ √ − √
3 a3 + b3 + 3 b3 + c3 + 3 c3 + a3 . 3 2 √ √ √
Đặt x = 3 a3 + b3, y = 3 b3 + c3, z = 3 c3 + a3. Suy ra: 1 1 1 1 P ≥ √ x3 + y3 + z3 + + − (x + y + z) . 3 2 x2 y2 z2
Mặt khác áp dụng bất đẳng thức AM-GM ta có được: 1 1 1 x2 + y2 + z2 + + ≥ 9 x2 y2 z2 1 1 1 9 ⇒ + + ≥ x2 y2 z2 x2 + y2 + z2 1 1 1 x3 + y3 + z3 ⇒ x3 + y3 + z3 + + ≥ 9 x2 y2 z2 x2 + y2 + z2 1 x3 + y3 + z3 ⇒ P ≥ √ 9 − (x + y + z) . 3 2 x2 + y2 + z2
Áp dụng bất đẳng thức Cauchy-Schwarz ta có:
(x + y + z)2 ≤ 3 x2 + y2 + z2 ⇒ (x + y + z)2 x2 + y2 + z2 ≤ 3 x2 + y2 + z22 mà √ √ √ √ √ √
x2 + y2 + z22 = x x. x + y y. y + z z. z2 ≤ (x + y + z) x3 + y3 + z3
⇒ (x + y + z)2 x2 + y2 + z2 ≤ 3 (x + y + z) x3 + y3 + z3 x3 + y3 + z3 1 ⇒ ≥ (x + y + z). x2 + y2 + z2 3 Do đó √ √ √ √ √ P ≥ 3 4 (x + y + z) = 3 4
3 a3 + b3 + 3 b3 + c3 + 3 c3 + a3 Vì √ √ √ √ √ √ √ √ √
t = 3 a3 + b3 + 3 b3 + c3 + 3 c3 + a3 ≥ 3 2 ab + bc + ca ≥ 3 3 2 3 abc 1 Suy ra abc ≤ t3. 54
Nên từ giả thiết ta suy ra: 1 3 ≤ t + t3 ⇔ t3 + 54t − 162 ≥ 0. (∗) 54 97 1. BẤT ĐẲNG THỨC AM-GM
Vì hàm số f (t) = t3 + 54t − 162 là hàm đồng biến và f (2)f (3) < 0 nên f (t) có nghiệm duy nhất x = m ∈ (2; 3). √ (a = b = c
Suy ra (∗) ⇔ t ≥ m ⇒ P ≥ 6 32.m. Đẳng thức xảy ra ⇔ √ ⇔ a = b = c = α. a3 + 3 3 2a − 3 = 0 √
Trong đó α là nghiệm duy nhất của phương trình X3 + 3 3 2X − 3 = 0 và α > 0. √ Vậy min P = 6 32.m (đpcm).
Câu 1.32. Ta chứng minh a + bc ≤ 2 (1). b2 + c2 Do bc ≤
< 2 nên 1 tương đương a2 ≤ (2 − bc)2 . Do 2
a2 ≤ 3 − (b2 + c2) và (2 − bc)2 − (3 − b2 − c2) = (bc − 1)2 + (b − c)2 ≥ 0 nên (1) đúng.
Ta chứng minh p(4 − a2)(4 − c2) ≥ ac + 2b (2). 3 a2 + b2 + c2 2 Ta có abc ≤ ≤ 1 nên 3
(4 − a2)(4 − c2) − (ac + 2b)2 = 16 − 4(a2 + b2 + c2 + abc) ≥ 0. Suy ra (2) đúng.
Ta chứng minh a2 + b2 + c2 ≥ ab + b2c + c2a (3). 1 Ta có a2 +
(ab + c2)2 ≥ a(ab + c2) = a2b + c2a (4) và 4 1 1 abc2 p
(4 − a2)b2 + (4 − c2)c2 ≥ bc (4 − a2)(4 − b2) ≥ bc2 + (5) 4 2 2
Cộng (4) và (5) theo vế suy ra (3) được chứng minh.
Quay trở lại bài toán. Từ (1) ta có a2b + ab2c ≤ 2ab, b2c + abc2 ≤ 2bc, c2a + a2bc ≤ 2ca. Suy ra
2(ab + bc + ca) ≥ a2b + b2c + c2a + abc(a + b + c)
⇔(a + b + c)2 ≥ a2 + b2 + c2 + abc(a + b + c) + a2b + b2c + c2a. (6) Từ (3) và (6) ta có
(a + b + c)2 ≥ abc(a + b + c) + 2(a2b + b2c + c2a)
⇔(a + b + c)(a + b + c − abc) ≥ 2(a2b + b2c + c2a).
Câu 1.33. Không mất tính tổng quát, giả sử a = max{a; b; c}.
Khi đó b + c + 1 ≤ c + a + 1 và b + c + 1 ≤ a + b + 1 b b c c Suy ra ≤ ; ≤
. Dấu bằng xảy ra khi a = b = c c + a + 1 b + c + 1 a + b + 1 b + c + 1
Mặt khác theo BĐT AM-GM ta có
1 − b + 1 − c + b + c + 1 3
(1 − b) (1 − c) (b + c + 1) ≤ = 1 3 Suy ra 1 − a
(1 − a) (1 − b) (1 − c) ≤ b + c + 1 Do đó a b c 1 − a P ≤ + + + = 1 b + c + 1 b + c + 1 b + c + 1 b + c + 1
Dấu = xảy ra khi a = b = c = 1. 98 1. BẤT ĐẲNG THỨC AM-GM Câu 1.34. √ √ √
• Cách 1: Theo BĐT AM-GM ta có: a + b ≥ 2 ab; b + c ≥ 2 bc; c + a ≥ 2 ca. 3a + 2b + c 3a + 2b + c 1 1 2 3 Ta có: P = ≤ √ √ √ = + + . (a + b)(b + c)(c + a) 2 ab.2 bc.2 ca 8 ab ac bc 3 4 5 3 4 5
Lại có: 3bc + 4ac + 5ab ≤ 6abc ⇔ + + ≤ 6. Đặt S = + + . a b c a b c Ta có 9 16 25 24 30 40 1 1 1 1 24 30 40 S2 = + + + + + = 9 + + 16 + + + + a2 b2 c2 ab ac bc a2 c2 b2 c2 ab ac bc 18 32 24 30 40 24 48 72 1 2 3 ⇒ S2 ≥ + + + + = + + = 24 + + ac bc ab ac bc ab ac bc ab ac bc 1 2 3 S2 3 ⇒ + + ≤ ≤ ab ac bc 24 2 1 3 3 ⇒ P ≤ . = 8 2 16
Dấu bằng xảy ra khi a = b = c = 2. 3
Vậy giá trị lớn nhất của P bằng khi a = b = c = 2. 6 3 4 5 3 4 5
• Cách 2: 3bc + 4ac + 5ab ≤ 6abc ⇔ + + ≤ 6. Đặt S = + + . a b c a b c Ta có:9 16 25 24 30 40 1 1 1 1 24 30 40 S2 = + + + + + = 9 + + 16 + + + + a2 b2 c2 ab ac bc a2 c2 b2 c2 ab ac bc 18 32 24 30 40 24 48 72 1 2 3 ⇒ S2 ≥ + + + + = + + = 24 + + . ac bc ab ac bc ab ac bc ab ac bc 1 4 1 4 1 4 ≥ ; ≥ ; ≥ ab (a + b)2 bc (b + c)2 ac a + c)2 4 8 12 1 2 3 ⇒ S2 ≥ 24 + + = 96 + + (a + b)2 (a + c)2 (c + b)2 (a + b)2 (a + c)2 (c + b)2 1 1 1 1 ⇒ S2 ≥ 96 + + 2 + (a + b)2 (b + c)2 (a + c)2 (b + c)2 2 4 ⇒ S2 ≥ 96 + (a + b)(b + c) (a + c)(b + c) 1 2 3a + 2b + c S2 ≥ 192 + = 192 (a + b)(b + c) (a + c)(b + c) (a + b)(b + c)(c + a) S2 36 3 P ≤ = = 192 192 16
Dấu bằng xảy ra khi a = b = c = 2. 3
Vậy giá trị lớn nhất của P bằng khi a = b = c = 2. 6
Câu 1.35. Biến đổi tương đương ta chứng minh được: a+b+c ≤ p3 (a2 + b2 + c2), ∀a, b,c > 0.
Dấu ” = ” xảy ra khi và chỉ khi a = b = c. Ta có v u ! u 1 1 1 V T ≤ t3. + + √ px3 + 2y3 + 6 py3 + 2z3 + 6 z3 + 2x3 + 6 v u s u 1 1 1 ≤ t3. 3. + + x3 + 2y3 + 6 y3 + 2z3 + 6 z3 + 2x3 + 6 99 1. BẤT ĐẲNG THỨC AM-GM
Theo bất đẳng thức TTBC-TBN ta có:
x3 + 2y3 + 6 = (x3 + y3 + 1) + (y3 + 1 + 1) + 3 ≥ 3xy + 3y + 3 = 3(xy + y + 1) Tương tự: y3 + 2z3 + 6 ≥ 3(yz + z + 1) z3 + 2x3 + 6 ≥ 3(xz + x + 1) Do đó 1 1 1 V T 4 ≤ 9 + + xy + y + 1 yz + z + 1 zx + x + 1 1 xy 1 = 9 + + xy + y + 1 xy(yz + z + 1) zx + x + 1 = 9. (do xyz = 1) √ Hay V T ≤
3. Dấu ” = ” xảy ra khi và chỉ khi x = y = z = 1. √ 2 3 + 1 1 1 1 1 1 y2 1 z z z Câu 1.36. Ta có P + = + + + x3 + + + + + + 3 3x 3x 3x 3 y2 3 z3 9 9 9
Áp dụng Bất đẳng thức AM-GM, ta có: s 1 1 1 1 1 3 x3 4 + + + x3 > 4 4 . = . (1) 3x 3x 3x 3 3x 3 3 √ 1 y2 2 3 + > . (2) y2 3 3 √ 1 z z z 4 3 + + + > (3) z3 9 9 9 9 √ √ √ √ 2 3 + 1 4 2 3 4 3 4 3 + 9
Từ (1), (2) và (3) ta có P + > + + hay P > 3 3 3 9 9
Dấu đẳng thức xảy ra khi và chỉ khi:  1 x3   =   3x 3     x = 1  1 y2    =  √ y2 3 ⇔ y = 4 3 √  1 z     z = 3  =   z3 9  √   x3 + y2 + z = 2 3 + 1. √ 4 3 + 9 √ √ Vậy min P = khi (x,y,z) = (1, 4 3, 3) 9
Câu 1.37. Ta có: a6b6 + b6c6 + c6a6 = a5b5.ab + b5c5.bc + c5a5.ca a5 + b5 + 1 + 1 + 1 b5 + c5 + 1 + 1 + 1 c5 + a5 + 1 + 1 + 1 ≤ a5b5 + b5c5 + c5a5 5 5 5 1 =
(a5b5(a5 + b5 + 3) + b5c5(b5 + c5 + 3) + c5a5(c5 + a5 + 3)) 5
Đặt x = a5,y = b5,z = c5. Ta có: x + y + z = 3;với x,y,z > 0.
Cần chứng minh: xy(x + y + 3) + yz(y + z + 3) + zx(z + x + 3) ≤ 15
hay xy(x + y) + yz(y + z) + zx(z + x) + 3(xy + yz + zx) ≤ 15
⇔ (xy + yz + zx)(x + y + z + 3) − 3xyz ≤ 15
⇔ 2(xy + yz + zx) − xyz ≤ 5 (1)
Mặt khác; xyz ≥ (3 − 2x)(3 − 2y)(3 − 2z) ⇒ 9xyz ≥ 12(xy + yz + zx) − 27 4 ⇒ xyz ≥ (xy + yz + zx) − 3. 3 4 2
Ta có: VT(1) ≤ 2(xy + yz + zx) − (xy + yz + zx) + 3 = (xy + yz + zx) + 3 ≤ 5. 3 3
Vậy bài toán được chứng minh. 100 1. BẤT ĐẲNG THỨC AM-GM 1 1
Câu 1.38. Với k = 1 thì bộ ; ; 2 không thỏa mãn. 2 2 4 4 7 Với k = 2 thì bộ ; ; không thỏa mãn. 5 5 5
Ta chứng minh với k = 3 thì bất đẳng thức đúng hay x3y3z3(x3 + y3 + z3) ≤ 3.
Không mất tính tổng quá ta có thể giả sử z nhỏ nhất suy ra z ≤ 1. Ta có x3 + y3 = (x + y)3 −
3xy(x + y) = (3 − z)3 − 3xy(x + y). Khi đó bất đẳng thức cần chứng minh tương đương 3 1 (3 − z)3 + z3 ≤
+ 3xy(x + y) ⇔ 3z2 − 9z + 9 ≤ + x2y + xy2 x3y3z3 x3y3z3 s 1 x3y3 3 Ta có + x2y + xy2 ≥ 3 3 =
, như vậy chỉ cần chứng minh x3y3z3 x3y3z3 z 3 3z2 − 9z + 9 ≤ ⇔ 3(z − 1)2 ≤ 0 z
Câu 1.39. Theo bất đẳng thức AM-GM, ta có: r r p a + c a + c a + b + 2(a + c) = (a + b) + + 2 2 v u r !2 r u a + c (a + b)(a + c) ≥ 3 3 3 t(a + b). = 3 . 2 2 Suy ra 1 2 ≤ . 3 27(a + b)(a + c) a + b + p2(a + c)
Tương tự với hai biểu thức còn lại. Do đó: X 1 X 2 4(a + b + c) ≤ = . 3 27(a + b)(a + c) 27(a + b)(b + c)(c + a) cyc a + b + p2(a + c) sym
Hơn nữa, ta thấy với mọi a, b, c dương: X
9(a + b)(b + c)(c + a) − 8(a + b + c)(ab + bc + ca) = a(b − c)2 ≥ 0 sym 8 ⇒ (a + b)(b + c)(c + a) ≥ (a + b + c)(ab + bc + ca). 9 Do đó: X 1 1 ≤ . (1) 3 6(ab + bc + ca) cyc a + b + p2(a + c)
Mặt khác, ta cũng có: (ab + ca + ca)2 ≥ 3abc(a + b + c)nên theo giả thiết: 1 1 1 ab + bc + ca 3(a + b + c) 3 16(a + b + c) ≥ + + = ≥ ⇒ ab + bc + ca ≥ . (2) a b c abc ab + bc + ca 16 Từ (1) và (2), suy ra: 1 1 1 8 + + ≤ . 3 3 3 (a + b + p2(a + c)) (b + c + p2(b + a)) (c + a + p2(c + b)) 9 101 1. BẤT ĐẲNG THỨC AM-GM Ta có đpcm. a,b,c > 0   a = b = c
Đẳng thức xảy ra khi dấu bằng ở tất cả các bất đẳng thức trên xảy ra hay: ⇒  1 1 1  16(a + b + c) = + + a b c 1 a = b = c = . 4
Câu 1.40. Gọi P là vế trái của bất đẳng thức cần chứng minh. Không mất tính tổng quát, ta giả sử a + b + c = 1.
Áp dụng bất đẳng thức AM-GM ta có : a a 2a √ + √ + a a2 + 8bc ≥ 3a ⇔ √ + a a2 + 8bc ≥ 3a. a2 + 8bc a2 + 8bc a2 + 8bc Tương tự : 2b 2c √ + b b2 + 8ca ≥ 3b ; √ + c c2 + 8ab ≥ 3c b2 + 8ca c2 + 8ab
Cộng ba bất đẳng thức trên lại với nhau ta được :
2P + a3 + b3 + c3 + 24abc ≥ 3 Mặt khác ta lại có :
1 = (a + b + c)3 = a3 + b3 + c3 + 3 (a + b) (b + c) (c + a) ≥ a3 + b3 + c3 + 24abc. Suy ra :
2P ≥ 3 − a3 + b3 + c3 + 24abc ≥ 3 − 1 = 2 ⇒ P ≥ 1 đpcm.
Câu 1.41. Áp dụng bất đẳng thức AM-GM ta có
a3b + 2b = a3b + b + b ≥ 3ab, b3c + 2c ≥ 3bc, c3a + 2a ≥ 3ca. Suy ra
a3b + b3c + c3a + 2(a + b + c) ≥ 3(ab + bc + ca), hay
a3b + b3c + c3a + 6 ≥ 3(ab + bc + ca). Mặt khác (a + b + c)2 ab + bc + ca ≤ = 3, 3 nên ta có
a3b + b3c + c3a + 9 ≥ 4(ab + bc + ca).
Câu 1.42. Áp dụng bất đẳng thức AM-GM ta có r 4 a4 1 a a b a 1 2a b a a = ≤ + + + = + + . abcd 4 b b c d 4 b c d Tương tự: 1 2b c b 1 2c d c 1 2d a d b ≤ + + , c ≤ + + , d ≤ + + . 4 c d a 4 d a b 4 a b c 102 1. BẤT ĐẲNG THỨC AM-GM Suy ra 3 a b c d 1 b c d a a + b + c + d ≤ + + + + + + + 4 b c d a 4 a b c d 3 1 b c d a ≤ (a + b + c + d) + + + + . 4 4 a b c d Nên b c d a a + b + c + d ≤ + + + . a b c d
Câu 1.43. Ta giả sử a = min {a,b,c}.
Ta có V T = Q (a + b) (a2 − ab + b2). Mà
(a + b) (b + c) (c + a) = (a + b + c) (ab + bc + ca) − 3abc
≤ (a + b + c) (ab + bc + ca) = 2 (ab + bc + ca) .
Suy ra V T ≤ 2 (ab + bc + ca) Q (a2 − ab + b2).
Do a = min {a,b,c} nên a2 − ab + b2 ≤ b2 và c2 − ca + a2 ≤ c2. Tiếp theo ta chứng minh
b2c2 (ab + bc + ca) b2 − bc + c2 ≤ 1. (∗)
Áp dụng bất đẳng thức AM-GM ta có
bc + bc + ab + bc + ca + b2 − bc + c2 4
b2c2 (ab + bc + ca) b2 − bc + c2 ≤ 4 1 = b2 + c2 + 2bc + ab + ca4 . 44 Lại có
b2 + c2 + 2bc + ca + ab = (b + c)2 + a (b + c) = (2 − a)2 + a (2 − a) = −2a + 4 ≤ 4. Từ đó, ta có đpcm.
Câu 1.44. Xét x2 − yz, y2 − zx, z2 − xy là các số thực dương. Áp dụng bất đẳng thức AM-GM ta có
x2 + y2 + z2 − (xy + yz + zx) 3 P ≤ 3 Mặt khác
x2 + y2 + z2 + 2(xy + yz + zx) = (x + y + z)2 ≥ 0, suy ra 1 xy + yz + zx ≥ − . 2 Do đó 1 P ≤ . 8
Xét x2 − yz > 0 > y2 − zx, z2 − xy,khi đó áp dụng bất đẳng thức AM-GM ta có
xy + xz + x2 − yz − y2 − z2 3
P = (x2 − yz)(xz − y2)(xy − z2) ≤ 3 103 1. BẤT ĐẲNG THỨC AM-GM Mặt khác
x2 + xy + xz − yz − y2 − z2 = x2 + xy + xz + yz − (y + z)2 ≤ x2 + xy + xz + yz 3 =
(x2 + y2 + z2) − (x − 2y)2 − (x − 2z)2 2 3 3 ≤ (x2 + y2 + z2) = . 2 2 √ √ 1 2 2 Suy ra P ≤
. Đẳng thức xảy ra chẳng hạn khi x = , y = − , z = 0. 8 2 2 1 Vậy GTLN của P bằng: . 8 Câu 1.45. Ta có x2y2 x2y2 1 (x + y)2 ≥ 4xy ⇒ = ≤ xy(x + y). 1 − z x + y 4 1 Do đó, V T ≤
(xy(x + y) + yz(y + z) + zx(z + x)) + 3xyz. Tiếp theo ta chứng minh 4 2 2(x + y + z)3
xy(x + y) + yz(y + z) + zx(z + x) + 12xyz ≤ = . (1) 3 3
Ta có (1) tương đương với
2(x3 + y3 + z3) + 3 (xy(x + y) + yz(y + z) + zx(z + x)) ≥ 24xyz. (2) Vì √ √ √
2(x3 + y3 + z3) ≥ 2xy xy + 2yz yz + 2zx zx, nên √ √ √ √
V T (2) ≥ 8(xy xy + yz zy + zx zx) ≥ 24 xyz ≥ 24xyz. √ (Do xyz ≤ 1 nên xyz ≥ xyz.).
Câu 1.46. Áp dụng bất đẳng thức AM – GM, ta có s a2 (b + 2c)a2 a2 (b + 2c)a2 2a2 + ≥ 2 = . b + 2c 9 b + 2c 9 3 Tương tự b2 (c + 2a)b2 2b2 c2 (a + 2b)c2 2c2 + ≥ , + ≥ . c + 2a 9 3 a + 2b 9 3 Suy ra: a2 b2 c2 F = + + b + 2c c + 2a a + 2b 2 1 ≥ a2 + b2 + c2 −
a2(b + 2c) + b2(c + 2a) + c2(a + 2b) . (1) 3 9
Lại áp dụng AM – GM, ta có a3 + a3 + c3 b3 + b3 + a3 c3 + c3 + b3 a2c + b2a + c2b ≤ + + = a3 + b3 + c3. (2) 3 3 3 Từ (1) và (2) suy ra: 2 1 F ≥ a2 + b2 + c2 − (a + b + c) (a2 + b2 + c2) 3 9 2 1 ≥ a2 + b2 + c2 −
a2 + b2 + c2 p3 (a2 + b2 + c2). 3 9 104 1. BẤT ĐẲNG THỨC AM-GM
Đặt t = p3 (a2 + b2 + c2) , từ giả thiết ta có:
25 a2 + b2 + c2 − 48 = 9 a4 + b4 + c4 ≥ 3 a2 + b2 + c22
⇒ 3 a2 + b2 + c22 − 25 a2 + b2 + c2 + 48 ≤ 0 16 ⇒ 3 ≤ a2 + b2 + c2 ≤ . 3 Do đó 2 1 F ≥ t2 − t3 = f (t). 9 27 với t ∈ [3; 4] (3) Mà min f (t) = f (3) = 1
(4). Từ (3) và (4) suy ra F ≥ 1. t∈[3;4]
Vậy min F = 1 xảy ra khi a = b = c = 1.
Câu 1.47. Bất đẳng thức cần chứng minh tương đương với √ √ X p 5a2 + 4bc − 2 bc > 3(a2 + b2 + c2)   X a2 1 ⇔ ≥  . (1) √ √  p3(a2 + b2 + c2) 5a2 + 4bc + 2 bc 5 Áp dụng đt AM-GM ta có p 8a2 + 3b2 + 3c2 + 4bc 3(a2 + b2 + c2)(5a4bc) ≤ 2 √
p3(a2 + b2 + c2).2 bc ≤ a2 + b2 + c2 + 3bc. Do đó √ √ p 10a2 + 5(b + c)2 3(a2 + b2 + c2) 5a2 + 4bc + 2 bc ≤ ≤ 5(a2 + b2 + c2). 2 Suy ra X a2 1 V T (1) ≥ = . 5(a2 + b2 + c2) 5
Câu 1.48. Gọi P là vết trái. Ta có 4a2b a2b (1 + a + b + c) a2bc = = a2b + . 1 + a + b 1 + a + b 1 + a + b Tương tự 4b2c ab2c 4c2a abc2 = b2c + , = c2a + . 1 + b + c 1 + b + c 1 + c + a 1 + c + a Do đó X a2b X X a 4P = 4 = a2b + abc . 1 + a + b 1 + a + b Mặt khác X a X a (1 + a + b + c) X ac 4 = = 3 + . 1 + a + b 1 + a + b 1 + a + b Lại có ac 3ac 1 2ac ac = ≤ + . 1 + a + b 2 (2a + b) + (2b + c) 3 2a + b 2b + c Nên X ac 1 ≤ (a + b + c) = 1. 1 + a + b 3 Suy ra
4P ≤ a2b + b2c + c2a + abc ≤ 4 ⇒ P ≤ 1. 105 1. BẤT ĐẲNG THỨC AM-GM
Câu 1.49. Ký hiệu P là vế trái của bất đẳng thức cần chứng minh theo yêu cầu đề bài.
Tiếp theo, có thể giải bài đã ra theo một trong các cách sau:
Cách 1 Vì a, b, c > 0 nên ta có: a2 + bc b2 + ca c2 + ab P = + + . pa(b + c).(a2 + bc) pb(c + a).(b2 + ca) pc(a + b).(c2 + ab)
Áp dụng bất đẳng thức Cauchy cho hai số thực dương a(b + c) và (a2 + bc), ta được: p ab + ac + a2 + bc (a + b)(a + c) a(b + c). (a2 + bc) ≤ = . 2 2 a2 + bc 2 (a2 + bc) Suy ra ≥ . (1.1) pa(b + c). (a2 + bc) (a + b)(a + c)
Chứng minh tương tự, ta được: b2 + ac 2 (b2 + ca) ≥ . (1.2) pb(c + a). (b2 + ca) (b + c)(a + c) c2 + ab 2 (c2 + ab) ≥ . (1.3) pc(a + b). (c2 + ab) (c + a)(c + b)
Cộng vế theo vế, các bất đẳng thức (1.1), (1.2) và (1.3), ta được: a2 + bc b2 + ca c2 + ab P ≥ 2 + + . (1.4) (a + b)(a + c) (b + c)(b + a) (c + a)(c + b) Ta sẽ chứng minh: a2 + bc b2 + ca c2 + ab 3 + + ≥ . (1.5) (a + b)(a + c) (b + c)(b + a) (c + a)(c + b) 2
Thật vậy, vì a, b, c > 0 nên
(1.5) ⇔ 2 a2 + bc (b + c) + b2 + ca (a + c) + c2 + ab (a + b) ≥ 3(a + b)(b + c)(c + a)
⇔ 4 a2b + ab2 + b2c + bc2 + c2a + ca2 ≥ 3 a2b + ab2 + b2c + bc2 + c2a + ca2 + 2abc
⇔ a2b + ab2 + b2c + bc2 + c2a + ca2 ≥ 6abc
⇔ a b2 + c2 − 2abc + b c2 + a2 − 2ac + c a2 + b2 − 2ab ≥ 0
⇔ a(b − c)2 + b(c − a)2 + c(a − b)2 ≥ 0. (1.6)
Hiển nhiên, (1.6) là bất đẳng thức đúng. Vì thế, (1.5) được chứng minh. Từ (1.4) và (1.5), suy
ra P ≥ 3. Từ các chứng minh trên ta thấy, đẳng thức xảy ra khi và chỉ khi dấu bằng xảy ra đồng
thời ở (1.1), (1.2), (1.3) và (1.6). Dễ thấy, điều cuối cùng có được khi và chỉ khi a = b = c.
Cách 2 Vì a, b, c > 0 nên ta có: p(a2 + bc) (ab + ac) p(b2 + ca) (bc + ba) p(c2 + ab) (ca + cb) P = + + . (2.1) a(b + c) b(c + a) c(a + b) √ √ √
Áp dụng bất đẳng thức Bunhiacopxki cho hai bộ 2 số a, bc và ab, ac , ta được: √ √ √ 2 a2 + bc (ab + ac) ≥ a. ab + bc. ac = ab. (a + c)2 . 106 1. BẤT ĐẲNG THỨC AM-GM q √ Suy ra (a2 + bc) (ab + ac) ≥ ab.(a + c). (2.2)
Bằng cách tương tự, ta cũng chứng minh được: q √ (b2 + ac) (bc + ba) ≥ bc.(b + a) (2.3) q √ (c2 + ab) (ca + cb) ≥ ca.(c + b). (2.4)
Từ (2.1), (2.2), (2.3) và (2.4), suy ra √ √ √ ab(a + c) bc(b + a) ca(c + b) P ≥ + + . (2.5) a(b + c) b(c + a) c(a + b)
Ký hiệu Q là vế phải của (2.5). Theo bất đẳng thức Cauchy cho 3 số thực dương, ta có: s √ √ √ ab(a + c) bc(b + a) ca(c + b) Q ≥ 3 + + = 3. (2.6) a(b + c) b(c + a) c(a + b)
Từ (2.5) và (2.6), suy ra P ≥ 3. Từ các chứng minh trên ta thấy, đẳng thức xảy ra khi và chỉ
khi dấu bằng xảy ra đồng thời ở (2.2),(2.3), (2.4) và (2.6). Dễ thấy, điều cuối cùng có được khi và chỉ khi a = b = c.
Cách 3 Theo bất đẳng thức Cauchy cho 3 số thực dương, ta có: s a2 + bc b2 + ca c2 + ab P ≥ 3 6 . . . (3.1) a(b + c) b(c + a) c(a + b)
Tiếp theo, ta sẽ chứng minh: a2 + bc b2 + ca c2 + ab . . ≥ 1. (3.2) a(b + c) b(c + a) c(a + b)
Thật vậy, do a, b, c > 0 nên
(3.2) ⇔ a2 + bc b2 + ca c2 + ab ≥ abc(a + b)(b + c)(c + a). (3.3) √ √
Áp dụng bất đẳng thức Bunhiacopxki cho hai bộ 2 số a, bc và b, bc , ta được:
a2 + bc b2 + bc ≥ (ab + bc)2 . Hay
b a2 + bc (b + c) ≥ b2 (a + c)2 . (3.4)
Chứng minh tương tự, ta cũng có:
c b2 + ca (c + a) ≥ c2 (b + a)2 . (3.5)
a c2 + ab (a + b) ≥ a2 (c + b)2 . (3.6)
Vì các vế của (3.4), (3.5) và (3.6) đều dương nên nhân các bất đẳng thức đó với nhau, vế theo
vế, rồi chia cả 2 vế của bất đẳng thức thu được cho abc(a + b)(b + c)(c + a), ta sẽ nhận được bất
đẳng thức (3.3). Vì thế, (3.2) được chứng minh. Từ (3.1) và (3.2), hiển nhiên suy ra P ≥ 3. Từ
các chứng minh trên ta thấy, đẳng thức xảy ra khi và chỉ khi dấu bằng xảy ra đồng thời ở (3.1),
(3.4), (3.5) và (3.6). Dễ thấy, điều cuối cùng có được khi và chỉ khi a = b = c. 107 1. BẤT ĐẲNG THỨC AM-GM
Câu 1.50. Áp dụng Cauchy ta có: 1 1 1
(a1 + a2 + · · · + a2018)2 = (a1 + a2 + · · · + a2018) + + · · · + a1 a2 a2018 √ 2018
≥ 2018 2018 a1a2 · · · a2018 · √ = 20182. 2018 a1a2 · · · a2018
Hay a1 + a2 + · · · + a2018 ≥ 2018. (1) a2 + 1 a3 + 2 a2018 + 2017 1 2 2017 Ta có P = a 2 3 2018 1 + + + · · · + − + + · · · + . (2) 2 3 2018 2 3 2018
Tiếp tục sử dụng Cauchy và sử dụng (1) ta thu được a2 + 1 a3 + 2 a2018 + 2017 a 2 3 2018 1 + + + · · · +
≥ a1 + a2 + · · · + a2018 ≥ 2018. (3) 2 3 2018
Dấu đẳng thức xảy ra khi a1 = a2 = · · · = a2018 = 1. 1 2 2017 1 1 1
Từ (2) và (3) suy ra P ≥ 2018 − + + · · · + = 1 + + + · · · + . 2 3 2018 2 3 2018 108
2. BẤT ĐẲNG THỨC CAUCHY-SCHWARZ
§2. Bất đẳng thức Cauchy-Schwarz
Câu 2.1. Bình phương hai vế và rút gọn, ta có q
(a2 + a2 + · · · + a2 ) (b2 + b2 + · · · + b2 ) ≥ (a 1 2 n 1 2 n
1b1 + a2b2 + · · · + anbn) .
Đây chính là bất đẳng thức Cauchy-Schwarz dạng đa thức. Đẳng thức xảy ra khi ai = kbi.
Câu 2.2. Áp dụng bất đẳng thức Cauchy – Schwarz ta có
a2 + 1 b2 + 1 = a2 + 1 1 + b2 ≥ (a + b)2 . Tương tự, ta cũng có
(b2 + 1) (c2 + 1) ≥ (b + c)2 , (c2 + 1) (a2 + 1) ≥ (a + c)2.
Nhân ba bất đẳng thức trên theo vế ta được
a2 + 1 b2 + 1 c2 + 1 ≥ (a + b) (b + c) (c + a) .
Đẳng thức xảy ra khi a = b = c = 1.
Câu 2.3. Áp dụng bất đẳng thức Cauchy – Schwarz ta có
a2 + b2 + 1 1 + 1 + c2 ≥ (a + b + c)2 = 9 hay là 9 a2 + b2 + 1 ≥ . c2 + 2 Tương tự 9 9 b2 + c2 + 1 ≥ và c2 + a2 + 1 ≥ . a2 + 2 b2 + 2
Cộng các bất đẳng thức trên theo vế, ta được 1 1 1 2 a2 + b2 + c2 + 3 ≥ 9 + + . a2 + 2 b2 + 2 c2 + 2
Đẳng thức xảy ra khi a = b = c = 1.
Câu 2.4. Áp dụng bất đẳng thức Cauchy – Schwarz ta có √ √ √ √ √ √ V T = a · a3 + 8abc + b · b3 + 8abc + c · c3 + 8abc
≤ p(a + b + c) (a3 + b3 + c3 + 24abc). Mặt khác
(a + b + c)3 = a3 + b3 + c3 + 3 (a + b) (b + c) (c + a) ≥ a3 + b3 + c3 + 24abc Suy ra q V T ≤
(a + b + c) (a + b + c)3 = (a + b + c)2 = 1. 1
Bài toán được chứng minh. Đẳng thức xảy ra khi a = b = c = . 3 109
2. BẤT ĐẲNG THỨC CAUCHY-SCHWARZ
Câu 2.5. Áp dụng bất đẳng thức Cauchy-Schwarz ta có X a4 (a2 + b2 + c2) V T = ≥ a(b + 2c)
a(b + 2c) + b(c + 2a) + c(a + 2b) (a2 + b2 + c2)2 a2 + b2 + c2 = ≥ = 1. 3(ab + bc + ca) 3
Câu 2.6. Áp dụng bất đẳng thức Cauchy-Schwarz ta có X a4 V T = √ a b2 + c2 + 7 (a2 + b2 + c2)2 ≥ √ √ √
a b2 + c2 + 7 + b c2 + a2 + 7 + c a2 + b2 + 7 (a2 + b2 + c2)2
≥ p(a2 + b2 + c2)(2a2 + 2b2 + 2c2 + 21) √ t t = √ , t = a2 + b2 + c2 ≥ 3. 2t + 21 Ta chứng minh √ t t √
≥ 1 ⇔ t3 ≥ 2t + 21 ⇔ (t − 3)(t2 + 3t + 9) ≥ 0 (đúng). 2t + 21
Câu 2.7. Áp dụng bất đẳng thức Cauchy-Schwarz ta có: 9 (a + b + c)2 = 4a2 + b2 + c2 2a2 + (a2 + b2) + (a2 + c2) a2 b2 c2 1 b2 c2 ≤ + + = + + . 2a2 a2 + b2 a2 + c2 2 a2 + b2 a2 + c2 Tương tự: 9 a2 c2 1 ≤ + + a2 + 4b2 + c2 a2 + b2 c2 + b2 2 và 9 a2 b2 1 ≤ + + . a2 + b2 + 4c2 a2 + c2 b2 + c2 2
Cộng 3 bất đẳng thức trên theo vế ta có đpcm.
Câu 2.8. Bất đẳng thức cần chứng minh tương đương với 3 3 3 6 1 − + 1 − + 1 − ≤ 2a2 + 3 2b2 + 3 2c2 + 3 5 a2 b2 c2 3 ⇔ + + ≤ . (1) 2a2 + 3 2b2 + 3 2c2 + 3 5
Áp dụng bất đẳng thức Cauchy-Schwarz ta có 25 25 = 3(2a2 + 3) 6a2 + (a + b + c)2 (2 + 2 + 1)2
= 2(2a2 + bc) + 2a(a + b + c) + a2 + b2 + c2 2 2 1 ≤ + + . 2a2 + bc a(a + b + c) a2 + b2 + c2 110
2. BẤT ĐẲNG THỨC CAUCHY-SCHWARZ Suy ra a2 3 2a2 2a a2 ≤ + + . 2a2 + 3 25 2a2 + bc a + b + c a2 + b2 + c2 Suy ra 3 2a2 2b2 2c2 V T (1) ≤ + + + 3 . 25 2a2 + bc 2b2 + ca 2c2 + ab Ta chứng minh a2 b2 c2 + + ≤ 1 2a2 + bc 2b2 + ca 2c2 + ab bc ca ab ⇔ + + ≥ 1. (2) 2a2 + bc 2b2 + ca 2c2 + ab
Áp dụng bất đẳng thức Cauchy-Schwarz ta có (ab + bc + ca)2 (ab + bc + ca)2 V T (2) ≥ = = 1.
2abc(a + b + c) + a2b2 + b2c2 + c2a2 (ab + bc + ca)2
Vậy bài toán được chứng minh.
Câu 2.9. Bất đẳng thức cần chứng minh tương đương với 1 1 1 3 1 − + 1 − + 1 − ≤ a2 + 1 b2 + 1 c2 + 1 2 a2 b2 c2 3 ⇔ + + ≤ a2 + 1 b2 + 1 c2 + 1 2 4a2 4b2 4c2 ⇔ + + ≤ 2. (1) 3a2 + ab + bc + ca 3b2 + ab + bc + ca 3c2 + ab + bc + ca Ta có 4a2 (a + a)2 a a = ≤ + . 3a2 + ab + bc + ca (2a2 + bc) + a(a + b + c) 2a2 + bc a + b + c Suy ra a2 b2 c2 V T (1) ≤ + + + 1 ≤ 1 + 1 = 2. 2a2 + bc 2b2 + ca 2c2 + ab Câu 2.10. Ta có 3 ab 2ab = 1 + = 3 − ab a2 + b2 + c2 − ab a2 + b2 + 2c2 + (a − b)2 (a + b)2 ≤ 1 + 2(a2 + b2 + 2c2) 1 a2 b2 ≤ 1 + + . 2 a2 + c2 b2 + c2 Tương tự 3 1 b2 c2 ≤ 1 + + 3 − bc 2 b2 + a2 c2 + a2 và 3 1 c2 a2 ≤ 1 + + . 3 − ca 2 c2 + b2 a2 + b2
Công ba bất đẳng thức trên theo vế ta có đpcm. 111
2. BẤT ĐẲNG THỨC CAUCHY-SCHWARZ Câu 2.11. Ta có 4 4 4 V T = + + x2 + y2 + 3yz y2 + z2 + 3zx z2 + x2 + 3xy 5 5 5 y + x + + z x2 + y2 + 3yz y2 + z2 + 3zx z2 + x2 + 3xy
Áp dụng bất đẳng thức Schwarz ta được: 1 1 1 2 5 √ + √ + √ 4.9 x y z V T ≥ +
2(x2 + y2 + z2) + 3(xy + yz + zx)
2(x2 + y2 + z2) + 3(xy + yz + zx) 36 45 ≥ +
2(x2 + y2 + z2) + 3(xy + yz + zx)
2(x2 + y2 + z2) + 3(xy + yz + zx) 84 162 = ≥ .
2(x2 + y2 + z2) + 3(xy + yz + zx) x2 + y2 + z2 + 27
Vậy bài toán được chứng minh.
Câu 2.12. Áp dụng bất đẳng thức Cauchy-Schwarz ta có a b c a2 b2 c2 + + = + + b + 2c c + 2a a + 2b a(b + 2c) b(c + 2a) c(a + 2b) (a + b + c)2 (a + b + c)2 ≥ = ≥ 1.
a(b + 2c) + b(c + 2a) + c(a + 2b) 3(ab + bc + ca) Do đó a b c 2 + + b + 2c c + 2a a + 2b 1 V T ≥ ≥ . 2(a + b + c) 2 √ √ √ Câu 2.13. Đặt x = ab, y = bc, z = ca, ta có √ √ √ b a ba x2 c b y2 a c z2 √ √ = √ = , √ √ = , √ √ = . 4b c − c a 4b ca − ca 4yz − z2 4c a − a b 4xz − x2 4a b − b c 4xy − y2
Áp dụng bất đẳng thức Cauchy-Schwarz ta có (x + y + z)2 3(xy + yz + zx) V T ≥ ≥ = 1.
4(xy + yz + zx) − (x2 + y2 + z2)
4(xy + yz + zx) − (xy + yz + zx) 1 1 1 Câu 2.14. Đặt = x, = y,
= z ta có x2 + y2 + z2 ≥ 1 và ta chứng minh bất đẳng thức a b c √ x3 x3 x3 3 + + ≥ y2 + z2 y2 + z2 y2 + z2 2 Ta có: x3 x3 x3 x4 y4 z4 + + = + + y2 + z2 y2 + z2 y2 + z2 x (y2 + z2) y (z2 + x2) z (x2 + y2) 2 (x2 + y2 + z2)
≥ z (y2 + z2) + y (z2 + x2) + z (x2 + y2) 112
2. BẤT ĐẲNG THỨC CAUCHY-SCHWARZ
Áp dụng bất đẳng thức AM-GM, ta có √ 1 p 2 3 x y2 + z2 = √ 2x2 (y2 + z2) (y2 + z2) ≤ x2 + y2 + z2 px2 + y2 + z2 2 9 √ 1 p 2 3 y z2 + x2 = √ 2y2 (z2 + x2) (z2 + x2) ≤ x2 + y2 + z2 px2 + y2 + z2 2 9 √ 1 p 2 3 z x2 + y2 = √ 2z2 (x2 + y2) (x2 + y2) ≤ x2 + y2 + z2 px2 + y2 + z2 2 9 Suy ra √ √ x3 y3 z3 3 p 3 + + ≥ x2 + y2 + z2 ≥ y2 + z2 z2 + x2 x2 + y2 2 2 √
Vậy bất đẳng thức được chứng minh. Đẳng thức xảy ra khi và chi chỉ a = b = c = 3.
Câu 2.15. Áp dụng bất đẳng thức Cauchy-Schwarz ta có 100 !2 100 ! 100 ! X X X (3S)2 = a 2 k+1(a2 + 2a ≤ a2 a2 + 2a k k+1ak+2) k+1 k k+1ak+2 k=1 k=1 k=1 100 ! 100 X X = 1. a2 + 2a 2 = a4 + 4a2a a2 k k+1ak+2 k k k+1ak+2 + 4a2 k+1 k+2 k=1 k=1 100 ! X ≤ a4 + 2a2(a2 + a2 ) + 4a2 a2 k k k+1 k+2 k+1 k+2 k=1 100 X = a4 + 6a2a2 + 2a2a2 . k k k+1 k k+2 k=1 Mặt khác 100 100 !2 X X a4 + 2a2a2 + 2a2a2 ≤ a2 k k k+1 k k+2 k k=1 k=1 và 100 50 ! 50 ! X X X a2a2 ≤ a2 a2 , k k+1 2i−1 2j k=1 i=1 j=1 nên ta có 100 !2 50 ! 50 ! 50 50 !2 X X X X X (3S)2 ≤ a2 + 4 a2 a2 ≤ 1 + a2 + a2 = 2. k 2i−1 2j 2i−1 2j k=1 i=1 j=1 i=1 j=1 √2 Suy ra S ≤ . 3
Câu 2.16. Bất đẳng thức cần chứng minh tương đương với X 1 1 ≤ a(a + b + c) + 2(ab + bc + ca) ab + bc + ca cyc X 2(ab + bc + ca) ⇔ ≤ 2 a(a + b + c) + 2(ab + bc + ca) cyc 113
2. BẤT ĐẲNG THỨC CAUCHY-SCHWARZ Hay X a(a + b + c) ≥ 1. (4) a(a + b + c) + 2(ab + bc + ca) cyc Ta có X a2 V T (4) = (a + b + c)
a2(a + b + c) + 2a(ab + bc + ca) cyc (a + b + c)3 ≥ = 1 P
[a2(a + b + c) + 2a(ab + bc + ca)] cyc
Vậy bài toán được chứng minh. r r r b2 c2 a2 Câu 2.17. Đặt S = a 3 + + b 3 + + c 3 + . c2 a2 b2
Áp dụng bất đẳng thức Cauchy-Schwarz ta có b2 b 2 (3 + 1) 3 + ≥ 3 + c2 c Suy ra r b2 1 ab a 3 + ≥ 3a + . c2 2 c Do đó 1 ab bc ca S ≥ 3(a + b + c) + + + . 2 c a b Lại có ab bc ca + + ≥ a + b + c. c a b
Nên ta có S ≥ 2(a + b + c).
Đặt x = ab, y = bc, z = ca, khi đó bất đẳng thức ab bc ca 2 + + ≥ S c a b √ ⇔ p p
2(x2 + y2 + z2) ≥ x x2 + 3z2 + y y2 + 3x2 + z z2 + 3y2 (4). Áp dụng AM-GM ta có √ 1 √ 1 1 x x2 + 3z2 = (2x) x2 + 3z2 ≤ (4x2 + x2 + 3z2) = (5x2 + 3z2). 2 4 4
Tương tự cho hai bđt còn lại và công 3 bđt đó theo vế ta có đpcm.
Câu 2.18. Đặt a = 1 + x2, b = 1 + y2, c = 1 + z2. Ta có x2 + 1 a b c x ≤ = , y ≤ , z ≤ . 2 2 2 2 Do đó a b c V T ≥ 2 + + b + 2c c + 2a a + 2b (a + b + c)2 ≥ 2 ≥ 1.
a(b + 2c) + b(c + 2a) + c(a + 2b) 114
2. BẤT ĐẲNG THỨC CAUCHY-SCHWARZ
Câu 2.19. Theo bất đẳng thức Cauchy-Shwarz, ta có a b c a2 b2 c2 + + = + + 3 p4(b3 + c3) c + a a + b a 3 p4(b3 + c3) bc + ab bc + ca (a + b + c)2 ≥ . a 3 p4(b3 + c3) + 2bc + a(b + c)
Từ đó, để chứng minh bất đẳng thức của bài ra, ta sẽ chứng minh (a + b + c)2 3 ≥ (1) a 3 p4(b3 + c3) + 2bc + a(b + c) 2 với mọi a, b, c > 0. Vì a, b, c > 0 nên
(1) ⇔ 2a2 + a(b + c) + 2(b2 − bc + c2) ≥ 3a 3 p4(b3 + c3). (2)
Vì b2 − bc + c2 > 0 nên áp dụng bất đẳng thức Cauchy cho 3 số thực dương 2a2, a(b + c) và b2 − bc + c2, ta được
2a2 + a(b + c) + 2(b2 − bc + c2) ≥ 3 3
p2a2 · a(b + c) · 2(b2 − bc + c2) = 3a 3p4(b3 + c3).
Vậy (2) được chứng minh; do đó (1) được chứng minh và vì thế bất đẳng thức của bài toán được
chứng minh. Dễ thấy, đẳng thức xảy khi và chỉ khi a = b = c.
Câu 2.20. Kí hiệu P là vế trái của bất đẳng thức cần chứng minh theo yêu cầu đề bài. b
• Nếu a = 0 thì từ các ràng buộc đối với a, b, c suy ra c > 0. Do đó P = + 2 > 1. c
• Nếu b = 0 thì từ các ràng buộc đối với a, b, c suy ra c > 0. Do đó a 2c a 2c P = + > + = 1. c c + 2a a + c 2c + 2a
• Nếu c = 0 thì từ các ràng buộc đối với a, b, c suy ra a,b > 0. a b r a b Do đó P = + ≥ 2 · ·
= 1. Dấu bằng xảy ra khi và chỉ khi b = 2a. b 4a b 4a
• Xét a,b,c > 0. Khi đó, theo bất đẳng thức Cauchy-Schwarz, ta có 4a2 b2 4c2 (2a + b + 2c)2 P = + + ≥ . (1.1) 4ab + 4ac bc + 4ab 2c2 + 4ac 8ab + 8ac + bc + 2c2 (2a + b + 2c)2
Tiếp theo, ta sẽ chứng minh > 1 (1.2) 8ab + 8ac + bc + 2c2 Thật vậy, ta có
(1.2) ⇔ 4a2 + b2 + 4c2 + 4ab + 8ac + abc > 8ab + 8ac + bc + 2c2 ⇔ (2a − b)2 + (3b + 2c)c > 0.
Bất đẳng thức cuối cùng hiển nhiên đúng và vì thế (1.2) được chứng minh.
Từ (1.1) và (1.2), hiển nhiên suy ra P > 1.
Vậy, tóm lại, bất đẳng thức đề bài được chứng minh. Dấu đẳng thức xảy ra khi và chỉ khi b = 2a và c = 0.
Câu 2.21. Ta có a + b + c ≤ p3(a2 + b2 + c2) = 3 Bất đẳng thức cần chứng minh tương đương với a b c + + ≥ 3. (1) 2 − a 2 − b 2 − c 115
2. BẤT ĐẲNG THỨC CAUCHY-SCHWARZ Ta có a2 b2 c2 (a + b + c)2 V T (1) = + + ≥ . a(2 − a) b(2 − b) c(2 − c) 2(a + b + c) − 3 (a + b + c)2 Ta chứng minh
≥ 3 ⇔ (a + b + c − 3)2 ≥ 0 (bđt luôn đúng). Đẳng thức xảy ra 2(a + b + c) − 3
khi và chỉ khi a = b = c = 1. 1 1 − 2ab Câu 2.22. Để ý rằng = 2 −
. Do đó bất đẳng thức cần chứng minh tương đương 1 − ab 1 − ab với 1 − 2ab 1 − 2bc 1 − 2cd 1 − 2da 8 + + + ≥ . 1 − ab 1 − bc 1 − cd 1 − da 3
Vì 1 − 2ab = (a − b)2 + c2 + d2 ≥ 0 nên ta có thể áp dụng bất đẳng thức Cauchy-Schwarz như sau 1 − 2ab 1 − 2bc 1 − 2cd 1 − 2da + + + ≥ 1 − ab 1 − bc 1 − cd 1 − da
[(1 − 2ab) + (1 − 2bc) + (1 − 2cd) + (1 − 2da)]2
≥ (1 − 2ab)(1 − ab) + (1 − 2bc)(1 − bc) + (1 − 2cd)(1 − cd) + (1 − 2da)(1 − da) 4[2 − (a + c)(b + d)]2 = .
4 − 3(a + c)(b + d) + 2(a2 + c2)(b2 + d2)
Bài toán được quy về chứng minh
3 [2 − (a + c)(b + d)]2 ≥ 2 4 − 3(a + c)(b + d) + 2(a2 + c2)(b2 + d2) .
Bất đẳng thức này tương đương với
4 − 6(a + c)(b + d) + 3(a + c)2(b + d)2 − 4(a2 + c2)(b2 + d2) ≥ 0, hay
3 [1 − (a + c)(b + d)]2 + 1 − 4(a2 + c2)(b2 + d2) ≥ 0. Ta có
1 − 4(a2 + c2)(b2 + d2) = (a2 + b2 + c2 + d2)2 − 4(a2 + c2)(b2 + d2)
= (a2 + c2 − b2 − d2)2 ≥ 0.
Nên bất đẳng thức cuối đúng. Bài toán được chứng minh. 2a2 2b2 2c2
Câu 2.23. Từ giả thiết, ta có thể đặt x = ,y = và z =
với a,b,c > 0. Thay vào bất bc ca ab
đẳng thức đã cho, ta có thể viết nó dưới dạng sau a4 b4 c4 + + ≥ 1. a4 + a2bc + b2c2 b4 + b2ca + c2a2 c4 + c2ab + a2b2
Sử dụng bất đẳng thức Cauchy- Schwarz, ta dễ thấy 2 (a2 + b2 + c2) V T ≥ ,
(a4 + b4 + c4) + abc(a + b + c) + (a2b2 + b2c2 + c2a2)
suy ra ta chỉ cần chứng minh bất đẳng thức sau nữa là đủ
a2 + b2 + c22 ≥ a4 + b4 + c4 + abc (a + b + c) + a2b2 + b2c2 + c2a2
⇔ a2b2 + b2c2 + c2a2 ≥ abc(a + b + c)
⇔ (ab − ac)2 + (bc − ba)2 + (ca − cb)2 ≥ 0.
Bài toán được chứng minh xong. 116
2. BẤT ĐẲNG THỨC CAUCHY-SCHWARZ 2a2 2b2 2c2
Câu 2.24. Từ giả thiết, ta có thể đặt x = ,y = và z =
với a,b,c > 0. Thay vào bất bc ca ab
đẳng thức đã cho, ta có thể viết nó dưới dạng sau a4 b4 c4 + + ≥ 1. a4 + a2bc + b2c2 b4 + b2ca + c2a2 c4 + c2ab + a2b2
Sử dụng bất đẳng thức Cauchy Schwarz, ta dễ thấy 2 (a2 + b2 + c2) V T ≥ ,
(a4 + b4 + c4) + abc(a + b + c) + (a2b2 + b2c2 + c2a2)
suy ra ta chỉ cần chứng minh bất đẳng thức sau nữa là đủ
a2 + b2 + c22 ≥ a4 + b4 + c4 + abc (a + b + c) + a2b2 + b2c2 + c2a2, hay
a2b2 + b2c2 + c2a2 ≥ abc(a + b + c) ⇔ (ab − ac)2 + (bc − ba)2 + (ca − cb)2 ≥ 0.
Bài toán được chứng minh xong. a2 b2 c2
Câu 2.25. Từ xyz = 1 ⇒ x = ,y = ,z =
. Bất đẳng thức cần chứng minh tương đương bc ca ab với a4 b4 c4 + + ≥ 1. (1) (a2 − bc)2 (b2 − ca)2 (c2 − ab)2
Áp dụng bất đẳng thức Schwarz ta có: 2 (a2 + b2 + c2) V T (1) ≥ .
(a2 − bc)2 + (b2 − ca)2 + (c2 − ab)2 Ta chứng minh:
(a2 + b2 + c2)2 ≥ (a2 − bc)2 + b2 − ca2 + c2 − ab2 ⇔ (ab + bc + ca)2 ≥ 0.
Vậy bài toán được chứng minh. 1 1 1 Câu 2.26. Đặt a = ; b = ; c =
⇒ x ; y ; z > 0 . Bất đẳng thức cần chứng minh tương x y z
đương với bất đẳng thức sau x y z 3 + + ≥ . pz (3x + y) px (3y + z) py (3z + x) 2
Đặt vế trái là P , sử dụng bất đẳng thức C - S ta có x2 y2 z2 P = + + x.pz (3x + y) y.px (3y + z) z.py (3z + x) (x + y + z)2
≥ xpz (3x + y) + ypx(3y + z) + zpy (3z + x)
Đặt Q = xpz (3x + y) + ypx (3y + z) + zpy (3z + x) Sử dụng bất đẳng thức Cauchy - Schwarz ta có √ √ √ p p p Q = x. xz (3x + y) + y. yx (3y + z) + z. zy (3z + x)
≤ p(x + y + z) [xz (3x + y) + xy (3y + z) + zy (3z + x)] p =
3 (x + y + z) (x2z + y2x + z2y + xyz) 117
2. BẤT ĐẲNG THỨC CAUCHY-SCHWARZ 4
Mặt khác, ta chứng minh được: x2z + y2x + z2y + xyz ≤
(x + y + z)3 Thật vậy, không mất 27
tính tổng quát ta giả sử y là số nằm giữa x và z. Khi đó
(y − x) (y − z) ≤ 0 ⇔ y2 + xz ≤ xy + yz ⇔ y2x + x2z ≤ x2y + xyz x + z x + z
⇒ x2z + y2x + z2y + xyz ≤ x2y + z2y + 2xyz = y (x + z)2 = 4.y. . 2 2  x + z x + z 3 y + + 4 ≤ 4. 2 2   = (x + y + z)3  3  27 2 3 Do đó Q ≤ (x + y + z)2 ⇒ P ≥
(đpcm). Đẳng thức xảy ra khi x = y = z. 3 2
Câu 2.27. Khi thay a, b, c bởi −a, − b, − c thì bất đẳng thức không đổi, do đó ta giả sử
a ≤ 0 ≤ b, c. Khi đó ta thay a bởi −a ta cần chứng minh BĐT
(a2 − ab + b2)(b2 + bc + c2)(c2 − ca + a2) ≥ 3(bc − ab − ac)3. (1) với a, b, c ≥ 0.
Ta chỉ xét bc − ca − ab ≥ 0. Do
b2 + bc + c2 − (bc − ac − ab) = b2 + c2 + a(b + c) ≥ 0, nên ta chứng minh
(a2 − ab + b2)(c2 − ca + a2) ≥ 3(bc − ab − ac)2. (2) Ta có
4(a2 − ab + b2) · 4(c2 − ca + a2) = (a + b)2 + 3(b − c)2 · (c + a)2 + 3(c − a)2
≥ [(a + b)(a + c) + 3(a − c)(a − b)]2 . Ta chứng minh
(a + b)(a + c) + 3(a − b)(a − c) ≥ 4(bc − ab − ac).
Bất đẳng thức này đúng vì nó tương đương với ⇔ a(2a + b + c) ≥ 0.
Câu 2.28. Áp dụng bất đẳng thức Cauchy-Schwarz ta có X a4 (a2 + b2 + c2)2 V T = ≥ . a2(2b2 − bc + 2c2) P a2(2b2 − bc + 2c2) Ta chứng minh X (a2 + b2 + c2)2 ≥ a2(2b2 − bc + 2c2), hay X X X a4 + abc a ≥ 2 a2b2. (1)
Áp dụng bất đẳng thức Schur ta có X X X X a4 + abc a ≥ ab(a2 + b2) ≥ 2 a2b2, nên (1) đúng. 118
2. BẤT ĐẲNG THỨC CAUCHY-SCHWARZ
Câu 2.29. Áp dụng bất đẳng thức Cauchy-Schwarz, ta có √ √ 2 P 2 = a(2 − bc) + 2. 2(b + c)
≤ (a2 + 2) (2 − bc)2 + 2(b + c)2 = (a2 + 2)(b2 + 2)(c2 + 2).
Lại áp dụng bất đẳng thức AM-GM, ta có 1 (a2 + 2)(b2 + 2)(c2 + 2) = 3(a2 + 2).2(b2 + 2).(c2 + 2) 6
1 3(a2 + 2) + 2(b2 + 2) + (c2 + 2) 3 ≤ = 36. 6 3
Từ đó suy ra P 2 ≤ 36 . Suy ra −6 ≤ P ≤ 6. Mặt khác với a = 0, b = 1, c = 2 thì P = 6;
a = 0, b = 1, c = 2 thì P = −6.
Vậy Pmax = 6 và Pmin = −6. a + b b + c c + a Câu 2.30. Đặt P = + +
. Áp dụng bất đẳng thức Cauchy (a + b + 1)2 (b + c + 1)2 (c + a + 1)2 - Schwarz ta có a + b b + c c + a 2 2(a + b + c)P ≥ + + . a + b + 1 b + c + 1 c + a + 1 Tiếp theo ta chứng minh a + b b + c c + a + + ≥ 2, a + b + 1 b + c + 1 c + a + 1 hay 1 1 1 + + ≤ 1. a + b + 1 b + c + 1 c + a + 1
Bất đẳng thức này đúng do 1 1 1 1 + + ≤ . a3 + b3 + abc b3 + c3 + abc c3 + a3 + abc abc
Câu 2.31. Áp dụng bất đẳng thức Cauchy-Schwarz ta có z2 (x4 + y)(
+ 1) ≥ (x2 + z)2 ⇒ (x4 + y)(z2 + y) ≥ y(x2 + z)2. y Tương tự:
(y4 + z)(x2 + z) ≥ z(y2 + x)2
(x4 + y)(z2 + y) ≥ y(x2 + z)2
(z4 + x)(y2 + x) ≥ x(z2 + y)2.
Nhân các bất đẳng thức trên theo vế ta được
(x4 + y)(y4 + z)(z4 + x) ≥ xyz(x + y2)(y + z2)(z + x2) ≥ (x + y2)(y + z2)(z + x2).
Câu 2.32. Áp dụng bất đẳng thức Cauchy-Schwarz ta có s a b c V T ≤ + + · (a + b + c). 3a + b 3b + c 3c + a 119
2. BẤT ĐẲNG THỨC CAUCHY-SCHWARZ Ta chứng minh a b c 1 + + ≤ 3a + b 3b + c 3c + a 2 X b 3 ⇔ ≥ . 3a + b 4 cyc Ta có X b X b2 (a + b + c)2 = ≥ . 3a + b 3ab + b2 a2 + b2 + c2 + 3(ab + bc + ca) cyc cyc Do đó, ta chứng minh (a + b + c)2 3 ≥ a2 + b2 + c2 + 3(ab + bc + ca) 4
⇔ a2 + b2 + c2 ≥ ab + bc + ca (đúng).
Câu 2.33. Từ giả thiết có
(1 − b2)(1 − c2) ≥ (a − bc)2.
Bất đẳng thức cần chứng minh tương đương với
(1 − b6)(1 − c6) ≥ (a3 − b3c3)2, hay
(1 − b2)(1 − c2)(1 + b2 + b4)(1 + c2 + c4) ≥ (a − bc)2(a2 + abc + b2c2)2. Như vậy cần chứng minh
(1 + b2 + b4)(1 + c2 + c4) ≥ (a2 + abc + b2c2)2
Áp dụng bất đẳng thức Cauchy-Schwarz, ta có
(1 + b2 + b4)(1 + c2 + c4) ≥ (1 + |bc| + b2c2)2
≥ (a2 + |abc| + b2c2)2 ≥ (a2 + abc + b2c2)2.
Do −1 ≤ a ≤ 1. Suy ra điều phải chứng minh. Dấu bằng xảy ra khi a = b = c = 1. Câu 2.34. Ta có 1 1 1 1 a b c 2 b c a 2! (a2 + b2 + c2) + + ≥ + + + + + a2 b2 c2 2 b c a a b c 1 a b c b c a 2 ≥ + + + + + 4 b c a a b c 6 a b c b c a ≥ + + + + + 4 b c a a b c 3 b + c c + a a + b = + + 2 a b c Suy ra 1 1 1 b + c c + a a + b 2 + + ≥ + + . a2 b2 c2 a b c 120
2. BẤT ĐẲNG THỨC CAUCHY-SCHWARZ
Câu 2.35. Bất đẳng thức cần chứng minh tương đương với n−1 2 n−1 P P n−1 (n − 2) x − 2 x k k X (n − 2)x2 + 2x 1 1 1 + ≥ 1 − k=1 k=1 (n − 1)x2 + 1 n − 1 n−1 2 k=1 1 (n − 1) P xk + 1 k=1 n−1 2 P n−1 xk + 1 X ((n − 1)x ⇔ k + 1)2 ≥ k=1 . (n − 1)((n − 1)x2 + 1) n−1 2 k=1 k (n − 1) P xk + 1 k=1
Áp dụng bất đẳng thức Cauchy-Schwarz ta có n−1 2 P n−1 xk + 1 X ((n − 1)xk + 1)2 ≥ k=1 . (n − 1)((n − 1)x2 + 1) n−1 k=1 k P x2 + 1 k k=1 Nên ta chứng minh n−1 2 n−1 2 P x P k + 1 xk + 1 k=1 ≥ k=1 , n−1 n−1 2 P x2 + 1 P k (n − 1) xk + 1 k=1 k=1 hay n−1 !2 n−1 X X (n − 1) xk ≥ x2. (1) k k=1 k=1
Câu 2.36. Áp dụng bđt Cauchy-Schwarz ta có 1 1 1 (a + b + c)2 + + ≥ 2a2 + bc 2b2 + ac 2c2 + ab ab + bc + ac 4(a + b + c)2 ≥ . P(2a2 + bc)(b2 + c2 + 2bc) Ta chứng minh
X(2a2 + bc)(b2 + c2 + 2bc) ≤ 4(a2 + b2 + c2)(ab + bc + ca), hay X X ab(a2 + b2) ≥ 2 a2b2.
Bất đẳng thức này đúng theo AM-GM s a2 + bc b2 + ac c2 + ab
Câu 2.37. Áp dụng bất đẳng thức AM − GM ta có P ≥ 3 · 3n · · . a(b + c) b(a + c) c(a + b) (1) a2 + bc b2 + ac c2 + ab Ta chứng minh · · ≥ 1. (2) a(b + c) b(a + c) c(a + b)
Thật vậy: Do a, b, c dương nên bất đẳng thức (1) đưa về
(a2 + bc)(b2 + ac)(c2 + ab) ≥ abc(a + b)(b + c)(c + a) (3). 121
2. BẤT ĐẲNG THỨC CAUCHY-SCHWARZ √ √
Áp dụng bất đẳng thức Cauchy - Schwarz cho 2 bộ 2 số a, bc và b, bc ta được
(a2 + bc)(b2 + bc) ≥ (ab + bc)2
hay b(a2 + bc)(b + c) ≥ b2(a + c)2. (4)
Chứng minh tương tự ta có c(b2 + ac)(a + c) ≥ c2(a + b)2 (5)
a(c2 + ab)(a + b) ≥ a2(c + b)2 (6)
Vì các vế của (4), (5), (6) đều dương. Nhân vế với vế các bất đẳng thức đó với nhau, rồi chia cả
2 vế của bất đẳng thức thu được cho abc(a + b)(b + c)(c + a) ta được (3), do đó có (2), suy ra P ≥ 3.
Dấu bằng xảy ra khi và chỉ khi dấu bằng xảy ra đồng thời ở (1), (4), (5) và (6), khi đó a = b = c.
Vậy giá trị nhỏ nhất của P là 3. √
Câu 2.38. a3 + 5 = (a3 + 1 + 1) + 3 ≥ 3 3 a3 · 1 · 1 + 3 = 3a + 3.
Tương tự: b3 + 5 ≥ 3b + 3; c3 + 5 ≥ 3c + 3. a3 + 5 b3 + 5 c3 + 5 3a + 3 3b + 3 3c + 3 + + ≥ + + a3(b + c) b3(c + a) c3(a + b) a3(b + c) b3(c + a) c3(a + b) Ta có: 3a + 3 3b + 3 3c + 3 3a(abc) + 3(abc)2 3b(abc) + 3(abc)2 3c(abc) + 3(abc)2 + + = + + a3(b + c) b3(c + a) c3(a + b) a3(b + c) b3(c + a) c3(a + b) 3(bc + b2c2) 3(ca + c2a2) 3(ab + a2b2) = + + ab + bc bc + ca ca + ab
Đặt x = bc, y = ca, z = ab; x,y,z > 0, xyz = 1. 3a + 3 3b + 3 3c + 3 x + x2 y + y2 z + z2 + + ≥ 3 + + a3(b + c) b3(c + a) c3(a + b) y + z z + x x + y x y z x2 y2 z2 = 3 + + + 3 + + y + z z + x x + y y + z z + x z + y x y z 3 + + ≥ y + z z + x x + y 2 √ x2 y2 z2 x + y + z 3 3 xyz 3 + + ≥ ≥ = . y + z z + x z + y 2 2 2 a3 + 5 b3 + 5 c3 + 5 Vậy + + ≥ 9. a3(b + c) b3(c + a) c3(a + b)
Dấu “=” xảy ra khi a = b = c = 1.
Câu 2.39. Biến đổi và áp dụng bất đẳng thức Cauchy - Schwartz: n n n X X X ak(bk + ak+1) = akbk + akak+1 k=1 k=1 k=1 v u n ! n ! n X X X ≤ u t a2 b2 + a k k k ak+1 k=1 k=1 k=1 v u n n X X = u t a2 + a k k ak+1 k=1 k=1 v u n !2 n u X X X = t ak − 2 aiaj + akak+1 k=1 1≤ik=1 122
2. BẤT ĐẲNG THỨC CAUCHY-SCHWARZ s n X X = 1 − 2 aiaj + akak+1. (1) 1≤ik=1 n X X Đặt x = aiaj và y = akak+1 ta có ngay x > y. (2) 1≤ik=1 Đồng thời cũng có n !2 n X X 1 = ak = a2 + 2x > y ⇒ y < 1. k k=1 k=1
Do (1) nên ta cần chứng minh:
√1 − 2x + y < 1 ⇔ 1 − 2x < 1 − 2y + y2 ⇔ 2(y − x) < y2 (hiển nhiên đúng). 123
3. MỘT SỐ BẤT ĐẲNG THỨC KHÁC
§3. Một số bất đẳng thức khác Câu 3.1. Ta có r 2a 2b 2c 3abc + + + 3 b + c c + a a + b a3 + b3 + c3 (a + b + c)2 9abc ≥ + ab + bc + ca q 3 3 (3abc)2(a3 + b3 + c3) (a + b + c)2 9abc ≥ + . ab + bc + ca a3 + b3 + c3 + 6abc Ta chỉ cần chứng minh (a + b + c)2 9abc − 3 ≥ 1 − ab + bc + ca a3 + b3 + c3 + 6abc 1 a + b + c ⇔ ≥ . ab + bc + ca a3 + b3 + c3 + 6abc
Bất đẳng thức này hiển nhiên đúng theo Schur.
Câu 3.2. Vì a + b + c = 1, nên ta có ab bc ca ab bc ca + + = (a + b + c) + + a + b b + c c + a a + b b + c c + a 1 1 1 = ab + bc + ca + abc + + a + b b + c c + a 9abc
≥ ab + bc + ca + 2(a + b + c) 9 = ab + bc + ca + abc. 2 Suy ra ab bc ca 2 + +
+ 1 ≥ 2(ab + bc + ca) + 9abc + 1. a + b b + c c + a Do đó, ta chứng minh 9abc + 1 ≥ 4(ab + bc + ca). (1)
Áp dụng bất đẳng thức Schur ta có 9abc a2 + b2 + c2 + ≥ 2(ab + bc + ca). a + b + c Suy ra 1 + 9abc ≥ 4(ab + bc + ca), hay (1) được chứng minh.
Câu 3.3. Bất đẳng thức cần chứng minh tương đương với X (a + b + c) 3
p(a2 + bc)(a2 + b2)(a2 + c2) ≥ 9 3p(a2 + b2)(a2 + c2)(b2 + c2)abc. cyc
Áp dụng bất đẳng thức Cauchy-Schwarz ta có
(a2 + b2)(a2 + c2) ≥ (a2 + bc)2, 124
3. MỘT SỐ BẤT ĐẲNG THỨC KHÁC và theo AM-GM X 9 3
p(a2 + b2)(a2 + c2)(b2 + c2)abc ≤ 3 c(a2 + b2). cyc Do đó, ta chứng minh X X (a + b + c) (a2 + bc) ≥ 3 c(a2 + b2), cyc cyc Đây chính là bđt Schur. Câu 3.4. Ta có
V T = a2b2c2 + 2(a2b2 + b2c2 + c2a2) + 4(a2 + b2 + c2) + 8. Mặt khác
a2b2 + b2c2 + c2a2 + 3 = a2b2 + 1 + b2c2 + 1 + c2a2 + 1 ≥ 2(ab + bc + ca) và √ 3abc
a2b2c2 + 2 = a2b2c2 + 1 + 1 ≥ 3 3 a2b2c2 = √ 3 abc 9abc ≥
≥ 2(ab + bc + ca) − (a2 + b2 + c2). a + b + c Suy ra
V T ≥ 2(ab + bc + ca) − (a2 + b2 + c2) + 2.2(ab + bc + ca) + 4(a2 + b2 + c2)
= 6(ab + bc + ca) + 3(a2 + b2 + c2) ≥ 6(ab + bc + ca) + 3(ab + bc + ca) ≥ 9(ab + bc + ca).
Bài toán được chứng minh.
Câu 3.5. Gọi P là vế trái của bất đẳng thức cần chứng minh. Không mất tính tổng quát, ta
giả sử a + b + c = 3. Áp dụng bất đẳng thức AM-GM ta có s s (a + b)3 (a + b)3 ab(4a + 4b + c) 1 + + ≥ (a + b) 8ab(4a + 4b + c) 8ab(4a + 4b + c) 27 2 Suy ra s (a + b)3 ab(4a + 4b + c) 1 + ≥ (a + b). 8ab(4a + 4b + c) 54 4 Tương tự s (b + c)3 bc(4b + 4c + a) 1 + ≥ (b + c) 8bc(4b + 4c + a) 54 4 và s (c + a)3 ca(4c + 4a + b) 1 + ≥ (c + a). 8ca(4c + 4a + b) 54 4
Cộng ba bất đẳng thức trên ta có 1 √ P + A ≥ B. 2 2 Với 1 A =
[ab(4a + 4b + c) + bc(4b + 4c + a) + ca(4c + 4a + b)] 54 1 =
[4ab(a + b) + 4bc(b + c) + 4ca(c + a) + 3abc] 54 1 1 1 =
[4(a + b + c)(ab + bc + ca) − 9abc] ≤ (a + b + c)3 = . 54 54 2 125
3. MỘT SỐ BẤT ĐẲNG THỨC KHÁC và 1 3 B = .2 (a + b + c) = . 4 2 Suy ra 1 3 1 √ √ P ≥ − = 1 ⇒ P ≥ 2 2. 2 2 2 2
Bài toán được chứng minh.
Câu 3.6. Sử dụng bất đẳng thức AM-GM,ta có:
12(a2 + b2 + c2) + 6abc + 48 − 30(a + b + c)
= 12(a2 + b2 + c2) + 3(2abc + 1) + 45 − 5.2.3(a + b + c) √
≥ 12(a2 + b2 + c2) + 9 3 a2b2c2 + 45 − 5.((a + b + c)2 + 9) 9abc = 7(a2 + b2 + c2) + √ − 10(ab + bc + ca) 3 abc 27 ≥ 7(a2 + b2 + c2) + − 10(ab + bc + ca) a + b + c
Mặt khác sử dụng bất đẳng thức Schur, 9
≥ 4(ab + bc + ca) − (a + b + c)2 = 2(ab + bc + ca) − (a2 + b2 + c2) a + b + c Do đó 27 7(a2 + b2 + c2) + − 10(ab + bc + ca) a + b + c
≥ 7(a2 + b2 + c2) + 6(ab + bc + ca) − 3(a2 + b2 + c2) − 10(ab + bc + ca)
= 4(a2 + b2 + c2 − ab − bc − ca) ≥ 0.
Bất đẳng thức được chứng minh. 1 1 1 Câu 3.7. Đặt x = , y = , z =
. Bất đẳng thức cần chứng minh trở thành a b c
x2 + y2 + z2 + 3 ≥ 2(xy + yz + zx)
⇔ (x + y + z)(x2 + y2 + z2 + 3) ≥ 2(x + y + z)(xy + yz + zx) Hay
x3 + y3 + z3 + 3(x + y + z) ≥ x2(y + z) + y2(z + x) + z2(x + y) + 6 (1). Ta có
x3 + y3 + z3 + 3(x + y + z) ≥ x3 + y3 + z3 + 9 = x3 + y3 + z3 + 3xyz + 6 ≥ V P (1).
Vậy bài toán được chứng minh. √ √ √
Câu 3.8. Đặt x = 3 a2, y = 3 b2, z = 3 c2, ta có xyz = 1 và bất đẳng thức cần chứng minh trở thành √ √ √
x3 + y3 + z3 + 3xyz ≥ 2 xy xy + yz yz + zx zx . (1)
Áp dụng bất đăng thức Schur ta có
x3 + y3 + z3 + 3xyz ≥ xy(x + y) + yz(y + z) + zx(z + x) √ √ √ ≥ 2xy xy + 2yz yz + 2zx zx.
Suy ra (1) đúng, hay bài toán được chứng minh. 126
3. MỘT SỐ BẤT ĐẲNG THỨC KHÁC 1 1 1
Câu 3.9. Áp dụng bất đẳng thức Holder với chú ý + + = 1 ta có 2 3 6 1 1 1 2 4a3 + 9b3 + 36c3 + + ≥ (a + b + c)3 = 1. 2 3 6 Từ đó ta có đpcm.
Câu 3.10. Đặt P là vế trái của bất đẳng thức. Áp dụng bất đẳng thức Holder ta có: X P 3 a(a + 2b) ≥ (a + b + c)4. Mặt khác X a(a + 2b) = (a + b + c)2, nên ta có
P 3 ≥ (a + b + c)2 = 1 ⇒ P ≥ 1.
Câu 3.11. Áp dụng bất đẳng thức Holder ta có a b c 2 √ √ √ 3 (x2 + y2 + z2) + + ≥ 3 a2 + 3 b2 + 3 c2 . x y z Từ đó ta có đpcm.
Câu 3.12. Áp dụng bất đẳng thức Holder ta có √ √ √ n−1 (xn + yn + zn) n−1 an + n−1 bn + n−1 cn ≥ (ax + by + cz)n = 1. Suy ra √ √ √ 1−n xn + yn + zn ≥ n−1 an + n−1 bn + n−1 cn . Câu 3.13. 1 1 1 √ ab + bc + ca √ (a + b + c) + + + 4 2 ≥ 9 + 4 2 a b c a2 + b2 + c2 1 1 1 √ √ ab + bc + ca ⇔(a + b + c) + + − 9 ≥ 4 2 − 4 2 a b c a2 + b2 + c2 (a − b)2 (b − c)2 (c − a)2
√ (a − b)2 + (a − c)2 + (b − c)2 ⇔ + + ≥ 2 2 . (*) ab bc ca a2 + b2 + c2 (a − b)2 (a − c)2
Không mất tính tổng quát giả sử a ≥ b ≥ c ⇒ =
Áp dụng bất đẳng thức b c Chebyshev ta có: (a − b)2 (a − c)2 +
(b + c) ≥ 2 (a − b)2 + (a − c)2 b c (a − b)2 (a − c)2 2 [(a − b)2 + (a − c)2] ⇒ + ≥ . (1) ab ac a(b + c)
Đẳng thức xảy ra khi và chỉ khi b = c.
Mặt khác theo bất đẳng thức AM − GM : √
2(a2 + b2 + c2) ≥ 2a2 + (b + c)2 ≥ 2 2a(b + c) 127
3. MỘT SỐ BẤT ĐẲNG THỨC KHÁC √ 2 2 2 ⇒ ≥ a(b + c) a2 + b2 + c2 2 [(a − b)2 + (a − c)2] √ (a − b)2 + (a − c)2 ⇒ ≥ 2 2 . (2) a(b + c) a2 + b2 + c2 √
Đẳng thức xảy ra khi và chỉ khi a = b = c hoặc a 2 = b + c. √
a2 + b2 + c2 ≥ 3bc > 2 2bc (do a ≥ b ≥ c) (b − c)2 √ (b − c)2 ⇒ ≥ 2 2 . (3) bc a2 + b2 + c2
Từ (1),(2),(3) suy ra bất đẳng thức (*) đúng. Suy ra điều phải chứng minh. 128 Chương 2
Một số phương pháp chứng minh bất đẳng thức §1. Phương pháp quy nạp 129
2. PHƯƠNG PHÁP PHÂN TÍCH BÌNH PHƯƠNG SOS
§2. Phương pháp phân tích bình phương SOS 130
2. PHƯƠNG PHÁP PHÂN TÍCH BÌNH PHƯƠNG SOS
Câu 5.1. Bất đẳng thức cần chứng minh tương đương với
2 a3 + b3 + c3 − ab (a + b) − bc (b + c) − ca (c + a) − a3 + b3 + c3 − 3abc ≥ 0
⇔ (a + b) (a − b)2 + (b + c) (b − c)2 + (c + a) (c − a)2 1 −
(a + b + c) (a − b)2 + (b − c)2 + (c − a)2 ≥ 0 2
⇔ (a + b − c) (a − b)2 + (b + c − a) (b − c)2 + (c + a − b) (c − a)2 ≥ 0
⇔ Sc (a − b)2 + Sa (b − c)2 + Sb (c − a)2 ≥ 0.
Không mất tính tổng quát giả sử a ≥ b ≥ c ⇒ Sb ≥ 0,Sc ≥ 0 Ta có Sa + Sb = 2c ≥ 0
Câu 5.2. Bất đẳng thức cần chứng minh tương đương với ab − bc + ca 1 bc − ca + ab 1 ca − ab + bc 1 − + − + − ≥ 0 b2 + c2 2 c2 + a2 2 a2 + b2 2 X (b + c)(2a − b − c) X (b + c)(a − b) X (b + c)(a − c) ⇔ ≥ 0 ⇔ + ≥ 0 b2 + c2 b2 + c2 b2 + c2 X (b + c)(a − b) X (c + a)(b − a) ⇔ + ≥ 0 b2 + c2 c2 + a2
X (a − b)2(ab + bc + ca − c2) ⇔ ≥ 0. (1) (b2 + c2)(c2 + a2) Mặt khác
ab + bc + ca − c2 = (b − c)(c − a) + 2bc ≥ (b − c)(c − a),
nên để chứng minh (1) ta chứng minh X (a − b)2(b − c)(c − a) X ≥ 0 ⇔
(a2 + b2)(a − b)2(b − c)(c − a) ≥ 0 (b2 + c2)(c2 + a2)
⇔ (a − b)2(b − c)2(c − a)2 ≥ 0.
Bất đẳng thức cuối hiển nhiên đúng.
Câu 5.3. BĐT cần chứng minh tương đương với
2 a3 + b3 + c3 − ab (a + b) − bc (b + c) − ca (c + a) − a3 + b3 + c3 − 3abc ≥ p p p ab 2 (a2 + b2) − a − b + bc 2 (b2 + c2) − b − c + ca 2 (c2 + a2) − c − a
⇔ Sc (a − b)2 + Sa (b − c)2 + Sb (c − a)2 ≥ 0, với 2ab Sc = a + b − c − , p2 (a2 + b2) + a + b 2bc Sa = b + c − a − , p2 (b2 + c2) + b + c 2ca Sb = c + a − b − . p2 (c2 + a2) + c + a
Do a, b, c bình đẳng, không mất tính tổng quát giả sử a ≥ b ≥ c. Ta có 2ab a ca Sc ≥ a + b − c − =
+ b − c ≥ 0, Sb ≥ c + a − b − ≥ 0 4b 2 a + c 131
2. PHƯƠNG PHÁP PHÂN TÍCH BÌNH PHƯƠNG SOS Ta cần chứng minh ! a b Sb + Sa = 2c 1 − − p2 (c2 + a2) + c + a p2 (b2 + c2) + b + c a b bc bc ≥ 2b 1 − − = + ≥ 0. 2 (a + c) 2 (b + c) a + c b + c
Câu 5.4. Bất đẳng thức cần chứng minh tương đương với 4a2 + 2bc − 3 (b2 + c2) 4b2 + 2ca − 3 (c2 + a2) 4c2 + 2ab − 3 (a2 + b2) + + ≥ 0 b2 + c2 c2 + a2 a2 + b2
2 (a2 − b2) + 2 (a2 − c2) − (b − c)2
2 (b2 − c2) + 2 (b2 − a2) − (c − a)2 ⇔ + b2 + c2 c2 + a2
2 (c2 − a2) + 2 (c2 − b2) − (a − b)2 + ≥ 0 a2 + b2
⇔ Sc (a − b)2 + Sa (b − c)2 + Sb (c − a)2 ≥ 0, với 2(a + b)2 1 Sc = − , (b2 + c2) (c2 + a2) a2 + b2 2(c + a)2 1 Sb = − , (b2 + c2) (b2 + a2) c2 + a2 2(b + c)2 1 Sa = − . (a2 + b2) (a2 + c2) b2 + c2
Giả sử a ≥ b ≥ c suy ra Sb ≥ 0,Sc ≥ 0, Sc ≥ Sb ≥ Sa Ta cần chứng minh Sa + Sb ≥ 0, hay
2 (c + a)2 c2 + a2 + 2 (b + c)2 b2 + c2 ≥ b2 + c2 b2 + a2 + a2 + b2 a2 + c2
Ta thấy hệ số c2 ở vế trái là 2 (c + a)2 + 2 (b + c)2 lớn hơn hoặc bằng vế phải là 2 (a2 + b2) nên
ta chỉ cần chứng minh khi c = 0. Hay
2 a4 + b4 ≥ a2 + b22 ⇔ a2 − b22 ≥ 0 (đúng).
Câu 5.5. Ta có : BĐT cần chứng minh tương đương với
4 a3 + b3 + c3 − 3abc + 6 ab2 + bc2 + ca2 − a3 − b3 − c3 ≥ 3 a2b + b2c + c2a − a3 − b3 − c3
⇔ 2 (a + b + c) (a − b)2 + (b − c)2 + (c − a)2
− 2 (2b + a) (a − b)2 + (2c + b) (b − c)2 + (2a + c) (c − a)2
+ (2a + b) (a − b)2 + (2b + c) (b − c)2 + (2c + a) (c − a)2 ≥ 0
⇔ (2a − b + 2c) (a − b)2 + (2b − c + 2a) (b − c)2 + (2c − a + 2b) (c − a)2 ≥ 0
⇔ Sc (a − b)2 + Sa (b − c)2 + Sb (c − a)2 ≥ 0.
Nếu a ≥ b ≥ c ⇒ Sa ≥ 0,Sc ≥ 0, mặt khác Sa + 2Sb ≥ 0,Sc + 2Sb ≥ 0.
Theo tiêu chuẩn 3 suy ra ĐPCM.
Nếu a ≤ b ≤ c ⇒ Sb ≥ 0,Sc ≥ 0.
Theo tiêu chuẩn 2 ta cần chứng minh Sa + Sb ≥ 0 ( hiển nhiên ) Suy ra điều phải chứng minh. 132
2. PHƯƠNG PHÁP PHÂN TÍCH BÌNH PHƯƠNG SOS
Câu 5.6. Trước hết ta thấy rằng : 1 1 1 X (x − y)2 (x + y + z) + + − 9 = , x y z xy x y z X (x − y)2 6 + + − 9 = . y + z z + x x + y (y + z)(z + x) Ta cần chứng minh : 1 1 1 x y z X (x − y)2 X 3(x − y)2 (x + y + z) + + ≥ 6 + + ⇔ ≥ . x y z y + z z + x x + y xy (y + z)(z + x) 1 3 1 3
với mọi số thực x, y, z thuộc đoạn [1; 2]. Đặt Sx = − , Sy = − , yz (x + y)(x + z) zx (y + x)(y + z) 1 3 Sz = − . xy (z + x)(z + y)
Bất đẳng thức đã cho viết dưới dạng tương đương là:
Sx(y − z)2 + Sy(z − x)2 + Sz(x − y)2 ≥ 0.
Không mất tính tổng quát, ta giả sử 2 ≥ x ≥ y ≥ z ≥ 1 . Ta sẽ chứng minh rằng Sx, Sy ≥ 0 . Thật vậy:
Sx ≥ 0 ⇔ x2 + xy + xz − 2yz ≥ 0, (đúng).
Sy ≥ 0 ⇔ y2 + yx + yz − 2zx ≥ x(y − z) + z(z + y − x) ≥ 0
(do x,y,z ∈ [1; 2] nên y + z − x ≥ 0).
- NếuSz ≥ 0 , ta có đpcm.
- Nếu Sz < 0, ta chứng minh được rằng Sx + 2Sz ≥ 0, Sy + 2Sz ≥ 0 .
Khi đó dễ dàng thấy rằng vì: (x − y)2 ≤ 2 (y − z)2 + (z − x)2 và Sz < 0 nên
Sx(y − z)2 + Sy(z − x)2 + Sz(x − y)2 ≥ (Sx + 2Sz)(y − z)2 + (Sy + 2Sz)(z − x)2 ≥ 0.
Vậy trong mọi trường hợp, ta luôn có đpcm.
Đẳng thức xảy ra khi x = y = z hoặc y = z = 1, x = 2 và các hoán vị của chúng. Câu 5.8. 2 a) Chứng minh
(a2 + b2 + c2) ≤ a3 + b3 + c3, (1). 3
(1) ⇔ (a + b + c) a2 + b2 + c2 ≤ 3 a3 + b3 + c3
⇔ a2b + a2c + b2c + b2a + c2a + c2b ≤ 2 a3 + b3 + c3
⇔ (b + c) (b − c)2 + (c + a) (c − a)2 + (a + b) (a − b)2 ≥ 0 2
Bất đẳng thức trên đúng nên (1) đúng. Đẳng thức xảy ra khi a = b = c = . 3 4
b) Chứng minh a3 + b3 + c3 ≤ (a2 + b2 + c2) − 3abc, (2) 3
(2) ⇔ 3 a3 + b3 + c3 ≤ 2 (a + b + c) a2 + b2 + c2 − 9abc
⇔ a3 + b3 + c3 + 9abc ≤ 2 a2b + a2c + b2c + b2a + c2a + c2b
⇔ Sa(b − c)2 + Sb(c − a)2 + Sc(a − b)2 ≥ 0, (3) .
trong đó Sa = 3a − b − c; Sb = 3b − a − c; Sc = 3c − a − b.
Không mất tính tổng quát giả sử 1 ≥ a ≥ b ≥ c. 133
2. PHƯƠNG PHÁP PHÂN TÍCH BÌNH PHƯƠNG SOS Ta có V T (3) = S a(b − c)2 + Sb
(c − a)2 − (a − b)2 + (Sb + Sc) (a − b)2
Mà Sa ≥ 0; Sb = 2(1 − a) + 2 (b − c) ≥ 0;
(c − a)2 − (a − b)2 ≥ 0;Sb + Sc = 4 (1 − a) ≥ 0.
Suy ra V T (3) ≥ 0 ⇒ (2) đúng. 2 1
Đẳng thức xảy ra khi a = b = c = ; a = 1,b = c = và các hoán vị. 3 2 134 3. PHƯƠNG PHÁP DỒN BIẾN
§3. Phương pháp dồn biến Câu 6.1.
a) Ta có bất đẳng thức sau với mọi a, b, c > 0
abc ≥ (a + b − c)(b + c − a)(c + a − b).
Thật vậy, giả sử a ≥ b ≥ c, ta có
abc − (a + b − c)(b + c − a)(c + a − b)
= (a + b − c)(a − b)2 + c(a − c)(b − c) ≥ 0. Do đó
abc ≥ (3 − 2c)(3 − 2a)(3 − 2b)
⇔ abc ≥ 27 − 18(a + b + c) + 12(ab + bc + ca) − 8abc
⇔ 3abc ≥ 4(ab + bc + ca) − 9 4 ⇔ abc ≥ (ab + bc + ca) − 3. 3 Suy ra 12 4 12 P = abc + ≥ (ab + bc + ca) + − 3 ≥ 8 − 3 = 5. ab + bc + ca 3 ab + bc + ca
Dấu “=” xảy ra khi a = b = c = 1.
Vậy giá trị nhỏ nhất của P là 5.
b) Do bất đẳng thức đã cho đúng với mọi a, b, c > 0 thỏa mãn a + b + c = 3 nên nó cũng đúng 3
với bộ số a = b = x, c = 3 − 2x với mọi x ∈ 0; . Khi đó 2 1 1 3 k −
≥ 1 − x2(3 − 2x), ∀x ∈ 0; x2 + 2x(3 − 2x) 3 2 3
⇔ k ≥ 3x(2 − x)(2x + 1), ∀x ∈ 0; . 2 4 88 Với x = ta có k ≥
> 9. Do k nguyên nên k ≥ 10. 3 9 10 13
Với k = 10, ta cần chứng minh f (a,b,c) := abc + ≥ . ab + bc + ca 3
Không mất tính tổng quát, giả sử a ≥ b ≥ c, khi đó 0 < c ≤ 1. a + b a + b
Ta sẽ chứng minh f (a, b, c) ≥ f ,
, c . Thật vậy, điều này tương đương với 2 2 10 10 (a + b)2 − ≥ c − ab ab + bc + ca (a + b)2 4 + bc + ca 4 10(a − b)2 ⇔ ≥ c(a − b)2. (a + b)2 (ab + bc + ca) + c(a + b) 4
Điều này đúng do c ≤ 1, ab + bc + ca ≤ 3 và (4a + 4b + 4c)2 3(a + b)(a + b + 4c) ≤ = 36. 4 135 3. PHƯƠNG PHÁP DỒN BIẾN a + b a + b 13 Ta sẽ chứng minh f , , c ≥
. Thật vậy, điều này tương đương với 2 2 3 c(3 − c)2 40 13 + ≥ 4 (3 − c)2 + 4c(3 − c) 3 c(3 − c)2 − 4 40 − 10(1 + c)(3 − c) ⇔ + ≥ 0 4 3(1 + c)(3 − c) c − 4 10 ⇔ (c − 1)2 + ≥ 0. 4 3(1 + c)(3 − c) Điều này đúng do
3(4 − c)(1 + c)(3 − c) − 40 =3c3 − 18c2 + 15c − 4 1
=3c(1 − c)(5 − c) − 4 < 3 ·
· 5 − 4 < 0, ∀c ∈ (0, 1]. 4
Vậy số nguyên k nhỏ nhất cần tìm là k = 10.
Câu 6.2. Từ điều kiện a + b + c = 3 ta sẽ nghĩ đến việc dồn biến về trung bình cộng. Bài toán
này ngoài trường hợp dấu bằng xảy ra khi a = b = c = 1. Còn xảy ra tại a = b = 0, c = 3
hoặc các hoán vị của bộ số (0,0,3). Không mất tính tổng quát, giả sử c = max{a,b,c}, suy ra
a ≤ c, b ≤ c và 1 ≤ c ≤ 3, 0 ≤ a + b ≤ 2. Đặt √ √ √ f (a,b,c) = 3a2 + 4bc + 9 + 3b2 + 4ca + 9 + 3c2 + 4ab + 9. a + b a + b
Ta sẽ chứng minh f (a,b,c) ≥ f , ,c . 2 2
Bước 1: Ta sẽ chứng minh rằng √ √ r 5(a − b)2 3a2 + 4bc + 9 + 3b2 + 4ca + 9 ≥ 3(a + b)2 + 8(a + b)c + 36 + . 2
Bình phương hai vế và sử dụng (x + y)2 = 2(x2 + y2) − (x − y)2, ta được √ √ 2
2(3a2 + 4bc + 9 + 3b2 + 4ca + 9 − 3a2 + 4bc + 9 − 3b2 + 4ca + 9 5(a − b)2
≥ 3(a + b)2 + 8(a + b)c + 36 + . 2 Tương đương với (a − b)2 √ √ 2 ≥ 3a2 + 4bc + 9 − 3b2 + 4ca + 9 . 2 Ta có √ √ (a − b) [3(a + b) − 4c] 3a2 + 4bc + 9 − 3b2 + 4ca + 9 = √ √ . 3a2 + 4bc + 9 + 3b2 + 4ca + 9
Do đó, ta chỉ cần chứng minh 1 [3(a + b) − 4c]2 ≥ √ √ . 2 3a2 + 4bc + 9 + 3b2 + 4ca + 92 Suy ra √ √ 2 3a2 + 4bc + 9 + 3b2 + 4ca + 9 ≥ 2 [3(a + b) − 4c]2 . (1) 136 3. PHƯƠNG PHÁP DỒN BIẾN
Kết hợp với c = max{a,b,c} và bất đẳng thức Minkowski, ta có √ √ √ √ 3a2 + 4bc + 9 + 3b2 + 4ca + 9 ≥ 3a2 + 4b2 + 9 + 3b2 + 4a2 + 9 r √ r √ 2 2 = 3a + (2b)2 + 32 + 3b + (2a)2 + 32
≥ p3(a + b)2 + 4(a + b)2 + 36 p = 7(a + b)2 + 36. Từ 1 ≤ c ≤ 3, ta có
[3(a + b) − 4c]2 ≤ [3(a + b) − 4]2 = 9(a + b)2 − 24(a + b) + 16.
Kết hợp với (1), ta chỉ cần chứng minh
7(a + b)2 + 36 ≥ 2 9(a + b)2 − 24(a + b) + 16 . Tương đương với
11(a + b) [3 − (a + b)] + 15(a + b) + 4 ≥ 0.
Bất đẳng thức cuối là đúng, bởi vì 0 ≤ a + b ≤ 2. Bước 1 được giải quyết hoàn toàn.
Bước 2. Để hoàn thành bước dồn biến, ta sẽ chứng minh r 5(a − b)2 √ 3(a + b)2 + 8(a + b)c + 36 + + 3c2 + 4ab + 9 2 ≥ p p 3(a + b)2 + 8(a + b)c + 36 + 3c2 + (a + b)2 + 9.
Bất đẳng thức trên tương đương với r 5(a − b)2 3(a + b)2 + 8(a + b)c + 36 +
− p3(a + b)2 + 8(a + b)c + 36 2 √ ≥ p3c2 + (a + b)2 + 9 − 3c2 + 4ab + 9.
Nhân liên hợp và phân tích thành tổng bình phương, ta đưa về chứng minh √ p 5
3c2 + (a + b)2 + 9 + 5 3c2 + 4ab + 9 r 5(a − b)2 ≥ p 2 3(a + b)2 + 8(a + b)c + 36 + + 2 3(a + b)2 + 8(a + b)c + 36. (2) 2 (i) Ta sẽ chứng minh r p 5(a − b)2 5 3c2 + (a + b)2 + 9 ≥ 2 3(a + b)2 + 8(a + b)c + 36 + . 2
Thật vậy, ta có a ≤ c, b ≤ c và 1 ≤ c ≤ 3. Do đó, ta có r 5(a − b)2 2 3(a + b)2 + 8(a + b)c + 36 + 2 p =
12(a2 + 2ab + b2) + 32(a + b)c + 144 + 10(a2 − 2ab + b2) p =
22(a2 + b2) + 2ab + 32(a + b)c + 144
≤ p22(c2 + c2) + 2c2 + 32(a + b)c + 144 p = 46c2 + 32(a + b)c + 144. 137 3. PHƯƠNG PHÁP DỒN BIẾN
Do đó, ta chỉ cần chứng minh rằng p 5
3c2 + (a + b)2 + 9 ≥ p46c2 + 32(a + b)c + 144.
Bình phương hai vế và rút gọn, ta được
29c2 + 25(a + b)2 + 81 − 32(a + b)c ≥ 0. Từ a + b + c = 3, ta có
29c2 + 25(a + b)2 + 9(a + b + c)2 − 32(a + b)c ≥ 0. hay
10c2 + 34(a + b)2 + 14c [2c − (a + b)] ≥ 0.
Bất đẳng thức cuối là đúng vì
2c − (a + b) = 2c − (3 − c) = 3c − 3 = 3(c − 1) ≥ 0, ∀1 ≤ c ≤ 3.
(ii) Tiếp theo, ta sẽ chứng minh √ p 5 3c2 + 4ab + 9 ≥ 2 3(a + b)2 + 8(a + b)c + 36.
Thật vậy, từ c = max{a,b,c}, ta có
p3(a + b)2 + 8(a + b)c + 36 ≤ p3(c + c)2 + 8(a + b)c + 36 p = 12c2 + 8(a + b)c + 36. Ta chỉ cần chứng minh √ p 5 3c2 + 4ab + 9 ≥ 2 12c2 + 8(a + b)c + 36.
Bình phương hai vế, ta được
25(3c2 + 4ab + 9) ≥ 4 12c2 + 8(a + b)c + 36 .
Chú ý rằng a + b + c = 3 nên
8c2 + 100ab + 9(a + b)2 + 14c [2c − (a + b)] ≥ 0.
Bất đẳng thức cuối là đúng tương tự trường hợp trên.
Do đó, bất đẳng thức (2) được chứng minh. Bước 2 được giải quyết hoàn toàn.
Bước 3. Để hoàn thành lời giải, ta chỉ cần chứng minh a + b a + b p p f , ,c = 3(a + b)2 + 8(a + b)c + 36 + 3c2 + (a + b)2 + 9 ≥ 12. 2 2 Từ a + b + c = 3, ta có p p
3(3 − c)2 + 8(3 − c)c + 36 + 3c2 + (3 − c)2 + 9 ≥ 12. Suy ra √ √ 63 + 6c − 5c2 + 4c2 − 6c + 18 ≥ 12.
Bình phương hai vế và khử căn hai lần, ta đưa về được
(c − 1)2(3 − c)(3c + 7) ≥ 0.
Bất đẳng thức cuối là đúng bởi vì ta có 1 ≤ c ≤ 3. Bước 3 được giải quyết hoàn toàn. Vậy bài
toán 1 được giải quyết hoàn toàn.
Đẳng thức xảy ra khi và chỉ khi a = b = c = 1 hoặc a = 3,b = c = 0 hoặc b = 3,c = a = 0 hoặc c = 3,a = b = 0. 138 3. PHƯƠNG PHÁP DỒN BIẾN
Câu 6.3. Bài toán này ngoài trường hợp dấu bằng xảy ra khi a = b = c = 1. Còn xảy ra tại 3 3 3 a = b =
, c = 0 hoặc các hoán vị của các bộ số , ,0
. Không mất tính tổng quát giả sử 2 2 2
rằng c = min{a,b,c}. Từ đây suy ra 0 ≤ c ≤ 1. Đặt √ √ √ f (a,b,c) = 9 − 6ab + a2 + b2 + 9 − 6bc + b2 + c2 + 9 − 6ca + c2 + a2. Ta sẽ chứng minh rằng a + b a + b f (a,b,c) ≥ f , ,c . 2 2
Bước 1. Ta sẽ chứng minh rằng √ √ r (a − b)2 9 − 6bc + b2 + c2 + 9 − 6ca + c2 + a2 ≥
36 − 12(a + b)c + (a + b)2 + 4c2 − . 5
Bình phương hai vế và biến đổi, ta thu được 6(a − b)2 √ √ 2 ≥ 9 − 6bc + b2 + c2 − 9 − 6ca + c2 + a2 . 5
Nhân liên hợp và phân tích tổng bình phương, ta đưa về chứng minh rằng 6 [6c − (a + b)]2 ≥ √ √ . 5 9 − 6bc + b2 + c2 + 9 − 6ca + c2 + a22 hay √ √ 2 6 9 − 6bc + b2 + c2 + 9 − 6ca + c2 + a2 ≥ 5 [6c − (a + b)]2 . (3)
Từ c = min{a,b,c} và a + b + c = 3, ta chứng minh được rằng
√9 − 6bc + b2 + c2 ≥ pb2 + (c + a)2 + (b − c)2. và
√9 − 6ca + c2 + a2 ≥ pa2 + (b + c)2 + (c − a)2.
Cộng hai đánh giá trên lại và áp dụng thêm bất đẳng thức Minkowski, ta có p p b2 + (c + a)2 + (b − c)2 + a2 + (b + c)2 + (c − a)2
≥ p(a + b)2 + (a + b + 2c)2 + (a − b)2 ≥ p(a + b)2 + (a + b + 2c)2 √ = 18 + 2c2.
Ngoài ra, [6c − (a + b)]2 = [6c − (3 − c)]2 = 49c2 − 42c + 9. Kết hợp với (3), ta đưa về
6(18 + 2c2) ≥ 5(49c2 − 42c + 9).
Bất đẳng thức cuối tương đương với −233c2 + 210c + 63 = (1 − c)(233c + 23) + 40 ≥ 0 đúng vì
c ≤ 1. Bước 1 được giải quyết hoàn toàn.
Bước 2. Để hoàn thành bước dồn biến, ta cần chứng minh √ r (a − b)2 9 − 6ab + a2 + b2 +
36 − 12(a + b)c + (a + b)2 + 4c2 − 5 ≥ p p 9 − (a + b)2 +
36 − 12(a + b)c + (a + b)2 + 4c2. 139 3. PHƯƠNG PHÁP DỒN BIẾN
Thật vậy, bất đẳng thức trên tương đương với
√9 − 6ab + a2 + b2 − p9 − (a + b)2 r (a − b)2
≥ p36 − 12(a + b)c + (a + b)2 + 4c2 −
36 − 12(a + b)c + (a + b)2 + 4c2 − . 5
Tiếp tục nhân liên hợp và phân tích tổng bình phương, ta chỉ cần chứng minh r p (a − b)2 10
36 − 12(a + b)c + (a + b)2 + 4c2 + 10
36 − 12(a + b)c + (a + b)2 + 4c2 − 5 √ ≥ p 9 − 6ab + a2 + b2 + 9 − (a + b)2. (4) (i) Ta sẽ chứng minh √ p 10
36 − 12(a + b)c + (a + b)2 + 4c2 ≥ 9 − 6ab + a2 + b2.
Thật vậy, từ a + b + c = 3 và c = min{a,b,c}, suy ra a ≥ c, b ≥ c, 0 ≤ c ≤ 1 và
a2 + b2 − 6ab = (a + b)2 − 2ab − 6ab = (a + b)2 − 8ab ≤ (a + b)2 − 8c2. Suy ra
√9 − 6ab + a2 + b2 ≤ p9 + (a + b)2 − 8c2.
Do đó, ta chỉ cần chứng minh p 10
36 − 12(a + b)c + (a + b)2 + 4c2 ≥ p9 + (a + b)2 − 8c2. Từ a + b + c = 3, ta có p 10
36 − 12(3 − c)c + (3 − c)2 + 4c2 ≥ p9 + (3 − c)2 − 8c2.
Khai triển và thu gọn, ta được
1707c2 + 4194(1 − c) + 288 ≥ 0.
Bất đẳng thức cuối cùng là đúng vì 0 ≤ c ≤ 1.
(ii) Tiếp theo, ta sẽ chứng minh r (a − b)2 10
36 − 12(a + b)c + (a + b)2 + 4c2 − ≥ p9 − (a + b)2. 5 Ta có: r (a − b)2 p 10
36 − 12(a + b)c + (a + b)2 + 4c2 − =
3600 − 1200(a + b)c + 80(a + b)2 + 400c2 + 80ab. 5 Ta cần chứng minh
p3600 − 1200(a + b)c + 80(a + b)2 + 400c2 + 80ab ≥ p9 − (a + b)2.
Bình phương hai vế và biến đổi, ta thu được
4086(1 − c) + 234 + 1681c2 + 80ab ≥ 0.
Bất đẳng thức cuối là đúng vì 0 ≤ c ≤ 1 và a,b là các số thực dương.
Do đó, bất đẳng thức (4) là được chứng minh. Bước 2 được giải quyết hoàn toàn.
Bước 3. Để hoàn thành lời giải, ta chỉ cần chứng minh a + b a + b √ p p f , ,c = 9 − (a + b)2 +
36 − 12(a + b)c + (a + b)2 + 4c2 ≥ 3 5. 2 2 140 3. PHƯƠNG PHÁP DỒN BIẾN Từ a + b + c = 3, ta có √ p p 9 − (3 − c)2 +
36 − 12(3 − c)c + (3 − c)2 + 4c2 ≥ 3 5. Suy ra √ √ √ 6c − c2 + 17c2 − 42c + 45 ≥ 3 5.
Lại bình phương hai vế và phân tích, ta thu được c(c − 1)2(10 − 3c) ≥ 0.
Bất đẳng thức cuối là đúng vì 0 ≤ c ≤ 1. 3 3
Dấu bằng xảy ra khi và chỉ khi a = b = c = 1 hoặc a = b = , c = 0 hoặc b = 0, c = a = . 2 2
Tương tự bài toán 1 ở trên, đánh giá ở bước 1 chính là chìa khóa để giải quyết bài toán này.
Câu 6.4. Bài toán này ngoài trường hợp dấu bằng xảy ra khi a = b = c = 1, còn xảy ra
tại a = b = 0, c = 3 hoặc các hoán vị của bộ số (0; 0; 3). Không mất tính tổng quát giả sử
c = max{a,b,c}, suy ra a ≤ c, b ≤ c. Từ a + b + c = 3, suy ra 1 ≤ c ≤ 3 kéo theo 0 ≤ a + b ≤ 2.
Bất đẳng thức được viết lại như sau √ p p p 12 + 22ab + 7(a2 + b2) + 12 + 22bc + 7(b2 + c2) +
12 + 22ca + 7(c2 + a2) ≤ 12 3. Ta đặt p p p f (a; b; c) = 12 + 22ab + 7(a2 + b2) + 12 + 22bc + 7(b2 + c2) + 12 + 22ca + 7(c2 + a2). a + b a + b
Ta sẽ chứng minh f (a; b; c) ≤ f ; ; c . 2 2
Bước 1. Ta sẽ chứng minh rằng p p 12 + 22bc + 7(b2 + c2) + 12 + 22ca + 7(c2 + a2)
≤ p48 + 44(a + b)c + 7(a + b)2 + 28c2 + 2(a − b)2.
Bình phương hai vế và biến đổi, ta thu được hp i2 5(a − b)2 ≤
12 + 22bc + 7(b2 + c2) − p12 + 22ca + 7(c2 + a2) . hay hp p i2 5 12 + 22bc + 7(b2 + c2) + 12 + 22ca + 7(c2 + a2) ≤ [7(a + b) + 22c]2.(5)
Áp dụng bất đẳng thức Cauchy-Schwar, ta có hp p i2 12 + 22bc + 7(b2 + c2) + 12 + 22ca + 7(c2 + a2)
≤(1 + 1) 12 + 22bc + 7(b2 + c2) + 12 + 22ca + 7(c2 + a2)
=48 + 44(a + b)c + 28c2 + 14(a2 + b2).
Kết hợp với (5), ta chỉ cần chứng minh
5 48 + 44(a + b)c + 28c2 + 14(a2 + b2) ≤ [7(a + b) + 22c]2 . Tương đương với
(a + b)[88c − 21(a + b)] + 240(c2 − 1) + 104c2 + 140ab ≥ 0. 141 3. PHƯƠNG PHÁP DỒN BIẾN
Bất đẳng thức cuối cùng đúng vì 0 ≤ a + b ≤ 2 và 1 ≤ c ≤ 3. Bước 1 được giải quyết hoàn toàn.
Bước 2. Để hoàn thành bước dồn biến ta cần chứng minh p p 12 + 22ab + 7(a2 + b2) +
48 + 44(a + b)c + 7(a + b)2 + 28c2 + 2(a − b)2 ≤p p 12 + 9(a + b)2 +
48 + 44(a + b)c + 7(a + b)2 + 28c2.
Thật vậy bất đẳng thức trên tương đương với
p48 + 44(a + b)c + 7(a + b)2 + 28c2 + 2(a − b)2 − p48 + 44(a + b)c + 7(a + b)2 + 28c2
≤p12 + 9(a + b)2 − p12 + 22ab + 7(a2 + b2).(6)
Bằng cách nhân liên hợp và phân tích tổng bình phương, ta đưa về chứng minh p p
48 + 44(a + b)c + 7(a + b)2 + 28c2 + 2(a − b)2 +
48 + 44(a + b)c + 7(a + b)2 + 28c2 ≥p p 12 + 9(a + b)2 + 12 + 22ab + 7(a2 + b2).(7) Đầu tiên ta chứng minh
p48 + 44(a + b)c + 7(a + b)2 + 28c2 + 2(a − b)2 ≥ p12 + 9(a + b)2.
Bình phương hai vế và thu gọn ta thu được
36 + 2(a + b)[22c − (a + b)] + 28c2 + 2(a − b)2 ≥ 0.
Bất đẳng thức cuối là đúng vì 0 ≤ a + b ≤ 2 và 1 ≤ c ≤ 3. Tiếp theo ta chứng minh
p48 + 44(a + b)c + 7(a + b)2 + 28c2 ≥ p12 + 22ab + 7(a2 + b2).
Bình phương hai vế và thu gọn ta thu được
36 + 44(a + b)c + 28c2 − 8ab ≥ 0.
Vì c = max{a,b,c} nên 28c2 − 8ab ≥ 0 và bất đẳng thức trên là đúng. Do đó, bất đẳng thức (7)
được chứng minh. Bước 2 được giải quyết hoàn toàn.
Bước 3. Để hoàn thành lời giải ta cần chứng minh a + b a + b √ p p f ; ; c = 12 + 9(a + b)2 +
48 + 44(a + b)c + 7(a + b)2 + 28c2 ≤ 12 3. 2 2
Từ a + b + c = 3 ta được điều phải chứng minh trở thành √ √ √ 93 − 54c + 9c2 + 111 + 90c − 9c2 ≤ 12 3
Bình phương hai vế và thu gọn ta được
(c − 1)2(c − 3)(c − 11) ≥ 0.
Bất đẳng thức cuối cùng đúng do 1 ≤ c ≤ 3. Bước 3 được giải quyết hoàn toàn hay bài toán được chứng minh xong.
Dấu bằng xảy ra khi và chỉ khi a = b = c = 1 hoặc a = 3,b = c = 0 hoặc b = 3,a = c = 0 hoặc c = 3,a = b = 0.
Bước đánh giá ở bước 1 là mấu chốt để ta giải quyết bài toán. 142 3. PHƯƠNG PHÁP DỒN BIẾN
Câu 6.5. Bài toán này ngoài trường hợp dấu bằng xảy ra khi a = b = c = 1 còn xảy ra tại 3 3 3 a = b =
,c = 0 hoặc các hoán vị của bộ số
; ; 0 . Không mất tính tổng quát giả sử 2 2 2
c = min{a,b,c}, khi đó 0 ≤ c ≤ 1.
Đặt f (a; b; c) = p2(a2 + b2) + 21c + p2(b2 + c2) + 21a + p2(c2 + a2) + 21b. Ta sẽ chứng minh rằng a + b a + b f (a; b; c) ≥ f ; ; c . 2 2
Bước 1. Ta sẽ chứng minh p p 2(b2 + c2) + 21a +
2(c2 + a2) + 21b ≥ p2(a + b)2 + 8c2 + 42(a + b) − 2(a − b)2.
Bình phương hai vế và khai triển ta có hp i2 4(a − b)2 ≥
2(b2 + c2) + 21a − p2(c2 + a2) + 21b .
Nhân liên hợp cho bất đẳng thức trên ta đưa về hp p i2 4 2(b2 + c2) + 21a + 2(c2 + a2) + 21b ≥ [21 − 2(a + b)]2.(8)
Áp dụng tính chất 2(x2 + y2) ≥ (x + y)2 và bất đẳng thức Minkowski, ta có p p 2(b2 + c2) + 21a + 2(c2 + a2) + 21b ≥p p (b + c)2 + 21a + (c + a)2 + 21b q √ q √ = (b + c)2 + ( 21a)2 + (c + a)2 + ( 21b)2 q √ √ ≥ (a + b + 2c)2 + ( 21a + 21b)2.
Từ a + b + c = 3 và c = min{a; b; c}, suy ra q √ √ (a + b + 2c)2 + ( 21a + 21b)2 q √ =
(3 − c + 2c)2 + 21(a + b + 2 ab) √
≥p(3 + c)2 + 21(3 − c + 2c) = c2 + 27c + 72.
Ngoài ra [21 − 2(a + b)]2 = [21 − 2(3 − c)]2 = 225 + 60c + 4c2,kết hợp với (8) ta cần có
4(c2 + 27c + 72) ≥ 225 + 60c + 4c2 ⇔ 16c + 21 ≥ 0.
Bất đẳng thức này đúng vì 0 ≤ c ≤ 1. Bước 1 được giải quyết hoàn toàn.
Bước 2. Để hoàn thành bước dồn biến, ta cần chứng minh p p 2(a2 + b2) + 21c +
2(a + b)2 + 8c2 + 42(a + b) − 2(a − b)2 ≥p p (a + b)2 + 21c + 2(a + b)2 + 8c2 + 42(a + b).
Bất đẳng thức tương đương với
p2(a + b)2 + 8c2 + 42(a + b) − p2(a + b)2 + 8c2 + 42(a + b) − 2(a − b)2
≤p2(a2 + b2) + 21c − p(a + b)2 + 21c. 143 3. PHƯƠNG PHÁP DỒN BIẾN
Nhân lượng liên hợp ta được p p 2(a + b)2 + 8c2 + 42(a + b) +
2(a + b)2 + 8c2 + 42(a + b) − 2(a − b)2 ≥ p p 2 2(a2 + b2) + 21c + 2 (a + b)2 + 21c.(9) Đầu tiên ta chứng minh p p
2(a + b)2 + 8c2 + 42(a + b) ≥ 2 2(a2 + b2) + 21c.
Thật vậy ta có c = min{a,b,c} suy ra 2(a2 + b2) = 2(a + b)2 − 4ab ≤ 2(a + b)2 − 4c2. Khi đó ta thu được p p 2 2(a2 + b2) + 21c ≥ 2 2(a + b)2 − 4c2 + 21c. Do đó ta cần chứng minh p p
2(a + b)2 + 8c2 + 42(a + b) ≥ 2 2(a + b)2 − 4c2 + 21c.
Từ a + b + c = 3, bình phương hai vế và biến đổi ta cần chứng minh
2(3 − c)2 + 8c2 + 42(3 − c) ≥ 8(3 − c)2 − 16c2 + 84c ⇔ (1 − c)(4 − c) ≥ 0.
Bất đẳng thức này đúng do 0 ≤ c ≤ 1.
Tiếp theo ta sẽ chứng minh p p
2(a + b)2 + 8c2 + 42(a + b) − 2(a − b)2 ≥ 2 (a + b)2 + 21c.
Để ý rằng p2(a + b)2 + 8c2 + 42(a + b) − 2(a − b)2 = p8c2 + 42(a + b) + 8ab nên ta cần chứng minh p p 8c2 + 42(a + b) + 8ab ≥ 2 (a + b)2 + 21c.
Từ a + b + c = 3 nên điều này tương đương với
8c2 + 42(3 − c) + 8ab ≥ 4[(3 − c)2 + 21c].
Từ c = min{a,b,c} ta có 8ab ≥ 8c2, do đó ta chỉ cần chứng minh
8c2 + 42(3 − c) + 8c2 ≥ 4[(3 − c)2 + 21c] ⇔ (1 − c)(15 − 2c) ≥ 0.
Bất đẳng thức cuối cùng đúng do 0 ≤ c ≤ 1. Đến đây thì bất đẳng thức (9) đã được chứng minh
hay bước 2 được giải quyết hoàn toàn.
Bước 3. Ta chỉ cần chứng minh a + b a + b p p f ; ; c = (a + b)2 + 21c +
2(a + b)2 + 8c2 + 42(a + b) ≥ 15. 2 2
Từ a + b + c = 3 thì ta chỉ cần chứng minh p p (3 − c)2 + 21c +
2(3 − c)2 + 8c2 + 42(3 − c) ≥ 15.
Bình phương khử căn ta đưa về c(c − 1)2(40 − 3c) ≥ 0.
Bất đẳng thức cuối đúng vì 0 ≤ c ≤ 1. Bước 3 được giải quyết hoàn toàn. Vậy bài toán đã được giải quyết. 3 3
Dấu bằng xảy ra khi và chỉ khi a = b = c = 1 hoặc a = b = ,c = 0 hoặc b = c = ,a = 0 hoặc 2 2 3 a = c = ,b = 0. 2 144 3. PHƯƠNG PHÁP DỒN BIẾN
Câu 6.6. Bài toán này ngoài trường hợp dấu bằng xảy ra a = b = c = 1. Còn xảy ra tại a =
b = 3 ,c = 0 hoặc các hoán vị bộ 3 , 3 ,0. Không mất tính tổng quát giả sử rằng c = min {a,b,c}. 2 2 2
Từ đây suy ra 0 ≤ c ≤ 1. √ √ √ Đặt f (a,b,c) = 3a2 − a + 1 + 3b2 − b + 1 +
3c2 − c + 1 − p6(a2 + b2 + c2) + 9.
Bước 1. Ta dễ chứng minh được √ √ √ 3a2 − a + 1 + 3b2 − b + 1 +
3c2 − c + 1 ≤ p6(a2 + b2 − 2(a + b) + 4 − (a + b)2.
Bước 2. Để hoàn thành bước dồn biến ta cần chứng minh
p6(a2 + b2 − 2(a + b) + 4 − (a + b)2 − p6 (a2 + b2 + c2) + 9 ≤ ≤ p p 3(a + b)2 − 2(a + b) + 4 + 3(a + b)2 + 6c2 + 9.
Nhân liên hợp và phân tích tổng bình phương, ta đưa về q q 3
6 (a2 + b2) − 2 (a + b) + 4 − (a − b)2 +
3(a + b)2 − 2 (a + b) + 4 ≥ q ≥ p 2 6 (a2 + b2 + c2) + 9 + 3(a + b)2 + 6c2 + 9 .
• Ta dễ chứng minh được q q 3
3(a + b)2 − 2(a + b) + 4 ≥ 2 3(a + b)2 + 6c2 + 9.
• Ta dễ chứng minh được q p 3
6 (a2 + b2) − 2(a + b) + 4 − (a − b)2 ≥ 2 6 (a2 + b2 + c2) + 9.
Bước 3. Ta chỉ cần chứng minh q √ q 3(a + b)2 − 2 (a + b) + 4 + 3c2 − c + 1 ≤ 3(a + b)2 + 6c2 + 9. Từ a + b + c = 3 ta suy ra √ √ 3c2 − 16c + 25 +
3c2 − c + 1 ≤ p9 (c2 − 2c + 4).
Bình phương hai vế và rút gọn, ta được 9c(c − 1)2(16 − 3c) ≥ 0.
Bất đẳng thức cuối đúng vì 0 ≤ c ≤ 1. Bước 3 được giải quyết hoàn toàn. Vậy bài toán được giải quyết.
Đẳng thức xảy ra khi a = b = c = 1 hoặc a = b = 3 ,c = 0, hoặc a = 0,b = c = 3 , hoặc 2 2 b = 0,c = a = 3 . 2
Câu 6.7. Bài toán này ngoài trường hợp dấu bằng xảy ra a = b = c = 2 . Còn xảy ra tại 3
a = b = 1 hoặc các hoán vị bộ (1,1,0). Không mất tính tổng quát giả sử rằng c = min {a,b,c}.
Từ đây suy ra 0 ≤ c ≤ 2 . 3 √ √ √ Đặt f (a,b,c) = a + b − 2ab + b + c − 2bc + c + a − 2ca.
Bước 1. Ta chứng minh được √ √ r 1 b + c − 2bc + c + a − 2ca ≥
2(a + b) + 4c − 4(a + b)c − (a − b)2. 2 145 3. PHƯƠNG PHÁP DỒN BIẾN
Bước 2. Để hoàn thành bước dồn biến, ta cần chứng minh √ r 1 a + b − 2ab +
2(a + b) + 4c − 4(a + b)c − (a − b)2 ≥ 2 r (a + b)2 ≥ p a + b + + 2(a + b) + 4c − 4c(a + b). 2
Nhân lượng liên hợp, ta được r p 1 2(a + b) + 4c − 4c(a + b) +
2(a + b) + 4c − 4(a + b)c − (a − b)2 ≥ 2 √ r (a + b)2 a + b − 2ab + a + b + . 2 (i) Ta chứng minh được √
p2(a + b) + 4c − 4c(a + b) ≥ a + b − 2ab. (ii) Ta chứng minh được r r 1 (a + b)2
2(a + b) + 4c − 4c(a + b) − (a − b)2 ≥ a + b − . 2 2
Bước 3. Ta chỉ cần chứng minh r 2c − c2 √ + 4 − 6c + 4c2 ≥ 2. 2
Bình phương hai vế và rút gọn, ta được c(3c − 2)2(16 − 9c) ≥ 0.
Bất đẳng thức cuối đúng vì 0 ≤ c ≤ 2 . Bước 3 được giải quyết hoàn toàn. Vậy bài toán đã cho 3 được giải quyết. 2
Đẳng thức xảy ra khi và chỉ khi a = b = c =
, hoặc a = b = 1,c = 0, hoặc a = 0,b = c = 1, hoặc 3 b = 0,c = a = 1.
Câu 6.8. Bài toán ngày ngoài trường hợp dấu bằng xảy ra khi a = b = c = 1. Còn xảy ra tại 2 5 2 2 5 a = b = ,c =
, hoặc các hoán vị của bộ số , ,
. Không mất tính tổng quát giả sử rằng 3 3 3 3 3
c = min {a,b,c}. Từ đây suy ra 1 ≤ c ≤ 3. Đặt √ √ √ 1 p f (a,b,c) = 2a2 − a + 1 + 2b2 − b + 1 + 2c2 − c + 1 − 21(a2 + b2 + c2) + 99. 3
Bước 1. Dễ dàng chứng minh được √ √ r 7 2a2 − a + 1 + 2b2 − b + 1+ ≥ 2(a + b)2 − 2(a + b) + 4 + (a − b)2. 16
Bước 2. Để hoàn thành bước dồn biến, ta cần chứng minh r 7 1 p 2(a + b)2 − 2(a + b) + 4 + (a − b)2 − 21(a2 + b2 + c2 + 99 ≥ 16 3 146 3. PHƯƠNG PHÁP DỒN BIẾN r 1 21
≥ p2(a + b)2 − 2(a + b) + 4 − 21c2 + (a + b)2 + 99. 3 2
Nhân lượng liên hợp và biến đổi ta đưa về r p 21 21(a2 + b2 + c2) + 99 + 21c2 + (a + b)2 + 99 ≥ 2 r 7 ≥ p 8 2(a + b)2 − 2(a + b) + 4 + (a − b)2 + 8 2(a + b)2 − 2(a + b) + 4. 16
Ta dễ dàng chứng minh được các kết quả sau: r 21 p 21c2 + (a + b)2 + 99 ≥ 8 2(a + b)2 − 2(a + b) + 4. 2 và r p 7 21(a2 + b2 + c2 + 99 ≥ 8 2(a + b)2 − 2(a + b) + 4 + (a − b)2. 16
Bước 3. Ta chỉ cần chứng minh √ r p 1 21 2(a + b)2 − 2(a + b) + 4 + 2c2 − c + 1 − 21c2 + (a + b)2 + 99 ≥ 0. 3 2 Từ a + b + c = 3, ta suy ra √ √ r 7c2 − 14c + 43 2c2 − 10c + 16 + 2c2 − c + 1 ≥ . 2
Bình phương hai vế và rút gọn, ta được 7(3c − 5)2(c − 1)2 ≥ 0.
Bất đẳng thức cuối cùng là đúng. Bước 3 giải quyết hoàn toàn và cũng kết thúc bài toán.
Đẳng thức xảy ra khi và chỉ khi a = b = c = 1 hoặc a = b = 2 ,c = 5 hoặc a = 5 ,bc = 2 hoặc 3 3 3 3 b = 5 ,c = a = 2 . 3 3
Câu 6.15. Chuẩn hóa ab + bc + ca = 3. Bất đẳng thức cần chứng minh trở thành 1 1 1 3 + + ≥ . (a + b)2 (b + c)2 (c + a)2 4 1 1 1 Đặt f (a, b, c) = + +
và t là số thực dương thỏa mãn 2at + t2 = 3. Xét (a + b)2 (b + c)2 (c + a)2 1 1 2 1 1
P = f (a, b, c) − f (a,t,t) = + − + − . (a + b)2 (a + c)2 (a + t)2 (b + c)2 4t2 Ta có
2at + t2 = ab + bc + ca ⇔ a(b + c − 2t) = t2 − bc. (1) a + b 2
Nếu b + c < 2t thì V T (1) < 0 và t2 >
≥ ab nên V P (1) ≥ 0, dẫn đến mâu thuẫn với 2 1
(1). Từ đó, suy ra b + c ≥ 2t ⇒ (b + c)2 ≥ 4t2 ⇒ (a + b)2 147 4. PHƯƠNG PHÁP P, Q, R §4. Phương pháp p, q, r
Câu 1.1. Bất đẳng thức cần chứng minh tương đương với 3 6 6p 1 + ≥
⇔ pq + 3q − 6p ≥ 0 ⇔ q ≥ . (1) p q p + 3 √
Ta có p2 ≥ 3pr = 3p, nên q ≥
3p. Do đó, để chứng minh (1) ta chứng minh 6p p3p ≥
⇔ p(p + 3)2 ≥ 12p2 ⇔ (p − 3)2 ≥ 0 (đúng). p + 3
Câu 1.2. Ta có q + 6r = 9 và bất đẳng thức cần chứng minh trở thành p + 3r ≥ 6 ⇔ 2p ≥ q + 3. (2) Theo BĐT Schur ta có 9 − q 27 + 2p3 9r ≥ 4pq − p3 ⇔ 9 · ≥ 4pq − p3 ⇔ q ≤ . 6 8p + 3
Do đó để chứng minh (2) ta chứng minh 27 + 2p3 2p ≥
+ 3 ⇔ p3 − 2p2 + 9p + 18 ≤ 0 ⇔ (p + 1)(p − 3)(p − 6) ≤ 0. (3) 8p + 3 p2 6p3 Ta có 9 = q + 6r ≤ +
⇔ p ≥ 3 nên (3) đúng khi p ≤ 6. Nếu p > 6 thì a + b + c + 3abc > 6. 3 27 a + b + c 3 64
Câu 1.3. Nếu a + b + c > 4 thì ta có a3 + b3 + c3 ≥ 3 > Áp dụng BĐT Schur, 3 9 ta có:
a3 + b3 + c3 + 3abc ≥ ab(a + b) + bc(b + c) + ca(c + a)
⇔ a3 + b3 + c3 + 6abc ≥ (ab + bc + ca)(a + b + c) = pq = 3p và p(4q − p2) p(12 − p2) r ≥ = . 9 9 Ta cần chứng minh: p(12 − p2) 3p + ≥ 10 9
(p − 3)[(16 − p2) + 3(4 − p) + 2] ⇔ ≥ 0. 9
Bất đẳng thức cuối hiển nhiên đúng nên ta có đpcm.
Đẳng thức xảy ra khi a = b = c = 1.
Câu 1.4. Bât đẳng thức cần chứng minh được viết lại như sau: 3r + 12 ≥ 5q. (1)
Mặt khác,theo BDT Schur,ta có: 3p(4q − p2) 3r ≥ = 4q − 9. 9 Nên ta chứng minh
4q − 9 + 12 ≥ 5q ⇔ q ≤ 3 (đúng). 148 4. PHƯƠNG PHÁP P, Q, R
Câu 1.5. Bất đẳng thức cần chứng minh tương đương với 8p + 3r ≥ 12 + 5q. (1) Áp dụng BDT Schur,ta có: p(4q − p2) p(2q − 3) 3r ≥ = . 3 3 p2 − 3
Từ giả thiết p2 − 2q = 3 ⇒ q = . 2
Nên ta chỉ cần chứng minh: p(p2 − 6) 5(p2 − 3) 8p + ≥ 12 +
⇔ (2p − 3)(p − 3)2 ≥ 0. 3 2
Bất đẳng thức cuối đúng nên ta có đpcm.
Câu 1.6. Biến đổi bất đẳng thức cần chứng minh và chuyển về dạng p,q,r,ta có:
8(243 − 18p + 3r) ≤ 3(729 − 81q + 27r − r2) ⇔ 243 − 99q + 57r − 3r2 ≥ 0. a + b + c Theo BDT AM-GM thì 3 = 3( )6 ≥ 3(abc)2 = r2. 3 Theo BDT Schur,ta có: p(4q − p2) 4q − 9 r ≥ = ⇒ 57r ≥ 19(4q − 9). 3 3 Nên ta cần chứng minh:
72 − 23q − 3r2 ≥ 0 ⇔ 3(1 − r2) + 23(3 − q) ≥ 0.
Vậy BDT được chứng minh.
Câu 1.7. Đưa bất đẳng thức về dạng p,q,r,từ giả thiết,ta có q + r = 4.
và lúc đó,bất đẳng thức trở thành
p2 − 2q + 5r ≥ 8 ⇔ p2 − 7q + 12 ≥ 0.
Nếu 4 ≥ p,sử dụng BDT Schur,ta có: p(4q − p2) r ≥ , 9 suy ra p(4q − p2) p3 + 36 4 ≥ q + ⇔ q ≤ 9 4p + 9 7(p3 + 36) ⇒p2 −
+ 12 ≥ 0 ⇔ (p − 3)(p2 − 16) ≤ 0. 4p + 9 √
điều này đúng vì 4 ≥ p ≥ 3q ≥ 3.
Nếu p ≥ 4, ta có p2 ≥ 16 ≥ 4q và p2
p2 − 2q + 5r ≥ p2 − 2q ≥ ≥ 8. 2
Vậy BDT được chứng minh.
Đẳng thức xảy ra khi x = y = z = 1 hoặc x = y = 2,z = 0 và các hoán vị 149
5. PHƯƠNG PHÁP TIẾP TUYẾN VÀ CÁT TUYẾN
§5. Phương pháp tiếp tuyến và cát tuyến
Câu 2.1. Vì BĐT đã cho thuần nhất nên ta chỉ cần chứng minh Bđt đúng với mọi số thực dương
a,b,c thỏa mãn a2 + b2 + c2 = 1, khi đó bđt cần chứng minh trở thành: f (a) + f (b) + f (c) ≥ 1trong √ 1 + 3 1 đó: f (x) = √ .
− x với 0 < x < 1. Dễ thấy hàm số f (x) có f 00(x) > 0 ∀x ∈ (0; 1). 3 3 x
Theo BĐT tiếp tuyến ta có : 1 √ 1 f (a) + f (b) + f (c) ≥ f 0 √ (a + b + c − 3) + 3f √ . 3 3  1  f 0 √ < 0 1 Do 3
⇒ f (a) + f (b) + f (c) ≥ 3f √ = 1. √ 3 
a + b + c ≤ p3(a2 + b2 + c2) = 3 n Câu 2.2. Đặt a P
i = tan xi (i = 1,2, . . . ,n) ⇒ ai > 0 i = 1,2, . . . ,n và ai ≤ n. i=1 Ta cần chứng minh : n Y ai 1 ≤ √ . p1 + a2 2n i=1 i x Xét hàm số f (x) = √ , x > 0 có 1 + x2 1 f 0(x) =
⇒ f 00(x) < 0 ∀x > 0. q (1 + x2)3 Do đó 1 1 1
f (x) ≤ f 0(1)(x − 1) + f (1) = √ (x − 1) + √ = √ (x + 1). 23 2 2 2 Suy ra  n n P n n n (ai + 1) Y ai Y 1 Y 1 2n 1 = f (a √ (a √  i=1  ≤ √ = √ . p i) ≤ i + 1) ≤   1 + a2 8n 8n  n  8n 2n i=1 i i=1 i=1
Đẳng thức xảy ra khi a1 = a2 = · · · = an = 1 hay tan x1 = tan x2 = · · · = tan xn = 1 ⇔ x1 = π x2 = · · · = xn = . 4 Câu 2.3. Ta có : b + c 1 − a a + c 1 − b b + a 1 − c bc ≤ ( )2 = ( )2; ca ≤ ( )2 = ( )2; ab ≤ ( )2 = ( )2 2 2 2 2 2 2 nên a b c 4a 4b 4c + + ≥ + + = f (a) + f (b) + f (c). 1 + bc 1 + ac 1 + ab a2 − 2a + 5 b2 − 2b + 5 c2 − 2c + 5 1 4x
Ta thấy đẳng thức xảy ra khi a = b = c =
và tiếp tuyến của đồ thị hàm số f (x) = 3 x2 − 2x + 5 1 99x − 3
tại điểm có hoành độ x = là :y = , nên ta xét 3 100 4x 99x − 3 (3x − 1)2(15 − 11x) − = ≥ 0 ∀x ∈ (0; 1). x2 − 2x + 5 100 100(x2 − 2x + 5) 150
5. PHƯƠNG PHÁP TIẾP TUYẾN VÀ CÁT TUYẾN Suy ra 4a 4b 4c 99(a + b + c) − 9 9 + + ≥ = . a2 − 2a + 5 b2 − 2b + 5 c2 − 2c + 5 100 10
Bài toán được chứng minh.
Câu 2.4. Vì Bđt cần chứng minh là thuần nhất nên ta chỉ cần chứng minh Bđt đúng với mọi
số thực dương a,b,c thỏa mãn a + b + c = 1. Khi đó Bđt đã cho trở thành: (1 − 2a)2 (1 − 2b)2 (1 − 2c)2 3 + + ≥ (1 − a)2 + a2 (1 − b)2 + b2 (1 − c)2 + c2 5 4a2 − 4a + 1 4b2 − 4b + 1 4c2 − 4c + 1 3 ⇔ + + ≥ 2a2 − 2a + 1 2b2 − 2b + 1 2c2 − 2c + 1 5 1 1 1 27 ⇔ + + ≤ 2a2 − 2a + 1 2b2 − 2b + 1 2c2 − 2c + 1 5 27 ⇔ f (a) + f (b) + f (c) ≤ . 5 1 Trong đó f (x) = với x ∈ (0; 1). 2x2 − 2x + 1 1 54x + 27
Tiếp tuyến của đồ thị hàm số y = f (x) tại điểm có hoành độ x = là y = . 3 25 Ta có 54x + 27 2(54x3 − 27x2 + 1) 2(3x − 1)2(6x + 1) − f (x) = = ≥ 0 ∀x ∈ (0; 1). 25 25(2x2 − 2x + 1) 25(2x2 − 2x + 1) Suy ra 54(a + b + c) + 81 27 f (a) + f (b) + f (c) ≤ = . (đpcm). 25 5 1
Câu 2.5. Phân tích. Với điều kiện a + b + c = 1 ta chọn điểm rơi tại a = b = c = = x0 và 3 1 11 1 16
đặt f (a) = a3 + 5a2. Khi đó f 0(a) = 3a2 + 10a, f 0 = , f = . 3 3 3 27 Ta chứng minh 11 1 16 a3 + 5a2 ≥ a − + 3 3 27 11 17 ⇔ a3 + 5a2 − a + ≥ 0 3 27 1 2 17 ⇔ a − a +
≥ 0 (BĐT cuối luôn đúng với điều kiện a > −1). 3 3
Chứng minh tương tự với các biểu thức còn lại: 11 1 16 b3 + 5b2 ≥ b − + , 3 3 27 11 1 16 c3 + 5c2 ≥ c − + . 3 3 27
Cộng vế với vế, ta được 11 16 16 S ≥ (a + b + c − 1) + = . 3 9 9 1 16 Khi a = b = c = thì S = . 3 9 16 1 Vậy min S = tại a = b = c = . 9 3 151
5. PHƯƠNG PHÁP TIẾP TUYẾN VÀ CÁT TUYẾN a
Câu 2.6. Phân tích. Chọn điểm rơi a = b = c = −2 và đặt f (a) = . Tính được a2 + a + 1 1 2
f 0(−2) = − , f (−2) = − . Ghép vào công thức (∗∗). 3 3 Ta sẽ chứng minh a 1 2 ≤ − (a + 2) − a2 + a + 1 3 3
⇔ −a3 − 5a2 − 8a − 4 ≥ 0
⇔ (a + 2)2(a + 1) ≤ 0 (luôn đúng với a < −1).
Chứng minh tương tự với các biến b, c, ta có b 1 2 ≤ − (b + 2) − , b2 + b + 1 3 3 c 1 2 ≤ − (c + 2) − . c2 + c + 1 3 3 1
Cộng vế với vế được S ≤ − (a + b + c) − 4 = 2 − 4 = −2. 3
Với a = b = c = −2 thì S = −2.
Vậy max S = −2 tại a = b = c = −2.
Câu 2.7. Phân tích. Lần này 3 biến không còn đối xứng (chỉ là hoán vị vòng quanh) và cũng
không nhóm dạng phân ly biến được. Ta sẽ nhóm riêng (a3 + 2a2b) để đánh giá và coi b là tham số
(hoặc có thể nhóm a3 + 2c2a). Từ điều kiện ta có thể nhẩm ra dấu bằng xảy ra khi a = b = c = 1
và đặt f (a) = a3 + 2a2b ⇒ f 0(a) = 3a2 + 4ab, f 0(1) = 3 + 4b, f (1) = 1 + 2b. Ta có sẽ chứng minh
a3 + 2a2b ≥ (3 + 4b)(a − 1) + (1 + 2b)
⇔ (a3 − 1) + (2a2b − 2b) − (3 + 4b)(a − 1) ≥ 0
⇔ (a − 1)2(a + 2b + 2) ≥ 0 (BĐT cuối luôn đúng với điều kiện ban đầu).
Chứng minh tương tự, ta có
b3 + 2b2c ≥ (3 + 4c)(b − 1) + (1 + 2c),
c3 + 2c2a ≥ (3 + 4a)(c − 1) + (1 + 2a).
Cộng vế với vế được S ≥ 4(ab + bc + ca) + a + b + c − 6 ≥ 15 − 6 = 9.
Với a = b = c = 1 thì S = 9.
Vậy min S = 9 tại a = b = c = 1.
Câu 2.8. Phân tích. Từ điều kiện đề bài nhẩm ra a = b = c = 1. Đặt f (a) = 2a3 − 3a2b suy
ra f 0(1) = 6 − 6b, f (1) = 2 − 3b. Ta có sẽ chứng minh
2a3 − 3a2b ≥ (6 − 6b)(a − 1) + 2 − 3b
⇔ (a − 1)2(2a − 3b + 4) ≥ 0 (BĐT cuối luôn đúng bởi điều kiện xác định).
Chứng minh tương tự, ta có
2b3 − 3b2c ≥ (6 − 6c)(b − 1) + 2 − 3c,
2c3 − 3c2a ≥ (6 − 6a)(c − 1) + 2 − 3a.
Cộng vế với vế ta được S ≥ 9(a + b + c) − 6(ab + bc + ac) − 12 ≥ 3 · 3 − 12 = −3.
Với a = b = c = 1 thì S = −3.
Vậy min S = −3 tại a = b = c = 1. 152
5. PHƯƠNG PHÁP TIẾP TUYẾN VÀ CÁT TUYẾN
Câu 2.9. Phân tích. Đặt f (a) = 5a3 + 2a2b, tính được f 0(1) = 15 + 4b, f (1) = 5 + 2b. Ta có
5a3 + 2a2b ≥ (15 + 4b)(a − 1) + 5 + 2b
⇔ 5(a − 1)(a2 + a + 1) + 2b(a − 1)(a + 1) − (15 + 4b)(a − 1) ≥ 0
⇔ (a − 1)[5a2 + 5a − 10 + 2b(a − 1)] ≥ 0
⇔ (a − 1)2(5a + 2b + 10) ≥ 0 (BĐT cuối luôn đúng vì điều kiện xác định).
Chứng minh tương tự rồi cộng vế với vế được S ≥ 13(a + b + c) + 4(ab + bc + ca) − 30.
Lại có a + b + c ≥ p3(ab + bc + ca) = 3.
Suy ra S ≥ 13 · 3 + 4 · 3 − 30 = 21.
Với a = b = c = 1 thì S = 21.
Vậy min S = 21 tại a = b = c = 1. 1 b
Câu 2.10. Phân tích. Đặt f (a) =
− , tính được f 0(1) = −2 + b, f (1) = 1 − b. a2 a Ta có 1 b −
≥ (−2 + b)(a − 1) + 1 − b a2 a 1 + a a − 1 ⇔ (1 − a) + b + (−2 + b)(1 − a) ≥ 0 a2 a 1 + a b ⇔ (1 − a) − 2 + b − ≥ 0 a2 a 2a + 1 b ⇔ (1 − a)2 −
≥ 0 (BĐT cuối luôn đúng vì điều kiện xác định). a2 a
Chứng minh tương tự rồi cộng vế với vế được T ≥ −4(a + b + c) + ab + bc + ca + 9 = 0.
Với a = b = c = 1 thì T = 0.
Vậy min T = 0 tại a = b = c = 1.
Câu 2.11. Phân tích. Đặt f (a) = a5 + 3a4b, tính được f 0(1) = 5 + 12b, f (1) = 1 + 3b. Ta có
a5 + 3a4b ≥ (5 + 12b)(a − 1) + 1 + 3b
⇔ (a − 1)(a4 + a3 + a2 + a + 1) + 3b(a − 1)(a3 + a2 + a + 1) − (5 + 12b)(a − 1) ≥ 0
⇔ (a − 1)[3b(a3 + a2 + a − 3) + a4 + a3 + a2 + a − 4] ≥ 0
⇔ (a − 1)2[3b(a2 + 2a + 3) + a3 + 2a2 + 3a + 4] ≥ 0 (BĐT cuối luôn đúng vì điều kiện xác định).
Chứng minh tương tự rồi cộng vế với vế được A ≥ 12(ab+bc+ca)−4(a+b+c)−12 ≥ 24−12 = 12.
Với a = b = c = 1 thì A = 12.
Vậy min A = 12 tại a = b = c = 1. √ 3
Câu 2.12. Phân tích. Đặt f (a) = a2 + b 3a + 1, tính được f 0(1) = 2 + b, f (1) = 1 + 2b. 4 Ghép vào công thức. Ta có √ 3 a2 + b 3a + 1 ≥ 2 + b (a − 1) + 1 + 2b 4 3b 3 ⇔ (a − 1) a + 1 + √ − 2 − b ≥ 0 3a + 1 + 2 4 9b ⇔ (a − 1)2 1 − √
≥ 0 (BĐT cuối luôn đúng vì điều kiện xác định). 4( 3a + 1 + 2)2 153
5. PHƯƠNG PHÁP TIẾP TUYẾN VÀ CÁT TUYẾN 3 13
Chứng minh tương tự rồi cộng vế với vế được B ≥ (ab + bc + ca) + (a + b + c) − 3 ≥ 9. 4 4
Với a = b = c = 1 thì B = 9.
Vậy min B = 9 tại a = b = c = 1. π
Câu 2.13. Xét hàm số f (x) = tan x, x ∈ 0;
, có f 0(x) = 1 + tan2 x, suy ra f 00(x) = 2 π
2 tan x(1 + tan2 x) > 0, ∀x ∈ 0; . 2
Áp dụng BĐT tiếp tuyến với ∆M N P nhọn, ta có : 1
f (A) ≥ f 0(M )(A − M ) + f (M ) = (A − M ) + tan M, cos2M suy ra 1 cos2 M.f (A) ≥ sin 2M + A − M. 2 Tương tự : 1 1 cos2 N.f (B) ≥
sin 2N + B − N ; cos2 P.f (C) ≥ sin 2P + C − P. 2 2 Suy ra sin 2M + sin 2N + sin 2P
cos2 M.f (A) + cos2 N.f (B) + cos2 P.f (C) ≥ . 2
Ta chọn các góc M, N, P sao cho √ √ cos M = k > 0; cos N = 2k; cos P = 3k.
Vì M, N, P là ba góc của tam giác nên ta có đẳng thức :
cos2 M + cos2 N + cos2 P + 2 cos M. cos N. cos P = 1, √ √ √ nên ta có (1 + 2 +
3)k + 2 6k3 = 1, do đó k là nghiệm dương của phương trình : √ √ √ 2 6x3 + (1 + 2 + 3)x − 1 = 0. (1) Suy ra √ √
sin 2M = 2 1 − cos2M · cos M = 2k 1 − k2;
sin 2N = 2kp2(1 − 2k2); sin 2P = 2kp3(1 − 3k2). Dẫn tới √ sin 2M + sin 2N + sin 2P
1 − k2 + p2(1 − 2k2) + p3(1 − 3k2) F ≥ = . 2k2 k
√1 − k2 + p2(1 − 2k2) + p3(1 − 3k2) Vậy min F =
đạt được khi A = M ; B = N ; C = P với k
M, N, P là ba góc của tam giác nhọn được xác định bởi √ √ cos M = k > 0; cos N = 2k; cos P = 3k,
trong đó k là nghiệm dương duy nhất của (1). 154
5. PHƯƠNG PHÁP TIẾP TUYẾN VÀ CÁT TUYẾN √ √
Câu 2.14. Ta có các hàm số f (t) = t3; g(t) =
1 + t2; h(t) = 4 1 + t4, t ∈ (0; 1) là những
hàm số có đạo hàm cấp hai dương trên khoảng (0; 1). Nên với a,b,c > 0 thỏa a + b + c = 1 áp
dụng BĐT tiếp tuyến, ta có:
f (x) ≥ f 0(a)(x − a) + f (a), h(y) ≥ h0(b)(y − b) + h(b), g(z) ≥ g0(c)(z − c) + g(c). Ta chọn a,b,c sao cho  r 3a2 = k k   a =     3  b     √ = k    k f 0(a) = g0(b) = h0(c) = k ⇔ 1 + b2 ⇔ b = √ (1) 1 − k2  c3  √    = k  3   k  q   4   (1 + c4)3 c = √  4 p1 − k 3 k Do a + b + c = 1 nên ta có √ r k k 3 k + √ + √ = 1. (2) 3 1 − k2 4 p1 − k 3 k
Dễ thấy phương trình (2) luôn có nghiệm trong khoảng (0; 1). Suy ra √ k 3k 1 1
P = f (x) + g(y) + h(z) ≥ f (a) + h(b) + g(c) = + √ + √ . 9 1 − k2 4 p1 − k 3 k
Đẳng thức xảy ra ⇔ x = a; y = b; z = c. √ k 3k 1 1 Vậy min P = + √ + √
với k là nghiệm nằm trong (0; 1) của (2). 9 1 − k2 4 p1 − k 3 k 155 Chương 3 Một số chuyên đề
§1. Ứng dụng đều kiện có nghiệm của phương trình bậc ba
Câu 1.1. Đặt n = ab + bc + ca, p = −abc Suy ra a, b, c là ba nghiệm của phương trình : x3 − mx + n = 0 (4) Ta có: 4 27 p2 ≤ − n3 ⇒ n3 ≤ − p2. 27 4 Do đó: 27 1 2
13p2 + 2p − 2 + 2n3 ≤ 13p2 + 2p − 2 − p2 = − p − 1 ≤ 0. 2 2 Suy ra:
13p2 + 2p − 2 ≤ −2n3 ⇔ 13a2b2c2 − 2ab − 2 ≤ −2 (ab + bc + ca)3 . Mà: 1
(a + b + c)2 = 0 ⇒ ab + bc + ca = − a2 + b2 + c2 , 2 dẫn tới: 1 1 13a2b2c2 − 2abc − 2 ≤ a2 + b2 + c23 ⇒ P ≤ . 4 4 (n = 2 Đẳng thức xảy ra ⇔
⇔ a,b,c là ba nghiệm của phương trình m = 3
x3 − 3x + 2 = 0 ⇔ (x − 1)2(x + 2) = 0 ⇔ x = 1,x = −2. 1 Vậy max P =
đạt được khi (a,b,c) = (1,1, − 2) và các hoán vị. 4
Câu 1.2. Đặt n = ab + bc + ca, p = −abc Ta có: 4 27 27 p2 ≤ − n3 ⇒ −n3 ≥ p2 ⇒ n3 ≥ p2. 27 4 4 Vì
a + b + c = 0 ⇒ a2 + b2 + c2 = −2(ab + bc + ca) = −2n ⇒ n ≤ 0. Do đó:
P = −32n5 − 32np2 − 8 |p| = 32 (−n)5 + (−n)p2 − 8 |p|
≥ 64 n3 |p| − 8 |p| ≥ 8 54|p|3 − |p| . 156
1. ỨNG DỤNG ĐỀU KIỆN CÓ NGHIỆM CỦA PHƯƠNG TRÌNH BẬC BA
Xét hàm số f (t) = 54t3 − t,t ≥ 0 ta có: √2
f 0(t) = 162t2 − 1,f 0(t) = 0 ⇔ t = . 18 √ √ ! 2 2
Lập bảng biến thiên ta có minf (t) = f = − . t≥0 18 27 √  √ 2  8 2 p =  Suy ra P ≥ − . Đẳng thức xảy ra khi 18
hay a,b,c là nghiệm của phương trình 27 1  n = − √  3 24 √ √ √ !2 ! 1 2 1 2 1 2 t3 − √ t + = 0 ⇔ t − √ t + √ = 0 ⇔ t = √ ,t = − √ . 3 24 18 p 3 p 3 6 3 3 9 6 3 3 9 √ √ 8 2 1 2 Vậy min P = − . Đạt được khi a = b =
√ ,c = − √ và các hoán vị. 27 p 3 6 3 3 9
Câu 1.3. Đặt m = −(a + b + c), n = ab + bc + ca, p = −abc. Từ giả thiết ta suy ra: m2
(a + b + c)2 = 4 (ab + bc + ca) ⇔ n = . 4 Suy ra m3 |m3| 27p − ≤ 4 4 ⇔ 108p − m3 ≤ m3
⇔ p(54p − m3) ≤ 0 ⇔ pm3 ≥ 54p2. Do đó: 1 1 1 r 1 P = pm3 + ≥ 54p2 + = 27p2 + 27p2 + ≥ 3 3 27p2.27p2 = 27 (đpcm). p4 p4 p4 p4  1  m2 p = √  n =    3    4    q √ Đẳng thức xảy ra ⇔ 1 3 27p2 = , chẳng hạn ta chọn m =
18 3 hay a,b,c là nghiệm của  p4  √     3   972 54p = m3  n =  4 phương trình: √ √ √ !2 ! q √ 3 3 p 3 3 972 1 18 3 4 t3 + 18 3t2 + t + √ = 0 ⇔ t + t + √ = 0. 4 3 6 3 √ √ 3 p18 3 3 4
Vậy min P = 27 đạt được khi a = b = − ,c = √ và các hoán vị. 6 3
Câu 1.4. Đặt m = −(a + b + c), n = ab + bc + ca, p = −abc. Từ giả thiết ta suy ra:
(a + b + c)2 = 3 (ab + bc + ca) + 4 ⇒ m2 = 3n + 4. Mặt khác : q 27p + 2m3 − 9mn ≤ 2 (m2 − 3n)3. 157
1. ỨNG DỤNG ĐỀU KIỆN CÓ NGHIỆM CỦA PHƯƠNG TRÌNH BẬC BA Suy ra
|27p + 2m (3n + 4) − 9mn| ≤ 18 ⇔ |27p − 3mn + 8m| ≤ 16 8m − 16 ⇒ mn − 9p ≥ . 3 Mặt khác:
P = 18 (ab + bc + ca)2 + 48 (ab + bc + ca) − (ab + bc + ca) (a + b + c) + 9abc
= 2 [3 (ab + bc + ca) + 4]2 − (ab + bc + ca) (a + b + c) + 9abc − 16
= 2 (a + b + c)2 − (ab + bc + ca) (a + b + c) + 9abc − 16 8m − 16 1
= 2m4 + mn − 9p − 16 ≥ 2m4 + − 16 = 6m4 + 8m − 64 . 3 3
Xét hàm số f (m) = 6m4 + 8m − 64, ta có: 1
f 0(m) = 24m3 + 8 ⇒ f 0(m) = 0 ⇔ m = − √ . 3 3 Suy ra 1 6 f (m) ≥ f − √ = − √ − 64. 3 3 3 3 1 2 Nên P ≥ − √ + 64 . 3 3 3  1 m = − √   3  3     1 1
Đẳng thức xảy ra khi và chỉ khi n = √ − 4
, suy ra a,b,c là nghiệm của phương trình 3 3 9     1 4 47  p = √ +   9 3 3 9 1 1 1 1 4 47 : t3 − √ t2 + √ − 4 t + √ + = 0. 3 3 3 3 9 9 3 3 9 1 2 Vậy min P = − √ + 64 . 3 3 3
Câu 1.5. Chuẩn hoá abc = 2 ⇒ a + b + c = 4. Đặt n = ab + bc + ca, suy ra q |18n − 91| ≤ (16 − 3n)3
⇔ 3n3 − 12n2 − 108n + 465 ≤ 0 √ −1 + 5 5
⇔ (n − 5)(3n2 + 3n − 93) ≤ 0 ⇔ 5 ≤ n ≤ . 2 Mặt khác:
a4 + b4 + c4 = a2 + b2 + c22 − 2 a2b2 + b2c2 + c2a2
= (16 − 2n)2 − 2 n2 − 16 = 2n2 − 64n + 288. Nên 1 1 P = a4 + b4 + c4 = n2 − 32n + 144 256 128√ " # 5 5 − 1
Vì hàm f (n) = n2 − 32n + 144 nghịch biến trên 5; nên ta suy ra 2 √ √ ! 1 9 1 5 5 − 1 383 − 165 5 max P = f (5) = và min P = f = . 128 128 128 2 2 158
2. BÀI TOÁN TÌM HẰNG SỐ TỐT NHẤT
§2. Bài toán tìm hằng số tốt nhất 2
Câu 2.1. Vì bất đẳng thức đúng với mọi giá trị a,b,cnên phải đúng với a = b = c = 1 ⇒ k ≤ . 3 2 Ta chứng minh k = là giá trị lớn nhất. 3 2 Xét k =
bất đẳng thức trở thành 3 2
a4 + b4 + c4 + abc (a + b + c) ≥ (ab + bc + ca)2 , 3 hay
3 a4 + b4 + c4 ≥ 2 a2b2 + b2c2 + c2a2 + abc (a + b + c) . (1)
Áp dụng bđt AM – GM ta có
a4 + b4 + b4 + c4 + b4 + c4 ≥ 2a2b2 + 2b2c2 + 2c2a2. Suy ra
3 a4 + b4 + c4 ≥ 3 a2b2 + b2c2 + c2a2 . (2) Mặt khác 1 1 1
a2b2 + b2c2 + c2a2 − abc (a + b + c) = (ab − bc)2 + (bc − ca)2 + (ca − ab)2 ≥ 0. (3) 2 2 2
Từ (2) và (3) suy ra (1) được chứng minh . 2 Vậy số k lớn nhất k = . 3
Câu 2.2. Không mất tính tổng quát, ta giả sử a = min {a; b; c}. Chọn b = c. Từ điều kiện, ta 1 1 có a + 2b = 1 ⇒ ≤ b ≤ . 3 2
Bất đẳng thức trở thành: a 2b 1 + ≥ 1 + 9b2 1 + 9ab + k(a − b)2 2 1 − 2b 1 2b 1 ⇔ − + − ≥ 0 1 + 9b2 6
1 + 9b (1 − 2b) + k(1 − 3b)2 3 (3b + 5) (1 − 3b)
(3b − 1) (6b + 1) − k(3b − 1)2 ⇔ + ≥ 0 6 (1 + 9b2)
3 9b − 18b2 + 1 + k(1 − 3b)2 6b + 1 − k (3b − 1) 3b + 5 ⇔ ≥ 9b − 18b2 + 1 + k(1 − 3b)2 6 (1 + 9b2) 54b3 + 27b2 + 12b − 1 ⇔ ≥ k 9b2 + 4b − 1 18b2 + 15b + 1 ⇔ ≥ k 9b2 + 4b − 1 7b + 3 ⇔ ≥ k − 2. 9b2 + 4b − 1 7b + 3 1 1 Ta xét hàm số f (b) = trên ; . 9b2 + 4b − 1 3 2 1 1 1
Ta dễ dàng thấy được f nghịch biến trên đoạn ; nên suy ra f (b) ≥ f = 2. 3 2 2 159
2. BÀI TOÁN TÌM HẰNG SỐ TỐT NHẤT
Do đó k − 2 ≤ 2 ⇔ k ≤ 4.
Ta chứng minh k = 4 là giá trị lớn nhất cần tìm. Tức là ta cần chứng minh: a b c 1 + + ≥ . (1) 1 + 9bc + 4(b − c)2 1 + 9ca + 4(c − a)2 1 + 9ab + 4(a − b)2 2
Áp dụng Bất đẳng thức Cauchy – Schwarz, ta có: (a + b + c)2
VT ≥ (a + b + c) + 27abc + 4a(b − c)2 + 4b(c − a)2 + 4c(a − b)2 1 = .
1 + 3abc + 4ab (a + b) + 4bc (b + c) + 4ca (c + a)
Do đó để chứng minh (1) ta cần chứng minh:
1 ≥ 3abc + 4ab (a + b) + 4bc (b + c) + 4ca (c + a)
⇔ (a + b + c)3 ≥ 3abc + 4ab (a + b) + 4bc (b + c) + 4ca (c + a)
⇔ a3 + b3 + c3 + 3abc ≥ ab (a + b) + bc (b + c) + ca (c + a)
(điều này đúng, do đây là bất đẳng thức Schur).
Do đó (1) đúng với mọi a, b, c không âm và thỏa a + b + c = 1 và với kmax = 4.
Vậy k = 4 là số cần tìm. a
Câu 2.3. Thử chọn b = c rồi xét hàm theo biến
, ta không dẫn đến kết quả cần tìm. Như vậy b
ta sẽ thử chọn một biến bằng 0, ở đây là một biến tiến dần đến 0.
Trong bất đẳng thức, ta cho c → 0, khi đó a3 + b3 ≥ M ab2. Nhưng lại theo bất đẳng thức AM – GM, ta có b3 b3 3 a3 + b3 = a3 + + ≥ √ · ab2. 2 2 3 4 3 3
Như vậy ta thấy M ≤ √ . Do đó ta chứng minh M = √
là giá trị lớn nhất cần tìm như sau: 3 4 3 4
Không giảm tính tổng quát, giả sử c = min {a,b,c}. Đặt a = u + c, b = v + c với u, v ≥ 0. Ta chứng minh:
(u + c)3 + (v + c)3 + c3 − 3c (u + c) (v + c)
≥ M (u + c) (v + c)2 + (v + c) c2 + c(u + c)2 − 3c (u + c) (v + c)
⇔ (3 − M ) u2 − uv + v2 + u3 + v3 − M uv2 ≥ 0.
Bất đẳng thức cuối cùng luôn đúng vì, 3 − M > 0, u2 − uv + v2 ≥ uv ≥ 0 và v3 v3 3 u3 + v3 = u3 + + ≥ √ uv2 = M uv2. 2 2 3 4
Từ đó ta hoàn tất phần chứng minh. 3
Vậy giá trị lớn nhất cần tìm là: M = √ . 3 4
Câu 2.4. Vì tính thuần nhất của bất đẳng thức nên ta chuẩn hóa xyz = 1. Từ đó ta biến đổi: a b c x = , y = , z =
. Khi đó bất đẳng thức trở thành: b c a a2 b2 c2 a b c + + + 3k ≥ (k + 1) + + bc ca ab b c a
⇔ a3 + b3 + c3 + 3k · abc ≥ (k + 1) ab2 + bc2 + ca2
⇔ a3 + b3 + c3 − 3abc ≥ (k + 1) ab2 + bc2 + ca2 − 3abc . 160
2. BÀI TOÁN TÌM HẰNG SỐ TỐT NHẤT
Đây là bất đẳng thức ở bài trên. 3 3
Vậy k + 1 = √ ⇔ k = −1 + √ . 3 4 3 4
Câu 2.5. Ta chọn b = c thì giả thiết trở thành: 2b − b2 a + 2b = b2 + 2ab ⇒ a = . 2b − 1 Vì a > 0 nên 2b − b2 1 > 0 ⇔ < b ≤ 2. 2b − 1 2
Bất đẳng thức trở thành: 1 2 (a + 2b) + ≥ k (a + 2b + 1) 2b a + b   2b − b2 1 2 2b − b2 ⇒ + 2b    +  ≥ k + 2b + 1 2b − 1  2b 2b − b2  2b − 1 + b 2b − 1 3b2 − b 2k ⇔ ≥ . 3b3 + 5b2 + b − 1 9 3b2 − b 1 Xét hàm số f (b) = trên ; 2 . 3b3 + 5b2 + b − 1 2 Ta có − (b2 − 1) (3b − 1)2 f 0 (b) = 0 ⇔ = 0 ⇔ b = 1. (3b3 + 5b2 + b − 1)2 2
Bằng cách lập bảng biến thiên ta tìm được min f (b) =
khi b = 2. Từ đó tìm được k ≤ 1.  1  9  ;2 2
Ta chứng minh k = 1 là giá trị lớn nhất cần tìm. Thực vậy ta cần chứng minh: 1 1 1 (a + b + c) + + ≥ a + b + c + 1 a + b b + c c + a 1 1 1 ⇔ (ab + bc + ca) + + ≥ a + b + c + 1 a + b b + c c + a ab bc ca ⇔ + + ≥ 1. a + b b + c c + a
Nhận thấy rằng dấu “=” đạt được tại một biến bằng 0 và hai biến bằng 2 nên nếu đánh giá bằng
những bất đẳng thức thông thường có dấu “=” tại tâm sẽ không dẫn đến kết quả. Để đảm bảo
dấu “=” ta sẽ loại bỏ dấu “=” tại biên này bằng cách sử dụng abc ≥ 0 như sau: Theo Cauchy - Schwarz: ab bc ca (ab + bc + ca)2 + + ≥ a + b b + c c + a
ab (a + b) + bc (b + c) + ca (c + a) (ab + bc + ca)2
= (a + b + c) (ab + bc + ca) − 3abc (ab + bc + ca)2 ≥ = 1. (a + b + c) (ab + bc + ca)
Chứng minh hoàn thành. Ta kết luận số k lớn nhất cần tìm là k = 1. 161
2. BÀI TOÁN TÌM HẰNG SỐ TỐT NHẤT
Câu 2.6. Cho b = c, ta có bất đẳng thức trở thành: 1 2 2ab + b2 (a + 2b) + − 9 ≥ k 1 − a b a2 + 2b2 2(a − b)2 (a − b)2 ⇔ ≥ k · ab a2 + 2b2 a2 + 2b2 k ⇔ ≥ . ab 2
Theo bất đẳng thức AM – GM, ta có: a2 + 2b2 √ ≥ 2 2. ab √ Suy ra k ≤ 4 2.
Ta chứng minh đây là giá trị k lớn nhất cần tìm. Sử dụng bất đẳng thức Schur – SOS như sau: 1 1 1 √ ab + bc + ca (a + b + c) + + − 9 ≥ 4 2 1 − a b c a2 + b2 + c2
(a + b + c) (ab + bc + ca) − 9abc √
a2 + b2 + c2 − ab − bc − ca ⇔ ≥ 4 2 · abc a2 + b2 + c2
2c(a − b)2 + (a + b) (a − c) (b − c) √
(a − b)2 + (a − c) (b − c) ⇔ ≥ 4 2 · abc a2 + b2 + c2
⇔ M · (a − b)2 + N · (a − c) (b − c) ≥ 0. √ √ 2c 4 2 a + b 4 2 Trong đó: M = − , N = − . abc a2 + b2 + c2 abc a2 + b2 + c2
Giả sử c = max {a,b,c} thì mọi việc hoàn tất nếu chỉ ra N ≥ 0, hay √
(a + b) a2 + b2 + c2 ≥ 4 2abc.
Do tính thuần nhất nên ta chuẩn hóa a + b + c = 1. Ta đi chứng minh kết quả mạnh hơn sau: √ √ a2 + b2 + c2 ≥ c (a + b)
2 ⇔ 1 − 2c (a + b) ≥ c (a + b) 2 + 2ab. Và mạnh hơn nữa: √ (a + b)2 1 − 2 · c (a + b) ≥ 2 · c (a + b) + 2 √ t2 ⇔ 1 − 2 · t (1 − t) ≥ 2 · t (1 − t) + , với t = a + b 2 √ √ !2 3 − 2 2 √ −2 + 2 t ⇔ t2 − 2 + 2 t + 1 ≥ 0 ⇔ − 1 ≥ 0. 2 2
Điều này đúng. Như vậy chứng minh hoàn tất. √
Vậy số k lớn nhất thỏa đề bài là k = 4 2.
Câu 2.7. Ta chọn a = b = t, a = b = t, c = 3 − 2t. Khi đó, ta có:
k 2t4 + (3 − 2t)4 − 3 ≥ 2t3 + (3 − 2t)3 + 3t2 (3 − 2t) − 6 −12t3 + 45t2 − 54t + 21
⇔ k ≥ 18t4 − 96t3 + 216t2 − 216t + 78 3(t − 1)2 (7 − 4t)
= (t − 1)2 (18t2 − 60t + 78) 7 − 4t = = f (t) . 2 (3t2 − 10t + 13) 162
2. BÀI TOÁN TÌM HẰNG SỐ TỐT NHẤT Ta có: 12 (2t2 − 7t + 3) 1 f 0 (t) = = 0 ⇔ t ∈ 3; . 4(3t2 − 10t + 13)2 2 2 1
Khoảng giá trị của t là (0; 3), từ đó ta thấy minf (t) = khi t = . (0;3) 7 2 1
Vậy trở lại bài toán, ta cho a = b =
, c = 2 thì ta được k ≥ 2 , ta chứng minh giá trị nhỏ nhất 2 7 2 của k là bằng cách chứng minh: 7
2 a4 + b4 + c4 − 3 ≥ a3 + b3 + c3 + 3abc − 6. 7
Đặt f (a,b,c) = V T − V P . Ta chứng minh: a + b a + b f (a,b,c) ≥ f , ,c 2 2 " # " # " # 2 (a + b)4 (a + b)3 (a + b)2 ⇔ a4 + b4 − − a3 + b3 − − 3c ab − ≥ 0 7 8 4 4
(a − b)4 + 6(a − b)2(a + b)2 3 (a + b) (a − b)2 (a − b)2 ⇔ − + 3c · ≥ 0 28 4 4 7a2 + 7b2 + 10ab 3 3c ⇔ − (a + b) + ≥ 0 28 4 4
⇔ 7 a2 + b2 + 10ab − 21 (a + b) + 21c ≥ 0
⇔ 7 a2 + b2 + 10ab − 21 (a + b) + 21 (3 − a − b) ≥ 0
⇔ 5 (a + b)2 + 2 a2 + b2 − 42 (a + b) + 63 ≥ 0.
Do 2 (a2 + b2) ≥ (a + b)2 nên ta sẽ chứng minh:
6(a + b)2 − 42(a + b) + 63 ≥ 0.
Và điều này sẽ đúng nếu ta giả sử c = max {a,b,c} vì khi đó a + b ≤ 2. Vậy ta chỉ cần chỉ ra: a + b a + b a + b f , ,c
≥ 0 ⇔ f (t; t; 3 − t) ≥ 0,t = 2 2 2 2 ⇔
2t4 + (3 − 2t)4 − 3 ≥ 2t3 + (3 − 2t)3 + 3t2 (3 − 2t) − 6. 7
Bất đẳng thức này đúng theo lập luận chặn t phía trên. 2 Vậy k =
là giá trị nhỏ nhất cần tìm. 7
Câu 2.8. Cho b = c giả thiết đã cho viết thành: b2 − 2b 1 a + 2b = b2 + 2ab ⇒ a = ≥ 0 ⇒ ≤ b ≤ 2. 1 − 2b 2
Lưu ý vì tìm số thực k lớn nhất nên xét k > 0.
Thay vào bất đẳng thức, ta được: b2 − 2b b2 (b2 − 2b) 2b + + k ≥ k + 3 1 − 2b 1 − 2b
⇔ 2b (1 − 2b) + b2 − 2b + kb2 b2 − 2b ≥ k (1 − 2b) + 3 (1 − 2b) 3
⇔ 3 (b − 1)2 ≥ k b2 − 1 (b − 1)2 ⇔ ≥ b2 − 1. k 163
2. BÀI TOÁN TÌM HẰNG SỐ TỐT NHẤT
Mà b2 − 1 ≤ 22 − 1 = 3 suy ra k ≤ 1. Ta chứng minh k = 1 là giá trị lớn nhất cần tìm.
Với giả thiết a + b + c = ab + bc + ca, ta chỉ cần chứng tỏ ab + bc + ca + abc ≥ 4.
Ta phản chứng ab + bc + ca + abc = 4 và đi chứng minh a + b + c ≥ ab + bc + ca.
Đây là một kết quả quen thuộc của kì thi VMO 1996.
Câu 2.9. Trong bất đẳng thức ban đầu, ta cho b = c, ta được: a3 + 2b3 b2 + 2ab 3 k + k · ≥ + 2b (a + b) (a + 2b)2 8 3 1 a3 + 2b3 1 1 b2 + 2ab ⇔ − ≥ k · − 2 b(a + b)2 4 3 (a + 2b)2 (a − b)2 (4a + 5b) (a − b)2 ⇔ ≥ k · 8b(a + b)2 3(a + 2b)2 8k (a + 2b)2 (4a + 5b) ⇔ ≤ . 3 b(a + b)2
Do tính thuần nhất nên ta có thể chọn b = 1. Khi đó, ta có: 8k (a + 2)2 (4a + 5) ≤ . 3 (a + 1)2 (a + 2)2 (4a + 5) Ta xét hàm f (a) = , a > 0. Ta có: (a + 1)2 √ 2 (a + 2) (2a2 + 2a − 1) −1 + 3 f 0 (a) = = 0 ⇔ a = (do a > 0). (a + 1)3 2 √ √ ! −1 + 3 √ 3 9 + 6 3
Lập bảng biến thiên ta được f (a) ≥ f = 9 + 6 3. Suy ra: k ≤ . 2 8 √ 3 9 + 6 3
Ta chứng minh giá trị k lớn nhất cần tìm là . 8
Ta sử dụng kĩ thuật Schur – SOS trong phần chứng minh. Bất đẳng thức cần chứng minh viết dưới dạng:
8 (a3 + b3 + c3 − 3abc) − 3 ((a + b) (b + c) (c + a) − 8abc)
a2 + b2 + c2 − ab − bc − ca ≥ k · . 8 (a + b) (b + c) (c + a) 3(a + b + c)2
Ta sử dụng các khai triển:
a3 + b3 + c3 − 3abc = (a + b + c) (a − b)2 + (a − c) (b − c)
(a + b) (b + c) (c + a) − 8abc = 2c (a − b)2 + (a + b) (a − c) (b − c)
a2 + b2 + c2 − ab − bc − ca = (a − b)2 + (a − c) (b − c) .
Từ đó ta nhóm được điều cần chứng minh thành:
M (a − b)2 + N (a − c) (b − c) ≥ 0, trong đó: 8a + 8b + 2c k M = − 8 (a + b) (b + c) (c + a) 3(a + b + c)2 5a + 5b + 8c k N = − . 8 (a + b) (b + c) (c + a) 3(a + b + c)2 164
2. BÀI TOÁN TÌM HẰNG SỐ TỐT NHẤT
Bây giờ, không giảm tổng quát ta giả sử c = min {a,b,c} nên a + b ≥ 2c.
Từ đó 8a + 8b + 2c ≥ 5a + 5b + 8c.
Từ đây ta thấy ngay M ≥ N . Hơn nữa cũng có (a − c) (b − c) ≥ 0.
Hơn nữa cũng có (a − c) (b − c) ≥ 0.
Như vậy bất đẳng thức chứng minh hoàn tất nếu ta chỉ ra được N ≥ 0. Tức là:
3 (5a + 5b + 8c) (a + b + c)2 ≥ 8k (a + b) (b + c) (c + a) .
Đổi biến (a + b,b + c,c + a) → (X,Y,Z). Khi đó cần chứng minh:
3 (X + 4Y + 4Z) (X + Y + Z)2 ≥ 32k · XY Z.
Ta chứng minh kết quả mạnh hơn là: √
3 (X + 4Y + 4Z) (X + Y + Z)2 ≥ 8k · X (Y + Z)2 = 27 + 18 3 X (Y + Z)2 .
Vì tính thuần nhất nên ta có thể chuẩn hóa X + Y + Z = 1. Từ đó ta cần chứng minh: √ √ 4 − 3X
3 (4 − 3X) ≥ 27 + 18 3 X (1 − X)2 ⇔ g (X) = ≥ 9 + 6 3, X(1 − X)2 trong đó 0 < X < 1. √
Khảo sát hàm g trên khoảng (0; 1) ta được g (X) ≥ 9 + 6 3.
Như vậy ta hoàn tất chứng minh. √ 3 9 + 6 3 Vậy k lớn nhất bằng . 8
Câu 2.10. Cho b = c ta được: 2 3 a b 1 + k + k ≥ k + . 2b b + a 2 Cho a → 0 ta được: √  −1 − 5 k ≤
8k (k + 1)2 ≥ (2k + 1)3 ⇔ 4k2 + 2k ≥ 1 ⇔  4  √ .  −1 + 5 k ≥ 4 2a 2b 2c
Ta chứng minh đây là toàn bộ giá trị k cần tìm. Đặt x = , y = , z = thì b + c c + a a + b xy + yz + zx + xyz = 4. Ta cần chứng minh:
(x + 2k) (y + 2k) (z + 2k) ≥ (2k + 1)3
⇔ xyz + 2k (xy + yz + zx) + 4k2 (x + y + z) + 8k3 ≥ (2k + 1)3 .
Theo kết quả VMO 1996, ta được: x + y + z ≥ xy + yz + zx. Từ đó:
xyz + 2k (xy + yz + zx) + 4k2 (x + y + z) + 8k3
≥ xyz + 2k (xy + yz + zx) + 4k2 (xy + yz + zx) + 8k3
= (xy + yz + zx + xyz) + 4k2 + 2k − 1 (xy + yz + zx) + 8k3
= 8k3 + 4 + 4k2 + 2k − 1 (xy + yz + zx) . 165
2. BÀI TOÁN TÌM HẰNG SỐ TỐT NHẤT Hơn nữa ta có: q q (xy + yz + zx)3
xy + yz + zx ≥ 3 3 (xyz)2 ⇒ xyz ≤ √ . 3 3 Từ đó: q (xy + yz + zx)3 t3
4 = xy + yz + zx + xyz ≤ xy + yz + zx + √ = t2 + √ , 3 3 3 3 √ √ với t = xy + yz + zx. Suy ra: t ≥
3⇒ xy + yz + zx ≥ 3. Suy ra:
xyz + 2k (xy + yz + zx) + 4k2 (x + y + z) + 8k3
≥ 8k3 + 4 + 4k2 + 2k − 1 (xy + yz + zx)
≥ 8k3 + 4 + 3 4k2 + 2k − 1 = (2k + 1)3 . Bài toán hoàn tất.
Câu 2.11. Trước hết ta đi tìm điều kiện cần cho k. 1 Thay a = b = x, c =
với x > 0, x 6= 1, bất đẳng thức đã cho có thể viết lại thành x2 2 k k x2 + + ≥ 3 + x 1 4 2x + + 1 x2 2 1 x2 ⇔ x2 + − 3 ≥ k − x 4 2x3 + x2 + 1 (x − 1)2 (x + 2) k(x − 1)2 (2x + 1) ⇔ ≥ x 4 (2x3 + x2 + 1) k (x + 2) (2x3 + x2 + 1) ⇔ ≤ 4 x (2x + 1) k 2 3 ⇔ ≤ x2 + 2x + − , ∀x > 0, x 6= 1. 4 x 2x + 1 √3 + 1 Bây giờ, ta sẽ cho x =
và tính giá trị của biểu thức 4 2 3 f (x) = x2 + 2x + − x 2x + 1 4x + 1 1
ở bên vế phải. Để ý rằng x là nghiệm của phương trình, do dó ta có x2 = , = 8x − 4, 8 x 3 = 4 − 4x. Suy ra 2x + 1 4x + 1 f (x) =
+ 2x + 2 (8x − 4) − (4 − 4x) 8 √ √ 45 95 45 3 + 1 95 45 3 − 50 = x − = · − = . 2 8 2 4 8 8
Với kết quả này, ta thu được √ √ 45 3 − 50 3 k ≤ = 45 ×
− 25 ≈ 45 × 0.866 − 25 ≈ 13.97. 2 2 166
2. BÀI TOÁN TÌM HẰNG SỐ TỐT NHẤT
Mặt khác, vì k là số nguyên nên từ đây có k ≤ 13. Tiếp theo, ta sẽ chứng minh k = 13 thỏa mãn
yêu cầu bài toán, tức là 1 1 1 13 25 + + + ≥ . a b c a + b + c + 1 4 Đặt 1 1 1 13 f (a,b,c) = + + + . a b c a + b + c + 1
Không mất tính tổng quát, giả sử a = max{a,b,c}, ta có √ √ 1 1 2 1 1 f (a,b,c) − f a, bc, bc = + − √ + 13 − √ b c bc a + b + c + 1 a + 2 bc + 1   √ √ 2 1 13 = b − c −  √  . bc (a + b + c + 1) a + 2 bc + 1 1
Do a = max{a; b; c} và giả thiết abc = 1 nên ta có bc ≤ 1, suy ra
≥ 1. Mặt khác, sử dụng bất bc
đẳng thức AM - GM, ta lại có 13 13 13 ≤ = < 1. √ √ √ (a + b + c + 1) a + 2 bc + 1 3 3 abc + 1 3 3 abc + 1 16
Từ đây ta đưa bài toán về chứng minh 1 25 f ,x,x ≥ x2 4 √ với x = bc, 0 < x ≤ 1.
Nếu x = 1 thì bất đẳng thức trên trở thành đẳng thức. Trong trường hợp 0 < x < 1 bằng cách
sử dụng lại biến đổi đã thực hiện trong quá trình tìm điều kiện cho k, ta thấy bất đẳng thức tương đương với (x + 2) (2x3 + x2 + 1) 13 ≥ x (2x + 1) 4
⇔ 4 (x + 2) 2x3 + x2 + 1 ≥ 13x (2x + 1)
⇔ 4 2x4 + 5x3 + 2x2 + x + 2 ≥ 26x2 + 13x
⇔ 8x4 + 20x3 − 18x2 − 9x + 8 ≥ 0. Ta có
8x4 + 20x3 − 18x2 − 9x + 8 = 8x4 − 8x2 + 2 + 20x3 − 20x2 + 5x + 10x2 − 14x + 6
= 2 2x2 − 12 + 5x (2x − 1)2 + 2 5x2 − 7x + 3 > 0. 2
Do 2 (2x2 − 1) ≥ 0,5 · x (2x − 1)2 ≥ 0 và 5x2 − 7x + 3 > 0 (tam thức bất hai có hệ số cao nhất
dương và biệt thức ∆ = −11 < 0).
Như vậy, bất đẳng thức cuối cùng hiển nhiên đúng. Vậy k = 13 là giá trị cần tìm. 167
2. BÀI TOÁN TÌM HẰNG SỐ TỐT NHẤT 1
Câu 2.12. Cho a = b = t > 0, c = ta được t2 1 2 2t6 + 1 t3 + 2 2t2 + + 3k ≥ (k + 1) + t2 ⇔ + 3k ≥ (k + 1) · t4 t t4 t t3 + 2 t3 + 2 2t6 + 1 3t − t3 − 2 −t6 + 2t3 − 1 ⇔ k 3 − ≥ − ⇔ k ≥ t t t4 t t4 2 (t3 − 1) ⇔ k (t − 1)2 (t + 2) ≤ t3 2 (t2 + t + 1) ⇔ k (t + 2) ≤ t3 1 1 2 2 1 + + (t2 + t + 1) t t2 ⇔ k ≤ ⇔ k ≤ . t4 + 2t3 2 1 + t 1 Đặt z = . Khi đó t 2 (z2 + z + 1) k ≤ . 2z + 1
Khi t → +∞ thì z → 0. Như vậy kmax = min f (z) , [0;+∞) 2 (z2 + z + 1) với f (z) = , với z ≥ 0. Ta có 2z + 1
2 (z2 + z + 1) (2z + 1)2 − 2 (z2 + z + 1) f 0 (z) = (2z + 1)2 2 (z2 + z + 1) (2z + 1)2 − 1 = ≥ 0, ∀z ≥ 0. (2z + 1)2
Suy ra hàm số f đồng biến trên [0; +∞), do đó với z ≥ 0 thì f (z) ≥ f (0) = 1. Do đó k ≤ 1.
Từ đó kmax = 1 nếu chỉ ra được 1 1 1 a2 + b2 + c2 + 3 ≥ 2 + + a b c
⇔ a2 + b2 + c2 + 3 ≥ 2 (ab + bc + ca) (do abc = 1)
⇔ a2 + b2 + c2 + 2abc + 1 ≥ 2 (ab + bc + ca) .
Đây là một kết quả quen thuộc được chứng minh bằng nguyên lí Diricle (ta cũng có thể dùng
dồn biến để chứng minh).
Dấu đẳng thức xảy ra khi (a,b,c) = (1; 1; 1); a = b → +∞, c → 0 và các hoán vị.
Câu 2.13. Thông thường trong những dạng toán này, đẳng thức đạt được ngoài giá trị tâm
(1; 1; 1) thì còn đạt được khi hai biến bằng nhau. Như vạy để chặn k, ta chọn a = b = t, c = 3−2t. Ta có: 2 1 +
− 3 ≥ k 2t2 + (3 − 2t)2 − 3 t 3 − 2t
6 − 4t + t − 3 (3t − 2t2) ⇔ ≥ k 6t2 − 12t + 6 3t − 2t2 6t2 − 12t + 6 1 ⇔ k ≤ ⇔ k ≤ . (3t − 2t2) (6t2 − 12t + 6) t (3 − 2t) 168
2. BÀI TOÁN TÌM HẰNG SỐ TỐT NHẤT
Như vậy giá trị c = 3 − 2t lớn nhất cần tìm chính là giá trị lớn nhất của hàm số 1 3 f (t) = , t ∈ 0; . t (3 − 2t) 2
Theo bất đẳng thức AM - GM, ta có 1 1 (2t + 3 − 2t)2 9 t (3 − 2t) = · 2t (3 − 2t) ≤ · = . 2 2 4 8 8
Từ đó kết luận được kmax = nếu chứng tỏ được 9 1 1 1 9 + + − 3 ≥ a2 + b2 + c2 − 3 . (1) a b c 8 1 1 1 9 Đặt f (a,b,c) = + + − 3 −
(a2 + b2 + c2 − 3). Xét hiệu a b c 8 a + b a + b f (a,b,c) − f , ,c 2 2 1 1 1 8 = + + − 3 − a2 + b2 + c2 − 3 − a b c 9 ! 4 1 8 (a + b)2 − − + 3 + + c2 − 3 a + b c 9 2 ! 1 1 4 8 (a + b)2 = + − − a2 + b2 − a b a + b 9 2 (a + b)2 − 4ab 8 a2 + b2 − 2ab = − · ab (a + b) 9 2 1 4 = (a − b)2 − . ab (a + b) 9
Ta giả sử c = max {a,b,c}. Khi đó a + b ≤ 2, suy ra (a + b)3 8 9 ab (a + b) ≤ ≤ < . 4 4 4 Như vậy a + b a + b f (a,b,c) ≥ f , ,c . 2 2 Ta chỉ cần chứng minh a + b a + b a + b f , ,c
≥ 0 ⇔ f (t,t,3 − 2t) ≥ 0, với t = . 2 2 2 Tức là chứng minh 2 1 8 + − 3 ≥ 2t2 + (3 − 2t)2 − 3 t 3 − 2t 9 8 1 24t − 16t2 − 9 ⇔ ≤ ⇔ ≤ 0 9 t (3 − 2t) t (3 − 2t) 16t2 − 24t + 9 (4t − 3)2 ⇔ ≥ 0 ⇔ ≥ 0 (đúng). t (3 − 2t) t (3 − 2t) 8
Vậy (∗) được chứng minh. Từ đó kết luận kmax = . 9 169
2. BÀI TOÁN TÌM HẰNG SỐ TỐT NHẤT
Câu 2.14. Kí hiệu (1) là bất đẳng thức đã nêu trong đề bài. Giả sử k là số thực sao cho bất
đẳng thức (1) đúng với mọi bộ ba số thực a, b, c, mà abc ≥ 0. 1
Trong (1) thay a = 0, b = c = 2, ta được 2 + 8k ≥ 4. Suy ra k ≥ . 4 1 Ta sẽ chứng minh k =
là giá trị nhỏ nhất cần tìm, tức chứng minh với mọi bộ ba số thực 4
không âm a, b, c, ta luôn có 1 abc +
(a − b)2 + (b − c)2 + (c − a)2 + 2 ≥ a + b + c. 4 hay
a2 + b2 + c2 + 2abc + 4 ≥ ab + bc + ca + 2 (a + b + c) . (2)
Xét 3 số (b − 1) (c − 1), (c − 1) (a − 1), (a − 1) (b − 1), ta có
a (b − 1) (c − 1) · b (c − 1) (a − 1) · c (a − 1) (b − 1) = abc (a − 1)2 (b − 1)2 (c − 1) ≥ 0.
Suy ra có ít nhất một số trong 3 số nêu trên không âm. Không mất tính tổng quát, giả sử
a (b − 1) (c − 1) ≥ 0. Khi đó, ta có abc ≥ a (b + c − 1).
Do vậy, bất đẳng thức (2) sẽ được chứng minh, nếu ta chứng minh được
a2 + b2 + c2 + 2a (b + c − 1) + 4 ≥ ab + bc + ca + 2 (a + b + c) , hay
a2 − (4 − b − c) a + b2 + c2 − bc − 2 (b + c) + 4 ≥ 0, (3)
với mọi a, b, c, mà abc ≥ 0.
Ta xem vế trái của (3) là một tam thức bậc hai theo ẩn a, ta có:
∆ = (b + c − 4)2 − 4b2 − 4c2 + 4bc + 8 (b + c) − 16 = −3b2 − 3c2 + 6bc = −3 (b − c)2 ≤ 0,
với mọi b, c nên (3) đúng với mọi a, b, c, mà abc ≥ 0. 1 Vậy k =
là giá trị cần tìm, theo yêu cầu của đề bài. 4
Câu 2.15. Rõ ràng, các giá trị x = y = z = 1 thỏa mãn ràng buộc nêu trong đề bài. Vì thế,
trong bất đẳng thức của đề bài, cho x = y = z = 1 ta được k + 1 ≥ 2. Suy ra k ≥ 1.
Tiếp theo, ta chứng minh với k = 1, bất đẳng thức của đề bài là một bất đẳng thức đúng; tức ta sẽ chứng minh x + y + z 2 3
p(x2 + 1) (y2 + 1) (z2 + 1) ≤ + 1, (1) 3
với mọi số thực dương x, y, z mà min {xy,yz,zx} ≥ 1.
Thật vậy, trước hết, ta chứng minh nhận xét sau: Nhận xét: Với a, b là hai số thực dương thỏa mãn ab ≥ 1, ta luôn có: ! a + b 2 a2 + 1 b2 + 1 ≤ + 1 . 2 a + b 2
Chứng minh: Vì ab ≥ 1 nên
− 1 ≥ ab − 1 ≥ 0. Do đó 2 !2 !2 a + b 2 a + b 2
a2 + 1 b2 + 1 = (ab − 1)2 + (a + b)2 ≤ − 1 + (a + b)2 = + 1 . 2 2 170
2. BÀI TOÁN TÌM HẰNG SỐ TỐT NHẤT
Nhận xét được chứng minh.
Không mất tổng quát, giả sử x ≤ y ≤ z. Khi đó, từ ràng buộc của đề bài, suy ra x ≥ 1. x + y + z Đặt t = . Ta có 3 x (x + y + z) x2 + xy + xz 1 + 1 + 1 xt = = ≥ = 1. 3 3 3
Do đó, áp dụng nhận xét lần lượt cho cặp (x,t) và cặp (y,z), ta được !2 x + t 2 x2 + 1 t2 + 1 ≤ + 1 , (2) 2 và !2 y + z 2 y2 + 1 z2 + 1 ≤ + 1 . (3) 2
Nhân (2) và (3), vế theo vế, ta được !2 !2 x + t 2 y + z 2
x2 + 1 y2 + 1 z2 + 1 t2 + 1 ≤ + 1 + 1 . (4) 2 2 x + t y + z √ √ Nhận thấy · ≥ xt ·
yz ≥ 1. Do đó, theo nhận xét, ta có 2 2 !2 !2 !4 x + t 2 y + z 2 x + y + z + t 2 + 1 + 1 ≤ + 1 . (5) 2 2 4 Từ (4) và (5), suy ra
x2 + 1 y2 + 1 z2 + 1 t2 + 1 ≤ t2 + 14 . Do đó x + y + z 2 3
p(x2 + 1) (y2 + 1) (z2 + 1) (t2 + 1) ≤ + 1. 3
Suy ra (1) được chứng minh và vì thế, giá trị k nhỏ nhất cần tìm theo yêu cầu bài là k = 1.
Câu 2.16. Kí hiệu (1) là bất đẳng thức đã nêu trong đề bài. Giả sử k là số thực sao cho bất
đẳng thức (1) đúng với mọi bộ ba số thực a, b, c là độ dài ba cạnh của một tam giác.
Trong (1) thay b = c > 0 ta được a 2b 2ab + b2 3 + + k · ≤ + k. 2b a + b a2 + 2b2 2 k 3
Cố định b và cho a tiến tới 0+ ta được 2 + ≤ + k. Suy ra k ≥ 1. 2 2
Ta sẽ chứng minh k = 1 là giá trị nhỏ nhất cần tìm, tức chứng minh với mọi bộ ba số thực a, b,
c là độ dài ba cạnh của một tam giác, ta luôn có a b c ab + bc + ca 5 + + + ≤ . (2) b + c c + a a + b a2 + b2 + c2 2 171
2. BÀI TOÁN TÌM HẰNG SỐ TỐT NHẤT Thật vậy, ta có: a b c 1 ab + bc + ca (2) ⇔ 1 − + 1 − + 1 − ≥ + b + c c + a a + b 2 a2 + b2 + c2 b + c − a c + a − b a + b − c (a + b + c)2 ⇔ + + ≥ b + c c + a a + b 2 (a2 + b2 + c2) (b + c − a)2 (c + a − b)2 (a + b − c)2 (a + b + c)2 ⇔ + + ≥ . (3) (b + c) (b + c − a) (c + a) (c + a − b) (a + b) (a + b − c) 2 (a2 + b2 + c2)
Do a, b, c là độ dài ba cạnh của một tam giác nên b + c − a > 0, c + a − b > 0, a + b − c > 0.
Do đó tất cả các phân thức nằm ở vế trái của (3) đều có mẫu thức dương. Vì thế, kí hiệu V T là
biểu thức nằm ở vế trái của (3), theo bất đẳng thức Cauchy – Schwarz dạng Engel, ta có: (a + b + c)2 V T ≥ . 2 (a2 + b2 + c2) Vì
(b + c − a) + (c + a − b) + (a + b − c) = a + b + c và
(b + c − a) (b + c) + (c + a − b) (c + a) + (a + b − c) (a + b) = 2 a2 + b2 + c2 .
Nên (3) được chứng minh và vì thế (2) được chứng minh.
Vậy k = 1 là giá trị cần tìm, theo yêu cầu của đề bài. 1
Câu 2.17. Rõ ràng, các giá trị x = y = z =
thỏa mãn ràng buộc nêu trong đề bài. Vì thế, 3 1 9 9k + 3
trong bất đẳng thức của đề bài, cho x = y = z = ta được ≤ , mà k > 0 nên suy 3 3k + 2 2 1 ra k ≥ . 3 1
Tiếp theo, ta chứng minh với k =
, bất đẳng thức của đề bài là một bất đẳng thức đúng; tức 3 ta sẽ chứng minh 1 1 1 + + ≤ 3, (1) 1 1 1 x + y + y + z + z + x + 3 3 3 √ √ √
với mọi số thực dương x, y, z mà xy + yz + zx = 1. Thật vậy, đặt √ √ √ T = x + y + z2 = x + y + z + 2.
Áp dụng bất đẳng thức Cauchy-Schwarz cho hai bộ 3 số dương, ta có 1 1 1 √ 1 √ 1 1 √ 2 T x + y + + + z ≥ x · √ + y · √ + √ · z = 3 3 3 3 3 3 3 hay 1 3z + 2 ≤ . 1 T x + y + 3 Tương tự ta có: 1 3x + 2 1 3y + 2 ≤ ; ≤ . 1 T 1 T y + z + z + x + 3 3 172
2. BÀI TOÁN TÌM HẰNG SỐ TỐT NHẤT
Cộng 3 bất đẳng thức vừa nêu trên, vế với vế, với lưu ý T = x + y + z + 2, ta được 1 1 1 3z + 2 + 3x + 2 + 3y + 2 + + ≤ = 3. 1 1 1 T x + y + y + z + z + x + 3 3 3 (1) được chứng minh. 1 Vậy k =
là giá trị cần tìm, theo yêu cầu của đề bài. 3
Câu 2.18. Kí hiệu (1) là bất đẳng thức cần chứng minh.
Trong (1) cho a = b = c = 1, suy ra k ≤ 729. Ta chứng minh (1) đúng với k = 729. Từ giả thiết ta có q
3 |abc| ≥ |ab| + |bc| + |ca| ≥ 3 3 (abc)2 ⇒ abc ≥ 1. Do đó
a2 + 4 b2 + c2 b2 + 4 a2 + c2 c2 + 4 a2 + b2
≥ a2 + 8 |bc| b2 + 8 |ac| c2 + 8 |ab| ≥ 93 (abc)2 ≥ 729.
Sử dụng bất đẳng thức Cauchy cho chín số a2 và 8 số |bc| . Vậy max k = 729. 1
Câu 2.19. Gọi (1) là bất đẳng thức cần tìm. Trong (1) cho a1 = 1; a2 = · · · = an = − ta n − 1 n được c ≤ . 2 n
Ta chứng minh BĐT (1) đúng với c = . Tức là cần chứng minh 2 n X n X |ai − aj| ≥ |ai| (2) 2 1≤ii=1
Ta có thể giả sử a1 ≥ a2 ≥ · · · ≥ an khi đó tồn tại k với (1 ≤ k ≤ n − 1) sao cho a1 ≥ a2 ≥ · · · ≥
ak ≥ 0 ≥ ak+1 ≥ · · · ≥ an. Khi đó ta có a1 + a2 + · · · + ak = − (ak+1 + · · · + an) và n n n X X X X X |ai − aj| = (ai − aj) = (n + 1 − 2i) ai; |ai| = 2 ai. 1≤i1≤ii=1 i=1 i=1 BĐT (2) trở thành n k k n k X X X X X (n + 1 − 2i) ai ≥ n ai ⇔ (n + 1 − 2i) ai + (n + 1 − 2i) ai ≥ n ai (3) i=1 i=1 i=1 i=k+1 i=1
Ta có a1 ≥ a2 ≥ · · · ≥ ak và n + 1 − 2.1 ≥ · · · ≥ n + 1 − 2.k.
Áp dụng bất đẳng thức Chebychev ta được k " k ! k !# k ! X 1 X X X (n + 1 − 2i) ai ≥ (n + 1 − 2i) ai = (n − k) ai . (4) k i=1 i=1 i=1 i=1
Tương tự thì ak+1 ≥ · · · ≥ an và n + 1 − 2(k + 1) ≥ · · · ≥ n + 1 − 2.n nên n " n ! n !# n ! k ! X 1 X X X X (n + 1 − 2i) ai ≥ (n + 1 − 2i) ai = −k ai = k ai . (5) n − k i=k+1 i=k+1 i=k+1 i=k+1 i=1
Từ (4) và (5) suy ra BĐT (3) đúng. n Vậy max cn = . 2 173
2. BÀI TOÁN TÌM HẰNG SỐ TỐT NHẤT
Câu 2.20. Ta chứng minh max ak < 2 và max bk < 2 . n−1 k=1,n k=1,n
Thật vậy, đặt L = max ak, từ (ii) suy ra L2 ≤ 1 + L ⇒ L < 2. k=1,n
Đặt bm = max bk, từ (i) dễ thấy k=1,n (k − 1) b b m + (m − k) b1 k ≥ , nếu 1 ≤ k ≤ m và m − 1 (k − m) b b n + (n − k) bm k ≥ , nếu m ≤ k ≤ n. n − m (k − 1) bm (n − k) bm Suy ra bk > nếu 1 ≤ k ≤ m và bk ≥ nếu m ≤ k ≤ n. m − 1 n − m Do đó n m n m ! X X X 1 X 1 = bk = bk + bk > (k − 1) bm m − 1 k=1 k=1 k=m+1 k=1 n ! 1 X n − 1 2 + (n − k) bm = .bm ⇒ bm < . n − m 2 n − 1 k=m+1 √ Đặt x0 = 1; xk = 1+Pk a ≤ x x i=1
ibi, k = 1,n thì xk − xk−1 = ak bk . Từ (ii) suy ra a2 k k ⇒ ak ≤ k .
Từ max ak < 2 suy ra xk − xk−1 < 2bk. Do đó với mọi k = 1,n thì k=1,n √ ! √ √ xk 1 x x k − xk−1 k − xk−1 < bk. √ √ = bk + √ x √ 2 k + xk−1 2 2 xk + xk−1 1 b 1 1 < b k k + < bk + . 2 4 2 2 (n − 1)
Lấy tổng k từ 1 đến n ta được √ √ n X 1 1 3 1 an ≤ xn < x0 + bk + = + . 2 2 (n − 1) 2 2 (n − 1) k=1 3
Cho n → +∞ ta được M = an ≤ . 2 k 1 3
Cuối cùng chọn bộ ak = 1 + ; bk =
thỏa các điều kiện đề bài. Vậy max M = . 2n n 2 174
Document Outline

  • Các bất đẳng thức cổ điển
    • Bất đẳng thức AM - GM
      • Bất đẳng thức AM - GM
      • Một số ví dụ áp dụng
      • Bài tập
    • Bất đẳng thức Cauchy - Schwarz
      • Bất đẳng thức Cauchy-Schwarz dạng đa thức
      • Bất đẳng thức Cauchy-Schwarz dạng phân thức
      • Các ví dụ minh họa
      • Bài tập
    • Một số bất đẳng thức khác
      • Bất đẳng thức Schur
        • Bất đẳng thức Schur
        • Các trường hợp đặc biệt
        • Bất đẳng thức Schur mở rộng
        • Các ví dụ
      • Bất đẳng thức Holder
        • Bất đẳng thức Holder
        • Trường hợp đặc biệt
        • Ví dụ minh họa
      • Bất đẳng thức Chebyshev
        • Bất đẳng thức Chebyshev
        • Ví dụ minh họa
      • Bài tập
    • Phương pháp quy nạp
      • Lý thuyết
      • Ví dụ minh họa
    • Phương pháp phân tích bình phương SOS
      • Lý thuyết
        • Một số tiêu chuẩn đánh giá
        • Một số biểu diễn cơ sở
      • Các ví dụ
      • Bài tập
    • Phương pháp dồn biến
      • Lý thuyết
      • Ví dụ minh họa
      • Bài tập
    • Các phương pháp chứng minh bất đẳng thức hiện đại
      • Phương pháp p, q, r
        • Lý thuyết
          • Bất đẳng thức Schur
          • Một số biểu diễn đa thức đối xứng ba biến qua p, q, r
          • Một số đánh giá giữa p, q, r
        • Một số ví dụ
        • Bài tập
      • Phương pháp sử dụng tiếp tuyến và cát tuyến
        • Lý thuyết
          • Hàm lồi - Dấu hiệu hàm lồi
          • Bất đẳng thức tiếp tuyến - Bất đẳng thức cát tuyến
        • Các ví dụ minh họa
        • Bài tập
      • Một số chuyên đề
        • Ứng dụng điều kiện có nghiệm của phương trình bậc ba trong chứng minh bất đẳng thức
          • Lý thuyết
            • Mở đầu
            • Một số kết quả
          • Ví dụ minh họa
          • Bài tập
        • Bài toán tìm hằng số tốt nhất trong bất đẳng thức
          • Lý thuyết
          • Ví dụ minh họa
          • Bài tập
        • Các bất đẳng thức cổ điển
          • Bất đẳng thức AM-GM
            • Bất đẳng thức Cauchy-Schwarz
              • Một số bất đẳng thức khác
                • Một số phương pháp chứng minh bất đẳng thức
                  • Phương pháp quy nạp
                    • Phương pháp phân tích bình phương SOS
                      • Phương pháp dồn biến
                      • Phương pháp p, q, r
                      • Phương pháp tiếp tuyến và cát tuyến
                    • Một số chuyên đề
                      • Ứng dụng đều kiện có nghiệm của phương trình bậc ba
                        • Bài toán tìm hằng số tốt nhất